Matematicas Para Olimpiadas

March 27, 2018 | Author: Abraham Hernandez | Category: Integer, Prime Number, Square Root, Natural Number, Factorization


Comments



Description

Taller de resoluciónde problemas de concurso Universidad de Puerto Rico Colegio Universitario de Cayey Dr. David A. SANTOS Versión del January 3, 2010 Contents Prefacio 1 2 3 4 v Técnicas elementales 1.1 Contradicción . . . . . . . . . . . . . . Tarea . . . . . . . . . . . . . . . . . . . . . . 1.2 Principio de las pichoneras de Dirichlet Tarea . . . . . . . . . . . . . . . . . . . . . . 1.3 Paridad . . . . . . . . . . . . . . . . . Tarea . . . . . . . . . . . . . . . . . . . . . . 1.4 Inducción . . . . . . . . . . . . . . . . Tarea . . . . . . . . . . . . . . . . . . . . . . 1.5 Buen orden . . . . . . . . . . . . . . . Tarea . . . . . . . . . . . . . . . . . . . . . . 1.6 Condiciones extremas . . . . . . . . . . Tarea . . . . . . . . . . . . . . . . . . . . . . . . . . . . . . . . . . . . . . . . . . . . . . . . . . . . . . . . . . . . . . . . . . . . . . . . . . . . . . . . . . . . . . . . . . . . . . . . . . . . . . . . . . . . . . . . . . . . . . . . . . . . . . . . . . . . . . . . . . . . . . . . . . . . . . . . . . . . . . . . . . . . . . . . . . . . . . . . . . . . . . . . . . . . . . . . . . . . . . . . . . . . . . . . . . . . . . . . . . . . . . . . . . . . . . . . . . . . . . . . . . . . . . . . . . . . . . . . . . . . . . . . . . . . . . . . . . . . . . . . . . . . . . . . . . . . . . . . . . . . . . . . . . . . . . . . . . . . . . . . . . . . . . . . . . . . . . . . . . . . . . . . . . . . . . . . . . . . . . . . . . . . . . . . . . . . . . . . . . . . . . . . . . . . . . . . . . . . . . . . . . . . . . . . . . . . . . . . . . . . . . . . . . . . . . . . . . . . . . . . . . . . . . . . . . . . . . . . . . . . . . . . . . . . . . . . . . . . . . 1 1 2 2 4 4 6 6 7 8 9 9 10 Álgebra y aritmética 2.1 Identidades algebraicas Tarea . . . . . . . . . . . . . 2.2 Los enteros . . . . . . Tarea . . . . . . . . . . . . . 2.3 Aritmética modular . . Tarea . . . . . . . . . . . . . . . . . . . . . . . . . . . . . . . . . . . . . . . . . . . . . . . . . . . . . . . . . . . . . . . . . . . . . . . . . . . . . . . . . . . . . . . . . . . . . . . . . . . . . . . . . . . . . . . . . . . . . . . . . . . . . . . . . . . . . . . . . . . . . . . . . . . . . . . . . . . . . . . . . . . . . . . . . . . . . . . . . . . . . . . . . . . . . . . . . . . . . . . . . . . . . . . . . . . . . . . . . . . . . . . . . . . . . . . . . . . . . . . . . . . . . . . 11 11 15 16 20 20 24 Combinatoria 3.1 Las reglas de la multiplicación y la suma Tarea . . . . . . . . . . . . . . . . . . . . . . 3.2 Métodos combinatorios . . . . . . . . . 3.2.1 Permutaciones sin repetición . . 3.2.2 Permutaciones con repetición . 3.2.3 Combinaciones sin repetición . 3.2.4 Combinaciones con repetición . 3.3 Principio de inclusión-exclusión . . . . Tarea . . . . . . . . . . . . . . . . . . . . . . . . . . . . . . . . . . . . . . . . . . . . . . . . . . . . . . . . . . . . . . . . . . . . . . . . . . . . . . . . . . . . . . . . . . . . . . . . . . . . . . . . . . . . . . . . . . . . . . . . . . . . . . . . . . . . . . . . . . . . . . . . . . . . . . . . . . . . . . . . . . . . . . . . . . . . . . . . . . . . . . . . . . . . . . . . . . . . . . . . . . . . . . . . . . . . . . . . . . . . . . . . . . . . . . . . . . . . . . . . . . . . . . . . . . . . . . . . . . . . . . . . . . . . . . . . . . . . . . . . . . . . . . . . . . . . . . . . . . . . . . . . . . . . . . . . . . . . . . . . . . . . . . . . . . . . . . . . . . . . . . . . . . . . . . . . . . . . . . . . . . . . 26 26 30 31 32 33 35 38 39 44 Sumas y recurrencias 4.1 Progresiones aritméticas . . . . . . . . . . . . . . . . . . . . . . . . . . . . . . . . . . . . . . . . . . . . . . Tarea . . . . . . . . . . . . . . . . . . . . . . . . . . . . . . . . . . . . . . . . . . . . . . . . . . . . . . . . . . . . 4.2 Progresiones geométricas . . . . . . . . . . . . . . . . . . . . . . . . . . . . . . . . . . . . . . . . . . . . . . 45 45 47 48 . . . . . . . . . . . . . . . . . . . . . . . . . . . . . . . . . . . . . . . . . . . . . . . . ii CONTENTS 5 6 7 iii Tarea . . . . . . . . . . . . . . . . . . . . . 4.3 Cancelación telescópica . . . . . . . . Tarea . . . . . . . . . . . . . . . . . . . . . 4.4 Recursiones y ecuaciones funcionales Tarea . . . . . . . . . . . . . . . . . . . . . . . . . . . . . . . . . . . . . . . . . . . . . . . . . . . . . . . . . . . . . . . . . . . . . . . . . . . . . . . . . . . . . . . . . . . . . . . . . . . . . . . . . . . . . . . . . . . . . . . . . . . . . . . . . . . . . . . . . . . . . . . . . . . . . . . . . . . . . . . . . . . . . . . . . . . . . . . . . . . . . . . . . . . . . . . . . . . . . . . . . . . . . . . . . . . . . . . . 51 52 58 59 62 Polinomios y ecuaciones 5.1 Ecuaciones . . . . Tarea . . . . . . . . . . . 5.2 Polinomios . . . . Tarea . . . . . . . . . . . . . . . . . . . . . . . . . . . . . . . . . . . . . . . . . . . . . . . . . . . . . . . . . . . . . . . . . . . . . . . . . . . . . . . . . . . . . . . . . . . . . . . . . . . . . . . . . . . . . . . . . . . . . . . . . . . . . . . . . . . . . . . . . . . . . . . . . . . . . . . . . . . . . . . . . . . . . . . 63 63 66 68 74 Desigualdades 6.1 Desigualdades del triángulo . . . . . . . . . . . Tarea . . . . . . . . . . . . . . . . . . . . . . . 6.2 El cuadrado de todo real es positivo . . . . . . Tarea . . . . . . . . . . . . . . . . . . . . . . . 6.3 Desigualdades de las medias . . . . . . . . . . Tarea . . . . . . . . . . . . . . . . . . . . . . . 6.4 Desigualdad de Cauchy-Schwarz-Bunyakovsky Tarea . . . . . . . . . . . . . . . . . . . . . . . 6.5 Desigualdad del reordenamiento . . . . . . . . Tarea . . . . . . . . . . . . . . . . . . . . . . . . . . . . . . . . . . . . . . . . . . . . . . . . . . . . . . . . . . . . . . . . . . . . . . . . . . . . . . . . . . . . . . . . . . . . . . . . . . . . . . . . . . . . . . . . . . . . . . . . . . . . . . . . . . . . . . . . . . . . . . . . . . . . . . . . . . . . . . . . . . . . . . . . . . . . . . . . . . . . . . . . . . . . . . . . . . . . . . . . . . . . . . . . . . . . . . . . . . . . . . . . . . . . . . . . . . . . . . . . . . . . . . . . . . . . . . . . . . . . . . . . . . . . . . . . . . . . . . . . . . . . . . . . . . . . . . . . . . . . . . . . . . . . . . . . . . . . . . . . . . . . . . . . . . . . . . . . . . . . . . . . . . . . . . . . . . . . . . . . . . . 75 75 79 80 84 85 90 90 93 93 97 Geometría plana 7.1 Ángulos . . . . . . . . . . . . . . . . . . . . . . . . . Tarea . . . . . . . . . . . . . . . . . . . . . . . . . . . . . . 7.2 Congruencia de triángulos y desigualdad del triángulo . Tarea . . . . . . . . . . . . . . . . . . . . . . . . . . . . . . 7.3 Trapecios y paralelogramos . . . . . . . . . . . . . . . Tarea . . . . . . . . . . . . . . . . . . . . . . . . . . . . . . 7.4 Perímetros y áreas . . . . . . . . . . . . . . . . . . . . Tarea . . . . . . . . . . . . . . . . . . . . . . . . . . . . . . 7.5 Teorema de Pitágoras . . . . . . . . . . . . . . . . . . Tarea . . . . . . . . . . . . . . . . . . . . . . . . . . . . . . 7.6 Proporcionalidad y semejanza . . . . . . . . . . . . . Tarea . . . . . . . . . . . . . . . . . . . . . . . . . . . . . . 7.7 Construcciones con regla y compás . . . . . . . . . . . Tarea . . . . . . . . . . . . . . . . . . . . . . . . . . . . . . 7.8 Repaso de Trigonometría . . . . . . . . . . . . . . . . Tarea . . . . . . . . . . . . . . . . . . . . . . . . . . . . . . 7.9 Repaso de Geometría Analítica . . . . . . . . . . . . . Tarea . . . . . . . . . . . . . . . . . . . . . . . . . . . . . . 7.10 Vectores . . . . . . . . . . . . . . . . . . . . . . . . . Tarea . . . . . . . . . . . . . . . . . . . . . . . . . . . . . . 7.11 Baricentros . . . . . . . . . . . . . . . . . . . . . . . Tarea . . . . . . . . . . . . . . . . . . . . . . . . . . . . . . 7.12 Transformaciones geométricas . . . . . . . . . . . . . Tarea . . . . . . . . . . . . . . . . . . . . . . . . . . . . . . 7.13 Teoremas de Ceva y de Menelao . . . . . . . . . . . . Tarea . . . . . . . . . . . . . . . . . . . . . . . . . . . . . . 7.14 Puntos y rectas notables de un triángulo . . . . . . . . Tarea . . . . . . . . . . . . . . . . . . . . . . . . . . . . . . 7.15 Potencia de un punto con respecto a un círculo . . . . . . . . . . . . . . . . . . . . . . . . . . . . . . . . . . . . . . . . . . . . . . . . . . . . . . . . . . . . . . . . . . . . . . . . . . . . . . . . . . . . . . . . . . . . . . . . . . . . . . . . . . . . . . . . . . . . . . . . . . . . . . . . . . . . . . . . . . . . . . . . . . . . . . . . . . . . . . . . . . . . . . . . . . . . . . . . . . . . . . . . . . . . . . . . . . . . . . . . . . . . . . . . . . . . . . . . . . . . . . . . . . . . . . . . . . . . . . . . . . . . . . . . . . . . . . . . . . . . . . . . . . . . . . . . . . . . . . . . . . . . . . . . . . . . . . . . . . . . . . . . . . . . . . . . . . . . . . . . . . . . . . . . . . . . . . . . . . . . . . . . . . . . . . . . . . . . . . . . . . . . . . . . . . . . . . . . . . . . . . . . . . . . . . . . . . . . . . . . . . . . . . . . . . . . . . . . . . . . . . . . . . . . . . . . . . . . . . . . . . . . . . . . . . . . . . . . . . . . . . . . . . . . . . . . . . . . . . . . . . . . . . . . . . . . . . . . . . . . . . . . . . . . . . . . . . . . . . . . . . . . . . . . . . . . . . . . . . . . . . . . . . . . . . . . . . . . . . . . . . . . . . . . . . . . . . . . . . . . . . . . . . . . . . . . . . . . . . . . . . . . . . . . . . . . . . . . . . . . . . . . . . . . . . . . . . . . . . . . . . . . . . . . . . . . . . . . . . . . . . . . . . . . . . . . . . . . . . . . . . . . . . . . . . . . . . . . . . . . . . . . . . . . . . . . . . . . . . . . . . . . . . . . . . . . . . . . . . . . . . . . . . . . . . . . . . . . . . . . . . . . . . . . . . . . . . . . . . . . . . . . . . . . . . . . . . . . . . . . . . . . . . . . . . . . . . . . . . . . . . . . . . . . . . . . . . . . . . . . . . . . . . . . . . . . . . . . . . . . . . . . . . . . . . . . . . . 98 98 112 113 118 118 125 125 126 128 130 131 137 138 143 144 152 154 159 160 167 168 172 173 178 178 184 185 192 192 . . . . . . . . . . . . . . . . . . . . . . . . . . . . . . . . . . . . . . . . iv CONTENTS Tarea . . . . . . . . . . . . . . . . . . . . . . . . . . . . . . . . . . . . . . . . . . . . . . . . . . . . . . . . . . . . 195 A Indicaciones y respuestas 196 Indicaciones y respuestas . . . . . . . . . . . . . . . . . . . . . . . . . . . . . . . . . . . . . . . . . . . . . . . . . 196 ya que muchos lectores se han comunicado conmigo. El hacerlas accesibles en la red me ha hecho percatar cuán útiles han sido para estudiantes de habla castellana. David A. de mes en mes. SANTOS [email protected] v . teoría de grafos. en un taller de resolución de problemas de concurso para maestros de secundaria en Puerto Rico. Bastante del material que aquí aparece es traducción de material que he escrito en lengua gringa. Todavía necesito añadir material en geometría. iré añadiendo material y las respuestas de varios ejercicios. enumeración y análisis. Quisiera pues pedir a los lectores el que me comunicasen errores que hallaren. etc. Por eso decidido el revisarlas frecuentemente. De semana en semana.Prefacio Escribí estas notas en el verano del 1996. Invito también a los lectores a contribuir material. BUT NOT LIMITED TO.0 or later (the latest version is presently available at http://www. INCLUDING.org/openpub/. EXPRESS OR IMPLIED.opencontent. interdit à tous. . aux riches comme aux pauvres de dormir sous les ponts. – Anatole France (Les Lys Rouge . THE IMPLIED WARRANTIES OF MERCHANTABILITY AND FITNESS FOR A PARTICULAR PURPOSE OR A WARRANTY OF NON-INFRINGEMENT.vi Aviso Legal La loi dans sa majestueuse égalité. de mendier dans la rue et de voler du pain. THIS WORK IS LICENSED AND PROVIDED “AS IS” WITHOUT WARRANTY OF ANY KIND. THIS DOCUMENT MAY BE FREELY DISTRIBUTED PROVIDED THE NAME OF THE ORIGINAL AUTHOR(S) IS(ARE) KEPT AND ANY CHANGES TO IT NOTED. THIS DOCUMENT MAY NOT BE SOLD FOR PROFIT OR INCORPORATED INTO COMMERCIAL DOCUMENTS WITHOUT EXPRESS PERMISSION FROM THE AUTHOR(S). version 1.1894) This material may be distributed only subject to the terms and conditions set forth in the Open Publication License. siendo el producto de cuatro enteros nones. Demuéstrese que el producto (a1 − 1)(a2 − 2) · · · (an − n) es par. Presúmase al contrario. . Luego entonces se concluye que 6 − 35 < . ◭ 2 Ejemplo El producto de 34 enteros es igual a 1. a2 . debe de haber un número par de −1’s. n. El lado diestro es non. ◭ 3 Ejemplo Demuéstrese. . Luego así habrán de haber diecisiete −1’s y diecisiete +1’s. Ya que el producto es 1. lo que conlleva a una contradicción. 10 √ √ √ 1 1 Presúmase que 6 − 35 ≥ . y. por ser la suma de cuatro enteros nones. ◮Resolución: Obsérvese primero que la suma de un número impar de enteros impares impar es. Esto resulta en una contradicción. . x y z w Demuéstrese que al menos uno de ellos es par. Si la suma de estos enteros fuese 0. Entonces 6 − ≥ 35. Luego yzw + xzw + xyw + xyz = xyzw. ◮Resolución: Presúmase que todos x. z. 1 . . ◭ 10 ◮Resolución: 4 Ejemplo Sea a1 . .Chapter 1 Técnicas elementales 1. an una permutación arbitraria de los enteros 1. El lado siniestro es par. Sólo tiene que demostrarse que al menos una de las diferencias ak − k es par. donde n es non. . . Al cuadrar uno y 10 10 √ 1 otro lado. w son nones. z. 59 ≥ 10 35. ◮Resolución: Forzosamente los enteros deberán ser ±1. Demuéstrese que la suma de éstos no puede ser 0. lo que no tiene sentido. que todas las diferencias ak − k impares son. 2. sin recurrir a una calculatriz. que 6 − √ 1 35 < . . y. w enteros satisfaciendo 1 1 1 1 + + + = 1. Es evidente que S = (a1 − 1) + (a2 − 2) + · · · + (an − n) = 0.1 Contradicción 1 Ejemplo Sean x. o sea. 3481 ≥ 3500. entonces debería haber tantos +1’s como −1’s. 1. −2. ◮Resolución: Hay siete sumas posibles. cada una un entero del conjunto {−3. o 1’s. tres filas y dos diagonales). . luego o bien ap2 + bpq o bien bpq + cq2 sería par. otra contradicción. Luego. . Esto contradice el hecho de que todo entero positivo mayor que 1 puede descomponerse en factores primos de forma única. si uno entre p y q fuese impar y el otro par. b. hace par al producto. ◭ 7 Ejemplo Si a. Se tiene que (2k)2 = 4(k2 ). . tal raíz racional d f racpq es ficticia. Por el principio de las pichoneras. Cada entero es o bien par. y ap2 + bpq + cq2 impar.999 . Demuéstrese que entre las ocho sumas resultantes (tres columnas. √ a 2 = . Demuéstrese que BC > AC. 1. c son enteros impares. tanto a2 b como b2 tienen un número par de primos en su factorización (contando repeticiones). por suposición. 2. hay al menos dos de ellas idénticas. ◭ Tarea b. .2 Chapter 1 ya que las ak ’s son un reordenamiento de 1. −1. ◭ . y por lo tanto = 0. con enteros positivos a. dos de las ocho sumas del cuadrado deberán de coincidir. o 2 al ser dividida por 4. Luego. así que nuestra suposición inicial es falsa y por lo tanto al menos una de las diferencias ak − k es par. d tales que x4 + 2x2 + 2x + 2 = (x2 + ax + b)(x2 + cx + d). con p. (2k + 1)2 = 4(k2 + k) + 1. (de la forma 2k) o non (de la forma 2k + 1). Esto es imposible. el cuadrado de cada entero o bien deja residuo 0 o bien deja residuo 1 al ser dividido por 4. ◭ 5 Ejemplo Demuéstrese que √ 2 es irracional. q q Si ambos p y p fuesen nones. q relativamente primos. 0. La suma de dos enteros entonces dejará residuo 0. b A>B 9 Problema Sea 0 < α < 1. 1. 11 Problema Demostrar que no existen enteros a. 2. ◮Resolución: Si la ecuación poseyere la solución racional p . . lo que por consiguiente. Luego 2b2 tiene un número impar de primos en su factorización y a2 tiene un número par de primos en su factorización. demuéstrese que la ecuación ax2 + bx + c = 0 no posee una solución racional. b. 0’s. S es. De esto se obtiene el resultado de inmediato. resultando en el entero par 0. la suma de un número impar de enteros impares. Demuéstrese que la expansión decimal de α también comienza con al menos 2000 nueves. 8 Problema En △ABC. Ahora bien. en donde hay al menos 2000 nueves. 10 Problema Sea α = √ 0. n. Demuéstrese que √ α > α.2 Principio de las pichoneras de Dirichlet 12 Ejemplo Las nueve casillas de un cuadrado 3 × 3 son llenadas aleatoriamente por −1’s. ◭ ◮Resolución: Presúmase que 6 Ejemplo Demúestrese que 2003 no es la suma de dos cuadrados. ◮Resolución: Primero se demostrará que la suma de dos cuadrados nunca deja residuo 3 al ser dividida por 4. De manera semejante. c. 3}. entonces q Å ã2 Å ã p p a +b + c = 0 =⇒ ap2 + bpq + cq2 = 0. Esto conlleva a 2b2 = a2 . b. entonces ap2 +bpq +cq2 sería también non. . . . y estos sumarán a 104. . . {7. 55}. . entonces habrá dos cuya diferencia será n. ◮Resolución: Obsérvese primero que si elegimos n + 1 enteros de cualquiera ristra de 2n enteros consecutivos. Si seleccionamos cincuenta y cinco enteros. . 100}. Demuéstrese que hay al menos tres puntos que pueden ser cubiertos por un cuadrado 1 1 × . Como se habrá de escoger veinte y se tiene diecienueve conjuntos. . . {3. 22. . 60}. 40}. . 16. ◭ 16 Ejemplo No importa cuales cincuenta y cinco enteros se seleccionen de {1. . 2. a + 3. . 97}. 5 5 1 1 Divídase al cuadrado en veinticinco subcuadrados × . a + 2n}.. ◭ paralelos al cuadrado original. .. 3. 100}. ◭ ... {15. {49. . . . . a + 2n} en los n pares {a + 1. 2. . 94}. 42. 126}. . . {a + n. . . 100. . ◮Resolución: Divídase el conjunto {1. 2. 30}. . . 82. cada uno de ellos con sus lados 5 5 51 = 3 al menos. . . 80} y {81. por la observación anterior (con n = 10). ◮Resolución: Agrúpese los treinticuatro enteros de esta progresión en los dicienueve grupos {1}. entonces. 5. 29. 62} y {63. 2}. {31. Demuéstrese que hay al menos dos enteros diferentes en A cuya suma es 104.Principio de las pichoneras de Dirichlet 3 13 Ejemplo Cincuenta y un puntos se distribuyen sobre un cuadrado 1 × 1. .. {7. . 126} en los seis subconjuntos {1.. habrá dos cuya differencia será 10. Del dicho grupo.. .. 4. . demuéstrese que habrá dos de ellos cuya diferencia será 10. . 4.. {52}. . 14}. 8. a + n + 2}. 64. 62. . . .20}. . 6}. {4. {61. . a + 2. 100}. Esto es patente al aparear los 2n enteros consecutivos {a + 1. ◭ 15 Ejemplo Demuéstrese que entre siete enteros positivos distintos ≤ 126. . a + n + 1}. Agrúpese pues los cien enteros como sigue: {1. Uno de estos subcuadrados posee 25 ◮Resolución: 14 Ejemplo (Putnam 1978) Sea A cualquier conjunto de veinte enteros escogidos de la progresión aritmética 1. . . . 13. siempre se puede conseguir dos de ellos a y b satisfaciendo b < a ≤ 2b.. . {a + 2. {41. 61. perteneciendo al mismo conjunto.. . se habrá de tomar dos enteros. . forzozamente habremos de seleccionar once del mismo grupo. 32. Dos de los siete enteros yacerán en el mismo subconjunto y satisfacerán las desigualdades mencionadas. {10. {21. . al menos. .. 4 Chapter 1 17 Ejemplo (AHSME 1994) Márquese a un disco con la etiqueta “1”, a dos discos con la etiqueta “2”, a tres discos con la etiqueta “3”, . . . , a cincuenta discos con la etiqueta ‘‘50”. Póngase a estos 1 + 2 + 3 + · · · + 50 = 1275 discos en una caja. Se sacan luego discos de la caja, al azar y sin remplazo. ¿Cuál es el número mínimo de discos que se debe sacar para garantizar al menos diez discos con la misma etiqueta? ◮Resolución: Si se saca todos los 1 + 2 + · · · + 9 = 45 discos con etiquetas “1”, . . . , “9” y cualquiera nueve discos con etiquetas “10”, . . . , “50”, se habrá sacado 45 + 9 · 41 = 414 discos. El 415-avo disco sacado garantizará que haya al menos diez discos con la misma etiqueta. ◭ 18 Ejemplo Dado cualquier subconjunto A de diez enteros del conjunto {1, 2, . . ., 98, 99} demuéstrese que siempre habrá dos subconjuntos disjuntos de A cuyos elementos tienen la misma suma. ◮Resolución: Hay 210 − 1 = 1023 subconjuntos no nulos que se pueden formar con un conjunto de diez elementos. A cada uno de estos subconjuntos le asociamos su suma. La máxima suma que puede ser obtenible es 90 + 91 + · · · + 99 = 945 < 1023. Luego, hay dos subconjuntos, digamos S, T (no necesariamente disjuntos) cuya suma de elementos es idéntica. Luego, S \ (S ∩ T ) y T \ (S ∩ T ) también tienen suma idéntica de elementos. ◭ 19 Ejemplo Dados cualesquiera 9 enteros cuyos factores primos yagan en el conjunto {3, 7, 11}, demuéstrese que habrá dos cuyo producto es un cuadrado perfecto. ◮Resolución: Para que un entero sea un cuadrado, todos los exponentes de los primos de su factorización en primos deben ser pares. Todo entero cuyos factores yagan en el conjunto dado es de la forma 3a 7b 11c . Los tríos (a, b, c) yacen en exactamente uno de los 8 patrones de paridad (par, par, par), (par, par, non), (par, non, par), (par, non, non), (non, par, par), (non, par, non), (non, non, par), (non, non, non). En un grupo de nueve tales enteros, habrá pues dos cuyo patrón de paridad sea idéntico. Luego el producto de estos dos enteros es un cuadrado, ya que la suma de cada exponente será par. ◭ Tarea 20 Problema Si se toman n + 1 enteros del conjunto {1, 2, . . . ,2n}, demuéstrese que siempre habrá dos que son relativamente primos. 21 Problema Si se toman n + 1 enteros del conjunto {1, 2, . . . ,2n}, demuéstrese que siempre habrá dos tales que el menor dividirá (sin dejar residuo) al mayor. 23 Problema (AHSME 1991) Una mesa circular tiene exactamente sesenta sillas en torno. Hay N personas ya sentadas de manera que la próxima persona a sentarse por fuerza se sentará al lado de alguien. ¿Cuál es el valor mínimo de N? 24 Problema Cinco puntos cualesquiera son colocados sobre un √ cuadrado de lado 1. Demuéstrese que dos de ellos están a una distancia de a lo sumo 2/2. 22 Problema Pruebe que entre n + 1 enteros, siempre habrá dos cuya diferencia será divisible por n. 1.3 Paridad 25 Ejemplo Dos esquinas diametralmente opuestas son cortadas de un tablero de ajedrez, que como se recordará, tiene 64 casillas. Demuéstrese que es imposible recubrir totalmente a las 62 casillas restantes de 31 dominós. ◮Resolución: Cada dominó cubre cuadrados de diferente color. Al eliminar dos casillas diametralmente opuestas, se eliminan dos casillas del mismo color. Por lo tanto quedan 32 casillas de un color y 30 de otras y luego los 31 dominós no las pueden cubrir a todas. ◭ 26 Ejemplo Los 28 dominós de un juego se enfilan observando las reglas del dominó. Si al principio de la cadena se observa un 6 ¿qué entero se observará al final de la cadena? Paridad 5 ◮Resolución: Se observará también a un 6. Cada número debe ocurrir un número par de veces de manera que se puedan enfilar. De los ocho 6’s: se tiene uno al principio de la cadena, seis de ellos se aparearán entre sí en medio de la cadena y finalmente, el restante quedará al final de la cadena. ◭ 27 Ejemplo Demuéstrese que para ninguna selección de signos en 1 ± 2 ± · · · ± 10, se obtendrá una suma 0. ◮Resolución: La suma 1 + 2 + · · · + 10 = 55, un entero impar. Ya que la paridad no es afectada por la elección de signo, para cualquier selección de signo ±1 ± 2 ± · · ·± 10 nunca será par, y en particular, nunca será 0. ◭ 28 Definición Llámase punto reticular en el plano al punto coordenado (m, n) en el plano cuyas coordenadas m y n son ambas enteras. 29 Definición El punto medio del segmento de recta que comienza en (x, y) y termina en (x1 , y1 ) es el punto x+x 2 1 , y + y1  . 2 30 Ejemplo Se seleccionan cinco puntos reticulares en el plano, al azar. Demostrar que existe dos de entre ellos que forman un segmento de recta cuyo punto medio es también un punto reticular. ◮Resolución: Hay cuatro patrones de paridad posibles para cada punto reticular en el plano: (par, par), (par, non), (non, non), (non, par). Por el principio de las pichoneras, dos de los cinco puntos reticulares compartirán el mismo patrón de paridad, y luego, su punto medio será también reticular.◭ Para los ejemplos siguientes necesitaremos algunas definiciones de tetrominós, las cuales se daremos en las figuras al calce. Figure 1.1: L-tetrominó Figure 1.2: T-tetrominó Figure 1.4: Tetrominó torcido Figure 1.3: Tetrominó recto Figure 1.5: Tetrominó cuadrado 31 Ejemplo Se posee tan sólo una copia de los cinco tetrominós arriba mostrados, pudiendo así cubrir 20 cuadrados. De- muéstrese que es imposible arreglarlos de tal manera que se cubra a un rectángulo. ◮Resolución: Si tal rectángulo existiese, tendría 20 cuadrados. Coloreése el rectángulo a la manera de un tablero de ajedrez, con diez cuadrados rojos y diez negros. El T-tetrominó siempre cubre un número impar de cuadrados rojos y los otros siempre cubren un número par de cuadrados rojos. Así pues el número de cuadrados rojos cubiertos es impar, contradicción.◭ 6 Chapter 1 32 Ejemplo Demuéstrese que un tablero de ajedrez (8 × 8) no se puede recubrir totalmente con 15 tetrominós rectos y un L-tetrominó. ◮Resolución: Coloreése las filas 1, 3, 5, 7 en blanco y las filas 2, 4, 6, 8 en azul. Un tetrominó recto siempre cubrirá un número par de cuadrados blancos y un L-tetrominó siempre cubrirá un número impar de cuadrados blancos. Si el recubrimiento fuese posible entonces se cubrirá tan sólo un número impar de cuadrados blancos, contradicción. ◭ Tarea 33 Problema Veinticinco niños y veinticinco niñas son sentados alrededor de una mesa circular. Demuéstrese que ambos vecinos de al menos una persona serán niñas. 36 Problema Demuéstrese que un tablero 10 × 10 nunca se podrá recubrir totalmente 34 Problema Se sueldan 2001 varillas (rectas) formando un camino. Demostrar que no existe ninguna línea recta—no pasando por un punto de soldadura del camino—que intersecte a todos los 2001 segmentos del camino. 37 Problema Demuéstrese que un tablero 8 × 8 nunca se podrá recubrir totalmente con con 25 tetrominós rectos. 15 T-tetrominós y un tetrominó cuadrado. 38 Problema Una urna tiene 900 boletas, numeradas del 100 al 999. Se sacan boletas al 35 Problema Se escribe los 1, 2, . . . ,2001 enteros en una pizarra. Se borran de dos en dos, remplazándolos con el valor absoluto de su diferencia. Demostrar que el último número obtenido nunca será 0. azar y sin remplazo, y se suman sus dígitos. ¿Cuál es el número menor de boletas que se necesitará sacar para garantizar que al menos tres de estas boletas tengan la misma suma de dígitos? 1.4 Inducción El principio de inducción matemática resta en la siguiente observación intuitiva. Supongamos que tenemos que efectuar una tarea que requiere cierto número de pasos sucesivos. Supongamos que siempre lograremos completar el paso n si ya hemos completado el paso n − 1. Así pues, si acaso pudiésemos comenzar (completando un paso base), entonces podríamos completar todos los pasos a partir del paso base. Así pues, en el principio de inducción matemática, tratamos de comprobar la veracidad de una aserción P(n) estableciendo primero su validez en un caso base k0 (usualmente k0 = 1). Luego tratamos de establecer si información sobre la validez de P(n − 1) conlleva a información favorable sobre P(n). 39 Teorema (Principio de inducción matemática) Si un conjunto S de enteros positivos posee al 1, y también se verifica que el entero n + 1 está toda vez que el entero n esté, entonces S = N. 40 Corolario Si el conjunto A de enteros positivos contiene al entero m y también contiene al entero n + 1 siempre que contenga a n, donde n > m, entonces A es el conjunto de todos los enteros positivos mayores o iguales a m. 41 Corolario (Inducción robusta) Si el conjunto A de enteros positivos contiene al entero m y también contiene a n + 1 siempre que contenga a m + 1, m + 2, . . . , n, donde n > m, entonces A es el conjunto de todos los enteros positivos mayores o iguales a m. 42 Ejemplo Demostrar que 2n > n, ∀n ∈ N. ◮Resolución: La aserción es cierta para n = 0, ya que 20 > 0. Presúmase que 2n−1 > n − 1 para n > 1. Ahora bien, 2n = 2(2n−1 ) > 2(n − 1) = 2n − 2 = n + n − 2. Pero n − 1 > 0 =⇒ n − 2 ≥ 0, ya que n + n − 2 ≥ n + 0 = n y entonces, 2n > n. Esto establece el resultado por inducción. ◭ Tarea 7 43 Ejemplo Demostrar que todo cuadrado se puede descomponer en n subcuadrados, no necesariamente del mismo tamaño, para toda n = 4, 6, 7, 8, . . .. ◮Resolución: Al dividir al cuadrado en cuatro, como en la figura 1.6, se incrementa el número de cuadrados por tres. Así pues, si n es asequible, también lo es n + 3. Así pues, si se demuestra que n = 6, n = 7 y n = 8 son factibles, entonces se conseguirá toda descomposición en n ≥ 6 cuadrados. Pero esto se deduce de las figuras 1.7 y 1.8 (para n = 7, se descompone uno de los subcuadrados de la figura 1.6), terminando la demostración. ◭ Figure 1.6: Ejemplo 43. Figure 1.7: Ejemplo 43. Figure 1.8: Ejemplo 43. 44 Ejemplo Demostrar que 33n+3 − 26n − 27 es un múltiplo de 169 para todo número natural n. ◮Resolución: Sea P(n) la aserción “∃T ∈ N with 33n+3 − 26n − 27 = 169T .” Demostrarase que P(1) es cierta y que P(n − 1) =⇒ P(n). Para n = 1 se asevera que 36 − 53 = 676 = 169 · 4 es divisible por 169, lo cual es evidente. Ahora bien, P(n − 1) se traduce en la existencia de un N ∈ N tal que 33(n−1)+3 − 26(n − 1) − 27 = 169N, i.e., para n > 1, 33n − 26n − 1 = 169N para algún entero N. Luego 33n+3 − 26n − 27 = 27 · 33n − 26n − 27 = 27(33n − 26n − 1) + 676n lo que simplifica a 27 · 169N + 169 · 4n, que claramente múltiplo de 169 es. Esto establece el resultado mediante inducción. ◭ Tarea 45 Problema Demostrar que para todo entero n ≥ 1, la cantidad √ √ (1 + 2)2n + (1 − 2)2n es un entero par y que √ √ √ (1 + 2)2n − (1 − 2)2n = b 2 para algún entero b. 48 Problema La sucesión de Fibonacci está dada por f0 = 0, f1 = 1, fn+1 = fn + fn−1 , n ≥ 1, 46 Problema Si k es impar, demostrar que 2n+2 divide a n k2 − 1 para todo natural n. 47 Problema Si se toman n + 1 enteros del conjunto {1, 2, . . . ,2n}, demuéstrese que siempre habrá dos tales que el menor dividirá (sin dejar residuo) al mayor. esto es, cada número luego del segundo es la suma de los dos precedentes. Así la sucesión de Fibonacci comienza por 0, 1, 1, 2, 3, 5, 8, 13, 21, . . . . b2 . demuéstrese que a = b = c = 0. Finalmente esto dá c = 2c1 . Sin pérdida de generalidad presúmase que a < b.bn enteros naturales no nulos. c). contradicción. las monedas sólo vienen en cantidades de 3 y 5 pe- sos. 51 Problema Utilícese inducción para demostrar que para entero natural n. Pero max(a1 . Demostrar que todos los elementos de C son idénticos. Supóngase que C tiene la propiedad siguiente: Si x ∈ C . . . n+2 . Así pues se debe tener que max(a. c) = 0 y todas las incógnitas son 0. llamémosle m. . . un cuadrado.5 Buen orden 57 Axioma (Axioma del buen orden) Todo conjunto no vacuo S de números naturales posee un elemento mínimo. y por lo tanto S = ∅. c) > 0 tan pequeño como fuere posible. demuestre que todo triángulo equilátero puede ser descompuesto en n subtriángulos equiláteros (no necesariamente del mismo tamaño) para toda n ≥ 6. 0 < m2 < m < 1. 1+ 1 1 1 + ··· + 2 ≤ 2 − . Demostrar que toda cantidad de 8 o más pesos se puede pagar con estas monedas. 49 Problema En el país Pesimista. b son enteros positivos tales que a2 + b2 a2 + b2 es entero. 2s] hay una potencia de 2. entonces existe particición de C \ {x} en dos subconjuntos A y B de n elementos cada uno tal que la suma de los elementos en A es igual a la suma de los elementos en B. c) lo que contradice la minimalidad de max(a. n + (−1) . c satisfaciendo la ecuación y con max(a. 54 Problema Sean a1 . la candidad n3 + (n + 1)3 + (n + 2)3 es siempre divisible por 9. 1[ no es nulo. ya que si a = b entonces then ◮Resolución: Supóngase que 0<k= forzando k = 1. Esto conlleva a 32a61 + b6 = 2c6 . 2(n + 1) Demostrar que siempre es posible suprimir unos cuantos términos de uno y otro lado (¡pero no todos!) y conservar la igualdad de la suma de los términos restantes. b. . c son enteros tales que a6 + 2b6 = 4c6 . ◮Resolución: Es evidente que podemos restringirnos al caso donde todas las incógnitas son mayores o iguales a 0. b. Si a6 + 2b6 = 4c6 entonces a deberá ser par. Por el axioma del buen orden. Pero esto quiere decir que S tiene un entero positivo m2 estrictamente menor que su mínimo elemento m. b. este conjunto debe poseer un elemento mínimo. a2 . 58 Ejemplo Demuéstrese que no existe ningún entero natural en el intervalo ]0. b.8 Chapter 1 Demuéstrese mediante inducción matemática . b. b) 1 + ab tan pequeño como fuere posible. a2 + 1 . a = 2a1 . 1[. Luego también b = 2b1 y así 16a61 + 32b61 = c6 . y por tanto m2 ∈ S . Ahora bien. c1 ) < max(a. Escójase un trío a. Supóngase que 50 Problema Utilícese inducción para demostrar que para todo número natural n > 0 Ä 1− 1 4 äÄ 1− ä  1 9 ··· 1 − 1 (n + 1)2  = a1 + a2 + · · · + an = b1 + b2 + · · · + bn < mn. por lo cual a61 + 2b61 = 4c61 . ◮Resolución: Presúmase al contrario que el conjunto S de enteros naturales en ]0. se tiene fn−1 fn+1 = fn2 53 Problema Sea n ≥ 1 un entero y sea C un conjunto constituido de 2n + 1 enteros positivos no nulos. Demostrar que en el intervalo cerrado [s.an y b1 . 1 + ab 1 + ab a2 + b2 = k es un contra-ejemplo de un entero que no es cuadrado. ◭ 60 Ejemplo (IMO 1988) Si a. . . 2a2 < 2. ◭ 59 Ejemplo Si a. Puede avalarse de la identidad 55 Problema Mediante inducción. con max(a. b. b1 . 1. no necesariamente distintos. . n 22 n 56 Problema Sea s un entero estrictamente positivo. 52 Problema Demostrar que para todo entero n ≥ 1. que para entero n ≥ 1. (a + b)3 = a3 + 3a2 b + 3ab2 + b3 . entonces demuestre que cuadrado es. La distancia de b a la recta ap es menor que d. Demuéstrese que los puntos están alineados. Sea f el pie de la perpendicular de p a L. como en la figura 1. Además b1 = a2 − k b2 − k < < b. Si a. c sobre L. a2 + b2 − k(ab + 1) = 0 es una ecuación cuadrática en b.10: Ejemplo 64. están del mismo lado que f y uno de ellos. ◭ A p c f q b a P C B D Figure 1. con suma de raíces ka y producto de raíces a2 − k. entre todos los pares (p. ¿Existirá un apareamiento para el cual ningún par de segmentos se intersecte? ◮Resolución: Sí existe. ◮Resolución: Si los puntos no estuviesen alienados. Por hipótesis hay al menos tres puntos a. digamos b. digamos a y b. 4. y. 1 + ab1 Tarea 62 Problema Demostrar que cuartetos de enteros estrictamente positivos (x. b b Entonces hemos encontrado otro entero b1 para el cual Entonces pues k cuadrado es. es más cercano a f . . ◭ a2 + b21 = k y que es menor que max(a. z. n son azules y ningún trío de entre ellos es colineal.9. . Sostenemos que bajo estas condiciones ningún par de segmentos se intersecará. . . El número de apareamientos es finito. el suponer que b1 < 0 es incompatible con a2 + b21 = k(ab1 + 1). Sortéanse los puntos en n pares de tal manera que cada par tiene un punto rojo y otro azul y se forman n segmentos uniendo cada par de puntos. 9. b. de modo que b1 + b = ka y b1 b = a2 − k. Sean b1 . .9: Ejemplo 63 Figure 1. L) de puntos p no sobre la línea L habrá uno minimizando la distancia d entre p y L. 16. luego ha de existir una manera de parear los puntos de modo que la distancia total de los segmentos sea mínima. w) sat- 61 Problema Demostrar que la serie infinita de cuadrados isfaciendo la ecuación x2 + y2 = 3(z2 + w2 ) no existen. 64 Ejemplo De 2n puntos en el plano. el suponer b1 = 0 es incompatible con a2 + 02 = k(0 · a + 1). Dos de éstos. 1. k son enteros positivos. n son rojos. b sus raíces.6 Condiciones extremas 63 Ejemplo (Problema de Sylvester) Un conjunto de n puntos en el plano posee la propiedad que toda línea pasando por dos de ellos siempre pasa por un tercero de ellos.Tarea 9 Ahora bien. no contiene ninguna progresión aritmética infinita. b). Como k no es cuadrado. 1. contradicción. contradicción. demostrar que el centro del disco pertenece a esta colección de puntos. contradicción.10 Chapter 1 Supóngase que bajo las condiciones de mínima distancia arriba estipuladas existe un par de segmentos AB y CD que se intersecan en el punto P. Si las distancias mutuas entre todos estos puntos son ≥ 1. minimizando la ya minimizada distancia. En virtud de la desigualdad del triángulo AD + BC < AP + PD + BP + PC = AB +CD. . ◭ Tarea 65 Problema (BMO 1975) Siete puntos se encuentran sobre un disco cerrado de radio 1. como en la figura 1.10. 1 Identidades algebraicas Una de las identidades más útiles en la resolución de problemas es la diferencia de cuadrados x2 − y2 = (x − y)(x + y). Muchas expresiones se pueden factorizar si se convierten en diferencias de cuadrados. Demostración: Primero demostraremos que si a = 1. entonces 1 + a + a2 + · · · an−1 = 1 − an .Chapter 2 Álgebra y aritmética 2. 11 . Por ejemplo x4 + x2 y2 + y4 = x4 + 2x2 y2 + y4 − x2 y2 = (x2 + y2 )2 − (xy)2 = (x2 − xy + y2)(x2 + xy + y2). Entonces xn − yn = (x − y)(xn−1 + xn−2 y + xn−3y2 + · · · + xyn−2 + yn−1 ). 1−a Póngase S = 1 + a + a2 + · · · + an−1. Si n es un entero positivo tenemos en general el siguiente teorema. También a4 + 4b4 = a4 + 4a2 b2 + 4b4 − 4a2b2 = (a2 + 2b2)2 − (2ab)2 = (a2 − 2ab + 2b2)(a2 + 2ab + 2b2) Otra identidad útil es la de diferencia de cubos x3 ± y3 = (x ± y)(x2 ∓ xy + y2). 66 Teorema Sea n un entero positivo. sin necesidad de hacer cálculos. 70 Ejemplo Si a2 + b2 = 1 y ab = 2. 69 Corolario Sean x. y y y y y Multiplicando por yn uno y otro lado Ç å Å ã ã Å x n−1 x x2 xn−1 xn y 1− y 1 + + 2 + · · · + n−1 = yn 1 − n . ❑ yn − xn = (y − x)(yn−1 + yn−2x + · · · + yxn−2 + xn−1 ). a4 + b4 . Otros resultados útiles son los siguientes 68 Teorema Si n es un entero positivo impar xn + yn = (x + y)(xn−1 − xn−2 y + xn−3y2 − xn−4 y3 + · · · + x2 yn−3 − xyn−2 + yn−1 ). ◭ . el corolario anterior nos dice que 781 = 1996 − 1215 divide a 19965 − 12155. halle (a + b)2 . ☞ El segundo factor tiene n términos y cada término tiene grado (peso) n − 1. Por ejemplo 129 = 27 + 1 divide a 2861 + 1 y 1001 = 1000 + 1 = 999 + 2 = · · · = 500 + 501 divide a 11997 + 21997 + · · · + 10001997. y al ver que (1 − a)S = S − aS = 1 − an. 1+ + = 1− y y y y å Å ãÇ x xn−1 x x2 xn 1− 1 + + 2 + · · · + n−1 = 1 − n . 67 Corolario Sean x. Poniendo ahora a = . y y y y y lo que da el resultado pedido. ◮Resolución: Tenemos (a + b)2 = a2 + b2 + 2ab = 5. y enteros con x = y y sea n un entero positivo impar. se ve que y Ä ä Å ãn−1 1 − x n Å ã2 x x x y + ··· + = 1+ + y y y 1 − yx de donde se obtiene Å 1− lo que equivale a x y ãÇ Å ãn−1 å Å ã2 Å ãn x x x x + ···+ . y enteros con x = y y sea n un entero positivo. x se obtiene el resultado. y a4 + b4 = (a2 + b2)2 − 2a2b2 = −7.12 Chapter 2 Entonces aS = a + a2 + · · · + an−1 + an. Entonces x + y divide a xn + yn . Por ejemplo. (a − b)2 . (a − b)2 = a2 + b2 − 2ab = −3. Luego S − aS = (1 + a + a2 + · · · + an−1) − (a + a2 + · · · + an−1 + an) = 1 − an. Entonces x − y divide a xn − yn . esto es n = 1. Vemos que n4 + 4 = n4 + 4n2 + 4 − 4n2 = (n2 + 2)2 − (2n)2 = (n2 − 2n + 2)(n2 + 2n + 2). ◭ 72 Ejemplo Demostrar que el único primo de la forma n4 + 4 es el 5. ◮Resolución: Rearreglemos la suma de la siguiente manera ã Å ã Å 1 1 1 + + 1+ 1978 2 1977 Å ã Å ã 1 1 1 1 + + + + ···+ 3 1976 989 990 = 1979 1979 1979 + + ···+ . Luego el único primo de la forma deseada es 23 − 1 = 7. donde n es un entero positivo. b 2 3 1978 entonces 1979 divide a u. vemos que el denominador divide a 1978!.Identidades algebraicas 13 71 Ejemplo Hallar todos los primos de la forma n3 − 1. Si este producto es un número primo entonces el factor más pequeño debe ser igual a 1. Luego. 1 · 1978 2 · 1977 989 · 990 Al sumar todas las fracciones arriba en la derecha . 2 3 4 2n − 1 2n n + 1 n + 2 2n ◮Resolución: La cantidad de la izquierda es ã Å 1 1 1 1 1 1 + + + + ···+ + 2 3 4 2n − 1 2n ã Å 1 1 1 1 + + + ···+ −2 2 4 6 2n = = = como queríamos demostrar. Así n2 − 2n + 2 = 1. 1979 divide al numerador de la fracción. demostrar que si 1 1 1 u = 1 + + + ···+ . el único primo de esta forma es 14 + 4 = 5. Así. Como 1979 es primo. o sea (n − 1)2 = 0. ◭ Å ã 1 1 1 1 1 + 1 + + + + ···+ 2 3 4 2n − 1 2n Å ã 1 1 1 1 1 −2 · 1 + + + + ···+ 2 2 3 4 n Å ã 1 1 1 1 1 1 + + + + ···+ + 2 3 4 2n − 1 2n ã Å 1 1 1 1 − 1 + + + + ···+ 2 3 4 n 1 1 1 + + ···+ . n+1 n+2 2n . ◭ 73 Ejemplo Dado que 1979 es primo. ◭ 74 Ejemplo Demostrar la siguiente identidad de Catalán: 1− 1 1 1 1 1 1 1 1 + − + ···+ − = + + ··· + . ◮Resolución: Podemos restringirnos a enteros positivos. ◮Resolución: Como n3 − 1 = (n − 1)(n2 + n + 1) y como n2 + n + 1 > 1. ningún factor de 1978! cancela al 1979 del numerador. deberemos tener n − 1 = 1. entonces √ √ 1 √ = n + 1 − n. » √ √ √ 5 + 2 6 = 2 + 3. de donde a2 − 2 = tan2 x + cot2 x. De aquí 1 + x + x2 + · · · + x80 = x81 − 1 . √ n+ n+1 . Entonces xS = x + x2 + x3 + · · · + x80 + x81 = S − 1 + x81 . ◮Resolución: Pongamos S = 1 + x + x2 + · · · + x80 . ◭ 78 Ejemplo Simplificar 1 1 1 1 √ √ √ +√ √ + ···+ √ √ +√ . √ √ √ √ 5 + 2 6 = 3 + 2 2 · 3 + 2 = ( 2 + 3)2 . 99 + 100 1+ 2 2+ 3 3+ 4 √ √ √ √ ◮Resolución: Como 1 = n + 1 − n = ( n + 1 − n)( n + 1 + n). x−1 Luego x81 − 1 x81 − 1 x27 − 1 x9 − 1 x3 − 1 = 27 · · · . x−1 x − 1 x9 − 1 x3 − 1 x − 1 Por lo tanto 1 + x + x2 + · · · + x80 = (x54 + x27 + 1)(x18 + x9 + 1)(x6 + x3 + 1)(x2 + x + 1). ◭ 77 Ejemplo Hallar la raíz cuadrada de ◮Resolución: Observe que Luego √ 5 + 2 6. Así tan3 x + cot3 x = (tan x + cotx)(tan2 x − tanx cot x + cot2 x) = a(a2 − 3). exprese tan3 x + cot3 x como un polinomio en a. ◭ 76 Ejemplo Factorizar 1 + x + x2 + · · · + x80 .14 Chapter 2 75 Ejemplo Si tan x + cotx = a. ◮Resolución: Primero observemos que a2 = (tan x + cotx)2 = tan2 x + cot2 x + 2. Por lo tanto. . 22225555 + 55552222 . √ 10 + 4i 6. 86 Problema Demostrar que 271958 − 108878 + 101528 es exactamente divisible por 26460. . 87 Problema Demostrar que si k es un entero positivo impar 1k + 2k + · · · + nk es divisible por 1 + 2 + · · · + n. Así pues. 90 Problema Halle la suma de los factores primos de 216 − 1. . 83 Problema Demostrar que el entero 11 . 84 Problema Demostrar que 7 divide a 91 Problema Dado que 1002004008016032 tiene un factor primo p > 250000. 92 Problema Si a3 − b3 = 24. . a − b = 2. = √ √ 100 − 99. √ √ y así 1 √ 99 + 100 = √ √ 2− 1 = √ √ 3− 2 = √ √ 4− 3 .. la expresión 2903n − 803n − 464n + 261n es siempre divisible por 1897. Además 2903n − 464n es divisible por 2903 − 464 = 2439 = 9 · 271 y −803n + 261n es divisible por −803 + 261 = −542 = −2 · 271. 1+ 2 2+ 3 3+ 4 99 + 100 ◭ 79 Ejemplo Demostrar que para todo entero positivo n. 82 Problema Halle a6 + a−6 dado que a2 + a−2 = 4. √ √ 1 1 1 1 √ +√ √ +√ √ + ···+ √ √ √ = 100 − 1 = 9. 2 81 Problema Calcule (123456789) − (123456791)(123456787) mentalmente. ◮Resolución: Por el Teorema 66. .. la expresión es divisible por 7. 85 Problema Demostrar que 100 divide a 1110 − 1..Tarea 15 Por lo tanto 1 √ 1+ 2 1 √ √ 2+ 3 1 √ √ 3+ 4 . 88 Problema Demostrar que 1492n −1770n −1863n +2141n es divisible por 1946 para todo entero positivo n. la expresión es pues divisible por 7 · 271 = 1897. . 2903n − 803n es divisible por 2903 − 803 = 2100 = 7 · 300 y 261n − 464n es divisible por −203 = (−29) · 7. como la expresión es divisible por 7 y por 271 y como estos son relativamente primos. ◭ Tarea 80 Problema Dado que 9877892 = 975727108521. es compuesto. hállelo. halle el valor de 9877902 . 93 Problema Hallar p 94 Problema Hallar p √ 11 + 72. . 11 | {z } 221 1′ s 89 Problema Dividir x128 − y128 por (x + y)(x2 + y2 )(x4 + y4 )(x8 + y8 )(x16 + y16 )(x32 + y32 )(x64 + y64 ). halle el valor de (a + b)2 . 1417 y 2312 se dividen por d > 1. 97 Problema Expandir el producto (1 + x)(1 + x2 )(1 + x4 )(1 + x8 ) · · · (1 + x1024 ). . Así pues. q2 .}. . 5. −8. . . 108 Ejemplo Sea r el residuo cuando 1059. 0. . (x + a)7 − x7 − a7 = 7xa(x + a)(x2 + xa + a2 )2 . Así pues todo entero es de la forma 2k o 2k + 1. ◮Resolución: Por el algoritmo de división. − 4. . −7. b enteros con a > 0. . . entonces n es un número primo. 3. Por ejemplo. . 4. 104 Problema Demostrar que A = x9999 + x8888 + x7777 + · · · + x1111 + 1 98 Problema Demostrar que si 2n − 1 es un número primo. . 9. 6. 2.2 Los enteros Una de las propiedades más útiles de los enteros es la expresada por el algoritmo de división: 107 Teorema (Algoritmo de división) Sean a. . 103 Problema Demostrar que 96 Problema Factorice 1 + x + x2 + x3 + · · · + x624 . . . Como 1059 = 5 · 179 + 164. . .}. Por ejemplo. 10. 358 = 2 · 179. . 895 = 5 · 179. 895 = d(q3 − q2 ) y 358 = d(q2 − q1 ).}. 3. . Observe que todo entero de la forma 4k + 1 es también de la forma 4t − 3 y que todo entero de la forma 4k + 3 es también de la forma 4t − 1. . 5. 8. −3. (a + b + c)3 − (−a + b + c)3 − (a − b + c)3 − (a + b − c)3 = 24abc. 0. 106 Problema Demostrar que 101 Problema Demostrar que a3 + b3 + c3 − 3abc = (a + b + c)(a2 + b2 + c2 − ab − bc − ca). 99 Problema Demostrar que si 2n + 1 es un número primo. 2.}. . existen enteros q1 . 105 Problema La diferencia p 100 Problema Demuestre que a2 + b2 + c2 − ab − bc − ca = Ð 1 (a − b)2 + (b − c)2 + (c − a)2 . q3 con 1059 = dq1 + r. r = 164.16 Chapter 2 95 Problema Evalúe la suma 102 Problema Demostrar que 1 1 1 √ √ +√ √ √ √ . B2 = {. Restando obtenemos 1253 = d(q3 − q1 ). −4. −6. 7. . Halle el valor de d − r. 4. 1417 = dq2 + r y 2312 = dq3 + r. B1 = {. . . 11. . 4k + 1. vemos que d = 179. Vemos pues que el algoritmo de división discrimina a los enteros según el residuo que dejan al ser divididos por a. −5. 1. −2. . descomponemos a los enteros en las dos familias A0 = {. . Finalmente. Primos de esta forma se llaman primos de Fermat. . 4k + 2 o 4k + 3. es divisible por B = x9 + x8 + x7 + · · · + x2 + x + 1. 2. . Hállelo. si a = 2. A1 = {. Entonces existen enteros q y r con b = aq + r. −1. . . . −5. . d − r = 15. −1.}. √ √ +√ 3 3 3 1+ 3 2+ 3 4 4+ 3 6+ 3 9 9 + 3 12 + 3 16 (x + y)5 − x5 − y5 = 5xy(x + y)(x2 + xy + y2 ). Como 7 · 179. −2. Observe que todo entero de la forma 2k + 1 es también de la forma 2t − 1. . Primos de esta forma se llaman primos de Mersenne. entonces n es una potencia de 2. . . los enteros son de la forma 4k.◭ .}. . B3 = {. 0 ≤ r < a. 39 = 4 · 9 + 3. 1. −3. Si a = 4 entonces descomponemos a los enteros en las cuatro familias B0 = {. 2 p √ √ 57 − 40 2 − 57 + 40 2 es un entero. todo número en esta sucesión es de la forma 4k − 1. 44} ·10n + 88 | . 00} 1. es el cuadrado de un entero. producen infinitos valores de n2 + 23 que son divisibles por 24.{z n 4′ s = . digamos 2t + 1. . . ±2. ◮Resolución: Si p > 3 es primo. .{z n 4′ s n−1 8′ s = = = = n−1 8′ s 8 4 · (10n − 1) · 10n + · (10n−1 − 1) · 10 + 9 9 9 4 1 4 2n n · 10 + · 10 + 9 9 9 1 2 n (2 · 10 + 1) 9 ã Å 2 · 10n + 1 2 3 Nos falta demostrar que esta última cantidad es entera.{z | . ◭ n−1 0′ s 113 Ejemplo Demostrar que el cuadrado de todo primo mayor que 3 deja residuo 1 al ser dividido por 12. que es de la forma 4k + 1. ◮Resolución: Tenemos que n2 + 23 = n2 − 1 + 24 = (n − 1)(n + 1) + 24.Los enteros 17 109 Ejemplo Demostrar que el cuadrado de todo entero es de la forma 4k o de la forma 4k + 1. ◮Resolución: Si el entero es par. que 3 divide a 2 · 10n + 1 = 2 00 . . 44448889. 444889.88} ·10 + 9 44 . 4489. y por lo tanto este entero es divisible por 3. 44 . 11 . las familias n = 24m ± 1. su cuadrado es (2a)2 = 4a2 . m = 0.◭ 112 Ejemplo Demostrar que todos los enteros en la sucesión . . Esto completa la demostración. ±3. 44} 88 | . | {z | . .{z suma de los dígitos de esta última cantidad es 3.{z n 1′ s n−2 1′ s n−2 1′ s Así pues. 4k − 1 no puede ser el cuadrado de ningún entero.11}+12 − 1. Para n > 2.◭ 111 Ejemplo Demuestre que n2 + 23 es divisible por 24 para un número infinito de números n. ±1. esto es. nos ocuparemos de los demás enteros en la sucesión. . entonces (2t + 1)2 = 4(t 2 + t) + 1. Si el entero es impar. . . ◮Resolución: Observe que . 1} = 11 . que es de la forma 4k. . . ◮Resolución: Como es obvio que 11 no es un cuadrado. . . Pero por el ejercicio anterior.{z | .◭ 110 Ejemplo Demostrar que ningún entero en la sucesión 11. 111. . 11111. entonces p es de la forma 12k ± 1. 88} 9 49. 12k ± 5. es decir de la forma 2a. 11} 00 + 12 − 1 = 100 · 11 . . 1111.{z n 4′ s n−1 8′ s son cuadrados. .{z | . Ahora bien (12k ± 1)2 = 12(12k2 ± 2k) + 1 . Luego.44}88 | . . Pero la | . 88} 9 44 .{z | . 62500. etc. donde ak . . b1 . De aquí n+1 = (3k ± 1)2 − 1 + 1 = 3k2 ± 2k + 1 = k2 + k2 + (k ± 1)2. como A + A′ = 10000000000. La condición del problema estipula que 1 y= (6 · 10n + y) . 250. Entonces x = 6 · 10n + y donde y es un entero positivo. . que es divisible por 6 y por lo tanto no es primo. ◮Resolución: Sea x el entero buscado. ◮Resolución: Claramente. A y A′ deberán tener 10 dígitos cada uno. luego 3n + 1 = (3k ± 1)2. 8p − 1 = 24k − 7 y 8p + 1 = 24k − 6.◭ 117 Ejemplo Hallar todos los enteros con dígito inicial 6 tales que si se les suprime este dígito incial. a1 y A′ = b10 b9 b8 . entonces 8p + 1 es compuesto.◭ 118 Ejemplo Sea A un entero positivo y A′ sea el entero positivo resultante de alguna permutación específica de los dígitos de A. .◭ 114 Ejemplo Demostrar que si ambos p y 8p − 1 son primos. entonces n +1 es la suma de dos cuadrados consecutivos. tenemos 2n + 1 = (2t + 1)2 para algún entero t. 3 como queríamos demostrar. ◮Resolución: Como 2n + 1 es un cuadrado impar. deberemos tener que a1 + b1 = a2 + b2 = · · · = ai + bi = 0 y ai+1 + bi+1 = 10. . la suma de dos cuadrados consecutivos. .◭ 116 Ejemplo Demostrar que si 3n + 1 es un cuadrado. 6250. Esto demuestra la aserción. 2. Si p = 3k + 1. . n= 2 Luego n + 1 = t 2 + (t + 1)2 .. luego la aseveración se cumple para p = 3. 2500. (2t + 1)2 − 1 = 2t 2 + 2t. 4 Esto requiere n ≥ 2 y por lo tanto y = 25. Demostrar que si A + A′ = 1010 entonces A es divisible por 10. ◮Resolución: Si p = 3. 8p − 1 = 24k − 15 no es primo. .. Luego x = 625.◭ 115 Ejemplo Demostrar que si n es un entero positivo tal que 2n +1 es un cuadrado. p es de la forma 3k + 1 o 3k + 2. Si p > 3. 25 o sea. 25000.el número resultante es 1/25 del número original. Si p = 3k + 2. bk . 625000. Resolviendo para n. ◮Resolución: Es claro que 3n + 1 no es un múltiplo de 3. k = 1.18 Chapter 2 y (12k ± 5)2 = 12(12k2 ± 10k + 2) + 1. ai+2 + bi+2 = · · · = a10 + b10 = 9. 8p − 1 = 23 y 8p + 1 = 25. 10 son los dígitos de A y A′ respectivamente. . entonces n + 1 es la suma de tres cuadrados. Pongamos pues A = a10 a9 a8 . 10n y= = 25 · 10n−2. etc. . Ahora. Sumando. n. 3 ó 4. Ahora bien. Entonces su producto P es P = (n − 1)n(n + 1)(n + 2) = (n3 − n)(n + 2) = n4 + 2n3 − n2 − 2n. P no puede ser un cuadrado. .◭ 119 Ejemplo ¿Cuántos ceros hay al final de 999!? ◮Resolución: El número de ceros está determinado por la potencia mayor de 10 que divide a 999!. ¿Cuál es el valor máximo de su producto? ◮Resolución: Tenemos enteros positivos a1 .Los enteros 19 para algún subíndice i. tenemos a1 + b1 + a2 + b2 + · · · + ai + bi + ai+1 + bi+1 + · · · + a10 + b10 = 2(a1 + a2 + · · · + a10). Pero como a1 + a2 + · · · + a10 = b1 + b2 + · · · + b10. . n + 2 los cuatro enteros consecutivos.◭ 120 Ejemplo La suma de enteros positivos es 1996. Como 1996 = 3(665) + 1 = 3(664) + 4. A y A′ son ambos divisibles por 10. Supongamos que a j > 4. ◮Resolución: Sean n − 1. 2. . . Como hay menos mútiplos de 5 en {1. jamás es un cuadrado. el número de ceros está pues determinado por la potencia mayor de 5 que divide a 999!. n. ◮Resolución: Sean n − 1. . . si i es par. ai + bi . an con a1 + a2 + · · · + an = 1996. Como P = 0 y P es 1 más que un cuadrado. esto es. a2 . un entero par. . 0 ≤ i ≤ 9. diferentes de 0. Luego el producto es divisible por 3 × 4 × 2 = 24. uno de ellos es de la forma 4k (y por lo tanto divisible por 4) y otro de ellos es de la forma 4a + 2 (y por ende divisible por 2). r donde r es un número racional. Note que si i = 9 no hay ninguna suma de las ai+2 + bi+2 . . 999} que múltiplos de 2. 10 + 9(9 − i) es impar y si i es impar 10 + 9(9 − i) es par. (n2 + n − 1)2 = n4 + 2n3 − n2 − 2n + 1 = P + 1 > P. los podemos substituir con 3’s. . n + 1. ninguna de las ak ’s puede ser igual a 1. n + 2 cuatro enteros consecutivos. Pero como 2 + 2 + 2 = 3 + 3 y 2 × 2 × 2 < 3 × 3. Esta es ò ï ò ï ò ï ò ï 999 999 999 999 + + + = 199 + 39 + 7 + 1 = 246. . si hay tres o más 2’s. Uno de ellos es divisible por 3. n + 1. . y si i = 0 no hay ninguna suma de las a1 + b1. Colegimos que i es impar. Si substituimos a j por los dos términos a j − 3 y 3 la suma no se afecta. pero el producto incrementa pues a j < 3(a j − 3). . ◭ 123 Ejemplo Hallar todos los enteros positivos de la forma 1 r+ . Demostraremos que para obtener un producto máximo deberemos tener la mayoría de las ak = 3 y a lo sumo dos a j = 2. Es claro que para maximizar a1 a2 · · · an . . ai+3 + bi+3 . el producto máximo es pues 3664 × 4.◭ 121 Ejemplo Demostrar que el producto de cuatro enteros consecutivos es siempre divisible por 24. 999! termina en 246 ceros. a1 + b1 + a2 + b2 + · · · + ai + bi + ai+1 + bi+1 + · · · + a10 + b10 = 10 + 9(9 − i). Ahora bien. 5 52 53 54 Por lo tanto.◭ 122 Ejemplo Demostrar que el producto de cuatro enteros consecutivos. . lo que necesariamente implica a1 = b1 = 0. . . Así pues las ak ’s son iguales a 2. 11 . si k ≥ 3. . . Ahora bien. 2 Como k es un entero. . Por ejemplo. . r puede ser entero si y sólo si k2 − 4 es un cuadrado de la misma paridad que k.20 Chapter 2 ◮Resolución: Demostraremos que la expresión r + 1/r es entero sólo cuando r = 1. r k un entero positivo. . (n + 10)2 = n2 + 20n + 100 tiene la misma paridad en su dígito de las decenas que el dígito de las decenas de n2 . 2. |{z} n 5′ s 128 Problema ¿Qué dígitos aparecen en el producto 3. 2. . 2 |{z} |{z} 2n 1′ s 126 Problema Demostrar que el cuadrado de un entero es de la forma 3k o 3k + 1. 0 5. . ◭ Tarea 131 Problema Demostrar que la suma de todos los enteros de n dígitos. . | {z } m−1 0′ s 132 Problema Demostrar que para todo entero positivo n. . |{z} |{z} m 1′ s n−3 9′ s y sea b el entero n−2 0′ s b = 1 000 . r + 1/r = 2. Esto termina la demostración. Luego demostrar que si los lados de un triángulo rectángulo son enteros. √ 133 Problema Demostrar que para todo número a = 0. 130 Problema ¿Cuál es la potencia mayor de 7 que divide a 1000!? es un entero compuesto.. la expresión 129 Problema Demostrar que no existe ningún entero con la propiedad de que si su n3 + (n + 2)3 4 dígito inicial se suprime. n 2′ s es un cuadrado. . 0 . es 125 Problema Sea a el entero a = 111 . Ahora. Sea pues 1 r + = k. . . a = ±i 3 se verifica la fórmula de Reyley (1825): 127 Problema Hallar la suma 5 + 55 + 555 + · · · + 5 .. decimos que a divide al entero b (escrito a|b) si existe un entero k con ak = b. ◭ 124 Ejemplo ¿Para cuántos enteros n en {1. Luego. . 9 55 00 . (k − 1)2 < k2 − 4 < k2 . hay 2 × 10 = 20 enteros n para los cuales se verifica la condición prescrita. 100} es el dígito de las decenas de n2 impar? ◮Resolución: En el subconjunto {1. . el entero resultante es 1/35 del entero inicial. 666 6′ s 134 Problema Demostrar que para n ≥ 2. . 11|99 porque 11 · 9 = 99. r = 1. esto es. k2 − 4 = 0.. . Si a es racional. 5 . 1 | {z } 494 99 . 6 ? |{z} |{z} 666 3′ s  a= a6 + 45a5 − 81a2 + 27 6a(a2 + 3)2 3  + −a2 + 30a2 − 9 6a(a2 + 3) 3  + −6a3 + 18a (a2 + 3)2 3 . Si k = 2. . 10} hay sólo dos valores de n (4 y 6) para los cuales el dígito de las decenas de n2 es impar. k2 − 4 está entre dos cuadrados consecutivos y por lo tanto no puede ser un cuadrado.. 2. 3.3 Aritmética modular Comenzaremos primero con la siguiente definición. 1 − 22 . entonces 3 divide a alguno de los lados. k2 − 4 no es real. Luego √ k ± k2 − 4 r= . . Si a = 0 es un entero. Luego. . . Si k = 1. . en cuyo caso r + 1/r = 2. . 3 · 6. p esto es. esto demuestra que todo número racional puede expresarse como la suma de tres cubos de números racionales. Demostrar que ab + 1 es un cuadrado perfecto. n ≥ 3. De manera semejante. x. nt = u − v. ◭ . es decir. Por ejemplo. x ≡ y mod n y j un entero positivo. Si u y v dejan el mismo residuo al ser divididos por n. el algoritmo de división distribuye los enteros en una de n clases dependiendo del residuo que deje el entero al ser dividido por n. ◭ 139 Ejemplo Hallar el residuo cuando 12233 · 455679 + 876533 es dividido por 4. Notamos de paso que si u ≡ v mod n. entonces decimos que u y v son congruentes módulo n y escribimos u ≡ v mod n. Así pues. 87653 = 87600 + 52 + 1 ≡ 1 mod 4. si u − v es divisible por n. Esto último es equivalente a ax + bu ≡ ay + bv mod n. n|(u − v) entonces hay enteros s. y enteros. 6|6 pero claramente 12 = 2 · 6 |6. n|(ax + bu − ay − bv). a|b. Dado un entero n ≥ 2.2) Por ejemplo. a|b. Por ejemplo. 11|99 y 33|330 implica que 11|330. 2|6. Entonces x j ≡ y j mod n. 61987 ≡ 6 · 61986 ≡ 6(62 )993 ≡ 6(−1)993 ≡ −6 ≡ 31 mod 37. 3 ≡ 24 mod 7 y 3 = 24 + (−3)7. El siguiente teorema es de suma utilidad. Demostración Como n|(x − y). entonces u = v + an para algún entero a. b. Si x ≡ y mod n y u ≡ v mod n entonces ax + bu ≡ ay + bv mod n. Sean a.t con ns = x − y. a|c =⇒ a|(xb + yc) (2. Si x ≡ y mod n y u ≡ v mod n entonces xu ≡ yv mod n. ◮Resolución: 62 ≡ −1 mod 37. 7|21 y 7|49 implica que 7 divide a 3 · 21 − 2 · 49 = −35. También. Demostración Pongamos a = u. Así 12233 · 455679 + 876533 ≡ 1 · 3 + 13 ≡ 4 ≡ 0 mod 4. Luego a(x − y) + b(u − v) = n(as + bt). 136 Corolario Sea n ≥ 2 un entero. 135 Teorema Sea n ≥ 2 un entero. 137 Corolario Sea n > 1 un entero. que 12233 · 455679 + 876533 es divisible por 4.1) a = 0. Entonces ab = 0.Aritmética modular 21 Las siguientes propiedades de divisibilidad son obvias. Si a no divide a b escribimos a |b. 3 ≡ 13 ≡ 26 ≡ −7 mod 10. Por ejemplo. Note además que a|c. b|c no necesariamente implica que ab|c. b|c =⇒ a|c (2. 455679 = 455600 + 76 + 3 ≡ 3. ◮Resolución: 12233 = 12200 + 32 + 1 ≡ 1 mod 4. b = y en el teorema anterior. c. O sea. o de manera equivalente. 138 Ejemplo Hallar el residuo cuando 61987 es dividido por 37. Pero 2 no es un cuadrado mod 5. entonces x2 ≡ 2 mod 5. ◮Resolución: Si x2 = 2 − 5y2. Luego 32n+1 + 2n+2 ≡ 7 · 2n ≡ 0 mod 7. a = 5. Ahora bien. 7 77 ≡ 74t+3 ≡ (74 )t · 73 ≡ 1t · 3 ≡ 3 mod 10. ◮Resolución: Observemos primero que sólo necesitamos cuadrar los enteros hasta 6. debemos tener 0 ≤ a. ◭ 145 Ejemplo Hallar los cuadrados mod 13. ◭ 143 Ejemplo Demostrar que 7 divide a 32n+1 + 2n+2 para todo número natural n. como 72 ≡ −1 mod 10.◭ . b = 1.22 Chapter 2 140 Ejemplo Hallar el último dígito de 3100 . para todo número natural n. b = 5. 3. Luego. 27 · 5 ≡ −1 mod 641 nos da 54 · 228 = (5 · 27)4 ≡ (−1)4 ≡ 1 mod 641. a = 6. 4. b = 2. ◮Resolución: Observemos que 32n+1 ≡ 3 · 9n ≡ 3 · 2n mod 7 y 2n+2 ≡ 4 · 2n mod 7. a = 2. Por lo tanto. b = 8 Luego 3080103. Luego 27 · 5 ≡ −1 mod 641 y 54 ≡ −24 mod 641. Cuadrando los enteros no negativos hasta el 6. Además. 3090503. 22 ≡ 4. Por inspección vemos que a = 8. 12. a = 3. 52 ≡ 12. Ensamblando todo esto. lo que significa que 641|(232 + 1). 72 ≡ 1 mod 4 y por lo tanto 77 ≡ (72 )3 · 7 ≡ 3 mod 4. Así. ◭ 7 142 Ejemplo Hallar el dígito de las unidades de 77 . lo que demuestra la aserción. 3060603. lo que quiere decir que hay un entero t tal que 77 = 3 + 4t. 9. a = 9. 42 ≡ 3. 32 ≡ 9. b ≤ 9. Así el último dígito es un 3. entonces tenemos 73 ≡ 72 · 7 ≡ −7 ≡ 3 mod 10 y 74 ≡ (72 )2 ≡ 1 mod 10. ◭ 141 Ejemplo Demostrar que 7|(22225555 + 55552222). b = 3. 3020303. los cuadrados mod 13 son 0. Esta útima congruencia y 54 ≡ −24 mod 641 nos da −24 · 228 ≡ 1 mod 641. ◮Resolución: 2222 ≡ 3 mod 7. y 10. porque r2 ≡ (13 − r)2 mod 13. ◮Resolución: Queremos hallar 3100 mod 10. Para que 30a0b03 sea divisible por 13 necesitamos 9b + 3a ≡ 7 mod 13. 3030703. ◮Resolución: Observemos que 641 = 27 · 5 + 1 = 24 + 54 . 7 ◮Resolución: Tenemos que hallar 77 mod 10. a = 0. 22225555 +55552222 ≡ 35555 +42222 ≡ (35 )1111 + (42 )1111 ≡ 51111 − 51111 ≡ 0 mod 7. 12 ≡ 1. 3100 = (32 )50 ≡ (−1)50 ≡ 1 mod 10. Aquí claro está. Observemos que 32 ≡ −1 mod 10. ◭ 147 Ejemplo Demostrar la siguiente observación de Euler: 232 + 1 es divisible por 641. 62 ≡ 10 mod 13. b = 7. ◭ 144 Ejemplo ¿Qué dígitos debe substituirse por a y b en 30a0b03 de tal manera que el entero resultante sea divisible por 13 ? ◮Resolución: Como 30a0b03 = 3 + 100b + 10000a + 3000000. Ahora bien. Ahora bien. observamos que 30a0b03 ≡ 3 + 9b + 3a + 3 ≡ 6 + 9b + 3a mod 13. 5555 ≡ 4 mod 7 y 35 ≡ 5 mod 7. 3050203. b = 6. 3000803 son todos divisibles por 13. el último dígito es el 1. ◭ 146 Ejemplo Demostrar que la ecuación x2 − 5y2 = 2 no tiene soluciones enteras. 1. obtenemos 02 ≡ 0. y tales que x3 = 2y + 15? ◮Resolución: No. . 24 ≡ 2. 1. . Así pues. 3. Esto produce una familia infinita de enteros n = 3k. (A y B se escriben en notación decimal. cada potencia de de 2 es congruente a 1. tenemos 10 j ≡ 1 mod 9. ◮Resolución: No hay tales soluciones. 2. ó 6 al ser dividido por 7. la suma de sus dígitos es A. . Por lo tanto. 153 Ejemplo (AHSME 1992) Los enteros de dos dígitos desde el 19 hasta el 92 se escriben consecutivamente para obtener el entero 192021222324 · · ·89909192.) . 22 ≡ 4. 152 Teorema Regla de los 9’s Un número natural n es divisible por 9 si y sólo si la suma de sus dígitos es divisible por 9. nunca deja residuo 1 cuando es dividido por 7. . 25 ≡ 4. Así pues. n2 . 23 ≡ 1 mod 7 y este ciclo de tres se repite. Hallar la suma de los dígitos de B. Colegimos que n = ak 10k + · · · + a1 10 + a0 ≡ ak + · · · + a1 + a0 . 2y + 15 ≡ 2. Pero 1599 ≡ 15 mod 16. 2k − 5 deja residuos 3. Ahora bien. Todas las cuartas potencias mod 16 son o bien ≡ 0 o bien ≡ 1 mod 16. ¿Cuál es la potencia mayor de 3 que divide a este número? ◮Resolución: Por la regla de los 9’s este número es divisible por 9 si y sólo si 19 + 20 + 21 + · · ·+ 92 = 372 · 3 lo es.Aritmética modular 23 148 Ejemplo Hallar un número infinito de enteros n tal que 2n + 27 sea divisible por 7. Como 10 ≡ 1 mod 9. . Demostración Sea n = ak 10k + ak−1 10k−1 + · · · + a1 10 + a0 la expansión en base-10 de n. el número es divisible por 3 pero no por 9. 1. ◭ Usando congruencias y el sistema de numeración decimal podemos obtener varias reglas de divisibilidad. Sea B la suma de los dígitos de A. . n14 ) de la ecuación diofántica n41 + n42 + · · · + n414 = 1599 de haberlas. . Esto es imposible. Esto significa que n41 + · · · + n414 es a lo sumo 14 mod 16. ◭ 151 Ejemplo (USAMO 1979) Determine todas las soluciones no negativas (n1 . ◭ 154 Ejemplo (IMO 1975) Cuando 44444444 se escribe en notación decimal. tal que 2n + 27 es divisible por 7. 2. k = 0. k = 1. ◭ 149 Ejemplo ¿Existen acaso enteros positivos x. 23 ≡ 1. 2. 4. de donde resulta la aserción. or 5 mod 7. 26 ≡ 1 mod 7 y así 23k ≡ 1 mod 3 para todos los enteros positivos k. ◭ 150 Ejemplo Demostrar que 2k − 5. La más famosa es quizás la siguiente. . ◮Resolución: 21 ≡ 2. . . 22 ≡ 4. y 6. Luego 23k + 27 ≡ 1 + 27 ≡ 0 mod 7 para todos los enteros positivos k. Los cubos mod 7 son 0. ó 4 mod 7. ◮Resolución: Observemos que 21 ≡ 2. . 875961·2753 = 2410520633. n 7|42 + 22 + 1 para todos los números naturales n. de donde colegimos que B ≤ 45. n > 3. n ∈ N. 160 Problema Demostrar que en cualquier subconjunto de 55 elementos tomado del conjunto {1. n |(k + c). . Por la regla de los 9’s. . . 174 Problema ¿Cuántos cuadrados hay mod 2n ? . que es patentemente falso. Por ejemplo. Por lo tanto n ≡ (−1)k ak + (−1)k−1 ak−1 + · · · − a1 + a0 mod 11. para enteros a. 1994) La sucesión creciente 3. c. 166 Problema Demostrar que si 157 Problema Hallar el último dígito de 2333333334 · 9987737 + 12 · 21327 + 12123 · 99987. 3. . . De todos los enteros naturales ≤ 45. 164 Problema Demostrar que si 9|(a3 + b3 + c3 ). de aquí A ≤ 159984. 173 Problema Demostrar que n 162 Problema Demostrar que para cualesquiera a. 161 Problema (AIME. son todos enteros. Sea C la suma de los dígitos de B. se sigue que C = 7. . es decir 12. entonces 3|abc. . Así la suma de los dígitos de B es a lo sumo 12. b. . mientras que 8924310064539 ≡ 8 − 9 + 2 − 4 + 3 − 1 + 0 − 0 + 6 − 4 + 4 − 3 + 9 ≡ 0 mod 11. ya que si esto fuese cierto entonces (8 + 7 + 5 + 9 + 6 + 1)(2 + 7 + 5 + 3) ≡ 2 + 4 + 1 + 0 + 5 + 2 + 0 + 6 + 3 + 3 mod 9. existe un en- tero k tal que n |(k + a). Pero como C ≡ 7 mod 9. 2. 2. tenemos 10 j ≡ (−1) j mod 11. 7 ≡ 44444444 ≡ A ≡ B ≡ C mod 9. . sólo si el número formado por sus últimos n dígitos es divisible por 2n . Determine el residuo cuando a83 se di- vide por 49. en notación decimal. Ahora bien. 172 Problema Demostrar que la suma de los dígitos. . 1 ≤ n ≤ 25 poseen la propiedad que n2 + 15n + 122 es divisible por 6. si y 165 Problema Describa todos los enteros n tal que 10|n10 + 1. b. . Como 10 ≡ −1 mod 11. 163 Problema (AIME 1983) Sea an = 6n + 8n . n |(k + b). no puede ser igual a 1991.100}. Tarea 155 Problema Si 62ab427 es un múltiplo de 99. 4444 log10 4444 < 4444 log10 104 = 17776. Sea n = ak 10k + ak−1 10k−1 + · · · + a1 10 + a0 . Por ejemplo. entonces a y b son también enteros. Así 44444444 = 44443(1481) · 4444 ≡ 1 · 7 ≡ 7 mod 9. n = 1. . de un cuadrado. . 3. n es divisible por 11 si y sólo si la suma alternante de sus dígitos es divisible por 11. 171 Problema Demostrar que no hay enteros con 800000007 = x2 + y2 + z2 . a2 − b2 . 158 Problema Demostrar que la ecuación x2 + 3xy − 2y2 = 122 no posee soluciones enteras. a3 − b3 . 2. Pero esto dice que 0 · 8 ≡ 8 mod 9. . c ∈ Z. 48. . 159 Problema Hallar cuántas n.50} tal que ningún par de elementos distintos de S tenga una suma divisible por 7? 169 Problema Demostrar que la ecuación x2 − 7y = 3 no tiene soluciones enteras. Esto significa que 44444444 tiene a lo sumo 17776 dígitos. ¿Cuál es el residuo cuando el 1994avo término de esta sucesión se divide por 1000? a − b. Entre los números naturales ≤ 159984 el que tiene la suma máximal de sus dígitos es 99999. 167 Problema Hallar los últimos dos dígitos de 3100 . 168 Problema (AHSME 1992) ¿Cuál es el tamaño del subconjunto mayor S de {1. y por lo tanto 44443 ≡ 73 ≡ 1 mod 9. 24. . 39 tiene la máxima suma dígitoal. a4 − b4 . siempre se encontrarán dos elementos que diferirán por 9. . 15. así la suma de los dígitos de 44444444 es a lo sumo 9 · 17776 = 159984. 912282219 ≡ 9 − 1 + 2 − 2 + 8 − 2 + 2 − 1 + 9 ≡ 7 mod 11 y así 912282219 no es divisible por 11. Se puede establecer un criterio de divisibilidad por 11 de una manera semejante. y así 8924310064539 es divisible por 11. 156 Problema Demostrar que un número natural es divisible por 2n . hallar los dígitos a y b. 170 Problema Demostrar que si 7|a2 + b2 entonces 7|a y 7|b.24 Chapter 2 ◮Resolución: Tenemos que 4444 ≡ 7 mod 9. o sea. ◭ Las congruencias mod 9 a veces pueden ser usadas para verificar multiplicaciones. consiste de aquellos mútiplos de 3 que son uno menos de un cuadrado. 2y. r no son necesariamente diferentes). demostrar que n − n + n − 1 es divisible por de $x4. . Si a0 . es un entero? 177 Problema Hallar todos los enteros a. p. donde x. q. 321 es lindo de tres dígitos ya que 1 divide a 3. . q.n} y sus primeros k dígitos forman un entero que es divisible por k para toda k. b. 1 ≤ k ≤ n. Por ejemplo. ¿Cuántos enteros lindos de seis dígitos hay? (a. . Dice que 88 pollos costaban un total n 2 178 Problema Si n > 1 es un entero. y son dígitos ilegibles. . an y f (1) son todos nones. 2. para el cual  12 + 22 + · · · + n2 n f (x) =  1/2 Xn ak xn−k k=0 un polinomio de grado n con coeficientes enteros. . demostrar que f (x) = 0 no tiene raíces racionales. b. a > 1 y todos los primos p.Tarea 25 175 Problema Demostrar que los no-múltiplos de 3 son potencias de 2 mod 3n . 181 Problema Un viejo recibo está algo borroso. ¿Cuánto costaba cada pollo? 2 (n − 1) . 179 Problema (Putnam 1952) Sea 176 Problema (USAMO 1986) ¿Cuál es el menor entero n > 1. r que satis- facen la ecuación pa = qb + ra 180 Problema (AHSME 1991) Un entero de n dígitos es lindo si sus n dígitos son una permutación del conjunto {1. 182 Problema Demostrar que un entero que consiste de 3n dígitos idénticos es divisible por 3n . 2 divide a 32 y 3 divide a 321. 2). (1. y) ∈ Z2 : |xy| = k} para k = 1. entonces su cardinalidad es el número de elementos que éste tiene. −1). −1). (1. . . (1. −3). 184 Regla (Regla de la suma: Forma disyuntiva) Sean E1 . (−1. Entonces card (E1 × E2 × · · · × Ek ) = card (E1 ) · card (E2 ) · · · card (Ek ) . esto es Ek ∩ E j = ∅ si k = j. 3). Entonces E1 = {(−1. (−2. (−1. −3). (−1. Comenzaremos nuestro estudio de la combinatoria con los siguientes dos principios fundamentales. (−1. (1. . −1). . 4). . (4. −1). ◭ 26 . Ek . (−1. 4). 1)} Por lo tanto 4 + 8 + 8 + 12 + 8 = 40 es el número deseado. Se denotará la cardinalidad de S por card (S). −1). E2 . (1. 1). 2). 5). −2). (2. conjuntos finitos mutuamente disjuntos. −1). (−1. (3. (−1. 185 Regla (Regla del producto) Sean E1 . 5. −4). Entonces card (E1 ∪ E2 ∪ · · · ∪ Ek ) = card (E1 ) + card(E2 ) + · · · + card(Ek ) . −2). −1). −5). −2). 5). −1). −2). 1)} E3 = {(−3. . (1. −5). (−4. (2. El número deseado es card (E1 ) + card(E2 ) + · · · + card (E5 ) . −1). 1). (−1. −1). (1. . . 3). 1). 1)} E2 = {(−2. (−2.1 Las reglas de la multiplicación y la suma 183 Definición (Cardinalidad de un conjunto) Si S es un conjunto. (1. (3. (1. . (2. (4. 1). 2). . (1. E2 . Ek . 1)} E5 = {(−5. . conjuntos finitos. 2). (−3. −4).Chapter 3 Combinatoria 3. (5. (−1. y) hay tales que 0 < |xy| ≤ 5? ◮Resolución: Póngase Ek = {(x. (−2. (−5. (2. 1)} E4 = {(−4. (5. . 1). 186 Ejemplo ¿Cuántos pares ordenados de enteros (x. un divisor de 400 deberá tener la forma 2α 5β con α ∈ {0. ◮Resolución: Divídase el diagrama como en la figura 3. 400.Las reglas de la multiplicación y la suma 27 187 Ejemplo Se escribe los divisores positivos de 400 en manera creciente: 1. 2. ◭ 190 Ejemplo Se escriben los enteros del 1 al 1000 en sucesión. 189 Ejemplo (AHSME 1977) ¿Cuántos caminos—consistiendo de una sucesión de segmentos verticales u horizontales y cada segmento uniendo un par adyacente de letras—en la figura 3. se contarán los caminos que comienzan con esta T hasta llegar a la C. cualquier divisor de 400 habrá de ser de la forma 2a 5b en donde 0 ≤ a ≤ 4 y 0 ≤ b ≤ 2. Si d(n) denota el número de divisores positivos de n. entonces d(n) = (a1 + 1)(a2 + 1) · · · (ak + 1). .2. 4. en donde los primos pi son distintos y las enteros ai son ≥ 1.1 deletrean CONT EST ? C C C C O C C O N O C C O N T N O C C O N T E T N O C C O N T E S E T N O C O N T E S T S T E N Figure 3. 8. O C C C O C O N C O N T C O N T E C O N T E S O N T E S T Figure 3. Hállese la suma de todos los dígitos. La otra porción del diagrama contribuirá 64 caminos más. Luego hay 5 maneras de elegir a a y 3 maneras de elegir a b. 188 Teorema Tenga el entero positivo n la factorización en primos a n = pa11 pa22 · · · pk k . hay 3 · 2 = 6 divisores de 400 que son además cuadrados. 2}. se obtiene el siguiente teorema. Para ser un cuadrado. 200. Pero la columna del medio es compartida por ambas porciones. 4} y β ∈ {0. 5. ¿Cuántos enteros hay en esta sucesión? ¿Cuántos de éstos son cuadrados? ◮Resolución: Como 400 = 24 · 52 . . Así pues. .2: Problem 189. dando un total de 5 · 3 = 15 divisores positivos.1: Problem 189. existen pues 26 = 64 caminos. 2. así que hay un total de 64 + 64 − 1 = 127 caminos. Ya que cada uno de los caminos requeridos debe utilizar la T de abajo a la derecha.. ◭ Arguyendo de la manera mostrada en el ejemplo 187. Ya que hay seis filas más a las que se podrá ir y en cada etapa se puede ir o hacia arriba o hacia la derecha. . 998). Hállese también la suma de estos números de 3-dígitos. Para obtener esta suma. Si 100x + 10y + z es uno de estos enteros. ◮Resolución: Hay 9 · 9 · 9 = 729 enteros positivos de 3-dígitos no poseyendo al 0 en su expansión decimal. Hállese también la suma de estos números de 3-dígitos. La suma de los dígitos del 000 al 999 es pues (0 + 1 + 2 + 3 + 4 + 5 + 6 + 7 + 8 + 9)(300) = 13500. así que cada dígito se utiliza 300 veces. ¿Qué dígito ocupa el 206790avo lugar? . Añadiendo 1 por la suma de los dígitos de 1000. la suma requerida es 81(1 + 2 + · + 9)100 + 81(1 + 2 + · + 9)10 + 81(1 + 2 + · + 9)1 = 404595. Aliter: Pareando los enteros del 0 al 999 de la siguiente manera (0. . hay 9 · 9 = 81 selecciones de las otras dos variables. 996). entonces para cada selección fija de una variable. se utilizan 3 × 1000 = 3000 dígitos. Cada uno de los 10 dígitos está representado de una manera igual y uniforme. (3. ◮Resolución: tres dígito es Utilizando el ejemplo 191. hay 900 − 729 = 171 tales enteros. ◭ 191 Ejemplo Determine cuántos enteros positivos de 3-dígitos pueden escribirse en notación decimal que no tengan al 0 en su expansión. 900(1099) = 494550. el total requerido es pues 27 · 500 + 1 = 13501. Luego. . (1. (499. ◭ 192 Ejemplo Determine cuántos enteros positivos de 3-dígitos pueden escribirse en notación decimal poseyendo al menos un 0 en su expansión. . la suma de los dígitos al escribir del 1 al 1000 es 13500 + 1 = 13501. . . 500). ◭ 2 193 Ejemplo Todos los enteros positivos se escriben en sucesión 123456789101112131415161718192021222324 . (2. La suma de todos los enteros de 100 + 101 + · · ·+ 998 + 999. observe que hay 900 sumandos y que la suma no cambia al cambiar el orden de los sumandos: dando S = S = 100 + 101 + ··· + 999 S = 999 + 998 + ··· + 100 2S = 1099 + 1099 + · · · + 1099 = 900(1099). Por lo tanto. 997).28 Chapter 3 ◮Resolución: Al escribir los enteros del 000 al 999 (con tres dígitos). La suma requerida es pues 494550 − 404595 = 89955. . 999). se ve que cada par tiene suma de dígitos 27 y que hay 500 pares. Las reglas de la multiplicación y la suma 29 ◮Resolución: Observemos que 1 · 9 + 2 · 90 + 3 · 900 + 4 · 9000 = 38819 y que 1 · 9 + 2 · 90 + 3 · 900 + 4 · 9000 + 5 · 90000 = 488819. Por lo tanto, el dígito buscado está entre los números de cinco dígitos. Si 5x+38819 ≥ 206790, entonces x ≥ 33595 (el entero x es cuánto nos adentramos en los números de cinco dígitos). Así pues, para llegar hasta el 206790avo dígito debemos de ir hasta el 33595avo número de cinco dígitos, es decir 43594 (el primero es 10000). Luego, hasta el dígito final de 43594 (el 4 de las unidades) hemos utilizado 38819 + 5 · 33595 = 206794 dígitos. Luego, el 4 ocupa la posición 206794ava, el 9 la 206793ava, el 5 la 206792ava, el 3 la 206791ava y el 4 la 206790ava. El dígito requerido es el 4. ◭ 194 Ejemplo ¿Cuántos enteros del 1 al 1000000 tienen al menos un 1 en su expansión decimal? ◮Resolución: Hay 8 enteros positivos de 1-dígito, 8 · 9 = 72 enteros positivos de 2-dígitos, 8 · 9 · 9 = 648 enteros positivos de 3-dígitos, 8 · 9 · 9 · 9 = 5832 enteros positivos de 4-dígitos, 8 · 9 · 9 · 9 · 9 = 52488 enteros positivos de 5-dígitos, 8 · 9 · 9 · 9 · 9 · 9 = 472392 enteros positivos de 6-dígitos, no poseyendo el dígito 1. Así pues hay 8 + 72 + 648 + 5832 + 52488 + 472392 = 531440 entre el 1 y el 999999 no poseyendo el dígito 1. Luego hay 999999 − 531440 = 468559 poseyendo al menos un 1 y así 468559 + 1 = 468560 enteros entre el 1 y el 1000000 teniendo al menos un 1 en su expansión decimal. Aliter: Analizaremos los enteros del 0 al 999999. Al resultado final le sumaremos 1, ya que 1000000 tiene un 1 en su expansión. Dividamos este conjunto de un millón de enteros como sigue: en 100000 decenas {0, 1, 2, . . ., 9} {10, 11, 12, . . ., 19} .. . {999990, 999991, . . ., 999999}. En 10000 centenas {0, 1, 2, . . ., 99} {100, 101, 102, . . ., 199} .. . 30 Chapter 3 {999900, 999901, . . ., 999999}, etc., hasta llegar a diez 100000enas {0, 1, 2, . . ., 99999} {100000, 100001, 100002, . . ., 199999} .. . {900000, 900001, . . ., 999999}. En la primera decena hay solamente un número, el 1, que tiene un 1 en su numeración decimal. En la segunda decena, los diez enteros tienen un 1 en su numeración decimal. En la primera centena, cada decena, excepto la segunda, contendrá exactamente un entero que tiene un 1 en su expansión. La segunda decena, claro está, tiene sus diez enteros con 1’s en sus expansiones. En consecuencia, la primera centena tiene 10 + 9 · 1 enteros que tienen el 1 en sus expansiones. En el primer millar, cada centena excepto la segunda tendrá exactamente 10 +9 ·1 enteros con el 1 en su expansión. La segunda centena, que consiste de los enteros 100, 101, . . .199 tendrá sus 100 enteros con el 1 en su expansión. Así pues, el primer millar tendrá exactamente 100 + 9(10 + 9 · 1) = 102 + 9 · 10 + 9 enteros con el 1 en su expansión. En la primera decena de millar, cada millar excepto el segundo, tendrá exactamente 102 + 9 · 10 + 9 enteros con el 1 en su expansión. El segundo millar tendrá sus 103 enteros con el 1 en su expansión. Luego, en la primera decena de millar hay 103 + 9(102 + 9 · 10 + 9) = 103 + 9 · 102 + 92 · 10 + 93 enteros con el 1 en su expansión. Un razonamiento semejante nos lleva a concluir que en la primera centena de millar hay 104 + 9(103 + 9 · 102 + 92 · 10 + 93) = 104 + 9 · 103 + 92 · 102 + 93 · 10 + 94 enteros con el 1 en su expansión y en los primeros millones hay 105 + 9 · 104 + 92 · 103 + 93 · 102 + 94 · 10 + 95 = 106 − 96 = 468559 10 − 9 enteros con el 1 en su expansión. Esto quiere decir que en los enteros del 0 al 999999 hay 468559 enteros con el 1 en su expansión y en los enteros del 1 al 1000000 hay 468559 + 1 = 468560 enteros con el 1 en su expansión. ◭ Tarea 195 Problema Se marcan n puntos, 1, 2, . . . ,n sobre una circunferencia, que colocamos a igual distancia unos de los otros. Si el punto marcado 15 está directamente opuesto al marcado 49, ¿cuántos puntos hay en total? 196 Problema ¿Cuántos de los factores de 295 hay que sean mayores que 1, 000, 000? 197 Problema Se escribe la sucesión de enteros positivos. 1, 2, 3, 4, 5, 6, 7, 8, 9, 10, 11, 12, 13, 14, 15, 16, 17, 18, 19, 20, . . . 199 Problema Para escribir un libro se utilizaron 1890 dígitos. ¿Cuántas páginas tiene el libro? 200 Problema Hallar d(1260), σ (1260) y φ (1260). 201 Problema Los enteros del 1 al 1000 se escriben en orden sobre un círculo. Comen- zando con 1, cada quinceavo número es marcado (esto es, 1, 16, 31, etc.). Este proceso se repite hasta que se marque un número por segunda vez. ¿Cuántos números sin marcar quedan? ¿Qué dígito ocupa la posición 3000-ava? 202 Problema ¿Cuántos enteros entre 1 y 3012 son divisibles por 5 o por 7 pero no por ambos números? 198 Problema ¿Cuántos divisores positivos tiene 28 39 52 ? ¿Cuál es la suma de estos divisores? Métodos combinatorios 31 203 Problema Escribir la versión de cuatro conjuntos del principio de inclusión- y que exclusión. max(x, y, z) = x + y + z − min(x, y) − min(y, z) − min(z, x) + min(x, y, z). 204 Problema ¿Cuántos números primos hay entre 1 y 100? ¿Qué relación nota entre estas fórmulas y el principio de inclusión-exclusión? 205 Problema Sean x, y, z números reales. Demostrar que 206 Problema ¿Cuántos enteros entre 1 y 1000000 no son ni cuadrados, ni cubos, ni max(x, y) = x + y − min(x, y) cuartas, ni quintas potencias? 3.2 Métodos combinatorios La gran mayoría de los problemas de conteo pertenecen a una de cuatro categorías, que explicaremos mediante el siguiente ejemplo. 207 Ejemplo Considérese el conjunto {a, b, c, d}. Selecciónese dos letras de entre estas cuatro. Dependiendo de la interpretación se obtendrá una de las siguientes cuatro respuestas. ➊ Permutaciones con repetición. El orden en que se listan las letras importa y se permite repetir letras. En este caso hay 4 · 4 = 16 selecciones posibles: aa ab ac ad ba bb bc bd ca cb cc cd da db dc dd ➋ Permutaciones sin repetición. El orden en que se listan las letras importa y no se permite repetir letras. En este caso hay 4 · 3 = 12 selecciones posibles: ab ba ca ac ad bc bd cb da db cd dc ➌ Combinaciones con repetición. El orden en que se listan las letras no importa y se permite repetir letras. En este caso 4·3 + 4 = 10 selecciones posibles: hay 2 aa ab ac ad bb bc bd cc cd dd ➍ Combinaciones sin repetición. El orden en que se listan las letras no importa y no se permite repetir letras. En este 32 Chapter 3 caso hay 4·3 = 6 selecciones posibles: 2 ab ac ad bc bd cd Consideraremos ahora ejemplos de cada situación. 3.2.1 Permutaciones sin repetición 208 Definición Definimos el símbolo ! (factorial), como sigue: 0! = 1, y para entero n ≥ 1, n! = 1 · 2 · 3 · · ·n. n! se lee n factorial. 209 Ejemplo Se tiene 1! = 1, 2! = 1 · 2 = 2, 3! = 1 · 2 · 3 = 6, 4! = 1 · 2 · 3 · 4 = 24, 5! = 1 · 2 · 3 · 4 · 5 = 120. 210 Definición Sean x1 , x2 , . . . , xn n objetos distintos . Una permutación de estos objetos es simplemente un rearreglo de ellos. 211 Ejemplo Hay 24 permutaciones de las letras de la palabra MAT H: MAT H MAHT MTAH MT HA MHTA MHAT AMT H AMHT AT MH AT HM AHT M AHMT TAMH TAHM T MAH T MHA T HMA T HAM HAT M HAMT HTAM HT MA HMTA HMAT 212 Teorema Sean x1 , x2 , . . . , xn n objetos distintos. Hay n! permutaciones de ellos. Demostración: La primera posición de puede elegir de n maneras, la segunda en n − 1 maneras, la tercera en n − 2, etc. Esto da n(n − 1)(n − 2) · · ·2 · 1 = n!. ❑ which es 24856274386944000. el número de arreglos buscados es 20! = 2432902008176640000. 214 Ejemplo ¿En cuántas maneras se pueden permutar las letras de la palabra MASSACHUSETT S ? ◮Resolución: Póngase subíndices a las repeticiones. ➍ Alíniense los libros alemanes y los libros españoles primero. Así el de permutaciones es (13)(12)(11)(10)(9)(8)(7)(6)12!. Al alinear estos 5 + 7 = 12 libros. Se permutan los libros alemanes en 5! maneras. “pegamos” los libros franceses y ponemos este bulto en uno de los espacios. se crean 12 + 1 = 13 espacios (incluyendo el espacio frente al primer libro.. S1 S2 S3 S4 puede permutarse en 4! maneras y T1 T2 puede permutarse en 2! maneras. ◭ 3. ➋ Agrúpese los libros por lengua lo que garantizará que los libros de una misma lengua estarán juntos. Así pues. Ahora. A1 A2 puede permutarse en 2! maneras. Luego el total number of permutaciones es (13)8!12! = 251073478656000. el octavo libro francés puede ponerse en algún de los restantes 6 espacios. Luego. Se permutan los 3 grupos en 3! maneras. los ponemos en estos espacios. ➌ Alíniense los libros alemanes y los libros españoles primero. Para asegurar que todos los libros franceses yagan uno al lado del otro. El primer libro francés puede ponerse en cualquiera de 13 espacios.2. En cada una de estas 13! permutaciones. .Métodos combinatorios 33 213 Ejemplo Un librero tiene 5 libros alemanes. los libros españoles en 7! maneras y los libros franceses en 8! maneras. etc. 7 libros españoles y 8 libros franceses. Luego el número total de maneras es 3!5!7!8! = 146313216000. el sobre-estimado de 13! es corregido y la cuenta final es 13! = 64864800. 2!4!2! ◭ Razonando de manera análoga al ejemplo 214. los libros nofranceses pueden ser permutados en 12! maneras. los espacios entre los libros y el espacio luego del último libro). Cada libro es diferente de cada otro. los espacios entre los libros y el espacio luego del último libro). Al alinear estos 5 + 7 = 12 libros. ➊ ¿Cuántos arreglos hay de estos libros? ➋ ¿Cuántos arreglos hay de estos libros si los libros deben estar agrupados por lengua? ➌ ¿Cuántos arreglos hay de estos libros si todos los libros franceses deben estar juntos? ➍ ¿Cuántos arreglos hay de estos libros si ningún libro francés está junto a otro libro francés? ◮Resolución: ➊ Se permutan 5 + 7 + 8 = 20 objetos. se podrá demostrar el siguiente teorema. gracias al teorema 212. formando MA1 S1 S2 A2CHUS3 ET1 T2 S4 .2 Permutaciones con repetición Consideramos ahora permutaciones con objetos repetidos. en cualquiera de 12. el segundo. Hay ahora 13 objetos distintos. Ahora. los libros franceses se permutan en 8! maneras y los libros no-franceses en 12! maneras. Para asegurar que ningún libro francés yaga junto a otro libro francés. los que pueden ser permutados en 13! maneras diferentes. se crean 12 + 1 = 13 espacios (incluyendo el espacio frente al primer libro. c) Nmero de permutaciones (1.34 Chapter 3 215 Teorema Si hay k tipos de objetos: n1 del tipo 1. ◮Resolución: Primero buscaremos soluciones del tipo a + b + c = 9. 1. H. etc. así pues 1 + 7 + 1 se cuenta aparte de 7 + 1 + 1. S. n2 del tipo 2. 4) 3! = 6 3! =1 3! (3. 5) (2. . dígase. 3. S. 3. 5) 3! = 6 3! =3 2! 3! =3 2! (1. H. T. T. 4. 6) 3! = 6 (1. dando un total de permutaciones de 10! = 907200. agregando luego las 9 letras A. U. ◭ 218 Ejemplo ¿En cuántas maneras pueden arreglarse las letras de la palabra MURMUR de tal manera que letras idénticas no sean adyacentes? ◮Resolución: Si comenzáramos con. n1 !n2 ! · · · nk ! 216 Ejemplo ¿En cuántas maneras se pueden permutar las letras de la palabra MASSACHUSETT S en tal manera que la partícula MASS esté siempre junta. E. C. U. 3. 2!2! ◭ 217 Ejemplo ¿De cuántas maneras se puede escribir el 9 como la suma de tres enteros positivos? Se tendrá cuenta del orden. 2. 2. S hay cuatro S’s y dos T ’s. 7) 3! =3 2! (1. con las letras en este orden? ◮Resolución: La partícula MASS se puede considerar como un bloque de una letra. Se tiene (a. T. 1 ≤ a ≤ b ≤ c ≤ 7 y veremos las permutaciones de cada trío. MU entonces las R se pueden disponer de la manera siguiente: M U R M U R R . S. Entonces el número de maneras en que estos n1 + n2 + · · · + nk objetos pueden permutarse es (n1 + n2 + · · · + nk )! . R . C. E. En A. 3) El número deseado es así 3 + 6 + 6 + 3 + 3 + 6 + 1 = 28. etc. T. b. 4) (2. Obsérvese también los siguientes valores de frontera Ç å Ç å n n = = 1. ◭ 3. k. 1 n−1 6·5·4 = 20. 5 7 222 Definición Considérese n distintos. 221 Ejemplo Ç å Ç å 11 11 = = 55. = k!(n − k)! 1 · 2 · 3 · · ·k k ☞ Obsérvese que en esta última fracción hay k factores tanto en el numerador como en el denominador. si se comenzase con MU se tendría 2 + 2 + 1 = 5 arreglos posibles. se puede elegir la primera letra de 3 maneras y la segunda en 2 maneras. Una k-combinación es una selección de k. (0 ≤ k ≤ n) objetos de entre los n sin tomar en cuenta se orden. en el segundo hay 2! = 2 y en el tercero hay solamente 1!. 0 n 220 Ejemplo Se tiene Ç å 6 3 Ç å 11 2 Ç å 12 7 Ç å 110 109 Ç å 110 0 = = = Ç å Ç å n n = = n. En el primer caso hay 2! = 2 de poner las M y la U restantes. En general. Así. El símbolo Ç å n (léase “n tomando k”) se define como k Ç å n · (n − 1) · (n − 2) · · ·(n − k + 1) n n! = .3 Combinaciones sin repetición 219 Definición Sean n.Métodos combinatorios 35 M U R R . 0 ≤ k ≤ n. k enteros no negativos. 1·2 12 · 11 · 10 · 9 · 8 · 7 · 6 = 792. Así pues el número de number of maneras pedidas es 3 · 2 · 5 = 30. la siguiente identidad de simetría Ç å Ç å n n n! n! . . = 1. = = = k k!(n − k)! (n − k)!(n − (n − k))! n−k Ésta se puede interpretar como sigue: el número de maneras de sacar k boletos de un sombrero que tiene n boletos distintos es él mismo número de maneras de elegir n − k boletos que permanezcan dentro del sombrero. con 0 ≤ k ≤ n. se satisface para enteros n. ☞ Ya que n − (n − k) = k. 9 2 Ç å Ç å 12 12 = = 792. 1·2·3·4·5·6·7 = 110.2. 1·2·3 11 · 10 = 55. O también podemos seleccionar a las 7 personas primero. el cual es Ç å 5 5·4·3 = = 10. XZ.YW Z. y luego al presidente de entre los 7. Luego. d. Lo que queremos es el número de maneras de seleccionar tres elementos de cinco. XZW. 225 Teorema Considérese n objetos distintos y sea k un entero satisfaciendo 0 ≤ k ≤ n. c. XW. Así el número de maneras es 7 = 542640. XYW. f }? ◮Resolución: Vea que aquí el orden carece de importancia. k Demostración: Tómese cualesquiera k objetos. 19 Así el número de maneras es 20 = 542640. 224 Ejemplo Las 3-combinaciones de la lista {X. 7 7 que concuerda con lo anterior. El número de k-combinaciones de Ç å estos n objetos es n . ya que hay n maneras de elegir el primer objeto.W } son XY. de 7 maneras.W } son XY Z. de 6 Ç å Ç å 20 20 maneras. etc.Y. b. entonces se produciría una lista con n(n − 1)(n − 2) · · ·(n − k + 1) entradas.36 Chapter 3 223 Ejemplo Las 2-combinaciones de la lista {X.Y. Luego a los otros 6 de entre los 19 restantes.YW. 3 1·2·3 ◭ 227 Ejemplo ¿De cuántas maneras podemos seleccionar un comité de tres personas de entre diez profesores? Ç ◮Resolución: å 10 = 120 ◭ 3 228 Ejemplo ¿De cuántas maneras podemos seleccionar un comité de siete personas con un presidente de entre veinte per- sonas? ◮Resolución: Primero se selecciona Ç å al presidente de 20 maneras. k! k ❑ 226 Ejemplo ¿Cuántos subconjuntos de tres elementos tiene el conjunto {a. n − 1 maneras de elegir el segundo. Z. ◭ 229 Ejemplo ¿De cuántas maneras podemos seleccionar un comité de siete personas con un presidente y un secretario de entre veinte personas? El secretario no sirve de presidente. . Z. el número total de k-combinaciones es Ç å n n(n − 1)(n − 2) · · ·(n − k + 1) = . Esta selección particular de k objetos se puede permutar en k! maneras. Si se tuviese en cuenta el order.W Z.Y Z. 3 obsérvese que cualquier camino de longitud mínima deberá consistir de 6 movimientos horizontales y de 3 verticales. se 5 cuenta el número de pasos de A a O (que son = 20 en número) y el número de caminos que hay de O a B ( que son 3 Ç å Ç åÇ å 4 5 4 = 4 en número). . 20}. Luego el número de maneras es Ç å Ç åÇ åÇ å Ç å Ç å 6 6 7 7 7 7 + + + = 384. uno deja residuo 1 y el tercero deja residuo 2 al ser divididos por 3. .Métodos combinatorios 37 Ç å Ç å 18 20 ◮Resolución: De 20 · 19 · = 7·6 = 3255840 maneras. Ç å 9 Hay así = 84 caminos. 2. . . . 2. . ◭ 5 7 230 Ejemplo Se toman tres enteros diferentes del conjunto {1. 7 números que dejan residuo 2 al ser divididos por 3. De estas 9 desplazamientos. las 3 verticales quedan completamente determinadas. 3 1 1 1 3 3 ◭ 231 Ejemplo Para contar el número de caminos de longitud mínima de A a B en la figura 3. La suma de tres enteros será divisible por 3 cuando (a) los tres enteros son divisibles por 3. . ¿En cuántas formas se pueden tomar de tal manera que su suma sea divisible por 3? ◮Resolución: En {1. Figure 3. 3 3 3 B B O A A Figure 3. 20} hay 6 números que dejan residuo 0 al ser divididos por 3. . (c) los tres dejan residuo 1 al ser divididos por 3. 6 232 Ejemplo Para contar el número de caminos Ç de å longitud mínima de A a B en la figura 3. 7 números que dejan residuo 1 al ser divididos por 3.4: Example 232. una vez se hallan elegido las 6 que serán horizontales.3: Example 231.4 que pasan por el punto O. El número total de caminos lo es pues = (20)(4) = 80. (b) uno de los enteros es divisible por 3. . para un total de 6 + 3 = 9 movidas. (d) los tres dejan residuo 2 al ser divididos por 3. b > 0. ◮Resolución: Este es el ejemplo 217. r−1 Demostración: Póngase xr − 1 = yr . El número de soluciones enteras no negativas de y1 + y2 + · · · + yr = n Ç es å n+r−1 . Entonces xr ≥ 1. r−1 Demostración: Escríbase n como n = 1 + 1 + · · ·+ 1 + 1. Para descomponer a n en r sumandos sólo se necesita escoger r − 1 +’s del total de n − 1. El número de soluciones enteras positivas de x1 + x2 + · · · + xr = n Ç es å n−1 . . lo cual demuestra el teorema. La ecuación x1 − 1 + x2 − 1 + · · · + xr − 1 = n es equivalente a x1 + x2 + · · · + xr = n + r. 3 ◭ 236 Corolario Sea n un entero positivo. que gracias al teorema 233 son en total Ç å 99 = 156849.4 Combinaciones con repetición 233 Teorema (De Moivre) Sea n un entero positivo. Por el teorema 233 esto es Ç å Ç å 9−1 8 = = 28. a > 0.2. c > 0. ❑ 234 Ejemplo ¿En cuántas maneras se puede escribir 9 como la suma de tres enteros estrictamente positivos? Aquí 1 + 7 + 1 se tomará como diferente de 7 + 1 + 1.38 Chapter 3 3. en donde hay n 1’s y n − 1 +’s. 3−1 2 ◭ 235 Ejemplo ¿En cuántas maneras se puede escribir 100 como la suma de cuatro enteros estrictamente positivos? ◮Resolución: Se buscan soluciones íntegras de a + b + c + d = 100. Se buscan soluciones íntegras de a + b + c = 9. Se necesita una descomposición de la forma 210 = 2a 2b 2c . d) de enteros que satisfagan a + b + c + d = 100. c. sea R2 el número de elementos en A pero no en B (i. b ≥ 0.. o sea.. en A \ B) y por R3 el número de elementos en B pero no en A (i. 3 ◭ 238 Ejemplo ¿En cuántas maneras se puede escribir 1024 como el producto de tres enteros positivos? ◮Resolución: Véase que 1024 = 210 . tiene Ç å n+r−1 r−1 soluciones.Principio de inclusión-exclusión 39 que por el teorema 233. Por el teorema 233 este número es Ç å 49 = 18424. se sabe que 10 tanto fuman como mascan tabaco.5. En esta sección se mostrará como deducir la cardinalidad de la reunión de conjuntos no necesariamente disjuntos. b.3 Principio de inclusión-exclusión La regla de la adición 184 da la cardinalidad de la reunión de conjuntos disjuntos. a ≥ 0. de a′ + b′ + c + d = 50. Entonces se desea el número de soluciones enteras estrictamente positivas de a′ + 29 + b′ + 21 + c + d = 100. ❑ 237 Ejemplo Hállese el número de cuartetos (a. Se tiene R1 + R2 + R3 = card (A ∪ B). c ≥ 0. Ç å Ç å 10 + 3 − 1 12 = = 66 en número.. lo cual demuestra el teorema. b > 21.e. en A ∩ B). a ≥ 30. 239 Teorema (Inclusión-exclusión para dos conjuntos) card (A ∪ B) = card (A) + card(B) − card(A ∩ B) Demostración: En el diagrama de Venn 3. 28 fuman y 16 mascan tabaco. b′ + 20 = b. sea R1 el número de elementos simultáneamente en ambos conjuntos (i. d ≥ 1. ◮Resolución: Póngase a′ + 29 = a. ◭ Por el teorema 236 éstas son 3−1 2 3. Además.e. ¿Cuántas personas ni fuman ni mascan tabaco? . esto es . c ≥ 1. en B \ A). El principio de inclusión-exclusión es atribuído tanto a Sylvester como a Poincaré. se necesitan soluciones íntegras de a + b + c = 10.e. ❑ 240 Ejemplo De 40 personas. . 396. 14. . Por lo tanto. 2. el número de personas que ni fuma ni masca tabaco es 40 − 34 = 6. . card (A ∩ B) = = 40. ◮Resolución: Si X es un conjunto finito. 3. Ahora bien. 12. 400.. ◭ 241 Ejemplo Cuatrocientos niños forman un círculo y los numeramos 1. ◭ Sea n = pa11 · pa22 · · · pas s . sólo 160 k’s provocan que todos los niños sean marcados. . 2. . . 3. R2 R3 R5 R7 R1 B Figure 3. . entonces por inclusión-exclusión se puede demostrar que φ (n) = (pa11 − p1a1 −1 )(pa22 − pa22−1 ) · · · (pas s − pas s −1 ). Marcamos cada k niños deteniéndonos cuando marcamos a un niño por segunda vez. . 16. 10. los enteros de la forma 6k ± 1. . que lo marcamos pues por segunda vez. entonces no marcamos a todos los niños. . . 242 Teorema (Inclusión-exclusión en tres conjuntos) card (A ∪ B ∪C) = card (A) + card(B) + card (C) −card (A ∩ B) − card(B ∩C) − card (C ∩ A) +card (A ∩ B ∩C) .exclusión card (A ∪ B) = card (A) + card (B) − card(A ∩ B). 3. Luego nos toca marcar al niño 2. .400} no son relativamente primos a 400 y 400 − 240 = 160 lo son. Ahora bien.. . 400}. . si k = 6. 9. Notamos que dejamos sin marcar a los niños 1. hay 34 personas o que fuman o que mascan tabaco. Así pues. Así pues. 400 = 24 52 . card (A) = 2 5 10 ï Luego card (A ∪ B) = 240 enteros en {1. 1 ≤ k ≤ 400 un entero fijo. 2. las k’s requeridas son aquellas k’s entre 1 y 400 inclusive que son relativamente primas a 400.6: Inclusión-exclusión para tres conjuntos. 1 ≤ k ≤ n relativamente primos a n. 6k + 3 entre 1 y 400. esto es. 20. 11. . pues el sexto luego 396 es el 2. ò ï ò ï ò 400 400 400 = 200. el número de elementos que hay en el conjunto. . ¿Para cuántos valores de k serán marcados todos los niños al menos una vez? ◮Resolución: Vemos que si k tiene un factor mayor que 1 en común con 400. . . donde las p’s son primos distintos.C 40 Chapter 3 R4 R6 B A R2 R1 R3 A Figure 3. . Sea k. . Nos toca ahora marcar a los niños 4. 2. Seguimos marcando a los niños 8.. . 7.. Sea A el conjunto de personas que fuman y B el conjunto de personas que mascan tabaco. 400. Por ejemplo. . Por inclusión. 400} y B el conjunto de múltiplos de 5 en {1. 5. denotaremos por card (X) su cardinalidad.. Sea A el conjunto los múltiplos de 2 en {1. . Para contar las k’s que no tienen factores primos en común con 400. card (B) = = 80. comenzamos marcando los ninõs 6. 398. 18. . El próximo niño a marcar es el sexto. Si φ (n) denota el número de enteros k. Como card (A ∪ B) = card (A) + card (B) − card (A ∩ B) = 28 + 16 − 10 = 34.399—esto es. .5: Inclusión-exclusión para dos conjuntos. contaremos las que sí tienen factores en común con 400 y las restaremos a 400. Véase también la figura 3. Luego. card (A ∪ B ∪C) = card (A ∪ (B ∪C)) = card (A) + card(B ∪C) − card (A ∩ (B ∪C)) = card (A) + card(B ∪C) − card ((A ∩ B) ∪ (A ∩C)) = card (A) + card(B) + card(C) − card (B ∩C) −card (A ∩ B) − card (A ∩C) +card ((A ∩ B) ∩ (A ∩C)) = card (A) + card(B) + card(C) − card (B ∩C) − (card (A ∩ B) + card(A ∩C) − card(A ∩ B ∩C)) = card (A) + card(B) + card(C) −card (A ∩ B) − card (B ∩C) − card(C ∩ A) +card (A ∩ B ∩C). card(A ∩ B) = 30. ◭ 244 Ejemplo ¿Cuántos números entre 1 y 600 inclusive no son divisibles ni por 3. 200 − 150 = 50 no utilizan ninguna droga. 7. Finalmente. card (B ∩C) = 50. 5. heroína y barbitúricos. En- . Esto demuestra el teorema. 75 usan cocaína. card (A ∩ B ∩C) = 10. 50 usan heroína y barbitúricos y 40 utilizan cocaína y barbitúricos. ni por 5. card (B) = 85. card(C ∩ A) = 40.6.Principio de inclusión-exclusión 41 Demostración: Combinando la propiedad asociativa y distributiva. Se nos es dado que card (A) = 75. ¿Cuántos de estos 200 políticos no usan drogas? ◮Resolución: Sean A. respectivamente. card (C) = 100. ni por 7? ◮Resolución: Denótese por Ak aquellos enteros en el intervalo [1. 85 usan heroína y 100 utilizan barbitúricos. Entre los 200. Por el principio de inclusión-exclusión card (A ∪ B ∪C) = 75 + 85 + 100 − 30 − 50 − 40 + 10 = 150 políticos utilizan al menos una droga. 600] que son divisibles por k = 3. ❑ 243 Ejemplo De 200 políticos entrevistados en la legislatura. 30 usan cocaína y heroína.C el conjunto de políticos entre los 200 que utilizan cocaína. 10 indulgen en el uso de las tres substancias. B. 9550 sin 8 Figure 3. 2. 8. 245 Ejemplo En un grupo de 30 personas . En la región común de A y B que no ha sido llenada ponemos 5 − 2 = 3.7. 6. 12 hablan castellano y 10 hablan francés. 8 hablan inglés .7: Ejemplo 245. ◭ 246 Ejemplo Considérese el conjunto de enteros naturales de cinco dígitos escritos en notación decimal. Llenamos sucesivamente los diagramas de Venn en la figura 3. 4. ¿Cuántos tienen exactamente cuatro 9’s? ¿Cuántos tienen exactamente cinco 9’s? ¿Cuántos no tienen ni un 8 ni un 9 en su expansión decimal? ¿Cuántos no tienen ni un 7. ¿Cuántos tienen o bien un 7. Se sabe que 5 hablan inglés y castellano . o un 9 en su expansión decimal? . ◭ card (A3 ) = ⌊ C sin 9 1 9550 2 4 14266 3 A 3 1 14266 14406 B 9550 2 14266 sin 7 Figure 3. 7}. 10. 5 600 card (A7 ) = ⌊ ⌋ = 85. En la parte restante de A ponemos 8 − 2 − 3 − 2 = 1. 600] divisibles por al menos uno de los enteros en {3. 3. 57} son 600 − 325 = 275 en total. 9. Los no divisibles por los enteros en {3. ni un 9 en su expansión decimal? 11.8: Ejemplo 246. 5 castellano y francés. 1. Las regiones disjuntas cuentan 1 + 2 + 3 + 4 + 1 + 2 + 3 = 16 personas. Fuera de estos tres círculos hay 30 − 16 = 14. y en la parte restante de C ponemos 10 − 2 − 3 − 4 = 1. 7 600 ⌋ = 40 card (A15 ) = ⌊ 15 600 ⌋ = 28 card (A21 ) = ⌊ 21 600 card (A35 ) = ⌊ ⌋ = 17 35 600 ⌋ = 5 card (A105 ) = ⌊ 105 Por inclusión-exclusión hay 200 + 120 + 85 − 40 − 28 − 17 + 5 = 325 enteros en [1. ¿Cuántas personas no hablan ninguno de estos idiomas? ◮Resolución: Sea A el conjunto de los anglófonos. y 7 inglés y francés. ni un 8. En la región común entre A y C que no ha sido llenada ponemos 5 − 3 = 2. 5. 3 600 card (A5 ) = ⌊ ⌋ = 120. en la parte restante de B ponemos 12 − 4 − 3 − 2 = 3. En la intersección de los tres ponemos 8. o un 8. B el conjunto de los hispanófonos y C el conjunto de los francófonos. Tres personas hablan los tres idiomas. 5. En la región común de B y C que no ha sido llenada ponemos 7 − 3 = 4. ¿Cuántos hay? ¿Cuántos no tienen un 9 en su expansión decimal? ¿Cuántos tienen al menos un 9 en su expansión decimal? ¿Cuántos tienen exactamente un 9? ¿Cuántos tienen exactamente dos 9’s? ¿Cuántos tienen exactamente tres 9’s? 7.42 Chapter 3 tonces 600 ⌋ = 200. entonces hayã8 selecciones para este primer dígito. Hay 6 selecciones para el primer dígito y 7 selecciones para los restantes 4 dígitos. En este caso hay 8 · 6 · 92 = 3888 tales números. Obviamente existe solamente 1 entero tal. Además hay =4 1 maneras de seleccionar donde el 9 estará. 4. Se condiciona en el primer dígito. 2.8. b ≥ 0. lo que da 94 = 6561 números. 7. b ≥ 0. En total hay 2916 + 3888 = 6804 enteros de cinco dígitos con exactamente dos 9’s en su expansión decimal. ◭ 247 Ejemplo ¿Cuántas soluciones enteras tiene la ecuación a + b + c + d = 100. Se condiciona en el primer dígito. Hay = 82160 soluciones íntegras 3 a + b + c + d = 100. lo que da 4 · 9 = 36 tales números. y hay 93 maneras de llenar los 3 espacios restantes. que satisfacen las siguientes restricciones: 1 ≤ a ≤ 10. y selec4 = 4 maneras. En total hay 6561 + 23328 = 29889 enteros de cinco dígitos con exactamente un 9 en expansión decimal. lo que da 6 · 74 = 14406 tales enteros. 3. Si el primer dígito no es un 9. a ≥ 1. Los otros dos dígitos restantes en 2 deberán de ser distintos de 9. El número de selecciones es así 8 · 94 = 52488. En total hay 36 + 8 = 44 enteros de cinco dígitos con exactamente tres 9’s en expansión decimal. Además hay Å 4 = 4 maneras de seleccionar donde los cuatro 9’s 4 estarán. c ≥ 2. De la figura 3. Además hay = 6 man2 eras de seleccionar donde los otros dos 9’s estarán. 11. así llenando todos estos espacios. Hay 8 selecciones posibles para el primer dígito y 9 selecciones posibles para los dígitos restantes. Si el primer dígito es un 9 entonces tres de los restantes cuatro serán Å ã9’s. los números dentro de los círculos suma a 85854. De nuevo. y la selección de posición se puede lograr 4 = 6 maneras. lo que da 4 · 93 = 2916 tales números. Se utiliza inclusión-exclusión. En este caso hay 8 · 4 · 93 = 23328 tales números. Hay 7 selecciones para el primer dígito y 8 selecciones para los restantes 4 dígitos. En este caso hay 8 · 4 · 9 = 288 tales números. y la selección de posición se puede 4 lograr en = 4 maneras. lo que da 7 · 84 = 28672 tales enteros. 5. d ≥ 20. 20 ≤ d ≤ 30? Ç å 80 ◮Resolución: Se utiliza inclusión-exclusión. b ≥ 0. 6. c ≥ 2. 9. Además hay Å 4 = 4 maneras de seleccionar donde los tres 9’s 3 estarán. Si el primer dígito no es un 9. y se tiene 9 maneras de llenar los espacios restantes. entonces hayÅ8 ã selec4 ciones para este primer dígito. 8. entonces hayÅ 8ãselecciones 4 para este primer dígito. Si el primer dígito es un 9 uno de los cuatro restantes Å será ã un 9. lo que da 6 · 92 = 486 tales números. Esto es la diferencia 90000 − 52488 = 37512. Si el primer dígito no es un 9. Si el primer dígito no es un 9. En total hay 486 + 288 = 774 enteros de cinco dígitos con exactamente tres 9’s en expansión decimal. Si el primer dígito es un 9 entonces los otros cuatro dígitos restantes deberán de ser distintos de 9. Si el primer dígito es un 9 entonces dos de los restantes cuatro Å serán ã 9’s. d ≥ 20 . El número de selecciones es así 9 · 104 = 90000. 10. Hay 9 selecciones posibles para el primer dígito y 10 selecciones posibles para los dígitos restantes. ción de posición se puede lograr en 1 Los otros tres dígitos restantes deberán de ser distintos de 9. En este caso hay 8 · 1 = 8 tales números. Luego el número deseado es 90000 − 85854 = 4146. condicionamos en el primer dígito.Principio de inclusión-exclusión 43 ◮Resolución: 1. entonces hayã8 selecciones para este primer dígito. Los otros restantes dígi3 tos deberán de ser distintos de 9. Sea A el conjunto de soluciones a ≥ 11. c ≥ 2. y hay 92 maneras de llenar los dos espacios restantes. ☞ Obsérvese que 37512 = 29889 + 6804 + 774 + 44 + 1. b ≥ 0. c ≥ 2. c ≥ 2. 20 ≤ d ≤ 30 es así Ç å Ç å Ç å Ç å 80 70 69 59 − − + = 7535. 0 1 2 n−1 n . b ≥ 0. å Ç å Ç å 70 69 59 . card (B) = . d ≥ 31. demuestre que       2n = 249 Problema En contando el número total de subconjuntos de un conjunto de n ele-     n n n n n + + + ··· + + . card (A ∩ B) = y así 3 3 3 Ç card (A ∪ B) = å Ç å Ç å 70 69 59 + − = 74625.44 Chapter 3 y B el conjunto de soluciones Ç Entonces card (A) = a ≥ 1. 3 3 3 El número total de soluciones a + b + c + d = 100 con 1 ≤ a ≤ 10. 3 3 3 3 ◭ Tarea 248 Problema ¿De cuántas maneras diferentes puede un estudiante adivinar un examen completo de verdadero/falso que tenga dieciocho preguntas? mentos. . Así: 10 = 6 + 4. . .Chapter 4 Sumas y recurrencias 4. 251 Ejemplo Hallar el 35avo término de una progresión aritmética cuyo 27avo término es 186 y cuyo 45avo término es 312. . Si el patrón de formación es respetado para los términos subsiguientes entonces deberíamos tener: décimo término = 6 + 9 · 4 = 42. −6. hallar los términos 10mo. 3. ¿Puede hallar el n-ésimo término? ◮Resolución: Observemos que cada término se obtiene sumándole 4 al término anterior. De igual manera deducimos que el enésimo término es = 6 + 4(n − 1). En general. 250 Ejemplo Si la sucesión de términos 6. entonces obtenemos la progresión aritmética a. 18 = 14 + 4. mientras que 1. . 7. 14. 0. . 12. Podemos expresar cada término en términos del primero. en donde la diferencia de términos consecutivos es constante se llama progresión aritmética. Así −18. −12. 18. a + 2d. .1 Progresiones aritméticas Consideremos el siguiente problema. . con término en la posición n igual a a + (n − 1)d. . 100avo. 50avo . 10. a + d. es una progresión aritmética de diferencia común 6. . Pero aún podemos ir más lejos. 45 . . . digamos a y si guardamos una diferencia común de d. a + 3d. ya que términos sucesivos no guardan diferencia constante.. Luego 6 = 6 + 0 · 4 primer t´ermino 10 = 6 + 1 · 4 segundo t´ermino 14 = 6 + 2 · 4 tercer t´ermino 18 = 6 + 3 · 4 cuarto t´ermino 22 = 6 + 4 · 4 quinto t´ermino. cincuentavo término = 6 + 49 · 4 = 202 y cienavo término = 6 + 99 · 4 = 402. 14 = 10 + 4. sigue la misma ley de formación. si comenzamos con un número arbitrario. 6. . no lo es. 97. . 22. ◭ Una progresión como la del ejemplo previo. 22 = 18 + 4. . 1. o sea.. 7 +8 ·1. el tercero −1/2 = 5/2 + 2(−3/2). .. Finalmente. . el diecinueveavo término 5/2 + 18(−3/2) = −49/2. 2 (4. ◮Resolución: La diferencia común es −3/2. 18d = 126 o d = 7. i. 2 puesto que hay n sumandos. + ··· + n + (n − 1) + ··· + 1 + (n + 1) + ··· + (n + 1) An = 1 + An = n 2An = (n + 1) = n(n + 1). ◮Resolución: Vemos que los términos están en progresión aritmética: 7. Operando como en los ejemplos anteriores. 252 Ejemplo Sumar la progresión aritmética 7 + 15 + 23 + · · ·+ 807.e. Luego el primer término es 5/2 = 5/2 + 0(−3/2). Así el primer término es a. etc. Así: 2S = (7 + 807) + (15 + 799) + (23 + 791) + · · ·+ (807 + 7) = 101 · 814 = 82214. −1/2. 7 +8 ·100. . Así. llamémosle a y a la diferencia común llamémosla d. S = 41107. Colegimos luego que S = −209. el segundo 1 = 5/2 + 1(−3/2). Sumando estas dos cantidades. . Finalmente el 35avo término. 2S = (5/2 − 49/2) + (1 − 46/2) + (−1/2 − 43/2) + · · ·+ (−49/2 + 5/2) = −44/2 − 44/2 − 44/2 − · · ·− 44/2 = 19(−44/2) = −418. el segundo a + d. Observe que si S = 7 + 15 + 23 + · · ·+ 807. ◭ La siguiente fórmula ocurre con regularidad y el lector hará bien en aprender a deducirla. Si An = 1 + 2 + 3 + · · ·+ n entonces también An = n + (n − 1) + · · ·+ 1. 7 +8 ·2. . el cuarto a + 2d + d = a + 3d. Pero la data del problema estipula que a + 26d = 186 y a + 44d = 312. la suma que queremos es S = 5/2 + 1 − 1/2 − · · ·− 49/2.46 Chapter 4 ◮Resolución: Tratemos de expresar la data que sabemos en términos del primer término y la diferencia común.1) . a + 34d está dado por a + 34d = 4 + 34(7) = 242. hasta 19 términos. De aquí (a + 44d) − (a + 26d) = 312 − 186 = 126. Pero entonces 186 = a + 26d = a + 26 · 7 = a + 182. De esto colegimos 1 + 2 + ···+ n = n(n + 1) . . ◭ Veremos ahora como sumar progresiones aritméticas. de donde a = 4. el tercero a + d + d = a + 2d. . . Así el 27avo término debe ser a + 26d y el 45avo término debe ser a + 44d. Como no se nos da el primer término. ◭ 253 Ejemplo Sumar 5/2. . . entonces también S = 807 + 799 + 791 + · · ·+ 7. Halle la suma del sexto. ◭ Tarea 256 Problema Hallar los términos 14 y 110 de la progresión 43. manera: (1) (3.. es un entero. ..2. cuyo primer término es −9 y cuyo último término es 14. 25.. 27. T2 T3 T4 Tn · · ··· . (13.. ... Conjeture y demuestre una fórmula para la suma del enésimo paréntesis.. .. . . hasta treinta términos. 19) (21. 11) n2 (n2 + 1) .. 29) ... ◭ 255 Ejemplo Hallar la suma 12 − 22 + 32 − 42 + 52 − 62 + · · · + 992 − 1002. . 128.. .. 23.Tarea 47 254 Ejemplo Hallar el valor de la suma 1 − 2 + 3 − 4 + 5 − 6 + · · ·+ 99 − 100. 261 Problema Sumar 64. −37. x + y. 273 Problema (AHSME 1991) Sea Tn = 1 + 2 + 3 + · · · + n y Pn = 263 Problema Sumar x − y. . a.... y octavo grupos. ◮Resolución: Como x2 − (x + 1)2 = −2x − 1.... . 1/2 − 2x.. 96. ...6. . es una progresión aritmética con diferencia común 1 y con a1 +a2 +a3 +· · · +a98 = 137. 1/2 − x. 270 Problema (AHSME 1994) Sumar la serie 2 1 20 + 20 + 20 + · · · + 40. 264 Problema Hallar el término 15 de una progresión aritmética cuyo término 14 es 9 y cuyo término 16 es −90. 1.. . tenemos (12 − 22 ) + (32 − 42 ) + (52 − 62 ) + · · · + (992 − 1002) = −(3 + 7 + 11 + · · ·+ 199) = −5050. 41. 1. a − d. 274 Problema Considere la tabla: 1=1 2+3+4 = 1+8 265 Problema Hallar el término 22 de una progresión aritmética cuyo término 11 es −1 y cuyo término 16 es 55.8. 42. x + 3y... .. x + 3y.. . . 15. .. séptimo... 5) 268 Problema Demostrar que (7. .. Luego agrupando la suma en cincuenta pares que suman −1. 5 + 6 + 7 + 8 + 9 = 8 + 27 10 + 11 + 12 + 13 + 14 + 15 + 16 = 27 + 64 266 Problema Hallar el número de términos y la diferencia común de una serie aritmética cuya suma es 30. Descubra el patrón de formación. 272 Problema Demostrar que 2 3 1995 1 + + + ··· + 1996 1996 1996 1996 260 Problema Hallar los términos 10 y 51 de la progresión x − y. x + y. . 5 5 257 Problema Hallar los términos 20 y 310 de la progresión −43. 262 Problema Sumar 1/2. −40. hasta cuarenta términos. 259 Problema Hallar los términos 13 y 150 de la progresión a − 2d. . 17. tenemos 1 − 2 + 3 − 4 + 5 − 6 + · · ·+ 99 − 100 = −50.. T2 − 1 T3 − 1 T4 − 1 Tn − 1 Hallar P1991 . 275 Problema Los enteros naturales impares se agrupan en paréntesis de la siguiente 267 Problema Sumar 1 + 2 + 3 + · · · + 100. . . . . a3 . a2 .. hasta diez términos.. . . . . 271 Problema (AIME 1984) Hallar el valor de a2 + a4 + a6 + · · · + a98 si a1 . 258 Problema Hallar los términos 12 y 1090 de la progresión 0. ◮Resolución: Observe que −1 = 1 − 2 = 3 − 4 = · · · = 99 − 100. 9. conjeture una ley general y demuéstrela. . 1 + 2 + 3 + · · · + (n − 1) + n = 2 2 2 269 Problema Demostrar que 1 + 3 + 5 + · · · + 2n − 1 = n2 .. Luego a(2)3 = 24 y así a = 3. −1.48 Chapter 4 4. . llamémosle r. Tampoco conocemos la razón común. . el cincuentavo término está dado por 2(3)49 . el cuarto por (−2) . el tercero por ar2 . 2. . 54. ◭ 278 Ejemplo Hallar la suma de la serie geométrica 1 + 2 + 4 + · · ·+ 1024. el cuarto por ar3 y siguiendo el patrón. etc. Así S = 2S − S = (2 + 4 + 8 · · · + 2048) − (1 + 2 + 4 + · · ·+ 1024) = 2048 − 1 = 2047. 162. ar3 24 de donde r = 2. el tercero por 2(3)2 . ◮Resolución: Pongamos S = 1 + 2 + 4 + · · ·+ 1024. 276 Ejemplo Hallar el décimo término de la progresión geométrica 1/2. llamémosle a. el segundo por ar. etc. El patrón indica pues que el décimo término 2 2 2 1 está dado por (−2)9 . el segundo por 2(3)1 . 6. 3 32 33 3 ◮Resolución: Tenemos 1 1 1 1 1 x = 2 + 3 + · · · + 99 + 100 . 1 ◮Resolución: Vemos que la razón común es −2. . De aquí tenemos r3 = ar6 192 = = 8. Así pues. Notamos que cada término es el triple del anterior. . 3 3 3 3 3 . el quinto por 2(3)4 . ar2 . y el enésimo término está dado por arn−1 . 277 Ejemplo Hallar el segundo término de una progresión geométrica con cuarto término 24 y séptimo término 192. . La data es que 24 = ar3 y 192 = ar6 . . el séptimo término es ar6 . ◭ 279 Ejemplo Hallar la suma de la serie geométrica x= 1 1 1 1 + + + · · · + 99 . Luego el primer término está dado por a. entonces va de la siguiente manera: a. Vemos que el primer término está dado por 2(3)0 . Entonces 2S = 2 + 4 + 8 + · · ·+ 1024 + 2048.. el 100-avo es 2(3)99 y el enésimo término está dado por 2(3)n−1 . Luego el primer término está dado por (−2)0 . ar3 . .. ar. ◭ 2 En general si una progresión geométrica tiene primer término a y razón común r. el cuarto por 2(3)3 . Decimos que 3 es la razón común de esta progresión geométrica. 18. Luego el segundo término está dado por ar = 6. el segundo está 2 1 1 1 1 2 3 dado por (−2) . ◮Resolución: No conocemos el primer término. el tercero por (−2) .2 Progresiones geométricas Consideremos ahora la progresión 2. Una progresión de este tipo se llama progresión geométrica. . 3a = −1 − 4 − 42 − 43 − · · · − 49 + 10 · 410. Calculamos ahora los siguientes valores: S1 = 2 − 1/20 = 1 S2 = 2 − 1/2 = 1.875 S5 = 2 − 1/24 = 1.9375 S6 = 2 − 1/25 = 1. a= 10 · 410 410 − 1 − .875 S4 = 2 − 1/23 = 1. ◮Resolución: Tenemos 1 Sn − Sn = (1 + 1/2 + 1/4 + · · ·+ 1/2n) − (1/2 + 1/4 + · · ·+ 1/2n + 1/2n+1) = 1 − 1/2n. − 2 2 · 399 280 Ejemplo Sumar a = 1 + 2 · 4 + 3 · 42 + · · · + 10 · 49.Progresiones geométricas 49 Luego 2 x 3 = = = 1 x− x 3 1 1 1 1 ( + 2 + 3 + · · · + 99 ) 3 3 3 3 1 1 1 1 −( 2 + 3 + · · · + 99 + 100 ) 3 3 3 3 1 1 . 2 Así Sn = 2 − 1/2n.5 S3 = 2 − 1/22 = 1. ◮Resolución: Tenemos Luego 4a − a nos da 4a = 4 + 2 · 42 + 3 · 43 + · · · + 9 · 49 + 10 · 410. Interpretar el resultado cuando n crece indefinidamente. Al sumar esta última serie geométrica.96875 S10 = 2 − 1/29 = 1.998046875 . 3 9 ◭ 281 Ejemplo Hallar la suma Sn = 1 + 1/2 + 1/4 + · · ·+ 1/2n. − 3 3100 De esto colegimos x= ◭ 1 1 . ◭ Supongamos que los procedimientos que utilizamos para sumar series geométricas finitas son válidos para las infinitas. . . ◮Resolución: Tenemos x = 1+ 40 40 40 30 + 4 + 6 + 8 + ··· . pues cada término es mayor que 1 y esto incrementa el valor de la serie cada vez más. + 10 102 103 104 2 2 2 1 2 + + + ··· = = .. Entonces podremos decir que S∞ = 1 + 1/2 + 1/22 + 1/23 + · · · hasta infinitos suma a 2. . ◮Resolución: Observemos que x= Pero S= 2 2 2 1 + + + ··· .. entonces decimos que la serie geométrica diverge. 282 Ejemplo Hallar la fracción que representa x = 0.1222222222 . esto sucederá cuando |r| < 1. . Si las sumas parciales no se acercan a un valor finito definitivo cuando n aumenta. 10 45 90 ◭ 283 Ejemplo Hallar la fracción que representa x = 1. S = 2 + 22 + 23 + · · · es obviamente infinitamente grande. . Del ejemplo anterior se vislumbra que esto sucede cuando |r| ≥ 1. i. nos acercamos cada vez más a 2. La teoría de series geométricas convergentes puede utilizarse para convertir un decimal periódico a una fracción. lo cual es un disparate mayúsculo.50 Chapter 4 Vemos que a medida que tomamos más términos de la serie. . Por ejemplo. la suma de números positivos da un resultado negativo. Como vimos en los ejemplos anteriores. ¿Qué sucede entonces? Entra ahora pues el concepto de convergencia. 102 103 104 90 45 Así 0.e.122222222222222 .304040404040 . La manipulación formal que utilizamos supra debe manejarse con sumo cuidado. ya que 1 S∞ − S∞ = (1 + 1/2 + 1/22 + 1/23 + · · · ) − (1/2 + 1/22 + 1/23 + 1/24 + · · · )) = 1 2 puesto que los términos luego del primero encuentran su opuesto en la serie de la derecha. Sin embargo. al operar formalmente como lo hicimos anteriormente obtenemos S = 2S − S = (22 + 23 + 24 + · · · ) − (2 + 22 + 23 + · · · ) = −2. = 1 1 11 + = . Diremos que la serie geométrica a + ar + ar2 + · · · converge hacia un valor finito S si cada suma parcial Sn = a + ar + ar2 + · · · + arn−1 se acerca más y más a S a medida que n aumenta. 2 10 10 10 10 . ¿Qué valor tendrá 9/10 + 9/100 + 9/1000 + · · · es 125. 0) y viaja sobre el plano una pulgada hacia el este. 2k k=1 3 + 5 · 4 + 7 · 42 + 9 · 43 + · · · + 99 · 448 . x+2y. 291 Problema Hallar todos los números complejos x. hallar el segundo término. 1/16 de pulgada hacia el este. . 22 y enésimo de la progresión geométrica 3/2. −3/8. 4. ¿Qué valor tendrá 1/2 + 1/4 + 1/8 + · · · hasta infinito? una progresión aritmética a la vez que (y + 1)2 . 1. 2x+y formen 289 Problema 1. etc. 1 − 1 + 1 − 1 + ··· 3. . 1/4 de pulgada hacia el oeste. xy + 5. . 5. Vemos que X = 1− 1 1 4 1 + − + ··· = 4 16 64 5 y 1 1 1 1 2 − + − + ··· = . 1/2 − 1/4 + 1/8 − · · · + 1/512 − 1/1024. . 1/2 pulgada hacia el norte. 2 8 32 128 5 Luego la mosca termina en el punto (4/5. hasta 10 términos. ¿Qué valor tendrá hasta 20 términos. . 1/8 de pulgada hacia el sur. . 3/32. Si la mosca hiciese esto ad infinitum. 2. la serie geométrica infinita S= 40 40 40 + + + ··· 104 106 108 suma a S= Así x = 1+ 2 40 = . Halle las tangentes de los ángulos agudos.Tarea 51 Ahora bien. y tales que x. hasta infinito? Discuta por qué 287 Problema Hallar la suma de las siguientes series geométricas: 1 = 0. ¿Qué valor tendrá 1/10 + 1/10 + 1/100 + · · · hasta infinito? 3.Y ) el punto donde la mosca terminaría. . 1/10 + 1/100 + 1/1000 + . 292 Problema Los lados de un triángulo rectángulo forman una progresión geométrica. ¿donde terminaría? ◮Resolución: Sea (X. 2/5). 286 Problema Si el término 15 de una progresión geométrica es −100 y el término 20 2.9999999999999999999 . 6 + 18 + 54 + . . hasta infinito? 290 Problema Hallar X∞ 288 Problema Hallar la suma de la serie k . 102 495 990 ◭ 284 Ejemplo Una mosca está en el origen (0. . ◭ Y= Tarea 285 Problema Hallar los términos 13. (x + 1)2 formen una progresión geométrica. 9900 495 2 30 1291 + = . . ¿Qué valor tendrá 1 + 1 + 1 + ··· hasta infinito? Discuta. b. ◮Resolución: Poniendo todo en una base común. hállelo. c.52 Chapter 4 293 Problema Sean a. 295 Ejemplo Simplificar Å 1+ ã Å ã Å ã Å ã 1 1 1 1 · 1+ · 1+ ··· 1 + . Si tal sucesión {vk } existe. . . Halle A y B. una progresión geométrica creciente. d las raíces de la ecuación x2 − 12x + B = 0. . a1 a2 an hallar su producto P = a1 a2 · · · an en términos de s y T. loga 2 de donde obtenemos el resultado. digamos a > 0. T = 1 1 1 + + ··· + . ◭ 296 Ejemplo Dado que (log2 3) · (log3 4) · (log4 5) · · · (log511 512) es un entero. Si s = a1 + a2 + · · · + an . diremos entonces que la serie a1 + a2 + · · · + an es una serie teléscopica. 2 3 4 99 lo que simplifica a 100/2 = 50. 294 Problema Los números a1 .3 Cancelación telescópica A veces podemos sumar una serie a1 + a2 + a3 + · · · + an si podemos encontrar una sucesión {vk } con ak = vk − vk−1 . en este orden. . ◭ 297 Ejemplo Simplificar ä ä Ä 99 ä Ä 3 Ä ä Ä 2 (2 + 1) · 22 + 1 · 22 + 1 · 22 + 1 · · · 22 + 1 . loga 2 loga 3 loga 4 loga 511 loga 2 Pero loga 512 = log2 512 = log2 29 = 9. a = 1: (log2 3) · (log3 4) · (log4 5) · · · (log511 512) = loga 3 loga 4 loga 5 loga 512 loga 512 · · ··· = . b las raíces de la ecuación x2 − 3x + A = 0 y sean c. Se sabe que a. Entonces a1 + a2 + a3 + · · · + an = v1 − v0 + v2 − v1 + · · · + vn−1 − vn−2 + vn − vn−1 = vn − v0 . .an forman una progresión geométrica. 4. a2 . d forman. 2 3 4 99 ◮Resolución: Sumando cada fracción tenemos: 3 4 5 100 · · ··· . ◭ 299 Ejemplo Hallar el valor exacto del producto P = cos ◮Resolución: obtenemos π 2π 4π · cos · cos .Cancelación telescópica 53 ◮Resolución: Utilizando la identidad x2 − y2 = (x − y)(x + y) y llamando P al producto: (2 − 1)P = = = = . 99 99 (22 − 1)(22 + 1) = 22 = de donde P = 22 100 100 − 1.. − 1. Como tan k◦ = 1/ tan(90 − k)◦. Finalmente.. ◭ 298 Ejemplo Simplificar S = log tan 1◦ + log tan 2◦ + logtan 3◦ + · · · + log tan 89◦ . . como tan 45◦ = 1. Luego. . 7 7 7 Multiplicando a uno y otro lado por sen π sen P = 7 = = = (sen π y haciendo uso de la identidad sen 2x = 2 sen x cos x 7 2π 4π π π cos ) · cos · cos 7 7 7 7 2π 2π 4π 1 (sen cos ) · cos 2 7 7 7 1 4π 4π (sen cos ) 4 7 7 8π 1 sen . colegimos S = log 1 + log1 + · · · + log 1 = 0. ◮Resolución: Observe que (90 − k)◦ + k◦ = 90◦ . 8 7 . tenemos S = log 1 + log1 + · · · + log 1 + logtan 45◦ . sumando el término k-ésimo con el 90 − k-ésimo obtenemos que la suma dada es S = log(tan 1◦ )(tan 89◦ ) + log(tan 2◦ )(tan 88◦ ) + log(tan 3◦ )(tan 87◦ ) + · · · + log(tan 44◦ )(tan 46◦ ) + logtan 45◦ . ä ä Ä 99 ä Ä 3 Ä ä Ä 2 (2 − 1) (2 + 1) · 22 + 1 · 22 + 1 · 22 + 1 · · · 22 + 1 Ä ä Ä ä Ä 2 ä Ä 3 ä Ä 99 ä 22 − 1 · 22 + 1 · 22 + 1 · 22 + 1 · · · 22 + 1 Ä 2 ä Ä 2 ä Ä 3 ä Ä 99 ä 22 − 1 · 22 + 1 · 22 + 1 · · · 22 + 1 Ä 3 ä Ä 3 ä Ä 4 ä Ä 99 ä 22 − 1 · 22 + 1 · 22 + 1 · · · 22 + 1 . deducimos que 7 7 1 P=− . . . podemos multiplicar una y otra columna para obtener 9999 2 4 6 10000 1 3 5 · · ··· < · · ··· . 2 4 6 10000 3 5 7 10001 o A < B. 8 ◭ 300 Ejemplo Demostrar que 9999 1 1 3 5 · · ··· < . 2! = 1 · 2 = 2. .54 Chapter 4 Como sen 8π π = − sen . Luego A2 = A · A < A · B. A2 < A · B = 1/10001. Pero entrelazando los factores de A y B. x x+1 Por tanto 1/2 < 2/3 3/4 < 4/5 5/6 < 6/7 . Por ejemplo 1! = 1. El símbolo n! (léase n factorial) significa n! = 1 · 2 · 3 · · ·n. ◭ A·B = Para nuestro siguiente ejemplo necesitaremos la siguiente definición. . 301 Ejemplo Sumar 1 · 1! + 2 · 2! + 3 · 3! + · · ·+ 99 · 99!. . 1 2 3 4 5 6 7 9999 10000 1 · · · · · · ··· · = . Observe que (k + 1)! = (k + 1)k!. 3! = 1 · 2 · 3 = 6... . 9999/10000 < 10000/10001 Como todas estas desigualdades son de números positivos. 3 5 7 10001 y Es claro que x2 − 1 < x2 para todo número real x.. 4! = 1 · 2 · 3 · 4 = 24. Tendremos por convención 0! = 1. De esto deducimos x−1 x < . De aquí A < 1/ 10001 < 1/100. 2 4 6 10000 100 ◮Resolución: Pongamos A= 1 3 5 9999 · · ··· 2 4 6 10000 B= 10000 2 4 6 · · ··· . 2 3 4 5 6 7 8 10000 10001 10001 √ y por consiguiente. . ◭ 302 Ejemplo Hallar una forma cerrada para la suma An = 1 + 2 + · · · + n. ◮Resolución: Observemos que k2 − (k − 1)2 = 2k − 1. ◮Resolución: Observemos que k3 − (k − 1)3 = 3k2 − 3k + 1. . . 1 + 2 + 3 + · · ·+ n = ·n2 /2 + n/2 = n(n + 1) . . 1 · 1! + 2 · 2! + 3 · 3! + · · ·+ 99 · 99! = 100! − 1! = 100! − 1. Así 1 · 1! = 2! − 1! 2 · 2! = 3! − 2! 3 · 3! = 4! − 3! . . 98 · 98 = 99! − 98! . n2 − 02 = 2(1 + 2 + 3 + · · ·+ n) − n. 99 · 99! = 100! − 99! Sumando una y otra columna. n2 − (n − 1)2 . . Luego 12 − 02 = 2·1−1 22 − 12 = 2·2−1 32 − 22 = 2·3−1 . Resolviendo para la suma. = 2·n−1 Sumando una y otra columna.Cancelación telescópica 55 ◮Resolución: De (k + 1)! = (k + 1)k! = k · k! + k! deducimos (k + 1)! − k! = k · k!. 2 ◭ 303 Ejemplo Hallar la suma 12 + 22 + 32 + · · · + n2 .. .. ..... . .. 3 2 3 Simplificando obtenemos 12 + 22 + 32 + · · · + n2 = n(n + 1)(2n + 1) . 1·2 2·3 3·4 99 · 100 100 100 305 Ejemplo Sumar 1 1 1 1 + + + ···+ . . Pero por el ejercicio anterior se tiene n3 − 03 = 3(12 + 22 + 32 + · · · + n2 ) − 3 · n(n + 1) + n. 12 + 22 + 32 + · · · + n2 = n3 1 n + · n(n + 1) − .. 2 Resolviendo para la suma.. 1 · 4 4 · 7 7 · 10 31 · 34 . n3 − 03 = 3(12 + 22 + 32 + · · · + n2 ) − 3(1 + 2 + 3 + · · ·+ n) + n. 1 1 − 99 100 1 1 1 1 99 1 + + + ··· + = 1− = . .56 Chapter 4 Luego 13 − 03 = 3 · 12 − 3 · 1 + 1 23 − 13 = 3 · 22 − 3 · 2 + 1 33 − 23 = . Sumando una y otra columna obtenemos ◭ 1 99 · 100 = = = . .. k(k + 1) k k + 1 Luego 1 1·2 1 2·3 1 3·4 .. 1·2 2·3 3·4 99 · 100 ◮Resolución: Observe que 1 1 1 = − . = 1 1 − 1 2 1 1 − 2 3 1 1 − 3 4 . 3 · n2 − 3 · n + 1 Sumando una y otra columna. 6 ◭ 304 Ejemplo Sumar la serie 1 1 1 1 + + + ···+ . 3 · 32 − 3 · 3 + 1 n3 − (n − 1)3 = .. . . . (3n + 1) · (3n + 4) · (3n + 7) 6 (3n + 1)(3n + 4) 6 (3n + 4)(3n + 7) Luego 1 1·4·7 1 4 · 7 · 10 1 7 · 10 · 13 .. . Sumando una y otra columna obtenemos 1 34 · 37 = = = = .. 1 25 · 28 · 31 Sumando una y otra columna obtenemos ◭ 1 1 − 6·1·4 6·4·7 1 1 = − 6 · 4 · 7 6 · 7 · 10 1 1 = − 6 · 7 · 10 6 · 10 · 13 . .. . .. 3 3 . . = 1 1 − 3 12 1 1 − 12 21 1 1 − 21 30 1 1 − 30 39 .. ◮Resolución: Observemos que 1 1 k(k + 1) = (k)(k + 1)(k + 2) − (k − 1)(k)(k + 1). . . = = 1 1 − 6 · 25 · 28 6 · 28 · 31 1 1 1 1 1 1 9 + + + ···+ = − = . 1 · 4 4 · 7 7 · 10 31 · 34 3 111 37 ◭ 306 Ejemplo Sumar 1 1 1 1 + + + ···+ . 1 · 4 · 7 4 · 7 · 10 7 · 10 · 13 25 · 28 · 31 ◮Resolución: Observe que 1 1 1 1 1 = · − · . 1 · 4 · 7 4 · 7 · 10 7 · 10 · 13 25 · 28 · 31 6 · 1 · 4 6 · 28 · 31 217 307 Ejemplo Sumar 1 · 2 + 2 · 3 + 3 · 4 + · · ·+ 99 · 100. 1 1 − 102 111 1 1 1 1 1 12 1 + + + ···+ = − = . (3n + 1) · (3n + 4) 3 3n + 1 3 3n + 4 Luego 1 1·4 1 4·7 1 7 · 10 1 10 · 13 ..Cancelación telescópica 57 ◮Resolución: Observe que 1 1 1 1 1 = · − · . un candidato hizo una trayectoria que presumimos yace en el plano. . .an números arbitrarios. 1−x n=1 consiste de 1’s separados por bloques de 2’s. 1·2 = 2·3 3·4 . 2. . con n bloques de 2’s en el n-ésimo bloque. k=2 313 Problema Hallar el valor exacto de 320 Problema Demostrar que csc 2 + csc 4 + csc8 + · · · + csc2n = cot 1 − cot 2n .58 Chapter 4 Por lo tanto 1 1 ·1·2·3− ·0·1·2 3 3 1 1 ·2·3·4− ·1·2·3 = 3 3 1 1 ·3·4·5− ·2·3·4 = 3 3 . tan π π π π π + 2 tan 99 + 22 tan 98 + · · · + 298 tan 2 = cot 100 . etc. 1. 1 · 2 + 2 · 3 + 3 · 4 + · · ·+ 99 · 100 = 1 1 · 99 · 100 · 101 − · 0 · 1 · 2 = 333300. 319 Problema Sean a1 . . en el tercero hacia el oeste. en el quinto hacia el este. 2. En el primer día él viajó hacia el este. 309 Problema Simplificar (1 + i)2004 . 2. hallar a y b. 2. 2. 2. . 322 Problema Demostrar que 315 Problema (AIME 1985) Calcular el producto x1 x2 · · · x8 si x1 = 97 y xn = n/xn−1 . 2. 2. = (1 + a1 )(1 + a2 )(1 + a3 ) · · · (1 + an ) − 1. Hallar la suma de los primeros 1234 términos de esta sucesión. Demostrar que a1 + a2 (1 + a1 ) + a3 (1 + a1 )(1 + a2 ) + a4 (1 + a1 )(1 + a2 )(1 + a3 ) 312 Problema Simplificar Ä X1023 log2 1 1+ k ä + · · · + an−1 (1 + a1 )(1 + a2 )(1 + a3 ) · · · (1 + an−2 ) . ¿a cuantas millas estaba él de su punto de partida en el 40avo día? Dn = 1 − 2 + 3 − 4 + · · · + (−1)n−1 n. 1 1 1 1 + + + ··· + . n > 1. . 2. 2. Si el candidato viajó n2 /2 millas en el n-ésimo día. . .. 2. 2. 318 Problema Simplificar con a y b números reales. Ä n(n + 1) ä2 2 . (1 − i)2000 317 Problema Demostrar que 13 + 23 + 33 + · · · + n3 = 310 Problema Si a + ib = 1 + 2i + 3i2 + 4i3 + · · · + 1995i1994 + 1996i1995 .. . . 3 3 ◭ Tarea 308 Problema Hallar una fórmula para 316 Problema (AIME 1993) Durante una campaña política reciente. 2.. él viajó hacia el norte. x2 n 1 − x2n+1 = x . 1. 23 + 1 33 + 1 43 + 1 1003 + 1 311 Problema Simplificar Ä 1− 1 22 äÄ · 1− 1 32 äÄ · 1− 1 42 ä Ä ··· 1 − 1 992 ä . en el segundo. 1. 23 − 1 33 − 1 43 − 1 1003 − 1 · · ··· . Demostrar que 314 Problema (AHSME 1996) La sucesión X∞ 1. 2. 2. 99 · 100 = 1 1 · 99 · 100 · 101 − · 98 · 99 · 100 3 3 Sumando una y otra columna. 1. 2100 2 2 2 22 . . log2 1996! log3 1996! log4 1996! log1996 1996! 321 Problema Sea 0 < x < 1. en el cuarto hacia el sur. a2 . 1. 2. 1998 < 1 + √ + √ + √ + · · · + √ 2 3 4 1000000 . .. 4 1 + n + n2 ä1/3 1 1 1 1 < 1999. n ≥ 1. .. x0 x1 · · · xn = 7 · 2nx1 x2 · · · xn−1 . . .. 2 n +n+1 k4 + k2 + 1 arctan n=0 k=1 326 Problema Demostrar que 324 Problema Evalúe el radical Ä 1 · 2 · 4 + 2 · 4 · 8 + 3 · 6 · 12 + · · · 1 · 3 · 9 + 2 · 6 · 18 + 3 · 9 · 27 + · · · 1 π = . 4..Recursiones y ecuaciones funcionales 59 323 Problema Demostrar que Xn 325 Problema Demostrar que X∞ n2 + n 1 k = · 2 . xn = xn−1 + n . En general aplicaremos las técnicas de la sección anterior.. 327 Ejemplo Sea x0 = 7 y xn = 2xn−1 . . Multiplicando una y otra columna. .4 Recursiones y ecuaciones funcionales Veremos ahora métodos para hallar formas cerradas de ciertas recursiones. ◮Resolución: Tenemos x0 = 7 x1 = x0 + 1 x2 = x1 + 2 x3 = x2 + 3 . xn = 2xn−1 . xn = 7 · 2 n . Cancelando factores comunes. . ◮Resolución: Tenemos x0 = 7 x1 = 2x0 x2 = 2x1 x3 = 2x2 . . . n ≥ 1. Hallar una fórmula cerrada para xn . ◭ 328 Ejemplo Sea x0 = 7 y xn = xn−1 + n. Hallar una fórmula cerrada para xn .. n ≥ 1.. ◭ 330 Ejemplo Una sucesión satisface u0 = 3. .. . 2 ◭ 329 Ejemplo Sea x0 = 7 y xn = 2xn−1 + 1. así que nos valdremos del siguiente artificio. . . Cancelando y simplificando. xn−1 = 2xn−2 + 1 xn = 2xn−1 + 1 Aquí no nos funcionan los métodos anteriores.. . u2n+1 = un . Aliter: Pongamos un = xn + 1 = 2xn−1 + 2 = 2(xn−1 + 1) = 2un−1. . obteniendo 2 n x0 = 2n · 7 2n−1x1 = 2n x0 + 2n−1 2n−2x2 = 2n−1 x1 + 2n−2 2n−3x3 = 2n−2 x2 + 2n−3 ... Luego la recursión un = 2un−1 la resolvemos como en nuestro primer ejemplo. 22 xn−2 = 23 xn−3 + 22 2xn−1 = 22 xn−2 + 2 xn = 2xn−1 + 1 Sumando una y otra columna y cancelando. ◮Resolución: Tenemos x0 = 7 x1 = 2x0 + 1 x2 = 2x1 + 1 x3 = 2x2 + 1 .60 Chapter 4 Sumando una y otra columna. obteniendo de esta forma un = 2n u0 = 2n (x0 + 1) = 2n · 8 = 2n+3 . Halle una forma cerrada para ella. xn = 7 + n(n + 1) . . Finalmente xn = un − 1 = 2n+3 − 1. xn = 7 · 2n + (1 + 2 + 22 + · · · + 2n−1) = 7 · 2n + 2n − 1 = 2n+3 − 1. Multipliquemos a la k-ésima fila por 2n−k . x0 + x1 + x2 + · · · + xn = 7 + x0 + x2 + · · · + xn−1 + (1 + 2 + 3 + · · ·+ n). . . Hallar una fórmula cerrada para xn . .. n ≥ 1. . Entonces v j+1 = a j+1 + 1 = (a j + 1)2 = v2j . Entonces vn = log un = log un−1 = n tenemos vn = v0 /2n. ◮Resolución: Sea un el número de maneras en que puede el duende subir una escalera de n escalones. ◮Resolución: Tenemos a j+1 + 1 = a2j + 2a j + 1 = (a j + 1)2 . j ≥ 0. Luego un = 31/2 .Recursiones y ecuaciones funcionales 61 1/2 ◮Resolución: Pongamos vn = log un . 2 2 331 Ejemplo Halle una forma cerrada para a0 = 5. Como 1996 = 1995 + 1 deja residuo 1 al ser dividido por 3. Es claro que u1 = 1. hallar f1996 (−1/3). Un duende puede subir la escalera de escalón en escalón o saltándose un escalón. = 1 x 1 − 1−x f2 (x) = f ( f1 (x)) = 1 =x 1 − x−1 x y 1 = f0 (x). o sea. o sea j j j a j+1 = v j+1 − 1 = v02 − 1 = (a0 + 1)2 − 1 = 62 − 1. f1 (x) = f4 (x) = f7 (x) = · · · y f2 (x) = f5 (x) = f8 (x) = · · · . . Como vn = vn−1 /2. De aquí v j+1 = v20 . Luego un = un−1 + un−2. Hallar una recursión para el número de maneras en que el duende puede subir la escalera. ◭ 334 Ejemplo Hallar todas las funciones que satisfacen f (x + y) + f (x − y) = 4x2 + 4y2 . 1−x ◮Resolución: Observe que f1 (x) = f ( f0 (x)) = x−1 1 . ◭ 333 Ejemplo Si f (x) = f0 (x) = 1 y fn (x) = f ( fn−1 (x)) para n > 0. Esto implica que f3 (x) = f0 (x) = f3 (x) = f6 (x) = · · · . f1996 (−1/3) = f1 (−1/3) = 4. a j+1 = a2j + 2a j . u2 = 2. ◭ 332 Ejemplo Una escalera tiene n escalones. El duende puede llegar al último escalón o bien desde el penúltimo o bien desde el antepenúltimo escalón. j Pongamos v j+1 = a j+1 + 1. 1−x Luego esta recursión es cíclica de orden 3. ◭ vn−1 1 log un−1 = . log un = (log u0 )/2n . 338 Problema Sea f (x) = f0 (x) = p 1 − x−2 . ◭ Tarea 335 Problema Sea x1 = 1. f (x) > 0∀ x > 0. f (m) > f (n) si m > n. hallar todos los enteros n = 1 tales que f (n) = n. xn = xn−1 + n . 345 Problema (AHSME 1979) La función f satisface f (x + y) = f (x) + f (y) − xy + 1 para todos los números reales x. 1 = 1. xn+1 = x2n − xn + 1. n > 0. fn (x) = f ( fn−1 (x)). Demostrar que f (n) = n. 3 3 344 Problema Hallar todas las funciones f que satisfagan f99 (4). ¿Para cuántos valores de x se cumple f (x) = f (−x)? . Demostrar que X∞ 342 Problema Una sucesión a1 . hallar el residuo cuando f (94) se divide por 1000. 340 Problema Demostrar que existe una función f única del conjunto R+ de los reales positivos a R+ tal que 346 Problema (AHSME 1981) La función f no está definida para x = 0.62 Chapter 4 ◮Resolución: Tomando y = 0. . a2 . Si f (1) = 1. 341 Problema Si u0 = 1/3 y un+1 = 2u2n − 1. 2 Demuéstrese que 337 Problema Hallar una forma cerrada para f (3x) = 3k−1 2 x0 = −1. . Hallar Ä ä 1 2 f (x) + f (x + ) + f (x + ) . 339 Problema Sea f una función con las siguientes propiedades: 1) 2) 3) 4) 5) f (n) está definida para todo entero positivo n. hallar una fórmula para un . f (2x) = 2k−1 Si f (19) = 94. f (2) = 2. n > 0. pero si x = 0 f ( f (x)) = 6x − f (x). f (mn) = f (m) f (n) para todo m y n. satisface a1 = 2 y am+n = 4mn am an ∀m. n > 0. xn Hallar el valor mínimo de n para el cual an tiene al menos 3000 dígitos. f (x)2 · f Ä1−xä 1+x = 64x. n. f (x) + f (x − 1) = x2 . f (n) es un entero. Ä ä 1 f (x) + f (x + ) . n=1 336 Problema (AIME 1994) La función f tiene la propiedad que para todo número real 343 Problema Sea k un entero no-negativo fijo y supóngase que x. y. satisface f (x) + 2 f Ä1ä x = 3x. Veamos que f (x) = 2x2 satisface la ecuación funcional: f (x + y) + f (x − y) = 2(x + y)2 + 2(x − y)2 = 2x2 + 4xy + 2y2 + 2x2 − 4xy + 2y2 = 4x2 + 4y2 . . obtenemos f (x) + f (x) = 4x2 o f (x) = 2x2 . escribimos x2 − 6x + 3 = x2 − 6x + 9 − 6 = √ (x − 3)2 − ( 6)2 = √ √ (x − 3 + 6)(x − 3 − 6). el método más eficiente es quizás la compleción del cuadrado.Chapter 5 Polinomios y ecuaciones 5. ◭ 3 348 Ejemplo Resolver 9x − 3x+1 − 4 = 0. 2 2 2 2 3 1 Luego. Para resolver ecuaciones cuadráticas.1 Ecuaciones A continuación veremos como resolver algunas ecuaciones. para resolver 2x2 + 6x + 5 = 0 escribimos 2x2 + 6x + 5 = = = 2x2 + 6x + 9 1 + 2 2 √ 3 1 ( 2x + √ )2 − (i √ )2 2 2 √ 3 3 1 √ 1 ( 2x + √ − i √ )( 2x + √ + i √ ). La mayoría de ellas son ecuaciones cuadráticas disfrazadas. Ésta es preferible a la fórmula cuadrática ya que crea “malicia” para identificar patrones. 63 . Por ejemplo. √ De aquí x = 3 ± 6. para resolver x2 − 6x + 3 = 0. 2 2 347 Ejemplo Resolver 9 + x−4 = 10x−2. 1 Luego x = ± y x = ±1. x = − ± i . ◮Resolución: Observe que x−4 − 10x−2 + 9 = (x−2 − 9)(x−2 − 1). De manera semejante. 13/6.2) ◮Resolución: Tenemos idénticamente (3x2 − 4x + 34) − (3x2 − 4x − 11) = 45. (1) se convierte 12(u2 − 2) − 56u + 89 = 0. 6. ◭ 352 Ejemplo Resolver p 3x2 − 4x + 34 − p 3x2 − 4x − 11 = 9. Por lo tanto 1 5 x+ = x 2 y 1 13 x+ = .64 Chapter 5 ◮Resolución: Observe que 9x − 3x+1 − 4 = (3x − 4)(3x + 1). De aquí x = −2. ◭ 351 Ejemplo Hallar las soluciones reales de p x2 − 5x + 2 x2 − 5x + 3 = 12. Luego u = 9 (descartamos u1/2 +5 = 0. 3/2. 12(x2 + 1 1 ) − 56(x + ) + 89 = 0. ◮Resolución: Reordenando 12x4 + 12 − 56(x3 + x) + 89x2 = 0. Como no existe ningún número real con 3x + 1 = 0. 56.1) Dividiendo por x2 . Así 3x − 4 = 0 nos da x = log3 4. Así (y − 20)(y − 42) = 504 o y2 − 62y + 336 = (y − 6)(y − 56) = 0. Pongamos y = x2 − x. Poniendo u = x2 −5x+3 obtenemos u +2u1/2 −15 = (u1/2 +5)(u1/2 −3) = 0. (5. Usando esto. x2 x Pongamos u = x + 1/x. ◭ 350 Ejemplo Resolver 12x4 − 56x3 + 89x2 − 56x + 12 = 0. −7. ◮Resolución: Observe que p x2 − 5x + 3 + 2 x2 − 5x + 3 − 15 = 0. este factor se descarta. (5. Por lo tanto x2 − 5x + 3 = 9 o x = −1. ¿por qué?). x 6 Concluimos que x = 1/2. Luego y = 6. Luego u2 − 2 = x2 + 1/x2 . lo que implica x2 − x = 6 y x2 − x = 56. 8. (5. 2. 2/3. ◭ 349 Ejemplo Resolver (x − 5)(x − 7)(x + 6)(x + 4) = 504. 4.3) . de donde u = 5/2. ◮Resolución: Reordenemos los factores y multipliquemos para obtener (x − 5)(x − 7)(x + 6)(x + 4) = (x − 5)(x + 4)(x − 7)(x + 6) = (x2 − x − 20)(x2 − x − 42). Para que x sea una solución. . Dividamos este intervalo en subintervalos de longitud 2π (con un último intervalo más corto): ]0. Sumando (2) y (4) p (5. cos 5 4 ◭ 355 Ejemplo ¿Cuántas soluciones reales tiene la ecuación sen x = x ? 100 ◮Resolución: Vemos que x = 0 es una solución. ◮Resolución: Usando la identidad cos(u ± v) = cos u cos v ∓ senu sen v par de veces obtenemos y cos 2θ = 2 cos2 θ − 1 (5. 5 de donde x = − .4) 3x2 − 4x + 34 = 7. 100]. (x − 1)(4x2 + 2x − 1) = 0. Por lo tanto podemos restringir x al intervalo ]0. Además si x > 0 es una solución. sólo contaremos las soluciones positivas. √ √ 3 14 + x. 4π ]∪]4π . 2π ]∪]2π . 100]. 6π ] ∪ · · ·∪]28π . −x < 0 lo es también.5) cos 3θ = 4 cos3 θ − 3 cos θ . vemos que x satisface la ecuación 4x3 − 2x2 − 3x + 1 = 0. ◭ 3 353 Ejemplo Resolver la ecuación ◮Resolución: Póngase u = √ √ 3 14 + x + 3 14 − x = 4. es decir √ 2π 5−1 = . y cos 2π /5 > 0. o sea. de donde 3 = (196 − x2)1/3 . v = 3 14 − x. (5. 3.6) Pongamos x = cos 2π /5. Como x = cos 2π /5 = 1. Así pues. x es la raíz positiva de la ecuación cuadrática 4x2 + 2x − 1 = 0. 100] =]0. ◭ 354 Ejemplo Halle el valor exacto de cos 2π /5. gracias a las dos identidades (5) y (6). Entonces 64 = (u + v)3 = u3 + v3 + 3uv(u + v) = 14 + x + 14 − x + 12(196 − x2)1/3 .Ecuaciones 65 Dividiendo cada miembro de (3) por los miembros correspondientes de (2) obtenemos p p 3x2 − 4x + 34 + 3x2 − 4x − 11 = 5. 30π ]∪]30π . se debe tener |x| = 100| senx| ≤ 100. Como cos 6π /5 = cos 4π /5. que al resolver nos da x = ±13. uv = 18. = −8.66 Chapter 5 De las gráficas de y = sen x.. u+v+x = v+x+y 0. 2. v = −7. 2 2 2 (5. z+x = 6. wu = 15. y+z = −3. w = 5. o z + x = −6. z = 2 o x = −4. −5 + x = −3. u = 5. Entonces el sistema se convierte en vw = 30. v = 6. ◭ Tarea . el total de soluciones reales es por lo tanto 31 + 31 + 1 = 63. y = −1. Luego y+z = 3. z = −2. El intervalo ]30π . En cada intervalo de la forma ]2π k. −8 + u = −3. v = −6. (y + z)(y + x) = 15. Combinando este resultado con cada una de estas ecuaciones en (7). o u = −3. 2(k + 1)π ]. y = x/100 vemos que en el intervalo ]0. v = z + x. Como 0 es también una solución. obtenemos u = 3. 100] tiene una onda completa de longitud π (ya que 31π < 100) en la cual hay dos soluciones. ◮Resolución: Sumando las cuatro ecuaciones y diviendo por 3. de donde x = 4. . (z + x)(z + y) = 18. x + y + u + v = −3. x + y = 5. w = −5. x+y = −5. Por consiguiente existen 1 + 2 · 14 + 2 = 31 Así pues. y = −3. De aquí x = 2. ◭ 356 Ejemplo Resolver el sistema de ecuaciones x+y+u = 4. ◭ 357 Ejemplo Resolver el sistema de ecuaciones (x + y)(x + z) = 30. . w = x + y.7) Multiplicando todas estas ecuaciones obtenemos u v w = 8100. hay 31 soluciones positivas y por ende 31 soluciones negativas. k = 1. . 14 existen dos soluciones. esto es. 0+y = −3. y = 1. Esto implica 4+v = −3. 2π ] existe sólo una solución. ◮Resolución: Pongamos u = y + z. uvw = ±90. y + u + v = −5. . c constantes reales con abc = 0. a b » x+ x+ p √ x + ··· = 2 hiciere sentido. hallar el valor de x. x(2x + 1)(x − 2)(2x − 3) = 63. y2 + 2xy + z = 168. 379 Problema Resolver el sistema de ecuaciones |x + 1| − |x| + 3|x − 1| − 2|x − 2| = x + 2. hallar el valor de x. √ √ x − x2 − 1 x + x2 − 1 360 Problema Resuelva x4 + x3 − 4x2 + x + 1 = 0. 363 Problema Resolver la ecuación log3 x + log9 y + log9 z = 2. 372 Problema Si la ecuación x hiciere sentido. = 1. Hallar esta progresión. =2 | q x+2 » x+2 p x + ··· + 2 {z n radicales √ x + 2 3x = x.Tarea 67 358 Problema Resolver 373 Problema Si la ecuación »x 2 a »a +3 x = q b 6a + . } . 2 2 2sen x + 5 · 2cos x = 7. 371 Problema Hallar el valor de √ 30 · 31 · 32 · 33 + 1. 6 6 x3 + 3x2 y + y3 = 8. p √ √ x+3−4 x−1+ x + 8 − 6 x − 1 = 1.. 365 Problema ¿Cuántas soluciones reales tiene la ecuación 378 Problema Encuentre una solución real para la ecuación sen x = ln x? 366 Problema Resolver la ecuación (x2 − 9x − 1)10 + 99x10 = 10x9 (x2 − 1).5 log√x (x2 −x) = 3log9 4 . y2 − (z − x)2 = b2 . 375 Problema Sean a. 364 Problema Resolver la ecuación  log1/3 log4 x + log16 y + log16 z = 2. 376 Problema Resolver el sistema log2 x + log4 y + log4 z = 2. z2 + 2yz + 2xz = 12480. z2 − (x − y)2 = c2 .b. 2x3 − 2x2 y + xy2 = 1. 359 Problema Resuelva (x − 7)(x − 3)(x + 5)(x + 1) = 1680. Resolver el sistema de ecua- 361 Problema Resolver la ecuación ciones 2 −8x+15)/(x−2) |x − 3|(x x2 − (y − z)2 = a2 . xx 382 Problema Hallar las raíces reales de la ecuación … . . x2 − yz = 3. 374 Problema Resolver la ecuación √ √ x + x2 − 1 x − x2 − 1 + = 98. 380 Problema Resolver el sistema 368 Problema Resolver la ecuación 2x + y + z + u = −1 6x4 − 25x3 + 12x2 + 25x + 6 = 0. 362 Problema Resolver la ecuación x 0. p z2 − xy = 5. 367 Problema Hallar las raíces reales de y2 − zx = 4. x + y + z + 2u = −1 381 Problema Resolver el sistema de ecuaciones 370 Problema Resolver la ecuación x2 + x + y = 8. x + 2y + z + u = 12 x + y + 2z + u = 5 369 Problema Una progresión geométrica de números reales satisface que la suma de sus primeros cuatro términos es 15 y la suma de los cuadrados de estos términos es 85. 377 Problema Resuelva el sistema √  √   5 5 cos x + + log1/3 cosx − = 2. (D) a0 + a4 + a8 + a12 + · · · + a800.68 Chapter 5 x − y = 2. Así pues. ◭ 390 Ejemplo Póngase (1 + x4 + x8 )100 = a0 + a1 x + a2x2 + · · · + a800x800 . Si las ak ’s son números reales entonces escribiremos p(x) ∈ R[x]. . Si las ak ’s pertenecen exclusivamente al conjunto de los números enteros diremos que p(x) ∈ Z[x]. (C) a1 + a3 + a5 + a7 + · · · + a799. Hallar: (A) a0 + a1 + a2 + a3 + · · · + a800. . . (x + 2)2 + (y + 3)2 + (x + 2)(y + 3) = 741. Finalmente. . (E) a1 + a5 + a9 + a13 + · · · + a797. Aquí la expresión de la fracción se repite n veces. escribiremos p(x) ∈ C[x] si las ak ’s son números complejos. p(x) es también la expresión p(x) = a0 + a1 x + a2x2 + · · · + a18400x18400 . 383 Problema Resolver la ecuación 1 1+ = x.2 Polinomios Recordemos que un polinomio es una expresión de la forma p(x) = a0 + a1 x + a2x2 + · · · + an xn . ¿Cuáles son las soluciones correctas? 385 Problema Resolver el sistema de ecuaciones x4 + y4 = 82. . x101 x1 = 101. 389 Ejemplo Hallar la suma de todos los coeficientes obtenidos luego de expandir y simplificar el producto (1 − x2 + x4 )109 (2 − 6x + 5x9)1996 . 1 1 1+ 1+ 386 Problema Resolver el sistema de ecuaciones . 388 Problema Dos estudiantes trataron de resolver la ecuación cuadrática x2 +bx +c = 0. 3. Así pues. 1. . (B) a0 + a2 + a4 + a6 + · · · + a800. Vemos entonces que la suma de los coeficientes de p(x) es p(1) = a0 + a1 + a2 + · · · + a18400. p √ √ x + x + 11 + x + x − 11 = 4. ◮Resolución: Pongamos p(x) = (1 + x4 + x8 )100 = a0 + a1x + a2 x2 + · · · + a800 x800 . x100 x101 = 100. ◮Resolución: Pongamos p(x) = (1 − x2 + x4 )109 (2 − 6x + 5x9)1996 . 387 Problema Resuelva para x 384 Problema Resolver el sistema x+2+y+3+ p p (x + 2)(y + 3) = 39. que también es p(1) = (1 − 12 + 14 )109 (2 − 6 + 5)1996 = 1. 1+ 1x x1 x2 = 1. . x2 x3 = 2. El segundo copió mal c y obtuvo las soluciones x = 2. Aquí los coeficientes ak de p(x) pueden ser cualquier número complejo. la suma deseada es igual a 1. Maguer ambos estudiantes ejecutaron todos los pasos correctamente. Vemos que p(x) un polinomio de grado 4 · 109 + 9 · 1996 = 18400. el primero copió mal el coeficiente b y obtuvo las soluciones x = −6. 5. una constante. Así pues. 4 (E) a1 + a5 + a9 + a13 + · · · + a797 = p(1) − p(−1) − ip(i) + ip(−i) = 0. Aquí 0 ≤ grado r(x) < grado a(x). 4 ◭ Otra propiedad de los polinomios que es a menudo útil es el algoritmo de división: si dividimos p(x) por a(x) obtendremos polinomios q(x). como x2 + x − 2 = (x − 1)(x + 2) es un polinomio de grado 2. ◭ 392 Ejemplo Un polinomio deja residuo −2 cuando se divide por x − 1 y residuo −4 cuando se divide por x + 2. de donde el cociente es q(x) = x3 + x2 − x − 1 y el residuo es r(x) = x + 2. 2 (D) a0 + a4 + a8 + a12 + · · · + a800 = p(1) + p(−1) + p(i) + p(−i) = 2 · 3100. el residuo r(x) al dividir p(x) por x2 + x − 1 es de grado 1 o menor. b que debemos determinar. es decir. de donde el residuo es r = 0. Ahora bien. De estas ecuaciones vemos que a = 2/3. ◮Resolución: Como estamos dividiendo por un polinomio de grado 1. r(x) con p(x) = a(x)q(x) + r(x). 2 (C) a1 + a3 + a5 + a7 + · · · + a799 = p(1) − p(−1) = 0. q2 (x) con p(x) = q1 (x)(x − 1) − 2 y p(x) = q2 (x)(x + 2) − 4. Por el algoritmo de división p(x) = q(x)(x2 + x − 1) + ax + b. Luego p(1) = −2 y p(−2) = −4. ◭ . (B) a0 + a2 + a4 + a6 + · · · + a800 = p(1) + p(−1) = 3100 . existe un polinomio q(x) y una constante r con (x + 3)5 + (x + 2)8 + (5x + 9)1997 = q(x)(x + 2) + r Si ponemos x = −2 obtenemos 0 = (−2 + 3)5 + (−2 + 2)8 + (5(−2) + 9)1997 = q(−2)(−2 + 2) + r = r.Polinomios 69 Entonces (A) a0 + a1 + a2 + a3 + · · · + a800 = p(1) = 3100 . es decir r(x) = ax + b para constantes a. b = −8/3. Por ejemplo. el residuo es un polinomio de grado 0. Hallar el residuo cuando este polinomio se divide por x2 + x − 2. al dividir x5 + x4 + 1 por x2 + 1 obtenemos x5 + x4 + 1 = (x3 + x2 − x − 1)(x2 + 1) + x + 2. Luego el residuo es r(x) = 2x/3 − 8/3. 391 Ejemplo Hallar el residuo cuando (x + 3)5 + (x + 2)8 + (5x + 9)1997 se divide por x + 2. Luego −2 = p(1) = a + b y −4 = p(−2) = −2a + b. ◮Resolución: De la información dada existen polinomios q1 (x). Pero g(4) = c(4 − 1)(4 − 2)(4 − 3) = 6c y g(4) = p(4) − 4 = 1. a = 111. Demostrar que p(x) = 14 para ningún entero x. para algún polinomio q(x) con coeficientes enteros. Cada sumando en paréntesis es divisible por x5 − 1 y por ende por f (x). el residuo al dividir p(x) por x − a es un polinomio de grado 0. p(2) = 2. g(x) = (x − a)(x − b)(x − c)(x − d)q(x). Luego g(x) = c(x − 1)(x − 2)(x − 3) para alguna constante c que debemos determinar. Entonces g(x) es un polinomio de grado 3 y g(1) = g(2) = g(3) = 0. 394 Teorema (Teorema de Ruffini) El polinomio p(x) es divisible por x − a si y sólo si p(a) = 0. El teorema se deduce de esto. hemos factorizado a 7 como el producto de al menos cuatro factores enteros distintos. De esta contradicción colegimos que no existe tal entero t. ◮Resolución: Observe que f (x)(x − 1) = x5 − 1 y f (x5 ) = x20 + x15 + x10 + x5 + 1 = (x20 − 1) + (x15 − 1) + (x10 − 1) + (x5 − 1) + 5. 3 y que satisfaga p(4) = 666. ◮Resolución: El polinomio debe tener la forma p(x) = a(x − 1)(x − 2)(x − 3). b. Luego. Como 666 = p(4) = a(4 −1)(4 −2)(4 −3) = 6a. 6 ◭ 396 Ejemplo El polinomio p(x) tiene coeficientes enteros y p(x) = 7 para cuatro valores enteros diferentes de x. Finalmente p(6) = g(6) + 6 = (6 − 1)(6 − 2)(6 − 3) + 6 = 16. p(3) = 3. . p(4) = 5. Hallar el residuo cuando f (x5 ) se divide por f (x). c. ◭ El algoritmo de división nos ayuda a demostrar el siguiente resultado. p(2) = 2. ❑ 395 Ejemplo Si p(x) un polinomio cúbico con p(1) = 1. una constante. p(4) = 5. Supongamos que p(t) = 14 para algún entero t. por el Teorema del factor. pues 7 es a lo sumo 7(−1)1 el producto de tres enteros distintos. a menudo conocido como el Teorema del factor. Luego el residuo es 5. p(3) = 3. Entonces g(t) = p(t) − 7 = 14 − 7 = 7. es decir. 2. Hallar p(6). ◭ 397 Ejemplo Hallar un polinomio cúbico p(x) que se anule cuando x = 1. Luego el polinomio deseado es p(x) = 111(x−1)(x−2)(x−3). esto es. lo que es imposible. De aquí p(a) = q(a)(a − a) + r = r. Así p(x) = q(x)(x − a) + r. Demostración: Como x − a es un polinomio de grado 1. De aquí 7 = g(t) = (t − a)(t − b)(t − c)(t − d)q(t). ◭ 398 Ejemplo Hallar un polinomio cúbico p(x) con p(1) = 1. ◮Resolución: El polinomio g(x) = p(x) − 7 se anula para cuatro enteros diferentes a. ◮Resolución: Pongamos g(x) = p(x) − x. d. donde a es una constante. de donde c = 1/6.70 Chapter 5 393 Ejemplo Sea f (x) = x4 + x3 + x2 + x + 1. 2 (x − 1)(x − 2)(x − 4) c(x) = − 2 b(x) = y d(x) = (x − 1)(x − 2)(x − 3) . Para obtener el coeficiente de x5 tomamos una x de cada binomio. Para introducir el tópico consideremos primero el siguiente ejemplo. 6 Así 1 p(x) = − · (x − 2)(x − 3)(x − 4) + (x − 1)(x − 3)(x − 4) 6 5 3 − · (x − 1)(x − 2)(x − 4) + (x − 1)(x − 2)(x − 3).Polinomios 71 ◮Resolución: Utilizaremos el siguiente método debido a Lagrange. 4. 4 y d(4) = 1. Para formar el término de x3 tomamos tres x de tres de los binomios y dos constantes de los dos binomios restantes. ◭ . 2 6 El lector podrá verificar que este polinomio cumple con las condiciones estipuladas. De manera semejante. Finalmente. b(x). 4. Para formar el término de x4 tomamos una x de cuatro de los binomios y una constante del binomio restante. b(2) = 1 y b(x) se anula cuando x = 1. 2. d(x) anulándose cuando x = 1. c(x). c(3) = 1 y c(3) = 1 se anula para x = 1. donde a(x). Así pues. el término constante es (1)(−2)(4)(−5)(6) = 240. ◮Resolución: Vemos que el producto es un polinomio de grado 5. 399 Ejemplo Hallar el producto (x + 1)(x − 2)(x + 4)(x − 5)(x + 6). Sea p(x) = a(x) + 2b(x) + 3c(x) + 5d(x). Utilizando el método del problema anterior hallamos a(x) = − (x − 2)(x − 3)(x − 4) . 3. 3. el coeficiente de x4 es 1 − 2 + 4 − 5 + 6 = 4. Así el coeficiente de x3 es (1)(−2) + (1)(4) + (1)(−5) + (1)(6) + (−2)(4) + (−2)(−5) + (−2)(6) +(4)(−5) + (4)(6) + (−5)(6) = −33. 6 (x − 1)(x − 3)(x − 4) . El producto pedido es entonces x5 + 4x4 − 33x3 − 134x2 + 172x + 240. ◭ Por último discutiremos las fórmulas de Viète y las identidades de Newton-Girard. 2. 3. Así pues el coeficiente de x5 es 1. d(x) son polinomios cúbicos con las siguientes propiedades: a(1) = 1 y a(x) se anula para x = 2. el coeficiente de x2 es (1)(−2)(4) + (1)(−2)(−5) + (1)(−2)(6) + (1)(4)(−5) + (1)(4)(6) + (−2)(4)(−5) +(−2)(4)(6) + (4)(−5)(6) = −134 y el coeficiente de x es (1)(−2)(4)(−5) + (1)(−2)(4)(6) + (1)(−2)(−5)(6) + (1)(4)(−5)(6) + (−2)(4)(−5)(6) = 172. . . .. la suma de las raíces tomadas de dos en dos..... Luego a2 + b2 + c2 = 02 − 2(−1/2) = 1. Si a0 = 0 y a0 xn + a1 xn−1 + a2 xn−2 + · · · + an−1x + an es un polinomio con raíces α1 .72 Chapter 5 Del ejemplo anterior vemos que cada término tiene un “peso” de 5. vemos que 1 1 1 ab + ac + bc −1/2 + + = = = 1/2. . ... 1≤ j<k≤n X α j αk αl . Finalmente. b.. k=1 α j αk . a b c abc −1 ◭ .... la suma de los cuadrados de las raíces y la suma de los recíprocos de las raíces de la ecuación 2x3 − x + 2 = 0. 2 Para hallar a2 + b2 + c2 recurrimos a la siguiente identidad a2 + b2 + c2 = (a + b + c)2 − 2(ab + ac + bc).. a n (−1)n = α1 α2 · · · αn .... Por las fórmulas de Viète la suma de las raíces es 0 a+b+c= − = 0 2 y la suma de las raíces tomadas de dos en dos es ab + ac + bc = −1 . .. αn entonces podemos escribir a0 xn + a1xn−1 + a2 xn−2 + · · · + an−1 x + an = a0 (x − α1 )(x − α2 )(x − α3 ) · · · (x − αn−1 )(x − αn ). α2 .. a0 400 Ejemplo Hallar la suma de las raíces. . 1≤ j<k<l≤n X α j αk αl αs .. ◮Resolución: Sean a.. pues de cada uno de los cinco binomios o bien tomamos el término de x o bien tomamos la constante..... c las raíces de 2x3 − x + 2 = 0... como abc = −2/2 = −1. De esto deducimos las fórmulas de Viète: − a1 X = a0 a2 = a0 − a3 = a0 a4 = a0 X n αk . 1≤ j<k<l<s≤n ... 2 x + y2 + z2 = 3. Hallar 1 1 1 + 2 + 2. xy + yz + zx = (x + y + z)2/2 − (x2 + y2 + z2 )/2 = 9/2 − 3/2 = 3 y de x3 + y3 + z3 − 3xyz = (x + y + z)(x2 + y2 + z2 − xy − yz − zx) se desprende que xyz = 1. Luego x = y = z = 1 es la única solución del sistema anterior. α2 β 2 γ2 ◭ Conjunto con las fórmulas de Viète tenemos las identidades de Newton-Girard para las sumas de potencias sk = α1k + α2k + · · · + αnk de las raíces: a0 s1 + a1 = 0. ◭ 403 Ejemplo (USAMO 1973) Determine todas las soluciones. b. ◮Resolución: Sean x. β . x3 + y3 + z3 = 3.Polinomios 73 401 Ejemplo Sean α . γ las raíces de x3 − x2 + 1 = 0. de donde a4 + b4 + c4 = a3 − 2a + b3 − 2b + c3 − 2c = a3 + b3 + c3 − 2(a + b + c) = −5 − 2(1) = −7. etc.. hallar a 2 + b 2 + c2 a 3 + b 3 + c3 y a 4 + b 4 + c4 . reales o complejas del sistema de ecuaciones x + y + z = 3. y. Ahora bien. z las raíces del polinomio p(t) = (t − x)(t − y)(t − z) = t 3 − (x + y + z)t 2 + (xy + yz + zx)t − xyz. Luego 1 1 1 + + = (1 − α ) + (1 − β ) + (1 − γ ) = 3 − (α + β + γ ) = 3 − 1 = 2. Luego p(t) = t 3 − 3t 2 + 3t − 1 = (t − 1)3 . 2 α β γ ◮Resolución: De x3 − x2 + 1 = 0 deducimos 1/x2 = 1 − x. 402 Ejemplo Si a. de x3 = x2 − 2 obtenemos x4 = x3 − 2x. Como x3 = x2 − 2. a0 s3 + a1 s2 + a2 s1 + 3a3 = 0. obtenemos a3 + b3 + c3 = a2 − 2 + b2 − 2 + c2 − 2 = a2 + b2 + c2 − 6 = 1 − 6 = −5. a0 s2 + a1s1 + 2a2 = 0. Finalmente. c son las raíces de x3 − x2 + 2 = 0. ◭ . ◮Resolución: Primero observamos que a2 + b2 + c2 = (a + b + c)2 − 2(ab + ac + bc) = 12 − 2(0) = 1. hallar el valor de p(n + 2). . Cuando p(x) es dividido por x − 3 el residuo es 6.rn son números reales. 406 Problema Sea (1 + x2 + x4 )100 = a0 + a1 x + · · · + a400 x400 . . α100 son las raíces de x100 − 10x + 10 = 0. ¿Cuál es el residuo cuando p(x) es dividido por x2 − 9? 413 Problema Hallar un polinomio p(x) de grado 4 con p(1) = −1. β safisfacen α 3 − 3α 2 + 5α − 17 = 0. β . .n + 1. hallar la suma 100 α1100 + α2100 + · · · + α100 . Hal- lar p + q. . Determine el valor de k. . . Demostrar que ω + ω + 1 = 0. p(2) = 2. 412 Problema Si α1 . . (n − 1)a21 ≥ 2na2 . donde r1 . 414 Problema Sean α . y 409 Problema (USAMO 1984) El producto de dos de las raíces de x4 − 18x3 + kx2 + 200x − 1984 = 0 415 Problema Los números reales α . r2 . . p(5) = 8. . . . Halle 1 1 1 + 3+ 3 α3 β γ 408 Problema La ecuación x4 − 16x3 + 94x2 + px + q = 0 tiene dos raíces dobles. ω 2 = √ 3 2 1/2 − i 3/2 y ω = 1. 2. √ 405 Problema Demostrar que las tres raíces de x3 − 1 = 0 son ω = 1/2 + i 3/2.74 Chapter 5 Tarea 404 Problema Sea 411 Problema Suponga que 2 n 2n xn + a1 xn−1 + a2 xn−2 + · · · + an = (x + r1 )(x + r2 ) · · · (x + rn ) (1 + x + x ) = a0 + a1 x + · · · + a2n x . . 410 Problema Si p(x) es un polinomio de grado n tal que p(k) = 1/k. p(4) = 5. k = 1. 407 Problema El polinomio p(x) satisface p(−x) = −p(x). Hallar a0 + a3 + a6 + · · · + a399 . es −32. α5 + β 5 + γ5. Demuestre que α + β = 2. α2 . p(−3) = 4. γ las raíces de x3 − x − 1 = 0. β 3 − 3β 2 + 5β + 11 = 0. Demuestre que Hallar a0 + a2 + a4 + · · · + a2n . . 418 Axioma (Conservación de las desigualdades en la adición) ∀(x. 416 Axioma (Ley de tricotomía) ∀(x. z) ∈ R3 . La función signum se define y se denota por 8 >> >> −1 >> < signum (x) = >> 0 >> >> : +1 El siguiente lema es inmediato. ☞ x < y es lo mismo que y > x. y. 419 Axioma (Conservación de las desigualdades en la multiplicación por factores positivos) ∀(x. que suelen utilizar palabrotas en neo-lengua como no-negativo y no-positivo. 417 Axioma (Transitividad) ∀(x. z) ∈ R3 . Este uso difiere de algunos autores. etc. si x > y entonces x + z > y + z. De manera semejante. si x > y y y>z entonces x > z. y. o bien y = x. o y > x. x = y. ☞ ¡Alerta al vocabulario! Se dirá que un número x es positivo si x ≥ 0 y que es estrictamente positivo si x > 0. se dirá que un número y es negativo si y ≤ 0 y que es estrictamente negativo si y < 0. x ≤ y quiere decir y > x. . if x > 0. si x > y y z>0 entonces xz > yz. 75 if x < 0.1 Desigualdades del triángulo Se presume que R está dotado de una relación > que satisface los siguientes axiomas. y. y) ∈ R2 exactamente una de las siguientes se cumple: x > y.Chapter 6 Desigualdades 6. if x = 0. 420 Definición (La función signum) Sea x un número real. z) ∈ R3 . x2 = |x| 7. |x| = x. o bien. 423 Teorema Sea x ∈ R. El valor absoluto de x se define y se denota por |x| = signum (x) x. y) − min(x. |−x| = |x|. y) ∈ R2 ). lo que significa que max(x. |x| = max(x. ❑ 426 Teorema Si (x. Ahora. y) = x + y. 422 Definición (Valor absoluto) Sea x ∈ R. y) ∈ R2 . 2 2 Demostración: Obviamente. |x|2 = |x2 | = x2 8.76 Chapter 6 421 Lema La función signum es multiplicativa. |x| = >> : x if x < 0. y) + min(x. 4. |xy| = |x| |y| . max(x. y) = . −x). Si |x| = x. y) = x + y + |x − y| x + y − |x − y| y min(x. o bien |x − y| = −(x − y) = y − x. Entonces |x| ≤ t ⇐⇒ −t ≤ x ≤ t. 3. y) ∈ R2 entonces signum (x · y) = signum (x) signum(y). en donde se ha utilizado el lema 421. o bien |x − y| = x − y y entonces x ≥ y. √ 6. si (x. |x| ≥ 0. 5. ❑ 425 Teorema Sea t ≥ 0. |x| = −x. Entonces 8 >> < −x 1. |x| ≤ t ⇐⇒ x ≤ t ⇐⇒ −t ≤ 0 ≤ x ≤ t. lo que significa que y ≥ x y . Si |x| = −x. |x| ≤ t ⇐⇒ −x ≤ t ⇐⇒ −t ≤ x ≤ 0 ≤ t. y) = x − y. − |x| ≤ x ≤ |x|. x = signum (x) |x| 424 Teorema (∀(x. Las siguientes propiedades del valor absoluto se deducen de inmediato de su definición. Demostración: O bien. if x ≥ 0. 2. esto es. Demostración: Se tiene |xy| = signum (xy) xy = (signum(x) x) (signum (y) y) = |x| |y| . entonces max(x. . . y) − min(x. . obteniendo −(|a| + |b|) ≤ a + b ≤ (|a| + |b|). ❑ 429 Corolario Sea (a. y) = x+y max(x. y) − min(x. 428 Corolario Sean x1 . y) + min(x.1) Demostración: De 5 en el teorema 423.2) . . Entonces. . Resolviendo el sistema de ecuaciones max(x. y) se obtiene el resultado. . y) = y − x. −|a| ≤ a ≤ |a| to −|b| ≤ b ≤ |b|. y) y para min(x. b) ∈ R2 . Entonces |x1 + x2 + · · · + xn | ≤ |x1 | + |x2 | + · · · + |xn | . y) = |x − y|. x2 . (6. para max(x. max(x.. de donde sigue el teorema. b) ∈ R2 . ❑ 427 Teorema (Desigualdad del triángulo) Sea (a. se obtiene la siguiente generalización. (6. Entonces |a + b| ≤ |a| + |b|. xn números reales. ||a| − |b|| ≤ |a − b| . Demostración: Se tiene |a| = |a − b + b| ≤ |a − b| + |b|. en aplicando 425. por adición. y) = |x − y|.Desigualdades del triángulo 77 entonces max(x. Demostración: Aplicando el teorema 427 n − 1 veces |x1 + x2 + · · · + xn | ≤ |x1 | + |x2 + · · · xn−1 + xn | ≤ |x1 | + |x2 | + |x3 + · · · xn−1 + xn | ≤ |x1 | + |x2 | + · · · + |xn−1 + xn | ≤ |x1 | + |x2 | + · · · + |xn−1 | + |xn | . En cualquier caso. ❑ Por inducción. y) − min(x. 78 Chapter 6 dando |a| − |b| ≤ |a − b|. k=1 la igualdad cumpliéndose porque xk ≥ 0. Xn k=1 1 1 1 < − . 1 1 1 1 1 + + + ···+ + 2 2 2 2 (x + 1) (x + 2) (x + 3) (x + n − 1) (x + n)2 < = = ◭ 1 1 1 + + x(x + 1) (x + 1)(x + 2) (x + 2)((x + 3)) 1 1 +··· + + (x + n − 2)(x + n − 1) (x + n − 1)(x + n) 1 1 1 1 1 1 − + − + − x x+1 x+1 x+2 x+2 x+3 1 1 1 1 − + − +··· + x+n−2 x+n−1 x+n−1 x+n 1 1 − . Sea xi ≥ 0 para toda i ∈ [1. (x + k)2 > (x + k)(x + k − 1) y por lo tanto 1 1 1 1 < = − . 2 (x + k) (x + k)(x + k − 1) x + k − 1 x + k De aquí. |b| = |b − a + a| ≤ |b − a| + |a| = |a − b| + |a|. Cuando n = 1 la igualdad es obvia. k=1 Estudiar el caso de igualdad. Si n > 1 la igualdad se cumple para Xn xi x j = 0. ◮Resolución: Expandiendo el producto Yn (1 + xk ) = 1 + k=1 Xn k=1 xk + Xn 1≤i< j≤n xi x j + · · · ≥ 1 + Xn xk . n]. (x + k)2 x x+n ◮Resolución: Obsérvese que k ≥ 1. Demostrar que Yn k=1 (1 + xk ) ≥ 1 + Xn xk . ❑ 430 Ejemplo (Desigualdad de Weierstrass) Sea n > 0 un entero. − |a − b| ≤ |a| − |b| ≤ |a − b| y se aplica ahora el teorema 425. x x+n . ◭ 1≤i< j≤n 431 Ejemplo Demostrar que para todo x > 0. De manera semejante. dando |b| − |a| ≤ |a − b| =⇒ − |a − b| ≤ |a| − |b| . Así. d) ∈ R2 . entonces max(x. b B b b+B B Aún más. z) = x + y + z − min(x. |x | = 1 − i þi nþ n n i=1 ◭ Tarea 439 Problema Sea a < b. i=1 þn Å ãã þ Å þ Xn þX 1 2 þ þ xi þ ≤ − 1+ þ þ þ i n i=1 i=1 þ þ ã Xn ã Å Å þ2 þ 1 þ − 1 − 1 þ|xi | ≤ 1 − 1 . . y. −y). Demostrar que 0 ≤ x < y ⇐⇒ x2 < y2 . y) ∈ R2 . √ 2 x 2 x+1 Utilícese este hecho para demostrar que si n > 1 es un entero.an reales tal que Xn i=1 Demostrar que X hallar el valor mínimo de q. 440 Problema Si x > 0. 2 √ √ 1 x+1− x = √ √ . z números reales. demuéstrese que p (a + c)2 + (b + d)2 ≤ p a2 + b2 + p 441 Problema Demostrar que si n > 2 es un entero. entonces c2 + d 2 . y) = demostrar que 436 Problema Sean x. Esto es. números reales. z) − min(z. y. a2 . y. b). Demostrar que 433 Problema Sean x. √ √ 1 1 1 2 n + 1 − 2 < 1 + √ + √ + ··· + √ < 2 n − 1 n 3 2 437 Problema Dar una demostración puramente geométrica de la desigualdad de Minkowski en dos dimensiones. nn/2 < n!. Demostrar que √ √ 1 1 < x+1− x < √ . si p y q son enteros positivos tales que 7 p 11 < < . ai = 0. (c. números reales. usando la identidad or (x ≤ −t). Igualdad ocurre si y sólo si ad = bc. 442 Problema Demostrar que 438 Problema Sean b > 0 y B > 0. |x − a| < |x − b| ⇐⇒ x < 434 Problema Sean t ≥ 0. Demostrar que 9999 1 1 3 5 · · ··· < . − min(−x. Demuéstrese que i=1 þ þn ã þX x þ 1 Å 1 þ iþ 1 − . ≤ þ þ þ iþ 2 n i=1 ◮Resolución: Para 1 ≤ i ≤ n. a+b . Ahora bien. Demostrar que |x| ≥ t ⇐⇒ (x ≥ t) 435 Problema Sean (x. .Desigualdades del triángulo 79 432 Ejemplo Sean xi ∈ R tales que X i=1 |xi | = 1 y que X xi = 0. x) + min(x. z). 2 4 6 10000 100 A a a+A A a < =⇒ < < . y) − min(y. . x+1+ x Demostrar que max(x. 1+ + ≤ 0 ⇐⇒ þ þi n n i n n i i n n i2 n þn þ þ ãã þþ þX x þ 1 þX n Å 2 Å 1 þ þ þ iþ − 1+ xi þ. i< j | ai − a j |≥ n 2 Xn i=1 | ai | . y números reales. . si (a. . se tiene þ þ Å Å ãã2 Å ã Å ã þ þ2 1 2 4 4 (i − n)(i − 1) 1 4 þ − 1 − 1 þ ≤ 1 − 1 ⇐⇒ 2 − 1 + 1 ≤ 1− ⇐⇒ 2 − ≤ 0. þ þ= þ þ þ i þ 2þ i n Thus ya que i=1 X i=1 xi = 0. 10 q 15 443 Problema Sea n ≥ 2 un entero y a1 . 2 El cuadrado de todo real es positivo 445 Teorema El cuadrado de todo real es positivo. . Ä√ √ ä2 a − b ≥ 0. obteniendo nuevamente el resultado. . b > 0. Entonces −a > 0 y se aplica lo venido de demostrar: −a > 0 =⇒ (−a)2 > 0 =⇒ 1 · a2 > 0 =⇒ a2 > 0. Demostración: Si a = 0. þ þ þ ak þ≤ max |ak | . mostrando el teorema cuando a > 0. · ≤ a b ⇐⇒ 1 1 a b 2 + a b . Entonces √ a+b . ❑ 1 1 + √ 1 1 2 ≤ ab. Entonces 2 1 1 + a b ≤ √ ab si y sólo si a = b. esto es.80 Chapter 6 444 Problema Dado un conjunto de números reales {a1 . ❑ 447 Teorema (Desigualdad de las medias armónica y geométrica) Sean a > 0. de la que se darán varias demostraciones y generalizaciones en las páginas subsiguientes. . si a = 0. entonces a2 ≥ 0. . con igualdad si y sólo si a = b. entonces 02 = 0 y no hay nada que demostrar. Presúmase ahora que a = 0.an } demostrar que ex- iste un índice m ∈ {0. . b ≥ 0. Presúmase ahora que a < 0. . Ä√ √ ä2 √ √ a+b . Presúmase primero que a > 0.n} tal que þ þX þ þ þ 1≤k≤m ak − X m<k≤n Si m = 0 la primera suma se toma como 0 y si m = n. o bien a > 0 o bien a < 0. 1. Demostración: Por el teorema 446. 446 Teorema (Desigualdad de las medias aritmética y geométrica) Sean a ≥ 0. . Gracias al axioma 419 con x = z = a y y = 0. Demostración: Por el teorema 445. . Por tricotomía. ❑ El teorema 445 probará ser extremadamente útil y será la base de las desigualdades clásicas de este capítulo. la segunda se toma como 0. a − b ≥ 0 ⇐⇒ a − 2 ab + b ≥ 0 ⇐⇒ ab ≤ 2 demostrando el teorema. Aún más. a2 . Expandiendo. Se comenzará con la desigualdad de las medias. þ 1≤k≤n 6. … demostrando el teorema. entonces a2 > 0. se tiene aa > a0 =⇒ a2 > 0. ∀a ∈ R. ab ≤ 2 con igualdad si y sólo si a = b. y > 0. Determinar el valor mínimo de a6 a4 a2 b6 b4 b2 + + + + + . ◭ ◮Resolución: Se tiene u1 u2 ≤ (6. Nótese que hay igualdad si y sólo si x= 1 ⇐⇒ x2 = 1 ⇐⇒ x = 1. Aplicando la desigualdad de las medias aritmética y geométrica de dos números. xy ◭ 451 Ejemplo Sean u1 . ◭ 449 Ejemplo Sean a.3) . establecer la desigualdad de las medias aritmética y geométrica de cuatro números: (u1 u2 u3 u4 )1/4 ≤ √ u1 + u2 + u3 + u4 4 u1 + u2 √ u3 + u4 y u3 u4 ≤ . ◭ 450 Ejemplo Demostrar que si x > 0. entonces x y 1 + ≤ . 2 2 Simplificando se obtiene el resultado deseado.El cuadrado de todo real es positivo 81 448 Ejemplo Demostrar que la suma de todo real estrictamente positivo y su recíproco es al menos 2. u4 reales positivos. x4 + y2 x2 + y4 xy ◮Resolución: Se tiene x y + x4 + y2 x2 + y4 ≤ ≤ = y x p + p 2 x4 y2 2 x2 y4 x y + 2 2x y 2xy2 1 . u2 . x ya que se presume x > 0. Entonces … 1 1 x+ x 1 1 = x· ≤ =⇒ 2 ≤ x + . b dos reales no nulos. Ahora. ◮Resolución: Sea x > 0. b6 b4 b2 a6 a4 a2 ◮Resolución: Se tiene a6 a4 a2 b6 b4 b2 + + + + + b6 b4 b2 a6 a4 a2 Ç = a6 b6 + b6 a6 ≥ 2+2+2 = 6. aplicando la desigualdad de las medias 2 2√ √ aritmética y geométrica de dos números dos veces a u1 u2 y u3 u4 se obtiene √ √ u1 +u2 »√ 4 + u3 +u u1 u2 + u3 u4 √ 2 u1 u2 u3 u4 ≤ ≤ 2 . å Ç + a4 b4 + b4 a4 å Ç + a2 b2 + b2 a2 å con igualdad si y sólo si a = b. x 2 x demostrando la aserción. u3 . números reales ligados por la relación a2 + b2 + c2 + d 2 = ab + bc + cd + da. 2 2 2 2 2 2 2 2 Descomponiendo en factores. ◭ 454 Ejemplo Sean a. 1 1 1 1 (a − b)2 + (b − c)2 + (c − d)2 + (d − a)2 = 0.82 Chapter 6 452 Ejemplo Sean u. b. o sea. c.5) se obtiene la desigualdad a2 + b2 + c2 ≥ ab + bc + ca. c = d. 4 ≤ u+v+w u+v+w + . 2 2 2 2 Como la suma de números positivos es cero solamente cuando cada número es cero. w reales positivos. 4 12 Luego de masajear la desigualdad anterior. a2 − ab + b2 − bc + c2 − dc + d 2 − da = 0. Demostrar que a = b = c = d. Utilizando el ejemplo 451. se obtiene (uvw)1/4  u + v + w  1/4 3 lo que equivale a (uvw)1/4 Multiplicando uno y otro lado por  u + v + w  1/4  u + v + w  −1/4 3 3 ≤ u+v+w .6) . a2 b2 b2 c2 c2 d2 d2 a2 − ab + + − bc + + − dc + + − da + = 0. ◮Resolución: Se tiene.4) 3 ◮Resolución: Por el ejemplo 451. b = c. ◭ 453 Ejemplo Demostrar que si a. demostrar la desigualdad de las medias aritmética y geométrica de tres números: u+v+w (uvw)1/3 ≤ (6. c son reales positivos entonces (a + b)(b + c)(c + a) ≥ 8abc. de donde se destila el resultado. 3 se obtiene (uvw)1/4 ≤  u + v + w  3/4 3 . b. d = a. ◭ ☞ Se nota de paso que de la identidad a2 + b2 + c2 − ab − bc − ca = ä 1Ä (a − b)2 + (b − c)2 + (c − a)2 2 (6. lo que demuestra la aserción. v. (6.  uvw  u + v + w   1/4 3 ≤ u + v + w + u+v+w 3 . se obtiene a = b. b +c ≥ 2 bc y c + a ≥ 2 ca. √ √ ◮Resolución: √ El resultado se obtiene en seguida en multiplicando las desigualdades a +b ≥ 2 ab. d. x. 2 2 w = a + b. b= u+w−v . √ ◮Resolución: Como 1 + a ≥ 2 a para a ≥ 0. se tiene (1 + a1)(1 + a2) · · · (1 + an) √ √ √ ≥ 2 a1 · 2 a2 · · · 2 an = √ 2n a1 a2 · · · an = 2n . x y x+y Se cumple igualdad si y sólo si ay = bx. Probar que a b c 3 + + ≥ .El cuadrado de todo real es positivo 83 455 Teorema Sean (a. a2 . . y) ∈ R4 con x > 0. (ay − bx)2 ≥ 0 =⇒ a2 y2 − 2abxy + b2x2 ≥ 0 =⇒ a2 y(x + y) + b2x(x + y) ≥ (a + b)2 xy =⇒ a2 b2 (a + b)2 + ≥ . demostrando la aserción. b+c c+a a+b 2 ◮Resolución: Esta es una de varias demostraciones que se dará de esta famosa desigualdad. 2 . x y x+y Hay igualdad si y sólo si a b = . b1 b2 bn 457 Ejemplo Sean a1 . Así v = c + a. se obtiene el corolario siguiente. 456 Corolario Sean ak . a21 a22 a2 (a1 + a2 + · · · + an )2 . an números reales estrictamente positivos satisfaciendo a1 a2 · · · an = 1. v+w−u u+v+w = a + b + c =⇒ a = . y > 0. bk números reales con bk > 0. Póngase u = b + c. 2 c= u+v−w . b. x y Demostración: Como el cuadrado de todo real es positivo. c reales estrictamente positivos. . Demostrar que (1 + a1)(1 + a2 ) · · · (1 + an) ≥ 2n . . + + ···+ n ≥ b1 b2 bn b1 + b2 + · · · + bn con igualdad si y sólo si a1 a2 an = = ··· = . Entonces a2 b2 (a + b)2 + ≥ .❑ Iterando el teorema 455. . ◭ 458 Ejemplo (Desigualdad de Nesbitt) Sean a. b. xk = 0 o a = k2 . con igualdad si y sólo si a = b. c. √ a+b ≤ p √ √ a+ b ≤ 2(a + b). usando b c a + + b+c c+a a+b v+w−u u+w−v u+v−w + + 2u 2v 2w    1 u v 1 v w 1  w u  3 = · + + + + · + · − 2 v u 2 w v 2 u w 2 3 ≥ 1+1+1− 2 3 . demostrar que 1 (b − a)2 a+b √ 1 (b − a)2 · ≤ − ab ≤ · 8 b 2 8 a 465 Problema Demostrar que ∀a ∈ R. x2 . 469 Problema La suma de dos reales positivos es 100. b. b.84 Chapter 6 a b + ≥ 2 para a > 0. . ◮Resolución: Se tiene 0 = a2 · a − 2a · a2 + a3 = a2 (x1 + 2x2 + 3x3 + 4x4 + 5x5 ) − 2a(x1 + 23x2 + 33x3 + 43x4 + 53 x5 ) + (x1 + 25x2 + 35 x3 + 45 x4 + 55 x5 ) = (a − 1)2 x1 + 2(a − 22)2 x2 + 3(a − 32)2 x3 + 4(a − 42)2 x4 + 5(a − 52)2 x5 . x2 . x2 . 3 a2 + ab + b2 x21 + x22 + · · · + x2n = x1 x2 + x2 x3 + · · · + xn−1 xn + xn x1 . Sea (x1 . 3a4 − 10a2 + 9 > 0. b. . 4. .xn ) ∈ Rn tal que a2 − ab + b2 1 ≥ . . . x5 satisfaciendo. 25. = 2 = obteniendo el resultado. Entonces. c − d 2 . 1}. 4 463 Problema Demostrar que si r ≥ s ≥ t entonces r2 − s2 + t 2 ≥ (r − s + t)2 . ◭ Tarea 460 Problema Sean (x1 . b c a 471 Problema Sean a.xn ) ∈ Rn tales que x21 + x22 + · · · + x2n = x31 + x32 + · · · + x3n 466 Problema Sean a ≥ 0 y b ≥ 0. ◭ 459 Ejemplo (IMO 1979) Hallar todos los reales a para los cuales existen reales positivos x1 . b − c2 . . 461 Problema Sea n ≥ 2 un entero. Demostrar que xk ∈ {0. d − a 1 ≤ . Demostrar que x1 = x2 = · · · = xn . 464 Problema Si 0 < a ≤ b. x3 . b. 462 Problema Sean a. x1 + 2 5 x2 + 3 5 x3 + 4 5 x4 + 5 5 x5 = a 3 . así que hay cinco valores para a: 1. entonces a b c + + ≥ 3. entonces (a2 + 1)(b2 + 1)(c2 + 1) ≥ 8abc. Maximizar su producto. 16. c números reales. Demostrar que a3 + b3 + c3 − 3abc = (a + b + c)(a2 + b2 + c2 − ab − bc − ca). se tiene b a Entonces. b > 0. Cada término debe ser cero por separado. x1 + 2x2 + 3x3 + 4x4 + 5x5 = a. x4 . c son reales positivos. 468 Problema Demostrar que si a. x1 + 2 3 x2 + 3 3 x3 + 4 3 x4 + 5 3 x5 = a 2 . Demostrar que = x41 + x42 + · · · + x4n . 9. b reales estrictamente positivos. 470 Problema Demostrar que si a. d números reales. . . de donde para cada k. 467 Problema Sean a. Demuéstrese que Ð 2 min a − b2 . . c son reales positivos. (a + b) (b + c) (c + a) ≥ 8 (a + b − c) (b + c − a) (c + a − b) . . tanto en esta sección como en secciones subsiguientes. son números reales positivos. . . Los casos k = 2 y k = 4 se han demostrado en el teorema 446 y en el ejemplo 451. b(1 − c). b. 2n−1 (6. √ √ √ √ √ √ a + b − c + b + c − a + c + a − b ≤ a + b + c. a−1+ 475 Problema Demostrar que si a. x2n−1 se tiene (x1 x2 · · · x2n−1 )1/2 n−1 ≤ x1 + x2 + · · · + x2n−1 . 474 Problema Demostrar que si x. 6. c son las longitudes de los lados de cualquier trián- Ð2 481 Problema Sean a. c las longitudes de los lados de un triángulo.3 Desigualdades de las medias 482 Teorema (Desigualdad de las medias aritmética y geométrica) Sean a1 . Nótese que . . entonces x2 + y2 + 1 > x p y2 + 1 + y p Demuéstrese que Ä x2 + 1. Demostrar que en el intervalo ]0. y2n . b. y2 . b. c números reales estrictamente positivos tales que abc = 1. . . Considérese ahora reales positivos y1 . π [. Demostración primera: La primera demostración es un argumento por inducción un tanto truculento debido a Cauchy. x2 .Desigualdades de las medias 85 Utilizar esta identidad para demostrar nuevamente que √ 3 uvw ≤ 476 Problema Sean x. Ð números reales estrictamente positivos tales que a2 + b2 + c2 > 2 a4 + b4 + c4 . . Luego demostrar 4 479 Problema Sean x. b. . Estas ilustrarán varias técnicas útiles. gulo. c son longitudes de los lados de un triángulo. b. y. . √ a + · · · + ak k a ···a ≤ 1 . 1 k k con igualdad a1 = · · · = ak . z reales estrictamente positivos. z números reales estrictamente positivos. . c las longitudes de los lados de un triángulo. 2 9x sen x + 4 x senx 478 Problema Sean a. presúmase que para reales positivos x1 . v. b. b. esto es. y. . y z x y2 z x para reales positivos u. . 1 . y. Se demostrará la desigualdad primero para potencias de 2 y luego se interpolará entre potencias de 2. entonces 1 b äÄ b−1+ 1 c äÄ c−1+ 1 a ä ≤ 1. Se darán varias demostraciones de tan importante resultado. Demostrar que u+v+w 3 x2 z y2 z2 x y + 2+ 2 ≥ + + . Demostrar que a. 4 480 Problema Sean a. d ≤ 1 entonces alguno de entre los productos a(1 − b). w. Su media geométrica es a lo sumo su media aritmética. 473 Problema Demostrar que si 0 ≤ x ≤ 1 entonces x(1 − x) ≤ que si 0 ≤ a. es ≤ 1 . 477 Problema Sean a. ak números reales positivos. Presúmase ahora que la desigualdad es válida para k = 2n−1 > 2. c.7) Se demostrará ahora la desigualdad para 2k = 2n . esto es. Demostrar que 472 Problema (AIME 1983) Minimizar la función 2 x→ 3 (ab + bc + ca) ≤ (a + b + c)2 < 4 (ab + bc + ca) . Demostrar que x2 − z 2 y2 − x2 z 2 − y2 + + ≥0 y+z z+x x+y c(1 − d). d(1 − a) y determinar cuando se cumple la igualdad. c. a2 . a2n con ak+1 = . . k ä1/k Ä . El truco es de aumentar esta colección hasta tener 2n números y utilizar el resultado obtenido para potencias de 2. . . . Por lo ya demostrado para 2n se tiene k  a + a + ···+ a  2 k Å  a + a + · · · + a  2n −k ã1/2n a1 + a2 + · · · + ak + (2n − k) 1 1 2 k k . . a2 . G = ak+1 Ak−1 k+1 Por la hipótesis inductiva. . A + Ak = 2 (k + 1)Ak+1 − kAk + (k − 1)Ak+1 + Ak k = Ak+1 . . considérese los 2n números reales a1 . Ahora bien. . Póngase A= ak+1 + (k − 1)Ak+1 . ak+1 . . . Obsérvese que ak+1 = (k + 1)Ak+1 − kAk . . 2n (y1 y2 · · · y2n−1 )1/2 n−1 en donde la primera desigualdad sigue del caso k = 2 y la segunda de la hipótesis inductiva (6. . = a2n = a1 + a2 + · · · + ak . .86 Chapter 6 î ó hay 2n − 2n−1 = 2n−1 (2 − 1) = 2n−1 enteros en el intervalo 2n−1 + 1 . 2 . k Gk = (a1 a2 · · · ak )1/k . el caso k = 2 ya se ha establecido en el teorema 446. Presúmase ahora que 2n−1 < k < 2n y considérense los k reales positivos a1 . La hipótesis inductiva es Ak ≥ Gk y se quiere demostrar que Ak+1 ≥ Gk+1 . 2n . ak . Póngase Ak = a1 + a2 + · · · + ak . (y1 y2 · · · y2n )1/2 n q (y1 y2 · · · y2n−1 )1/2 = ≤ ≤ = n−1 y2n−1 +1 · · · y2n Ð1/2n−1 Ð1/2n−1 + y2n−1 +1 · · · y2n 2 y1 + y2 + · · · + y2n−1 y2n−1 +1 + · · · + y2n + 2n−1 2n−1 2 y1 + · · · + y2 n . . ❑ k Demostración segunda: La segunda demostración es también por inducción. Queda así demostrado el teorema para potencias de 2. . . a1 a2 · · · ak ≤ k 2n de donde  a + a + · · · + a  1−k/2n k n 1 2 k (a1 a2 · · · ak )1/2 ≤ k lo que implica (a1 a2 · · · ak )1/2 Resolviendo para n  a + a + ···+ a  a1 + a2 + · · · + ak 1 2 k + (2n − k) k k . A ≥ G. . Así pues. 2n  a + a + · · · + a  1−k/2n  a + a + · · · + a  1 2 1 2 k k ≤ . a2 . Se tiene. Como en la demostración anterior. El “relleno” será el promedio de los números a1 . . ak . ak .7). k k a1 + a2 + · · · + ak da la desigualdad deseada. j. ❑ 483 Ejemplo Sea f (x) = (a + x)5 (a − x)3 . a′j = ai + a j − A. 48 a+x a−x en donde se satisface la igualdad si y sólo si = . Ä ä k−1 1/2k Gk+1 k+1 Ak+1 = Se ha establecido que Ä ä k−1 1/2k Ak+1 ≥ Gk+1 =⇒ Ak+1 ≥ Gk+1 . Habrá desigualdad estricta si al menos dos de las a’s son desiguales. Al final se verá que todas las ai serán iguales y que su media aritmética A será igual a su media geométrica G. a′i a′j = A (ai + a j − A) = ai a j + (a j − A) (A − ai) > ai a j ya que a j − A > 0 y A − ai > 0. pero que incrementarán el producto a1 a2 · · · an .Desigualdades de las medias 87 Así pues. Por otra parte. k+1 Ak+1 lo cual completa la inducción. ❑ Demostración tercera: La tercera demostración depende de la noción de continuidad. El procedimiento terminará eventualmente logrando igualar a todas las a’s a la media aritmética e incrementando a la media geométrica.◭ 5 3 f (x) ≤ 484 Ejemplo Para todo entero n > 1 se tiene 1 · 3 · 5 · · · · (2n − 1) < nn . tales que ai < A < a j . a]. . lo cual es imposible. nA a1 + a2 + · · · + an > = A. a1 = a2 = · · · = a5 = y a6 = a7 = a8 = . ◮Resolución: Si x ∈ [−a. Luego deben de haber dos subíndices i. Póngase a′i = A. Ak+1 = A + Ak 2 ≥ (AAk )1/2 ≥ (GGk )1/2 . 5 3 8 4 de donde 55 33 a8 . x ∈ [−a. Obsérvese que ai + a j = a′i + a′j . las ai Si todas las ai fuesen > A entonces n n no pueden ser todas < A. por lo tanto se puede utilizar la desigualdad de las a+x a−x medias con n = 8. Hallar el valor máximo de f por medio de la desigualdad de la media. de donde reemplazar las a’s originales con las a’s primas no altera la media aritmética. a]. entonces a + x ≥ 0 y a − x ≥ 0. Se deduce que 5 3 Ö   a − x  è8 a + x  a8  a + x5  a − x3 +3 5 5 3 = ≤ . Se hará una serie de substituciones que conservarán la suma a1 + a2 + · · · + an . Nótese ahora que hay a lo sumo n a’s que reemplazar. De igual manera. 1 + . se tiene 2 b c  x + x + x 3 x2 y2 z2 1 1 2 3 = x x x ≤ = . De hecho. Å 1 · 3 · 5 · · · · (2n − 1) < 1 + 3 + 5 + · · ·+ (2n − 1) n Ç ãn = n2 n ån = nn . Obsérvese que como los factores son desiguales habrá desigualdad estricta. demostrando la aserción. . 1 1 1 + + ···+ a1 a2 an x2 . el volumen máximo es 8abc 8xyz ≤ √ . a > 0. 2z las dimensiones de esta caja. . . a2 . Se quiere maximizar 8xyz. an > 0. x3 = 2 . 486 Ejemplo Hallar el volumen de la caja rectangular mayor con lados paralelos a los ejes que se pueda inscribir en el elipsoide  x 2 a +  y 2 b +  z 2 c = 1. . . n n n tiene media aritmética 1+ y media geométrica Å Por lo tanto. . a2 > 0. b > 0. es estrictamente creciente. Aquí. ◮Resolución: Sean 2x. 1 + . Å ã 1 n/(n+1) 1 . > 1+ n+1 n Å 1+ 1 n+1 ãn+1 Å ã 1 n > 1+ . . su media armónica está dada por n . el conjunto de n + 1 números n 1 1 1 1. 1 + . Poniendo n = 3.88 Chapter 6 ya que por la desigualdad de las medias. 1+ esto es 1 1+ n 1 n+1 ãn/(n+1) . 1 2 3 a 2 b 2 c2 3 27 Por lo tanto. 2. en la desigualdad de las medias. . c > 0. . 3 3 ◭ 487 Definición Dados los reales a1 > 0. . Å ã 1 n 485 Ejemplo La sucesión xn = 1 + . . n = 1. n o xn+1 > xn . x1 = x2 = z2 y2 . 2y. P n ≤ n−1 s s − ak . geométrica y aritmética) Sean b1 > 0. . . se deduce 489 Corolario (Desigualdad de las medias armónica. Demostrar que Xn k=1 y que Xn k=1 ◮Resolución: Póngase bk = n2 s ≥ s − ak n−1 ak n ≥ . n ❑ Combinando el teorema 482 y el corolario 488. n−1 . b2 > 0. Entonces s − ak X n s − ak Xn 1 = = n−1 bk s k=1 k=1 y por el corolario 489. . se tiene s − ak s − ak Xn k=1 ak s − ak = = ≥ = Xn Å ã s −1 s − ak k=1 Xn Å s ã −n s − ak k=1 n2 −n n−1 n . ≤ (b1 b2 · · · bn )1/n ≤ 1 1 1 n + + ···+ b1 b2 bn 490 Ejemplo Sea ak > 0 y póngase s = a1 + a2 + · · · + an. 1 1 1 + + ···+ b1 b2 bn Demostración: Esto se sigue en poniendo ak = Å 1 1 1 ··· b1 b2 bn 1 en el teorema 482. b2 > 0. bn > 0. . . . Como s ak −1 = . Entonces n b1 + b2 + · · · + bn . . Entonces. bn > 0. n n k=1 demostrando la primera desigualdad. bk ã1/n 1 1 1 + + ···+ b1 b2 bn ≤ . s − ak n−1 s . . Entonces n ≤ (b1 b2 · · · bn )1/n .Desigualdades de las medias 89 Como colorario al teorema de las medias se obtiene 488 Corolario (Desigualdad de las medias armónica y geométrica) Considérense los reales b1 > 0. n ak = 1. bk números reales. a2 .90 Chapter 6 demostrando la segunda desigualdad. . Entonces Xn !2 ak bk = k=1 ! Xn a2k Xn ! b2k k=1 k=1 X − (ak b j − a j bk )2 . se puede relajar la estrictura de la desigualdad en la última suma. Demostrar que 493 Problema Sean x1 . a2 . 27 27 6. z números reales estrictamente positivos satisfaciendo x + y + (a + b + c)3 ≥ a3 + b3 + c3 + 24abc.xn reales estrictamente positivos.. Determinar el valor mínimo de k=1 Demostrar que 1 x X (n − 1)n .xn = 1.. b..an a1 a2 . y. . ..xn reales estrictamente positivos satisfa- ciendo x1 x2 . k=1 494 Problema Sean a. c. . x2 ..an reales estrictamente positivos. z = 1...an = 1. b. . ❑ a2k b2j − X ak bk a j b j + 1≤k≤ j≤n !2 Xn ak bk k=1 X 1≤k≤ j≤n . . De- estrictamente positivos satisfaciendo (2 + ai ) ≥ 3n . reales Yn 492 Problema Sean a.. k=1 501 Problema Sean a. c reales positivos. satisfaciendo X Yn äÄ 1+ 1 y äÄ 1+ 1 z ä ≥ 64.xn reales estrictamente positivos y a1 . x2 . Demostrar que que b √ 1 a + b + 2 + ≥ 3. Demostrar que 499 Problema Sean x... 500 Problema Sea n ≥ 2 un entero y x1 . n! < Ä n + 1 än 2 a1 . i=1 mostrar que a2 + b2 + c2 + d 2 + ab + ac + ad + bc + bd + cd ≥ 10. x1 . Demostrar 496 Problema Sean a.. a2j b2k . ◭ Tarea 491 Problema Demostrar que si n > 1 es un entero. reales con a = 0. Demuéstrese que Ä 1+ 495 Problema Sean a1 .. X (ak b j − a j bk )2 = 1≤k< j≤n X (a2k b2j − 2ak bk a j b j + a2j b2k ) 1≤k≤ j≤n = X a2k b2j − 2 1≤k≤ j≤n = X n Xn k=1 j=1 demostrando el teorema. 498 Problema Sea n > 1 un entero. b. .4 Desigualdad de Cauchy-Schwarz-Bunyakovsky 502 Teorema (Identidad de Lagrange) Sean ak .. Demostrar que 497 Problema Sean . ak (1 − ak ) ≤ n2n 1≤i< j≤n x9i + x9j 6 xi + x3i x3j + x6j . d reales positivos satisfaciendo a+b+c+d = 1. c. a2 . ... Demostrar que (x1 + x2 + · · · + xn ) Ä1 1 1 + + ··· x1 x2 xn ä Xn 2 ≥n .an reales positivos. ! Yn ak (s − xk ) ≥ n(n − 1) xk 1 n ak k=1 . b. Ahora bien. 1≤k< j≤n Demostración: Como para j = k se tiene ak b j − a j bk = 0. d reales estrictamente positivos satisfaciendo abcd = 1.... a a 2 2 abc + bcd + cda + dab ≤ 176 1 + abcd.. x2 . . !2 ! ! Xn Xn Xn 4 xk yk ≤4 x2k y2k . yk números reales. y21 + y22 + · · · + y2n de donde se obtiene la desigualdad deseada. þ þ þ þXn Xn þ þ |x | xk yk |yk | þ þ » »P k ≤ · »P Ðþ Ð P n P n þ n n 2 2 2 2 þk=1 x k=1 k=1 yk þ k=1 xk k=1 yk å Ç k=12 k n X 1 x y2 ≤ P nk 2+P nk 2 2 k=1 xk k=1 yk k=1 = de donde se deduce la desigualdad requerida. en donde se tiene desigualdad porque se suman cuadrados de números reales.❑ Demostración cuarta: La desigualdad es obvia si ! Xn x2k k=1 Xn ! y2k = 0. an ) = t(b1 . k=1 k=1 k=1 dando la desigualdad deseada. . . Luego este polinomio cuadrático es positivo para todo real t de donde se desprende que tiene raíces complejas y su discriminante b2 − 4ac es negativo. Entonces þn þ þX þ þ þ þ xk yk þ ≤ þ þ ! 1/2 Xn k=1 ! 1/2 Xn x2k y2k . . . k=1 k=1 con igualdad si y sólo si se cumple la proporción (a1 . 1 ≤ k ≤ n. bn ) para alguna constante real t. ❑ 1≤k< j≤n Demostración segunda: Póngase a = Xn x2k . b = k=1 at 2 + bt + c = t 2 Xn Xn xk yk y c = k=1 x2k − 2t k=1 Xn Xn y2k . Demostración primera: La desigualdad sigue de inmediato de la identidad de Lagrange. así que presúmase al contrario. . k=1 Utilizando la desigualdad del triángulo y la desigualdad de la media. a2 . b2 . x21 + x22 + · · · + x2n = ≥ x21 y21 x22 y22 x2n y2n + + · · · + y2n y21 y22 (x1 y1 + x2 y2 + · · · + xn yn )2 . .Desigualdad de Cauchy-Schwarz-Bunyakovsky 91 503 Teorema (Desigualdad de Cauchy-Bunyakovsky-Schwarz) Sean xk . . . ❑ Demostración tercera: Gracias al corolario 456. Considérese el polinomio cuadrático k=1 xk yk + k=1 Xn y2k = k=1 Xn k=1 (txk − yk )2 ≥ 0. ! ! !2 X Xn Xn Xn (xk y j − x j yk )2 xk yk = y2k − x2k k=1 X (teorema 502). . ❑ 1. Así. ya que 1≤k< j≤n k=1 k=1 (xk y j − x j yk )2 ≥ 0. yk números reales. 5 5 ◭ 506 Ejemplo Hallar todos los reales positivos a1 ≤ a2 ≤ . k=1 Xn a2k = 144... Xn k=1 a2k ≤ a3k = 216. ◮Resolución: Por CBS.92 Chapter 6 504 Ejemplo Demostrar que si x1 .+ x1 x2 xn ã Xn √ 1 xi √ xi ≥ !2 i=1 2 n . . . . = ◭ 505 Ejemplo (USAMO 1978) Sean a. a2 . + x1 x2 xn ã ≥ n2 . . Ä ä 16 (8 − e)2 ≤ 4 16 − e2 ⇐⇒ e (5e − 16) ≤ 0 ⇐⇒ 0 ≤ e ≤ . a3n ) para algún real t. c. e reales satisfaciendo a2 + b2 + c2 + d 2 + e2 = 16. . .. k=1 Como hay igualdad (a1 . 3 y n = 32. . son números reales estrictamente positivos. x2 . De aquí. . Maximizar el valor de e. Ä ä (a + b + c + d)2 ≤ (1 + 1 + 1 + 1) a2 + b2 + c2 + d 2 = Ä ä 4 a 2 + b 2 + c2 + d 2 . a32 . . ◭ 2 . . . . . Å 1 1 1 (x1 + x2 + . . = an = a. b. entonces Å 1 1 1 (x1 + x2 + . d. . . Entonces Xn k=1 ! 1/2 2 (xk + yk ) ≤ Xn k=1 ! 1/2 x2k + Xn k=1 ! 1/2 y2k . an ) = t(a31 . Xn ! a3k k=1 Xn ! ak . 5 16 6 El valor máximo e = es alcanzado cuando a = b = c = d = . de donde na = 96. . k=1 k=1 ◮Resolución: Obsérvese que 96 · 216 = 1442 y por CBS. + xn ) + + . ◮Resolución: Por CBS. .. a + b + c + d + e = 8. . xn . ≤ an satisfaciendo Xn Xn ak = 96. . Así a1 = a2 = . na2 = 144 resulta en a = 507 Teorema (Desigualdad de Minkowski) Sean xk . + xn ) + + . . . . tales que Ð2 p √ √ x−1+ y − 1 + z − 1. . Demostrar que xyz ≥ 3 (x + y + z). . a2 . x2 . d números reales estrictamente positivos tales que a2 + b2 = Ð3 . los números ai . xn }. se dice que ´son similarmente sorteadas si ambas son crecientes o si ambas son decrecientes.5 Desigualdad del reordenamiento 515 Definición Dado un conjunto de números reales {x1 . y > 1. . . k = 1. 513 Problema Sean x.❑ Tarea 508 Problema Si ak . Demostrar que d3 c3 + ≥ 1.Desigualdad del reordenamiento 93 Demostración: Se tiene. . con igualdad si y sólo si x1 = x2 = · · · = xn . . a21 + a22 + . b. . a2 .n.an números reales estrictamente positivos. . z > 1 tales que 509 Problema Sea xi ≥ 0 para 1 ≤ i ≤ n. z números reales estrictamente positivos tales que xyz ≥ xy + yz + zx.an . . S1 = > (n − 1) a41 + a42 + . 510 Problema Sea n ≥ 3 un entero y considérese reales estrictamente positivos a1 . . a b . y. Demostrar que x y z Ð 514 Problema Sean a1 . . k=1 512 Problema Sean x > 1. . 517 Ejemplo Las sucesiones 1 ≤ 2 ≤ · · · ≤ n and 12 ≤ 22 ≤ · · · ≤ n2 son similarmente sorteadas. k. ak son las longitudes de los lados de un triángulo. a j . . denótese por xˇ1 ≥ xˇ2 ≥ · · · ≥ xˇn el reordenamiento decreciente de las xi y por xˆ1 ≤ xˆ2 ≤ · · · ≤ xˆn el reordenamiento creciente de las xi . . c. S1 − ak k=1 6. 516 Definición Dadas dos sucesiones de números reales {x1 . . Se dice que son diferentemente sorteadas si una es creciente y la otra es decreciente. mientras que las sucesiones 1 1 1 ≥ 2 ≥ · · · ≥ 2 y 13 ≤ 23 ≤ · · · ≤ n3 son diferentemente sorteadas. . y2 . yn } de la misma longitud n. Xn y2k k=1 k=1 donde la desigualdad deseada se sigue por CBS. bk . . + a2n x+y+z ≥ 1 1 1 + + = 2. . . ck . xn } y {y1 . Xn Póngase a2i . Xn ai y S2 = i=1 Demostrar que para todo trío i. dos a dos distintos. . Xn (xk + yk )2 Xn = k=1 x2k + 2 k=1 Xn ≤ x2k + 2 k=1 Ñ = Xn xk yk + Xn k=1 ! 1/2 k=1 Xn x2k k=1 Xn ! 1/2 x2k y2k Xn + k=1 Xn ! 1/2 y2k + k=1 é ! 1/2 2 2 yk . x2 . . son reales positivos. . demuéstrese que !4 Xn ak bk ck ≤ k=1 Xn ! a4k k=1 Xn ! b4k k=1 c2 + d 2 !2 Xn c2k 511 Problema Sean a. i=1 Demuéstrese que Xn S2 − a2k ≥ S1 . . + a4n . Demostrar que x1 + x2 + · · · + xn ≤ n  x2 + x2 + · · · + x2  1/2 1 n 2 n p . . j. 2 1 2 n . . 1≤k≤n Demostración: Sea {σ (1). an } y {b1 . . Si hay dos subíndices i. j. x2 yn = 1 = 1. bn } se tiene X 1≤k≤n Así la suma X aˇk bˆ k ≤ X 1≤k≤n ak bk ≤ X aˆk bˆ k . ... Presúmase pues que todas son estrictamente positivas. Obsérvese que para 2 ≤ k ≤ n.94 Chapter 6 518 Teorema (Desigualdad del reordenamiento) Dados números reales {a1. mayor será la suma. a2 . . xk yk−1 = ak (a1 a2 · · · an )(k−1)/n a1 a2 · · · ak = · . . ◮Resolución: Si alguna de las ak es cero. . ❑ 519 Ejemplo Dar una demostración de la desigualdad de las medias utilizando la desigualdad del reordenamiento. tales que las sucesiones halan en direcciones opuestas. . σ (n)} un reordenamiento de {1. . (a1 a2 · · · an )n/n a1 a2 . b2 . ☞ Obsérvese que X aˆk bˆ k = 1≤k≤n X aˇk bˇ k . k/n a1 a2 · · · ak−1 (a1 a2 · · · an ) (a1 a2 · · · an )1/n Las xk y las yk son diferentemente sorteadas. y en virtud de la desigualdad de reordenamiento. si a1 .. x1 y2 = . . . . . . 1≤k≤n ak bk se minimiza cuando las sucesiones son diferentemente sorteadas y se maximiza cuando las sucesiones 1≤k≤n son similarmente sorteadas. n con igualdad si y sólo si a1 = · · · = an . .. n}.. . xn y y1 = 1 . Esta última desigualdad demuestra que mientras más cerca estén las a’s y las b’s de halar en la misma dirección. (a1 a2 · · · an )1/n (a1 a2 · · · an )1/n (a1 a2 · · · an )1/n . (a1 a2 · · · an )2/n . entonces considérese las sumas S = a1 bσ (1) + a2bσ (2) + · · · + ai bσ (i) + · · · + a j bσ ( j) + · · · + anbσ (n) S′ = a1 bσ (1) + a2bσ (2) + · · · + ai bσ ( j) + · · · + a j bσ (i) + · · · + anbσ (n) Entonces S′ − S = (ai − a j )(bσ ( j) − bσ (i) ) > 0. . 2. . .. (a1 a2 · · · an )1/n x2 = a1 a2 · · · an = 1. σ (2). 1 + 1 + ···+ 1 = x1 y1 + x2 y2 + · · · + xn yn ≤ x1 yn + x2 y1 + · · · + xn yn−1 = a2 an a1 + + ··· + . no hay nada que demostrar. Esto demuestra el resultado. an son reales positivos. . xn = 1 . demuéstrese que √ a1 + · · · + an n a1 · · · an ≤ . Esto es. . por ejemplo ai > a j y bσ (i) < bσ ( j) . . Póngase x1 = a1 . Desigualdad del reordenamiento 95 o sea. c) ∈ R3 . sin x cos x Por otra parte. son similarmente sorteadas. Entonces a ≥ b ≥ c es similarmente sorteada que ella misma. cos3 x y . Entonces a ≥ b ≥ c es similarmente sorteado con a2 ≥ b2 ≥ c2 y en virtud de la desigualdad del reordenamiento. ◭ 522 Ejemplo Demostrar que si ∀(a. 2 ◮Resolución: Sin pérdida de generalidad. . b. cosx sin x 2 ã i πh Ä ä Å 1 1 ◮Resolución: Sea x ∈ 0 . a2 c + b2a + c2b). 2 4 2 2 2 Se puede también utilizar la desigualdad de las medias tres veces a2 + b2 ≥ 2ab. a3 + b3 + c3 ≥ 3abc. Así 2 cosx sin x pues. ◮Resolución: Sin pérdida de generalidad. a2 + b2 + c2 ≥ ab + bc + ca. b. supóngase que a ≥ b ≥ c. c ≥ 0. a3 + b3 + c3 = aa2 + bb2 + cc2 ≥ a2 b + b2c + c2 a. b ≥ 0. Luego. . a2 + b2 + c2 = aa + bb + cc ≥ ab + bc + ca. La desigualdad deseada también se sigue de inmediato de la identidad Å ã b+c 2 3 a + b + c − ab − bc − ca = a − + (b − c)2 . n≤ a1 + a2 + · · · + an . ◭ 521 Ejemplo Demostrar que ∀(a. Las sucesiones sin3 x. b2 + c2 ≥ 2bc. por la desigualdad del reordenamiento. f π 4 = 1. c2 + a2 ≥ 2ca. y sumar. con a ≥ 0. supóngase que a ≥ b ≥ c. y así por la desigualdad del reordenamiento. c) ∈ R3 . entonces se cumplen las siguientes desigualdades: a3 + b3 + c3 ≥ max(a2 b + b2c + c2a. . (a1 a2 · · · an )1/n de donde se obtiene el resultado. y a3 + b3 + c3 = aa2 + bb2 + cc2 ≥ a2 c + b2a + c2b. f (x) ≥ sin3 x 1 1 + cos3 x = sin2 x + cos2 x = 1. ◭ 520 Ejemplo Minimizar f : x → i πh sin3 x cos3 x + sobre 0 . el mínimo deseado es 1. ä 1Ä 2 a 3 + b 3 + c3 ≥ a (b + c) + b2(c + a) + c2(a + b) . b2 . Usando la desigualdad del s−a s−b s−c b c a b c a + + ≥ + + . Sumando se obtiene la desigualdad deseada. de la identidad a3 + b3 + c3 − 3abc = (a + b + c)(a2 + b2 + c2 − ab − bc − ca). . . s−a s−b s−c s−c s−a s−b a b c a b c + + ≥ + + . 1 1 1 ≥ ≥ s−a s−b s−c 1 1 1 . 2 a3 + b3 + c3 = aa2 + bb2 + cc2 ≥ Otra vez. . bn } prove that 1 X n 1≤k≤n Ñ aˇk bˆ k ≤ 1 X n éÑ ak 1≤k≤n 1 X n é ≤ bk 1≤k≤n 1 X n aˆk bˆ k . b. c y reordenamiento dos veces.96 Chapter 6 Sumando. b+c c+a a+b 2 ◮Resolución: Presúmase que a ≥ b ≥ c. . . a2 . Demostrar que a b c 3 + + ≥ . c reales estrictamente positivos. y así a3 + b3 + c3 =≥ a2 b + b2c + c2a = (ab)a + (bc)b + (ac)c ≥ (ab)c + (bc)a + (ac)b = 3abc. s−a s−b s−c s−b s−c s−a . 1≤k≤n ◮Resolución: Se aplica la desigualdad del reordenamiento n veces: aˇ1 bˆ 1 + aˇ2bˆ 2 + · · · + aˇn bˆ n ≤ a1 b1 + a2 b2 + · · · + an bn ≤ aˆ1 bˆ 1 + aˆ2bˆ 2 + · · · + aˆnbˆ n aˇ1 bˆ 1 + aˇ2bˆ 2 + · · · + aˇn bˆ n ≤ a1 b2 + a2 b3 + · · · + an b1 ≤ aˆ1 bˆ 1 + aˆ2bˆ 2 + · · · + aˆnbˆ n aˇ1 bˆ 1 + aˇ2bˆ 2 + · · · + aˇn bˆ n ≤ a1 b3 + a2 b4 + · · · + an b2 ≤ aˆ1 bˆ 1 + aˆ2bˆ 2 + · · · + aˆnbˆ n aˇ1 bˆ 1 + aˇ2bˆ 2 + · · · + aˇn bˆ n ≤ a1 bn + a2 b1 + · · · + an bn−1 ≤ aˆ1 bˆ 1 + aˆ2bˆ 2 + · · · + aˆnbˆ n . Póngase s = a + b + c. an } y {b1 . ◭ 524 Ejemplo (Desigualdad de Nesbitt) Sean a. Esta última desigualdad también resulta de la desigualdad de las medias ya que (a3 b3 c3 )1/3 ≤ a 3 + b 3 + c3 . . 3 u otra vez. si a ≥ b ≥ c entonces ab ≥ ac ≥ bc. . ä 1Ä 2 a (b + c) + b2(c + a) + c2(a + b) . ◭ 523 Ejemplo (Desigualdad de Chebyshev) Dados reales {a1 .. Entonces −a ≤ −b ≤ −c =⇒ s − a ≤ s − b ≤ s − c =⇒ y por tanto las sucesiones a. y la identidad del ejemplo 521. . b. . . están similarmente sorteadas. satisfaciendo ab + bc + cd + da = 1. . c. 1978) Sea ak . ≥ yn números reales. .Desigualdad del reordenamiento 97 Sumando estas dos desigualdades. . b+c c+a a+b 2 1 . + an . y y1 ≥ y2 ≥ . Considérese una permutación (z1 . b. . . . Demostrar que 526 Problema Sean a. + n−1 + n ≥ a1 + a2 + . 1 ≤ k ≤ n una sucesión de enteros distintos estrictamente positivos.+xn n . Demostrar que a2 a21 a2 a2 + 2 + . .. . i=1 528 Problema Sean a1 . De- mostrar que an bn cn an−1 + bn−1 + cn−1 + + ≥ . De- x x x11 x22 . k k=1 530 Problema Sean x1 . . . . 4 (1 + y) (1 + z) (1 + z) (1 + x) (1 + x) (1 + y) Xn i=1 Xn (xi − yi )2 ≤ (xi − zi )2 . . b. . .an reales estrictamente positivos. a2 . xn ) mostrar que x1 +x2 +. . a2 a3 an a1 Demostrar que . . . . c reales estrictamente positivos y sea et n > 0 un entero. b3 c3 d3 1 a3 + + + ≥ . z2 .. y2 . xxnn ≥ (x1 x2 . x2 . . Demostrar que y3 z3 3 x3 + + ≥ . d reales positivos. . .yn ) . ≥ s−a s−b s−c s−a s−b s−c Å de donde 2 ã a b c + + ≥ 3.zn ) de (y1 . . ≥ xn . . . ◭ Tarea 525 Problema (IMO. . . z reales estrictamente positivos tales que xyz = 1. b+c c+a a+b estableciendo el resultado.xn reales estrictamente positivos. b+c+d c+d +a d +a+b a+b+c 3 531 Problema (IMO 1998) Sean x. Demostrar que Xn ak ≥ k2 k=1 Xn 529 Problema Sean a. . y. Å ã a b c b+c c+a c+a 2 + + + + . 527 Problema (IMO 1975) Sean x1 ≥ x2 ≥ . . El segmento de recta con punto inicial A y punto final B se denotará por [AB] y su longitud o distancia positiva entre A y B por AB. A Figure 7.g. Los puntos generalmente ← → se denotarán por mayúsculas. Se utilizarán tanto métodos sintéticos. A algunos resultados se darán múltiples demostraciones. B P O A Figure 7.3: Un rayo.1 Ángulos Se supondrán por conocidas las nociones elementales de punto. sino más bien. recta. etc. plano y segmento de recta. notando que AB = BA.4: Ángulo. la progresión de tópicos no será necesariamente lineal. como vectoriales. 534 Presunción Dado un punto y una recta. analíticos y trigonométricos. 7. Una gran parte de estos resultados son clásicos y se estudiarán con métodos clásicos: esto es.g. A B Figure 7. P. presentar una serie de resultados útiles para la resolución de problemas tipo olimpíada. o bien el punto no yace sobre la recta. Las rectas generalmente se denotarán por mayúsculas con flechas supra. e. sin introducir las nociones de distancia o distancia dirigida. si coinciden cuando una es sobreimpuesta a la otra. 533 Presunción Dos puntos distintos determinan una recta única. B Figure 7. El deseo no es de dar una presentación rigurosa y axiomática de la Geometría. o bien el punto yace sobre la recta. Figure 7.5: Ángulos convexo (en rojo) y cóncavo (en verde).Chapter 7 Geometría plana Se intentará aquí recoger una cantidad de resultados y métodos útiles para resolver problemas de tipo concurso en Geometría. rayo. Por tanto.1: Un punto. La recta que contiene los puntos A y B se denotará por AB. 532 Definición Se dirá que dos figuras son congruentes. Q. ← → ← → M . para evidenciar la utilidad de diversos puntos de vista. 98 .2: Una recta. e. El rayo con punto inicial A y que pasa por B se denotará por [AB[. L . [OB[) y ¤ ([OB[ . y esta a otra tercera recta. Mide 180◦ o π radianes. [OC[) = ([OA[ . 542 Definición (Ángulo) Sean [OA[ y [OB[ dos rayos de origen común O. • es simétrica. Se utilizará tanto el grado como el radián para medir ángulos. 545 Definición (Ángulo llano) Un ángulo llano es el ángulo formado por un rayo y el rayo con el mismo punto inicial pero viajando en dirección opuesta. ☞ Como el rayo inicial puede tanto girar en sentido levógiro como en sentido dextrógiro. Se utilizará la misma notación para denotar tanto a un ángulo como a su medida.Ángulos 99 535 Presunción Dos puntos yacen sobre una recta única. También se utiliza la ‘ b notación AOB o O. 180 π (7. 544 Definición (Revolución) Una revolución es el ángulo obtenido al rotar un rayo hasta que yaga otra vez sobre sí mismo. existe una recta única L′ que contiene a P y que satisface L L′ . 536 Presunción Dos rectas sobre el mismo plano o bien se intersecan en un punto único o bien no se intersecan. [OC[) son adyacentes y se cumple ¤ ¤ ¤ ([OA[ . 537 Presunción El paralelismo es una relación de equivalencia. hay ambigüedad al nombrar ángulos. Recuerdése que se verifica la identidad r A = . Así pues la misma notación se puede utilizar tanto para nombrar el ángulo convexo o cóncavo. esto es. 539 Definición (Puntos colineales) Tres puntos o más puntos se dicen colineales si yacen en la misma recta. la otra lo es a la primera y • es transitiva. ya que si una recta es paralela a otra. 540 Presunción (Relación de Chasles) Si A. ya que toda recta es paralela a sí misma. B. Mide 360◦ o 2π radianes. Véase la figura 7. [OB[). [OC[). la primera lo es a la tercera. . en cuyo caso ← → → ← → ← → ← se denominan paralelas. 538 Presunción Dado un punto P y una recta L que no contenga a P. • es reflexiva. se dice que los ángulos ([OA[ . La mayoría de las veces la notación se referirá al ángulo producido cuando el rayo inicial viaja en sentido levógiro. La región barrida por el rayo [OA[ cuando este gira ¤ sobre el vértice O hasta llegar al rayo [OB[ se denomina ángulo dirigido y se denota por ([OA[ . ¤ 543 Definición (Ángulo adyacente) Si el rayo [OB[ está entre los rayos [OA[ y [OB[.5. ya que si una recta es paralela a una segunda recta. Si la recta L es paralela a la recta L′ se escribirá L L′ . C son tres puntos colineales y si B está entre A y C entonces AC = AB + BC.1) en donde A es la medida del ángulo en grados ◦ y r es la medida del ángulo en radianes. pero en caso contrario se harán notar las excepciones con el símbolo . 541 Definición (Rectas concurrentes) Tres o más rectas se dicen concurrentes si pasan por un punto en común. [OB[) + ([OB[ . B′ M B A Figure 7. 548 Definición (Ángulo obtuso) Un ángulo obtuso es aquél que mide más que un ángulo recto pero menos que un ángulo llano.10: Ángulos opuestos por el vértice. Estas últimas rectas pueden ser o no paralelas. 549 Definición (Ángulo reflejo) Un ángulo reflejo es aquél que mide más que un ángulo llano. A′ B Figure 7. si el ángulo entre ellas es recto.6: Ángulos adyacentes. 2 547 Definición (Ángulo agudo) Un ángulo agudo es aquél que mide menos que un ángulo recto.1. 554 Definición (Rectas perpendiculares) Dos rectas L y L′ se dicen perpendiculares. en virtud del teorema 7.1.100 Chapter 7 546 Definición (Ángulo recto) Un ángulo recto es la mitad de un ángulo llano. Por formar ángulos llanos adyacentes. 551 Definición (Ángulo suplementario) Dos ángulos se dicen suplementarios si la suma de sus medidas es un ángulo llano. 555 Definición (Transversal) Una transversal es una recta que cruza a otras dos o más rectas. 553 Teorema Dos ángulos opuestos por el vértice son congruentes.9: Ángulo recto. ′ OA = π = B ′ OA + A ′ OB′ =⇒ AOB ′ OB′ . . 552 Definición (Ángulos opuestos por el vértice) Cada par de ángulos opuestos cuando dos rectas se intersecan se llaman ángulos opuestos por el vértice. ☞ Es evidente que si un ángulo recto se descompone en dos ángulos adyacentes. ’ ’ ’ ‘ +B ‘ = A’ AOB ❑ Si dos rectas se cortan y uno de los ángulos en el corte es recto. Mide 90◦ o π radianes.7: Revolución. el uno es suplementario al otro. entonces todos los demás serán ángulos rectos. entonces forman un ángulo recto.8: Ángulo llano. si dos ángulos adyacentes son suplementarios. De aquí la siguiente definición. Recíprocamente. 550 Definición (Ángulo complementario) Dos ángulos se dicen complementarios si la suma de sus medidas es un ángulo recto. B O A Figure 7. Demostración: Véase la figura 7. C O A Figure 7. denotado por L ⊥ L′ . B O A O A Figure 7. Véase a figura 7. γ . Véase a figura 7. entonces las rectas cortadas son paralelas. El lado opuesto al ángulo recto se llama hipotenusa. b o a veces. y los otros dos lados catetos.13. y equilátero si su tres lados tienen la misma longitud 563 Definición Un triángulo rectángulo es aquél que posee un ángulo recto. entonces las rectas cortadas son paralelas. los ángulos opuestos a los lados congruentes son congruentes. Recíprocamente. obtenida al unir. D B A D O′ O B′ B A′ A O′ O D′ Figure 7. α . 557 Definición (Ángulos alternos internos) Ángulos entre rectas cortadas por una transversal y en lados opuestos de la transversal son llamados ángulos alternos internos. tres puntos no alineados en el plano. Un triángulo se dice isósceles si dos de sus lados tienen la misma longitud. lo ángulos alternos externos son congruentes. letras griegas. por A. β . si una transversal cortare dos rectas y si los ángulos alternos internos fueren congruentes. 561 Presunción Si una transversal corta a dos paralelas. sus ángulos interiores CAB. si dos de los ángulos de un triángulo son congruentes. si una transversal cortare dos rectas y si los ángulos correspondientes fueren congruentes.11.Ángulos 101 556 Definición (Ángulos correspondientes) Ángulos homólogos en rectas cortadas por una transversal son llamados ángu- los correspondientes. . por Dado un triángulo △ABC. Recíprocamente. lo ángulos correspondientes son congruentes. por segmentos de recta. ‘ se denotarán. 562 Definición (Triángulo) Un triángulo es una figura en el plano. Recíprocamente. B′ B A′ A B′ O′ A′ O D′ Figure 7. 560 Presunción Si una transversal corta a dos paralelas.12. el triángulo es isósceles. 558 Definición (Ángulos alternos externos) Ángulos fuera de rectas cortadas por una transversal y en lados opuestos de la transversal son llamados ángulos alternos externos. b Bb y C.13: Ángulos alternos externos.11: Ángulos correspondientes.12: Ángulos alternos internos. 3 ‘ ABC ‘ y BCA. entonces las rectas cortadas son paralelas. Un triángulo equilátero es equiángulo. si una transversal cortare dos rectas y si los ángulos alternos externos fueren congruentes. esto es. Se presumirá conocido lo siguiente. respectivamente. 564 Presunción Si un triángulo es isósceles. Recíprocamente. 559 Presunción Si una transversal corta a dos paralelas. Véase a figura 7. Se presumirá que el lector conoce los siguientes resultados. lo ángulos alternos internos son congruentes. π cada uno de sus tres ángulos mide radianes. D D′ Figure 7. ← → Demostración: Constrúyase por A una recta paralela a BC.17. y el punto Y está sobre esta recta. 566 Definición (Ángulos exteriores de un triángulo) Un ángulo exterior o externo de un triángulo es el ángulo suplemen- tario formado al extender un lado del triángulo.102 Chapter 7 565 Teorema La suma de los ángulos internos de un triángulo es un ángulo llano (180◦ o π radianes). 568 Ejemplo Hallar la suma de ángulos de los vértices b + Bb + Cb + D b + Eb + Fb A en la figura 7. [BC] y [AB] son los catetos. presúmase que el punto X está sobre esta recta. Figure 7. La siguiente aserción es obvia. Por ser ángulos alternos externos a rectas paralelas.16: Un ángulo externo. 567 Teorema La medida de un ángulo exterior de un triángulo es la suma de las medidas de los dos ángulos internos opuestos del triángulo. C Figure 7. a la izquierda de A.15: Teorema 565.17: Ejemplo 568. ❑ A C B Figure 7. b π = XAB como se quería demostrar. como en la figura 7.15.14: Triángulo △ABC rectángulo en B. b YAC Por ser ángulos adyacentes en una línea recta ‘ +A b + YAC ‘ = Bb + A b + C. ‘ = B. a la derecha de A. Para simplificar la nomenclatura. Véase la figura 7. b XAB ‘ = C. [AC] es la hipotenusa.16. A A X Y B E F G B C B C H A X I D Figure 7. . b3 = 3x. A ◭ D b 2 c3 B a2 c1 a1 A c2 b1 C Figure 7. a3 = π − b1 − c2 = 3x.18: Ejemplo 569. G b + Bb + Cb + D b+H b + Eb + Fb + G “ + Ib = 3π . e I. demostrar que tan sólo se puede construir la figura 7. luego pues se tiene b+B b = π. . Como los triángulos han de ser isósceles. 569 Ejemplo Se construye una sucesión de triángulos isósceles.. G.Ángulos 103 ◮Resolución: Añádanse los vértices G. como se muestra en la figura. luego BC = CD. Esto conlleva a c1 = π − a1 − b1 y así a2 = π − c1 = 2x. ◮Resolución: Sea n el número de tales triángulos que puede ser construido.18. . c2 = π − a2 − b2 = π − 4x. H. Sea a1 = x. b1 = a1 = x. A b + Bb + Cb + D b + Eb + Fb = 3π − π = 2π .. b+G A y así de donde b + Ib = π . b2 = a2 = 2x. etc. tal como en ‘ = x > 0. Si BAC π 2x triángulos. Eb + Fb + H b+H “ + Ib = π . Considérese la figura . an = nx bn = nx. Por ser ángulos opuestos por el vértice. e I son también los ángulos internos del triángulo en el centro. H. . Cb + D “ = π. comenzando con AB = BC. El polígono es simple si sus lados no se cruzan. . de donde π . Es convexo si para cada dos puntos dentro del cuadrilátero.21: Cuadrilátero no convexo. Demostración: De la figura 7. C G E B H D J F A Figure 7. vemos que si el cuadrilátero es convexo. Presúmase primero que el polígono es convexo. pero no convexo. Pn n puntos distintos en el plano.104 Chapter 7 En otras palabras. Presúmase ahora que el polígono es simple. k < n y Pn con P1 . el segmento de recta uniéndolos está dentro del cuadrilátero. El cuadrilátero es simple si sus lados no se cruzan. entonces se puede elegir un vértice y conectarlo con otro vértice no consecutivo. Demostración: Por inducción. se denomina polígono. . . Se puede construir triángulos en tanto 2nx < π . .19: Cuadrilátero simple y convexo. y así. Figure 7. 2π o 360◦ . . la suma de los ángulos interiores del polígono es la suma de los ángulos de estos n − 2 triángulos. Para n = 3 esto es el teorema 565. creando así dos triángulos. no tres de ellos alineados. . pero no convexo. Es convexo si para cada dos puntos dentro del polígono. esto es. la suma de los ángulos es la de dos triángulos. a menos que se indique lo contrario. n= 2x ◭ 570 Definición (Cuadrilátero) Un cuadrilátero es una figura en el plano obtenida al unir cuatro puntos. en cada caso. P2 . De esta manera se han formado n − 2 triángulos. ❑ 572 Definición (Polígono) Sean P1 . La figura de n lados P1 P2 . creando una diagonal y dos triángulos. Un rayo emanando desde este vértice deberá chocar con otro . entonces deberá tener un ángulo reflejo. ya que el polígono no es convexo. Luego. la palabra cuadrilátero tan sólo denotará la noción de cuadrilátero simple. (n − 2)π . Si el cuadrilátero es simple. el segmento de recta uniéndolos está dentro del polígono. 573 Teorema La suma de los ángulos internos de un polígono de n lados es n − 2 ángulos llanos ((n − 2)180◦ o (n − 2)π radianes). ninguno trío de entre ellos en línea recta. como en la figura ??.20: Cuadrilátero no simple.19. Pn obtenida al unir Pk con Pk+1 . 571 Teorema La suma de los ángulos internos de un cuadrilátero simple es dos ángulos llanos (360◦ o 2π radianes). el enésimo triángulo tiene dos ángulos iguales a nx. la palabra polígono tan sólo denotará la noción de polígono simple. Desde el vértice de este ángulo reflejo se traza una diagonal. es decir. a menos que se indique lo contrario. K I L Figure 7. Elíjase cualquier vértice y únase a este vértice con los otros n − 2 vértices que no inciden en él. ☞ De ahora en adelante. ☞ De ahora en adelante. Entonces al menos uno de sus vértices tiene un ángulo reflejo. o bien. 1 ≤ k ≤ n las medidas de los ángulos internos del polígono. . Normalmente se referirá al arco nombrado en sentido levógiro. etc. haciéndose notar salvedades con el símbolo . Este procedimiento deberá terminar eventualmente en formando un triángulo. Cuando se llega al punto original.22). Como la suma de los ‘ + EFA ‘ = π . Así Xn Xn θk = π (n − 2). ABC ◭ 579 Definición (Arco) Dos puntos A y B en la circunferencia de un círculo dividen al círculo en dos partes.23. La siguiente aserción es evidente. En el último caso. (π − θk ) = 2π =⇒ k=1 k=1 ❑ 574 Definición (Ángulos externos de un polígono) Si se extiende un un lado de un polígono simple. llamadas arcos. el ángulo suplementario al ángulo del vértice es el ángulo externo del polígono. O bien ambos sub-polígonos son convexos. porque de otro modo el polígono tendría área infinita. Al viajar en sentido dextrógiro por el perímetro del polígono. como en la figura 7. n n ← → ← → EF. 575 Teorema La suma de los ángulos externos de un polígono de n lados es dos ángulos llanos (360◦ o 2π radianes). se ha dado una revolución completa. Hay ambigüedad al nombrar arcos.Ángulos 105 vértice creando una diagonal interior. 576 Definición (Polígono regular) Un polígono regular es aquél cuyos lados son congruentes y cuyos ángulos son congru- entes. en cuyo caso ya se termina la demostración por inducción. se divide a este subpolígono en dos regiones. AB ‘ + BCD ‘ + CDE ‘ + DEF. 577 Teorema La medida de un ángulo interior de un polígono regular es (n − 2)180◦ π (n − 2) o radianes. al menos uno de ellos no lo es. ❑ Segunda demostración: Sean θk . Esta diagonal divide al polígono en dos sub-polígonos. se tiene ángulos interiores de un hexágono es 4π y como FAB ‘ + BCD ‘ + CDE ‘ + DEF ‘ = 4π − π = 3π . Hállese la suma de ángulos 578 Ejemplo En la figura adjunta (figura 7. pues al moverse a través de los ángulos exteriores de un polígono se ha dado una revolución. se hace un giro de π − θk una vez pasado el k-ésimo vértice. ı denotado por AB. comenzando desde un punto que no es un vértice. ‘ ABC ◮Resolución: Trácese una recta perpendicular a ambas paralelas. ya que el número de vértices es finito. La siguiente aserción es entonces obvia. Así pues BAC exterior al △OAB. es llamado ángulo central. O O A B dOA. (III) cuando el centro del círculo está en el exterior del ángulo. (II) cuando el centro del círculo está en el interior del ángulo. 580 Definición (Cuerda) Dados dos puntos en la circunferencia de un círculo.22: Ejemplo 578. F Figure 7. esto es. C B O A Figure 7. ‘ = OBA. Se desarrollarán ahora una serie de resultados útiles en la caza de ángulos. 584 Definición (Secante a un círculo) Una secante es una recta que pasa por dos puntos de una circunferencia. COB 2 . Véanse las figuras 7. el segmento de recta que los une es llamado cuerda. una tangente a un círculo es perpendicular al radio trazado desde el punto de contacto. el △OAB es isósceles en A. la mitad de un diámetro se llama radio. ya que OA y OB son radios. 585 Presunción Una recta perpendicular en su extremo a un radio de un círculo le es tangente al círculo.25: Cuerda [AB]. O A B Figure 7. 583 Definición (Tangente a un círculo) Una tangente es una recta que pasa por sólo un punto de la circunferencia.28. B A Ángulo periférico Se utilizará el siguiente resultado.30. de fácil demostración.B B A C A C 106 Chapter 7 D D E F E Figure 7. ‘ Por ser ángulo En el primer caso.23: Ejemplo 578. Un segmento desde el centro del círculo hasta la circunferencia. 581 Definición (Ángulo central) Un ángulo cuyo vértice es el centro del círculo y cuyos lados son lados del círculo. Recíprocamente. Una cuerda que pasa por el centro del círculo se llama diámetro.24: Arco AÙB. 7.27: dCA.29 y 7. Demostración: Se dividirá la demostración en tres casos: (I) cuando uno de los lados es un diámetro. 582 Definición (Ángulo periférico) Un ángulo cuyo vértice está en la circunferencia del círculo y cuyos lados son cuerdas es llamado ángulo periférico o inscrito en el círculo. Figure 7.26: Ángulo central B B Figure 7. 586 Teorema La medida de un ángulo periférico es la mitad de la medida del ángulo central que subtiende el mismo arco. ‘ ‘ = BAC ‘ + OBA ‘ =⇒ CAB ‘ = COB . Caso II.34: Corolario 590.33. Por lo tanto [BV ] ⊥ [PB].33: Teorema 589. .Ángulos 107 En el segundo caso. Caso I. P A Figure 7. Ä ä ‘ ‘ = π − AOV ‘ = π − 1 · π − BOA ‘ = π − ABV ‘ = BOA . Caso III.35: Teorema 591 P B′ B Figure 7. Corolario Los siguientes corolarios son ahora inmediatos. Figure 7.29: Teorema 586.32: 588. ❑ 590 Corolario Tangentes trazadas desde un punto exterior hasta un círculo son congruentes. 587 Corolario Dos ángulos periféricos que subtienden el mismo arco son congruentes. Figure 7.28: Teorema 586. Demostración: En la figura 7. ‘ = CAD ‘ + DAB ‘ = COD + DOB = COB . A′ B A O O C O A Corolario Figure 7.36: Teorema 592 589 Teorema El ángulo entre una tangente a un círculo y una cuerda es la mitad del ángulo central subtendido por la cuerda.31: 587. Figure 7. Así.30: Teorema 586. utilizando el primer caso. CAB 2 2 2 ❑ B B B B C C C A D A O O D A C Figure 7. A′ O O ′ B′ Figure 7. PBA 2 2 2 2 2 como se tenía que demostrar. A A B B B O X O V A P A Figure 7. CAB 2 2 2 Para el tercer caso se utiliza el primer y el segundo caso: ‘ = DAB ‘ − DAC ‘ = DOB − DOC = COB . trácese el diámetro [BV ]. 588 Corolario Un ángulo periférico que subtiende a un semicírculo es un ángulo recto. 594 Teorema El cuadrilátero simple ABMN es cíclico si y sólo si sus ángulos opuestos son suplementarios.36. AXB 2 2 ð Ł ‘ es suplemento del ángulo exterior del triángulo △A′ XB′ . ❑ 592 Teorema El ángulo entre dos secantes intersecándose fuera de una circunferencia es el promedio de la diferencia de los ángulos centrales de los arcos subtendidos. sea X = AA′ ∩ BB′ . extendiendo el lado A′ X hasta A. 2 Ä ä ‘ + PBA ‘ π − PAB Ä ä ′ AB + B ′ BA ‘ = π − A‘ Ä ä ′ AB + B ′ BA ‘ = π − A‘ ‘ = PAB = = = = ! ′ OB ’ ’′ A AOB π− + 2 2 ! ′ ‘ ’′ AOA BOB ′ ′ ’ π− + A OB + 2 2 Ç å ′ ′ ’ ‘ AOB A OB π− − 2 2 ′ ′ ’ ‘ AOB A OB − 2 2 terminando la demostración. ❑ 593 Definición Un cuadrilátero se llama cíclico si sus cuatro vértices yacen en un círculo. 2 2 ! terminando la demostración. ← → ← → Demostración: En la figura 7. . Se tiene que demostrar que ′ ′ ’ ‘ ‘ = AOB + A OB . ← → ← → Demostración: En la figura 7. − ′ OB′ A’ . Así Ahora AXB Ä ä ′ B′ + XB ′ A′ ’ ’ π − XA Ä ä ′ B′ + BB ′ A′ ’ ’ = π − AA ‘ = AXB = = ’′ ’′ BOA AOB + π− 2 2 ′ ′ ’ ‘ AOB A OB + .108 Chapter 7 Demostración: Se sigue de inmediato de la congruencias △OPB ∼ = △OPA. ❑ 591 Teorema El ángulo entre dos cuerdas intersecándose dentro de una circunferencia es el promedio de los ángulos centrales de los arcos subtendidos.35. sea X = AA′ ∩ BB′ . Se tiene que demostrar que ‘ ‘ = AOB AXB 2 Del △PAB. Demostración: Sean T = R el (otro) punto de intersección de los círculos circunscritos de los triángulos △ARQ y △BPR. T‘ PC = π − BPT . y así tendrá 2ABM π = = = ‘ + MNA ’ ABM ‘ + 1 BOA ‘ ‘ + 1 QOP ABM 2 2 1‘ π + QOP.37 y 7. esto es. O A B A Figure 7. o bien el mismo arco Demostración: Si ABMN es cíclico. la aserción es clara. R puntos sobre los lados [BC].39. dando nuevamente la demostración. En este caso también se ‘ + BOA ‘ = 2π . π = = = ‘ + MNA ’ ABM ‘ − 1 QOP ‘ ‘ + 1 AOB ABM 2 2 1‘ π − QOP. tal como en la figura 7. a menos que QOP circunferencia. ‘. ❑ de donde se colige que QOP N M N P B N P M Q B A Figure 7. Presúmase ahora que los ángulos opuestos del cuadrilátero ABMN son suplementarios. T‘ RB = π − ART ‘. a la Lo anterior es una contradicción.40: Teorema 594.39: Teorema 594. 595 Teorema (Miquel) En el △ABC sean P. Presúmase primero que N está fuera del círculo circunscrito al △ABM. Por la colinealidad de los puntos involucrados. presúmase que N está en el interior del círculo. △BPR.Ángulos 109 ‘ y AMB ‘ subtienden.37: Teorema 594. Véase las figuras 7. [CA].40. 2 ‘ = 0. o bien. Entonces los círculos circunscritos de los triángulos △ARQ. Se supondrá que N está fuera tanto como dentro del círculo y se obtendrá una contradicción en cada caso. arcos complementarions. en efecto.38. Q. ya que ANB del círculo. △CQP pasan por un punto común. Trácese un círculo circunscrito al △ABM de centro O. N Q M Figure 7. Por hipótesis y en considerando ABM los ángulos periféricos.38: Teorema 594. a menos que N pertenezca. [AB]. Finalmente. 2 ‘ = 0. T‘ QA = π − CQT ‘. Obsérvese que ‘ y BOA ‘ juntos subtienden la circunferencia entera y luego 2ABM ‘ + BOA ‘ = 2π . como en la figura 7. M O B A Figure 7. respectivamente. Se demostrará que N también yace en este círculo. BOF 2 2 2 2 π α ‘ = − . de donde T al círculo circunscrito del △CQP pertenece. D en una circunferencia. . Como [BD] y [BF] son tangentes al círculo. PR ⊥ QS.C. como en la figura ‘ = α .41.◭ El ángulo buscado es por lo tanto 2(BOF) 2 2 597 Ejemplo (Canadá. T‘ QA = π − ART ‘. Véase la figura 7.41: Teorema 595.42: Ejemplo 596. B. Los puntos P.42. ˆ DA.43: Ejemplo 597. DOA Ahora. ◮Resolución: Obsérvese que ADO ‘ = π − α . ˆ Demostrar que ← de la circunferencia son. Q. respectivamente. [AO] biseca al HAG 2 2 2 ’ Luego biseca al DOF. a la recta AH en E y a la recta BC en F. [OB] ‘ y así DAO ‘ = α . ‘. Si HAG ‘ y que AEO ‘ son rectos.   ‘ ‘ = DOA = 1 π − α . ˆ CD.110 Chapter 7 Porque los cuadriláteros involucrados son cíclicos. 7. Figure 7. T‘ RB = π − BPT Se deduce que T‘ PC = ‘ π − BPT = π − T‘ RB = ‘ π − ART = π − T‘ QA = ‘ π − CQT y por consecuencia el cuadrilátero CQPT es cíclico. hállese BOC ‘ en términos de α .❑ A G Q D R B B T Q F B P C P O T C A C E H A S O D R Figure 7. 1975) Se toman cuatro puntos “consecutivos” A. los puntos medios de los arcos AB. S → → ← ı BC. Figure 7. ← → ← → ← → 596 Ejemplo El círculo de centro O es tangente a la recta AG e D. R. B. intersecándose en los puntos P. De manera semejante se demuestra que PV ⊥ UW y PW ⊥ UV . Demuéstrese que P es el ortocentro del △UVW .45. B. Del △PT S. Pero esto se desprende de ◮Resolución: Es suficiente demostrar que CAD ‘ = APB ‘ + ADB ‘ CAD y de que estos últimos dos ángulos son constantes. Luego pues. U. 599 Ejemplo Tres círculos congruentes pasan por un punto común P.44: Ejemplo 598.44.Ángulos 111 → ← → ← ◮Resolución: Sea PR ∩ QS = T como en la figura 7.1 ◭ 598 Ejemplo Dos círculos se intersecan en A y B.46. Además PU es una cuerda común de los círculos con centro en A y B y así. AVW B es un paralelogramo. V y W como en la figura 7. ◭ 600 Ejemplo Sean A. ◮Resolución: Sean A.) AC 1 ¿Rectal? . El punto P viaja alrededor de uno de los círculos. AC y BC. (Figura 7. véase la figura 7. Luego AV BW y así. ‘ es constante. como en la figura 7. paralelogramos. respectivamente.C los centros de los círculos.47. Las rectas PA y PB se extienden de tal manera que corten al otro círculo en C y D. ◭ C A P B D Figure 7. Note que AVCP y BWCP son rombos y por tanto. Constrúyanse círculos con diámetros en AB.43 y sea O el centro de la circunferencia.C puntos colineales. lo es también a VW . perpendicular a AB. ˆ Demuéstrese que BEFD es un rectángulo. 2 = mostrando la perpendicularidad de las rectas. de donde se obtiene el resultado. Sea D un punto en el arco ı tal que AD ⊥ AC y sea EF la tangente común a los arcos AB ı y BC. Obsérvese que ¯ y RDS ¯ juntos comprenden la mitad de la circunferencia. PBQ ‘S PT = ‘ + SPT) ‘ π − (PST ‘ + SPR) ‘ π − (PSQ Ä 1 ‘ ‘ä = π− POQ + ROS 2 1 = π− π 2 π = . Demuéstrese que la longitud de la cuerda CD es independiente de P. ambos triángulos son equiláteros.48. Figure 7. x 605 Problema Se remueve de una pared un ladrillo que tiene forma poligonal regular. Si MBD ′ F ′ = 90 entonces ME ′ F ′ = NF ′ E ′ = 90 . ¿Cuál es el menor número de lados de este polígono? 75◦ 65◦ b+ Cb + bD + Eb de la 606 Problema Hallar la suma de los ángulos de los vértices b A+B estrella de cinco puntas de la figura 7. como en la dDH. Hállese la medida de x. así MBE ◦ ◦ ′ F ′ y NBD ′ E ′ . cabría otra vez en el hueco original. F = F . así NBF ′ B. Figure 7. ’ ’ F ′ BD = BE ′F ′ ’ ’ E ′ B.45: Ejemplo 599. y entonces E = E .49). Como ADC ′ BD = BF ′ E ′.47: Ejemplo 600. Figure 7.49: Problema 604. o en otras ÷ ÷ ÷ ÷ ÷ ’ = ME ‘ = NF ’ = ME tanto MBD ′ ′ ′ ′ ı ˆ palabras. Se concluye que BEFD es un rectángulo. demostrar que B AC = π . y sea F ′ la intersección de DC con el semicírculo BC. 607 Problema El △ABC es isósceles en A. 5 . ¿Cuánto mide E” AF en grados? 604 Problema Tres cuadrados idénticos se construyen consecutivamente.48: Problema 601. Sea Sea E ′ la intersección de AD con el semicírculo AB.112 Chapter 7 ◮Resolución: Relájese la condición AC ⊥ BD y permítase a D recorrer el semicírculo AC. en grados.46: Ejemplo 599. De igual manera. Se observa que si el ladrillo sufriese un giro de 40◦ o de 60◦ en torno a su centro. Por lo ’′ = ME ’ ’′ = NF ’ El △ME ′ B es isósceles. Figure 7.50: Problema 606. Si P es el punto medio del segmento [AB] y ” si AP = PB = BC. figura adjunta (figura 7. Se tiene ’ ’ ‘ = AE puntos medios de AB y BC respectivamente. BE ′ DF ′ es un rectángulo. Sean M y N los ′ B = BF ′C = 90◦ . Calcúlese la suma de ángulos A” CH + A D C E A B Figure 7. 602 Problema ¿Qué ángulo forman las agujas del reloj a cuarto para las cinco? 603 Problema DEFG es un cuadrado que se ha trazado fuera del pentágono regular ABCDE. ◭ D U U B A P P W B A W V F E C V C Figure 7. Tarea A B C D H G F E 601 Problema En la figura 7. △NBF ′ es isósceles. E F es tangente a ambos semicírculos AB y BC.50. 610 Problema Dos círculos se intersecan en dos puntos. demostrar que AM + BM +CM < P. 611 Problema Un círculo está inscrito en el triángulo △ABC. F D Figure 7. De- mostrar que AB − DE = CD − FA = EF − BC. .52. Hallar D AE. como en C F la figura 7. Hallar el radio del círculo.△A1 A2 A3 es equilátero y An+3 es el punto medio del segmento [An An+1 ] para todo entero estrictamente positivo n.Congruencia de triángulos y desigualdad del triángulo 113 Figure 7. Demostración: En la figura 7. ❑ 616 Ejemplo Sea M un punto en el interior del △ABC. A5 G A1 A4 A2 C E A B Figure 7. 608 Problema (AHSME. mostrar que A÷ 44 A45 A46 = A3 3 A6 ” 613 Problema En la figura AB = BC = CD = DE = EF = FG = GA. Demostrar que B AC = 614 Problema ABCDEF en la figura 7.2 Congruencia de triángulos y desigualdad del triángulo Los siguientes resultados auxiliarán en el desarrollo del tema de esta sección.53: Problema 613. B P E D Figure 7.55. [CA] y [BC] respectivamente. 1978) En la figura 7. El círculo es tangente a la hipotenusa [AB] en P. Demuéstrese que la recta que une los centros de dos círculos que se intersecan es bisecada por la cuerda común de los círculos.54 es un hexágono convexo equiangular. 2 612 Problema Un círculo es inscrito en el triángulo △ABC. Demostrar que AB + AC > MB + MC. en donde AP = 20 y BP = 6. si P el perímetro del △ABC. F los puntos de tangencia a los lados [AB]. Demostrar que B A 1 BD = (AB + BC −CA). De2π .52: Problema 612. 2 ” mayor en C = B. A C 7. Una tangente exterior común toca al círculo de menor radio en B y al círculo de radio π . rectángulo en C. la suma de las longitudes de cualesquiera dos lados es mayor que la longitud del tercer lado. E. 615 Teorema (Desigualdad del triángulo) En todo triángulo no degenerado.54: Problema 614.51. En consecuencia.51: Problema 608. siendo D. 609 Problema Dos círculos de radios desiguales son tangentes externamente en el punto A. AB + AC > BC ya que una línea recta es la distancia más corta entre dos puntos en el plano. 56: Triángulos congruentes. AB + AC > MB + MC. ◭ A / B C C B /// D /// / A E F Figure 7.55: Teorema 615. b. Figure 7. b+c c+a a+b . AB + BC +CA < 2(AM + BM +CM) =⇒ P < AM + BM +CM. BM +CM > BC. 2 La segunda desigualdad se obtiene del problema anterior. Así pues. BA + BC > MA + MC. Demuéstrese que b c a + + < 2. CA +CB > MA + MB =⇒ 2(AB + BC +CA) > 2(MA + MB + MC) =⇒ AM + BM +CM < P. CA +CB > MA + MB. Entonces AB + AC = AB + AN + NC > BN + NC = BM + MN + NC > BM + MC. Si M es un punto en el interior del triángulo. demostrar que P < AM + BM +CM < P. CM + AM > CA. 617 Ejemplo Sea P el perímetro del △ABC.114 Chapter 7 ← → ◮Resolución: Sea N el punto en el cual BM corta al [AC]. demostrando la primera aserción. ◭ 618 Ejemplo Sean a. c las longitudes de los lados de un triángulo. De manera semejante se puede demostrar que BA + BC > MA + MC. Sumando. 2 ◮Resolución: Del △MAB se tiene Del △MCA se tiene Del △MBC se tiene AM + BM > AB. . que D es el punto medio del segmento [AB] y que F está fuera de la ← → recta CD. presúmase que CD ⊥ AB. [CD] ⊥ [AB]. 621 Corolario Todo punto que equidista de los extremos de un segmento de recta yace en la mediatriz de este segmento de recta. A D B Figure 7. ➋ cualquier punto que no esté en la perpendicular está a distancias desiguales de las extremidades del segmento. Entonces ← ‘ = ADE. Sobrepóngase ahora △BDE en △ADE. completando la demostración. En la figura 7. ❑ F C C E A E D B Figure 7.Congruencia de triángulos y desigualdad del triángulo 115 ◮Resolución: Se tiene b c a + + b+c c+a a+b = < = = 2a 2b 2c + + (b + c) + (b + c) (c + a) + (c + a) (a + b) + (a + b) 2b 2c 2a + + a + (b + c) b + (c + a) c + (a + b) 2 (a + b + c) a+b+c 2.58.57. A cae sobre B. Por hipótesis BD = AD y así.58: Teorema 620. 620 Teorema Si se erige una perpendicular desde el punto medio de un segmento de recta entonces ➊ cualquier punto en la perpendicular es equidistante de las extremidades del segmento de recta. BDE BD coincide con AD.57: Teorema 620. Pero esto quiere decier que las rectas BE y AE coinciden y por lo tanto AE = BE. al → ← → ← doblar. Como una recta es la distancia más corta entre dos puntos. utilizando [DE] como eje de simetría. ◭ 619 Definición (Mediatriz de un segmento de recta) La mediatriz de un segmento de recta es la recta que pasa por el punto medio del segmento y es perpendicular a éste. BE + EF > BF. Los siguientes corolarios se deducen fácilmente. de donde BE + EF > BF =⇒ AF = AE + EF > BF. Se tiene ← → → ‘ ya que ambos son ángulos rectos. Pero BE = AE en virtud de ➊. Demostración: Presúmase que en la figura 7. donde D es el punto medio del [AB] y sea E cualquier punto en [CD]. b Sobreimpóngase △ABC en △DEF Demostración: En la figura. 626 y 627. esto es. Bb deberá caer sobre E y C sobre F. A = △DEF. b=D b y Bb = E. 625 Teorema (Criterio LAL) Si dos de los lados de un triángulo y el ángulo comprendido por estos dos lados son congruentes a los lados y al ángulo homólogos de otro triángulo. Demostración: Presúmase en la figura 7. Póngaseð al △DEF Ł sobre △ABF ′ con el lado [DE] coincidiendo con [AB] y F cayendo sobre F ′ . b el lado [AC] deberá de tal manera [AB] coincida con [DE]. 7. ‘ ❑ AED 624 Definición Los triángulos △ABC y △DEF se dicen congruentes. se sigue que deberá caer en la intersección de estos dos segmentos. b Cb = Fb y por lo tanto △ABC ∼ En la figura 7. Así. Ya que C cae simultáneamente en [DF] y [EF].60: Teorema 627. 627 Teorema (Criterio LLL) Si los tres lados de un triángulo son congruentes a los tres lados homólogos de otro triángulo. ambos triángulos son congruentes. AC = DF. BC = EF. Como AB = DE y AC = DF. Como A b b caer en el lado [DF] y C deberá caer en alguna parte sobre [DF].59. y que CA = FD. denotado por △ABC ∼ = △DEF si existe una correspon- dencia tal que sus lados correspondientes y ángulos correspondientes sean iguales. AC = DF y A byD b coincidan. presúmase que AB = DE. F′ F Figure 7. b = D. en F. ambos triángulos son congruentes. A b = D. el lado [BC] deberá caer en alguna ← → parte de la recta EF. Como B = E. ❑ A A D C B C E F D B E Figure 7.59 se tiene AB = DE.116 Chapter 7 622 Corolario Dos puntos equidistantes de los extremos de un segmento de recta determinan la recta perpendicular a este segmento de recta que pasa por el punto medio del segmento de recta. ambos triángulos son congruentes. BC = EF.60 que AB = DE. ‘ = EBD ‘ y Demostración: Sobreimpóngase △BDE en △ADE en la demostración del teorema 620. que el segmento [AB] caiga sobre el [DE] y que el [AC] caiga sobre △DEF de tal manera que A sobre [DF]. Por .❑ 626 Teorema (Criterio ALA) Si dos de los ángulos de un triángulo y el lado comprendido por estos dos ángulos son congru- entes a los ángulos y al lado homólogos de otro triángulo. Trácese el segmento CF ′ .59: Teoremas 625. que A caiga en D y que B caiga en E. b = D. el lado [BC] coincide con el lado [EF] demostrando el teorema. 623 Corolario Si se trazan rectas desde cualquier punto de la mediatriz de un segmento de recta hasta los extremos éste entonces ➊ las rectas hacen ángulos iguales con el segmento de recta.56 se tiene AB = DE. ➋ las rectas hacen ángulos iguales con la perpendicular. Luego EAD ‘ = BED. b Sobreimpóngase △ABC Demostración: Presúmase que en la figura 7. b Bb = E. A′ ′ A C C′ Y M B′ B Figure 7. HB . 12 Constrúyase el △AFB equilátero. Demostrar que △CDO es equilátero. MC . La ceviana es propia si no coincide con un lado del triángulo. Además DAO = △CBO gracias al criterio LAL.63: Puntos menelaicos.Congruencia de triángulos y desigualdad del triángulo 117 ð Ł ð Ł ð Ł hipótesis [AC] = AF ′ y [BC] = BF ′ . Por lo tanto es suficiente demostrar que DOA 12 tendría DC = DA = DO = CO. Véase la figura 7. ‘ = OBA ‘= 632 Ejemplo En el cuadrado ABCD de la figura 7. Luego △DAO ∼ △AOB es isósceles. y 2 12 12 ‘ = 5π . 631 Definición (Altura) La ceviana que va desde un vértice de un triángulo y es perpendicular al segmento opuesto se llama altura del triángulos. CA. AB del △ABC. así AA′ . Obsérvese que AO = OB. ◮Resolución: 2 Que no puntos melenudos.66. C P C X A C′ B A Figure 7.61. BB′ . Esto implica que ∼ △ABC = △DEF. [CHC ] y sus respectivas medidas por hA .65. ’′ . por virtud del corolario 622. mB y mC . ya que entonces △DAO sería isósceles y se así DO = CO. 629 Definición A un punto P que yaga sobre la recta determinada por un lado del △ABC se le llama punto menelaico2 de este lado. de donde resultaría que △CDO es equilátero. CA. L A B Figure 7. Si el punto no es un vértice del triángulo entonces es un punto menelaico propio. [CMC ] y sus respectivas medidas por mA . Las respectivas medianas serán entonces por [AMA ]. se denotarán respectivamente por HA . 630 Definición (Mediana) Una ceviana que va desde un vértice de un triángulo al punto medio del segmento opuesto se llama mediana. Pero entonces △ABC ∼ ‘ = BAF Entonces BAC = △ABF ′ por el criterio LAL (teorema 625). [AB] ⊥ CF ′ en su punto medio. hB y hC . CC′ son cevianas. AB del △ABC. HC . MB .62: Cevianas. N .61: Cevianas. ☞ Los pies de las perpendiculares de los lados BC. ❑ 628 Definición Una recta que pasa por el vértice de un triángulo se llama ceviana de este vértice. OAB π . [BHB ]. como en la figura 7. ☞ Se adoptará la convención de marcar la intersección de la ceviana con el lado opuesto a su vértice con una prima ′ . [BMB ]. ☞ Los puntos medios de los lados BC. Las respectivas alturas serán entonces por [AHA ]. se denotarán respectivamente por MA . ya que el ‘ = CBO ‘ = π − π = 5π . Así. Como DAO = y OAF = + = mediatriz del segmento [DC]. Recíprocamente. Figure 7. Demostrar que si MA BMA = AMA . A A M B Figure 7.64: Mediatriz de [AB]. [PN] ⊥ [BC].En el △ABC. satisfaciendo que no tres de ellos son colineales. b A = 100◦ .69: Problema 639. ← → ‘ esto es que PBM ‘ = Demostración: En la figura 7. Véase la figura 7.66: Ejemplo 632. presúmase que P está en la bisectriz BD del ángulo ABC. si [PM] ⊥ [AB]. B H Figure 7. la recta OF biseca a los ángulos AOB 5 5π ‘ π π π π π π π ‘= − ‘ =π− − = . M C 636 Problema En el triángulo △ABC. 7. B b = 30◦ . Demostrar que B 638 Problema Considérese n puntos verdes y n puntos amarillos en el plano. 635 Problema Sea P un punto en el interior del △ABC. Recíprocamente. [PN] ⊥ [BC] y PM = PN entonces △BPM ∼ = △BPN. la suma de la longitud de las alturas es menor que el perímetro del triángulo.A D 118 / A C D C M Chapter 7 D P C / B O A B O B N B A F Figure 7.67: Teorema 633. Demostrar que se puede parear cada punto verde con uno amarillo de tal manera que ningún segmento se cruce. Recíprocamente. 637 Problema Demostrar que en un triángulo arbitrario.65: Ejemplo 632. Así PM = PN. [BH] es una altura y [BM] es una mediana. entonces el triángulo es rectángulo en A.68: Problema 636. Figure 7. ‘ siendo además la Como △AOB and △AFB son ambos isósceles.3 Trapecios y paralelogramos 640 Definición (Trapecio) Un trapecio es un cuadrilátero en el cual al menos un par de lados son paralelos. 642 Teorema Si ABCD es un paralelogramo entonces C . ◭ 633 Teorema Todo punto en la bisectriz de un ángulo equidista de los lados del ángulo. ‘ Presúmase que [PM] ⊥ [AB]. si un punto equidista de los lados del ángulo entonces yacerá sobre la bisectriz angular. DOA 12 12 demostración. Además. demostrar que si los ángulos de un triángulo son desiguales. ❑ Tarea 634 Problema Demostrar que si los lados de un triángulo son desiguales.69. 639 Problema (AHSME 40) Véase la figura 7. Siendo triángulos rectángulos con ángulos idénticos e hipotenusa PBN. Figure 7. Así. los lados opuestos son también desiguales y el lado mayor está opuesto al ángulo mayor. Así pues AOB 12 6 12 2 12 12 12 3 5π ‘ = AOF ‘ = 5π . completando la y como además DA = AF. Hállese la medida de C dHC.68. 641 Definición (Paralelogramo) Un paralelogramo es un cuadrilátero en el cual ambos pares de lados son paralelos. ‘ y AFB. △DAO ∼ = △FAO gracias al criterio LAL. b = 50◦ .67. ” PC > b A. idéntica se concluye que △BPM ∼ = △BPN. se traza la mediana [AMA ]. los ángulos opuestos son también desiguales y el ángulo mayor está opuesto al lado mayor. siendo ambos triángulos rectángulos con un cateto e hipotenusa iguales. por el criterio ALA. A′ A B′ B Figure 7.71.70 y 7. • las diagonales se bisecan la una a la otra. • los lados opuestos son congruentes. Demostración: Véase la figura 7. Como la suma de los ángulos interiores de un paralelogramo es 2π se tiene b + Bb + Cb + D b + B) b + B) b = 2(A b =⇒ (A b = (Cb + D) b = π. 643 Teorema Sea ABCD un cuadrilátero convexo. algunas de las propiedades arriba mencionadas son suficientes para que un cuadrilátero paralelogramo sea. Los ángulos interiores opuestos son congruentes. Por la igualdad de ángulos arriba establecida y por el criterio △OAB ∼ = △OCD. Por ser ángulos alternos internos a dos paralelas. Las diagonales se bisecan la una a la otra. un par de lados opuestos son congruentes y paralelos.Trapecios y paralelogramos 119 • los ángulos opuestos son congruentes. 2. de donde los lados opuestos son congruentes. ‘ DAC ‘ = CDB. 3. 2π = A de donde los ángulos interiores adyacentes son suplementarios. Las siguientes propiedades son mutuamente equivalentes: 1. ❑ D D′ D C C C′ O B A Figure 7. ‘ BAC ‘ = BCA. 5.71: Teorema 643. Demostración: Refiérase a la figuras 7. En efecto. Los lados opuestos son iguales. Ahora. de donde AO = OC y BO = OD. .70. ‘ = DCA. ‘ ABD ‘ = ADB. A congruentes. ‘ CBD b = DAC ‘ + BAC ‘ = DCA ‘ + BCA ‘ = Cb y Bb = ABD ‘ + CBD ‘ = CDB ‘ + ADB ‘ =D b y así los ángulos opuestos son Ahora. 4. ABCD es un paralelogramo. de donde las diagonales se bisecan la una a la otra. • los ángulos interiores adyacentes son suplementarios.70: Paralelogramo. △DAC ∼ = △BCA y así AD = BC y AB = CD. ← → ← → ‘ = CBD. Por ser ángulos opuestos por el vértice O. Presúmase que en cuadrilátero ABCD. A b + Bb + Cb + D b = 2π . ABD = CDB. Los ángulos verticales DOC = △COD. 3. De aquí. Los cuatro lados son congruentes. y se concluye que AD BC. En △DBA y △BDC se tiene ADB ← → ‘ ‘ alternos internos entre paralelas por la transversal DB. se tiene DC AB. De aquí. Así ‘ + ABD ‘ = CBD ‘ + CDB ‘ =⇒ D b = B. Las diagonales se bisecan la una a la otra en ángulos rectos. △DAC ∼ = △BCA. Los cuatro ángulos internos son todos rectos. AD = A′ D′ y BC = B′C′ . ❑ Las siguientes definiciones y presunciones son análogas. Las siguientes propiedades son mutuamente equivalentes: 1. BD a las rectas AD y se tiene A → ← → ← ← → BC son iguales. 2. b ADB b = C.71. AO = OC y DO = OB. b Como 3 =⇒ 4 En la figura 7. En efecto. Las siguientes propiedades son mutuamente equivalentes: 1. ‘ y BOA ‘ son congruentes y por tanto △AOB ∼ 4 =⇒ 5 Sean AD = BC y AB = CD. Las diagonales se bisecan el ángulo en cada vértice. se obtiene entonces que los ángulos alternos internos producidos por la diagonal [AC] a estas rectas son iguales. b De la misma manera se puede establecer que A → ← − → ← → ←− ← → b = Cb y Bb = D. DOA = BOC y △DOA = △BOC por LAL. Por la misma razón. ABCD es un rectángulo. todos los ángulos de un rectángulo serán rectos.120 Chapter 7 1 =⇒ 2 Que un par de lados opuestos son congruentes se sigue por definición de paralelogramo. 2. Luego. 644 Definición (Rombo) Un rombo es un paralelogramo en el cual dos lados consecutivos son congruentes. 645 Presunción Sea ABCD un cuadrilátero simple. . 646 Definición (Rectángulos) Un rectángulo es un paralelogramo en el cual al menos uno de sus ángulos es recto. 4. Luego ODA = OBC ← → ← → ← → siendo ángulos alternos internos de la transversal DB. por el criterio ALA. se tiene AD CB. en efecto. De igual manera. AD = BC y AB = CD. ABCD es un rombo. A ← → → b Luego los ángulos alternos internos de la transversal ← b = π −D b = π − Bb = C. ‘ Luego 5 =⇒ 1 Supóngase AO = OC y BO = OD. Se concluye que ABCD es un paralelogramo. Luego OAB = OCD y siendo ángulos alternos internos de la transversal = ← → ← → ← → ∼ ‘ ‘ ‘ ‘ y AC. Que este par de lados opuestos son congruentes se sigue por el teorema 642. COD ‘ ‘ △COD ∼ △AOB por el criterio LAL. Como es bien sabido. ‘ = AOB. 647 Presunción Sea ABCD un cuadrilátero simple. ← → ← → Habiendo probado que AD BC. ‘ por ser ángulos 2 =⇒ 3 Presúmase que AD BC y que AD = BC. es fácil demostrar lo siguiente. Así. se obtiene el siguiente teorema. ❑ Aplicando el teorema anterior y su corolario a los triángulos obtenidos por las diagonales de un trapecio. MC MB BC. donde F es la intersección de → ←−−→ ← ←−−→ ←−→ la dicha recta con el lado [AC]. 648 Definición (Distancia entre rectas paralelas) La distancia entre dos rectas paralelas es la longitud de cualquier seg- mento perpendicular de una a otra recta. Por ser ángulos opuestos por el vértice MC . → ← → ← ←−−→ Demostración: Constrúyase BD AC e intersecando MB MC en D.❑ 2 2 651 Corolario La recta que biseca a un lado de un triángulo y es paralela a otro de sus lados. Como por ∼ hipótesis AMC = MC B se concluye por el criterio ALA que △AMC MB = △BMC D. se tiene F = MB . Tanto MB como F yacen sobre [AC]. la recta MC F es paralela a BC. 649 Definición (Cuadrado) Un cuadrado es un paralelogramo que es tanto un rombo como un rectángulo.73: Teorema 653. MB MC = MC D y AMB = DB. Se utilizarán ahora los resultados obtenidos arriba para demostrar varias propiedades de triángulos y cuadriláteros. 652 Teorema El segmento de recta que une los puntos medios de los lados no paralelos de un trapecio es paralelo a las bases y mide el promedio de las longitudes de las bases. ← → ←−→ Demostración: Supóngase que en la figura 7. Luego se tiene MC MB MC F. Las diagonales son congruentes y se bisecan la una a la otra. M B Figure 7. ← → ← → b ÷ ◊ ÷ M B MC B = DM B B. Se demostrarán ahora importantes teoremas de concurrencia.Trapecios y paralelogramos 121 3.72: Teorema 650 Figure 7. C C O D A MC D MB MB MA N O C B P H G A MC B A Figure 7. biseca también al tercer lado. Demostrar a su vez. que la mediana del trapecio biseca las diagonales del trapecio. Por ser ángulos alternos internos de las paralelas AC DB se tiene que A = M C BD. . Finalmente. MB MC = = .74: Teorema 655.72. 650 Teorema El segmento de recta que une los puntos medios de dos lados de un triángulo es paralelo al tercer lado y mide la mitad de éste. Del teorema 650. Dicho segmento se denomina la mediana del trapecio. DMB BC Luego BDMBC es un paralelogramo. Luego MA MB HG es un paralelogramo y por el teorema 642. 2CMC . Entonces 3P < AMA + BMB +CMC < P.122 Chapter 7 653 Teorema Las medianas de un triángulo concurren. 3 AO′ = 2AMA . MA MB = 2 AB ←→ ← → y HG AB. 654 Ejemplo Sea P el perímetro del △ABC. llamado baricentro divide a la mediana en razón 2 : 1 comenzando desde el vértice. AO + BO > AB =⇒ 2 2 3 AMA + BMB > AB =⇒ AMA + BMB > AB. sus diagonales se se tiene GH = 2 bisecan en O. ❑ C MB MA O A B MC C′ Figure 7. Sean G y H los puntos medios de [OA] y [OB]. Del AB ←−−→ ← → y MA MB AB y en considerando el △OAB y aplicando nuevamente el teorema 650 teorema 650. BH = HO = OMA Demostración: de donde se deduce AO = 2AMA . Por lo arriba demostrado. 3 BO = 2BMB . 3 Pero entonces CO′ = de donde las tres medianas concurren. 3 ←−→ ←−→ Supóngase ahora que AMA ∩ CMC = O′ . El punto de concurrencia. 4 ◮Resolución: Usando el teorema 653. 3 3 2 .75: Ejemplo 654. ←−→ ←−→ Sea AMA ∩ BMB = O. respectivamente. 3 AO′ = 2AMA = AO =⇒ O = O′ . Luego AG = GO = GMA . 2 Considerando △CDB y △ADB se tiene [ON] [DB] . Como CC′ = 2CMC y AC′ = BC. 2 [PM] [DB] .75. considerando △ACD y △ABC se tiene [PO] [CA] . AB + BC > 2BMB . C Figure 7. 3 3 (AB + BC +CA) =⇒ AMA + BMB +CMC > (AB + BC +CA) .Trapecios y paralelogramos 123 De manera semejante. 3 BMB +CMC > BC. y sean M. 2(AB + BC +CA) > 2(AMA + BMB +CMC ) =⇒ AMA + BMB +CMC < P. 2 4 2(AMA + BMB +CMC ) > deduciendo la primera desigualdad. Figure 7. O. Refléjese el punto C a través del punto MC y sea C′ la imagen de esta reflexión. 655 Teorema (Varignon) Sea ABCD un cuadrilátero convexo. Nótese que CMC = MCC′ y que por lo tanto. P. CAC′ B es un paralelogramo. Véase la figura 7. 2 Sumando. PO = CA = MN. De la misma manera. ON = DB = DB. [CD] y [DA]. se tiene AC + AC′ > CC′ =⇒ AC + BC > 2CMC .76: Teorema 656. Entonces MNOP. 2 3 CMC + AMC > CA. respectivamente. [BC].78: Teorema 658. ❑ . es un paralelogramo. Se desprende que MNOP es un paralelogramo. Sumando. Demostración: Por el teorema 650. AC + ABC > 2AMA .77: Teorema 657. [MN] [CA] . ◭ C B′ A′ HB HA A H C E A D HC B MC I MB O A F B B C′ MA Figure 7. N. los puntos medios de los lados [AB]. dando la segunda desigualdad. ❑ ☞ De la demostración del teorema 656. que BA′ = AC y C′ B = AC. Como ya se ha demostrado que las mediatrices perpendiculares concurren. Luego. Demostración: Denótense los puntos medios de los lados BC. ∼ Por lo tanto △ICD = △ICF. se intersecarán en un sólo punto. como los lados opuestos de un paralelogramo son congruentes. Falta demostrar que B es el punto medio de A′C′ . Se tiene que demostrar que OMA ⊥ BC. E y F los pies de las perpendiculares desde I a BC. Sea O el punto de intersección de las mediatrices perpendiculares de AB y AC. El punto de concurrencia es llamado el incentro del triángulo. Obsérvese que △IDB ∼ = △IEB y △IEA ∼ = △IFA.80: El circuncírculo. Se deberá demostrar que IC biseca al ángulo de vértice C. la altura del △ABC en B es perpendicular AC y como. Los casos de las otras dos alturas se demuestran de manera semejante. De manera semejante. Por definición.CA. ya que forman una línea recta. Véase la figura 7. AB por MA . Refiérase a la figura 7. Luego B es la mediatriz de A′ B′ y queda demostrada la aserción. al ser ambos triángulos rectángulos con dos catetos iguales. Figure 7. ❑ 658 Teorema Las mediatrices de los lados de un triángulo concurren.79. Nótese que tanto ABA′C como ACBC′ son paralelogramos. . Notando ángulos correspondientes OM A B = OMAC = 2 Queda demostrado el teorema. MB . Pero que cualquier par de alturas se intersecan en un punto es obvio.77. ID = IE = IF = r. El punto de concurrencia es llamado el ortocentro del triángulo. 657 Teorema Las alturas de un triángulo concurren. Obsérvese que △OBMC ∼ = △OAMC . ❑ ☞ De la demostración del teorema 658 se sigue que el circuncentro de un triángulo dado equidista de sus vértices. A F MC E MB O I B B MA D C C Figure 7. digamos. esta altura también es perpendicular a A′C′ . Se sigue. Sean D. por construcción. es el centro de su círculo circunscrito. △IDC ∼ = △IFC. MC . AC A′C′ . se sigue que en tanto las alturas se intersequen. El punto de concurrencia es llamado el circuncentro del triángulo. respectivamente. △OBMA = △OCMA . Se demostrará que cada altura del △ABC es un segmento de recta yaciendo en una mediatriz perpendicular del △A′ B′C′ . Luego I es el centro del círculo inscrito en A el △ABC de radio r. como se debía demostrar.79: El círculo inscrito. En consecuencia. Demostración: Denótese por I la intersección de las bisectrices angulares de los ángulos de los vértices A y B. Demostración: Por cada uno de los vértices del △ABC trácese una recta paralela al lado opuesto del vértice.124 Chapter 7 656 Teorema Las bisectrices de los ángulos interiores de un triángulo concurren. CA y AB. ∼ ÷ ÷ π . y fórmese el △A′ B′C′ . ya que no son rectas paralelas! Se ha demostrar ahora que la altura del △ABC en B yace en la mediatriz perpendicular de A′C′ . Se sigue que ID = IE = IF. de donde el perímetro 3 buscado es π = π s.82: Ejemplo 665. C A Figure 7.81. En particular. El área de un sector circular que subtiende un 4 θ r2 . si a = 0 se obtiene que el área de un triángulo de base b y altura h es 2 √ 3 2 triángulo equilátero de lado s es s . La longitud de un arco de círculo que subtiende un arco central de θ radianes es rθ .Tarea 125 Tarea 659 Problema Si las medianas de dos lados de un triángulo son iguales. El área de un trapecio de bases paralelas de longitud a y b unidades lineales y de altura h es 2 bh .83: Ejemplo 666. 663 Presunción (Fórmulas de área de varias figuras planas) El área de un rectángulo de lados a y b unidades lineales es (a + b)h ab unidades cuadradas. se presumirán conocidas las siguientes fórmulas. Figure 7. 6 4 . Hallar el perímetro y el área de un triángulo de Reuleux. 660 Problema Si dos de las alturas de un triángulo son iguales. El perímetro o circunferencia de un círculo es 2π r. como en la figura 7. En particular.4 Perímetros y áreas Se presumirán conocidos los conceptos de perímetro y área. demostrar que el triángulo es isósceles. 661 Problema Si un triángulo no es isósceles. B Figure 7.81: Ejemplo 664. El área de un unidades cuadradas. 7. El área de uno de dichos segmentos es √ π s2 s2 3 − . ángulo central de θ radianes en un círculo de radio r es 2 ☞El área del triángulo △ABC se denotará por [△ABC]. cada uno midiendo s . 664 Ejemplo Un triángulo de Reuleux es la figura obtenida al trazar arcos de radio s con centro en cada uno de los vértices de un triángulo equilátero de lado s. demostrar que el trián- gulo es isósceles. en donde r es el radio del círculo. cada arco de 60◦ . 662 Presunción (Fórmulas de perímetro de varias figuras planas) El perímetro de un polígono es la suma de las longitudes de sus lados. demostrar que las bisectrices internas angulares son desiguales. El área de un círculo de radio r es π r2 . π ◮Resolución: El perímetro es la suma de los tres arcos circulares. 3s 3 El área deseada es el área del triángulo equilátero contenido más tres veces el área de uno de los segmentos circulares. respectivamente. Así pues el área en rojo es 1−2· El área de la cruz patea es luego π · 4 Ç √ å2 Ç √ å2 √ 2 2 1 π 1 −2· 1− = 2− − . de lado a. Por el Teorema de Pitágoras. Como 1 π π R2 es de la circunferencia. − = + 6 4 4 2 ◭ 665 Ejemplo Tres círculos de radio R son mutuamente tangentes. 4 6 en donde R y r son los radios de los círculos. Mostrar que el perímetro del área sombreada es a+ πa 3 y por lo tanto independiente de los radios de los círculos. [CA]. T los pies de las perpendiculares desde P hasta los lados [AB]. Demostrar que PR + PS + PT = h. Sean R. ¿Cuál es el área de la figura acotada por los tres círculos? ◮Resolución: El área buscada es el área del triángulo equilátero formado al unir los tres centros de los círculos menos tres veces el área de uno de los sectores angulares formados. Hallar el área sombreada.126 Chapter 7 Ç de donde el área buscada es 3 √ å √ √ ä π s2 s2 3 s2 3 s2 Ä π− 3 . cada sector angular tiene área de . A B Figure 7. (2r)2 = 12 + 12. [BC].84: Problema 667. Como el triángulo equilátero tiene lado de longitud 2R. 668 Problema (Teorema de Viviani) Sea h la longitud de una altura del triángulo equilátero △ABC y sea P cualquier punto en el interior del triángulo. 2 ◭ 666 Ejemplo El cuadrado de la figura 7. . Luego el área buscada es 3 6 6 √ R2 π R2 3 − . 2 2 2 2 4 √ π 2 2−1− .83 tiene lado 1. 2 ◭ Tarea C 667 Problema El △ABC es equilátero. Círculos diagonalmente opuestos son tangentes. su área es √ R2 3. ◮Resolución: El área de la figura es el doble del área en rojo. como se muestra en la figura 7. La diagonal del cuadrado mide 2r. S. La región en rojo tiene como área el área del cuadrado menos dos de los cuartos de círculos y de donde r = 2 dos pequeños triángulos isósceles en las esquinas. Dos círculos tangentes con centros en B y C respectivamente se trazan. √ 2 .84. como en la figura 7. La región sombreada es formada por el área solapada de cuartos de círculos centrados en los vértices del cuadrado. Demostrar además que el área de esta región es √ a2 3 π 2 − (R + r2 ).82. Primero se halla el radio de cada uno de los cuartos de círculo. Tarea 127 669 Problema En el trapecio ABCD. Dea2 mostrar que el área del cuadrilátero EJCK es y no depende de la posición de J (o 4 K). C F Figure 7. 674 Problema El punto P está en el interior del triángulo equilátero △ABC de lado 3.88. los seis círculos pequeños tienen radio 1 y cada uno es tangente a sus dos vecinos y al círculo mayor que los encierra.86: Problema 671.87 tiene lado 4. BB′ y CC ′ son diámetros de la circunferencia y [ACB′] = 1. la distancia de P a [AC] es 2a y la distancia de P a [CB] es 3a. 670 Problema En la figura 7. Si AB = 4. BC = 5 y DE = 3. Se traza [MN] [AB] con M ∈ [AD] y N ∈ [BC]. H K B A G E J D Figure 7.87: Problema 672. 671 Problema Cuatro cilindros de diámetro 1 están pegados apretadamente por una cuerda muy fina. ¿cuál es el área del rectángulo? c6 c8 Figure 7. π . Figure 7.85: Problema 670. 673 Problema (AHSME. como en la figura 7. tal que las diagonales AA′ . Demostrar que el área de la rosa de cuatro pétalos mostrada es 8π − 16.89 se diseca en nueve cuadrados. Si el cuadrado sombreado tiene área 1. 4 c1 Demostrar también que el área sombreada entre los cilindros es 1 − c5 c2 c7 c4 c3 Figure 7. B A D E C C F B A′ A B′ C′ Figure 7. hallar MN. Hallar a. hallar EF. ¿Cuál es el área de la región estrellada acotada por los seis círculos internos? 675 Problema El rectángulo en la figura 7.88: Problema 673. como en la figura 7. [AB] [CD].90. 677 Problema El hexágono ABCA′ B′C ′ en la figura 7. La distancia de P a [AB] es a.85.86. 1984) Un rectángulo interseca a un círculo. como en la figura 7. Si [MN] biseca el área del trapecio. Figure 7. La rosa es la intersección de semicírculos de radio 2 con diámetros en los lados del cuadrado.91 está inscrito en una circunferencia.89: Problema 675 672 Problema El cuadrado en la figura 7. Demostrar que la cuerda tiene longitud 4 + π . están colocados en manera tal que un vértice de uno está en el centro del otro.91: Problema 677. 676 Problema Dos cuadrados ABCD y EHGF. ambos de lado a. . Calcular [ABCA′B′C ′ ].90: Problema ??. casi todas basadas en la disección de figuras. En todas las figuras utilizadas se presumirá que los catetos miden a.96. C b c A a B Figure 7.95: Pitágoras. cada uno de área 2 . de área (a − b)2 .93. como en la figura 7. Hállese el área de R. cada una de las cuerdas divide al círculo exterior en dos regiones cuyas áreas están en razón 1 : 3.94. el cuadrado mayor tiene área ab (a + b)2 . En la figura 7. Figure 7. el área azul del uno es la suma de las áreas rojas del otro.97 tiene área c2 . Véase la figura 7. 680 Teorema (Pitágoras) La suma de los cuadrados de las longitudes de los catetos de un triángulo rectángulo es igual a la longitud del cuadrado de la hipotenusa. El cuadrado interno azul tiene área c2 . (a + b)2 = c2 + 4 2 El cálculo algebraico efectuado se puede ver geométricamente con un reordenamiento de las piezas.93: Problema 679.92: Problema 678. A O B Figure 7. La primera demostración es atribuida al mismísimo Pitágoras. Demostración: Se presentarán varias demostraciones aquí. Los arcos O la región sombreada. Demostrar 1 que la razón del área sombreada al área del círculo interno es . El cuadrado mayor en la figura 7. Este está ab y de un cuadrado magenta. ya que ambos cuadrados tienen la misma área y descontando los triángulos amarillos. 2π ÙB es un cuarto de circunferencia del círculo de centro O que tiene radio 679 Problema A ÙA y OÙB son semicírculos congruentes y de diámetro R. La segunda demostración es atribuida a Bhaskara.128 Chapter 7 678 Problema En la figura 7. 7. b con b ≤ a y que c es la medida de la hipotenusa.94: Dimensiones. a Figure 7.92.95. compuesto de cuatro triángulos rectángulos.96: Igual en área a la figura 7. 2 Luego Å ã ab =⇒ a2 + b2 = c2 .95. Cada uno de los triángulos rectángulos amarillos tiene área .5 Teorema de Pitágoras b Se discutirá ahora lo que es quizás el más famoso teorema en todas las matemáticas. a+b Figure 7. La intersección de las cuerdas forma un cuadrado concéntrico con el círculo exterior e inscrito en el círculo interior. como en la figura 7. Figure 7. y BAF = BAC + CAF = CAB + BAE = CAE. Como △XY Z es rectángulo en Z.98 tiene bases a y b y altura a + b.❑ G F H C K B A E Figure 7. La tercera demostración es atribuida al presidente yanqui James A. △AEC tiene el lado [AE] y altura desde C igual a desde B mide AC. Luego. Por lo tanto.97: Bhaskara. Así. entonces el triángulo es rectángulo en C. Y Z = BC = a y Zb = 90◦ . Primero. a2 +b2 = c2 . ❑ . Garfield. de donde su área es (a + b)2 . El △ABF tiene base [AF] y su altura AC2 . se puede aplicar el Teorema de Pitágoras y XY 2 = XZ 2 + ZY 2 = b2 + a2 = c2 =⇒ XY = c. De manera semejante.99: Euclides 681 Teorema (Recíproco del Teorema Pitágoras) Si en el triángulo △ABC. de donde Cb = Zb = 90◦ . Esto quiere decir que el área AC2 del cuadrado cuyos lados tienen longitud AC es igual al área del rectángulo AELM. Su área es por lo tanto 2 ← → ← → ← → ← → AM. Finalmente. los dos rectángulos AELM y BMLD componen el cuadrado en la hipotenusa AB. Por otra parte. en donde M = AB∩ CL y CL AE. La cuarta demostración es una de las de los Elementos de Euclides. △ABF ∼ = △AEC por el criterio ALA.98: Garfield L D Figure 7. + 2 2 2 dando otra vez el resultado. △XY Z ∼ = △ABC. 2 ab c2 Pero el mismo trapecio puede ser descompuesto en dos triángulos de área y uno de área . Demostración: Constrúyase el △XY Z tal que XZ = AC = b. dando de nuevo el teorema. el área BC2 del cuadrado cuyos lados tienen longitud BC es igual al área BMLD. El trapecio en la figura 7. por el criterio LLL. AF = AC. el área del △AEC es la mitad del área del rectángulo AELM. 2 2 Å ã 2 2 (a + b) ab c =2 =⇒ a2 + b2 = c2 .Teorema de Pitágoras 129 Luego Å c2 = (a − b)2 + 4 ab 2 ã =⇒ c2 = a2 + b2. ‘ ‘ ‘ ‘ ‘ ‘ ya que AE = AB. como en la figura 7. B C 687 Problema Se inscribe un círculo dentro de un cuarto de círculo. Tarea 683 Problema El △ABC es rectángulo en C. Figure 7. r12 + r22 . Véase entonces que la escalera llega desde la base de la columna hasta su tope.103: Problema 686. Hallar la longitud de la escalera. 684 Problema Se inscribe un círculo de radio 2 en un cuadrado. donde el último peldaño de la escalera está directamente arriba del primero.104. p Luego la longitud de la escalera es H 2 +C2 .102: Problema 685.104: Problema 687. tal como en la figura 7. formando así un triángulo rectángulo de catetos que miden H y C. Figure 7. A 4 Figure 7. demuéstrese que r= p 686 Problema En la figura adjunta.102. Si el radio del círculo inscrito al △ABC es r.130 Chapter 7 682 Ejemplo Una escalera en caracol de anchura muy fina enrosca a una columna de altura H y circunferencia C. Hallar el área del anillo sombreado. Hallar r. . hallar BD. B A de radio r es tangente tanto al círculo mayor como a dos lados del cuadrado dentro del cuadrado. [AB] es tangente al círculo interno y AB = 20. Se inscribe un círculo de radio r1 en el △ACD y otro de radio r2 en el △ADB. Sea D el pie de la perpendicular desde el vértice C hasta el lado [AB]. Si el círculo mayor tiene radio R. ◮Resolución: Desenróllese la escalera. Un círculo menor. los círculos son concéntricos. Si [AD] y [BD] son tangentes.101: Problema 684. ◭ Figure 7. 685 Problema Dos círculos de radio 2 y de centros O y P son mutuamente tangentes.100: Ejemplo 682.100. como en la figura 4 7. O A D B A O P C Figure 7. hallar el radio del círculo menor. en el trapecio ACC′ A′ se tiene. 690 Teorema Considérese una serie de paralelas cortando dos rectas. Ahora bien. a : b = (a + c) : (b + d).Proporcionalidad y semejanza 131 7. considérese una sucesión de números racionales r1 . supóngase que CA = r. . A′ B′ ð Ł a Presúmase primero que r = .❑ La recíproca de este último teorema se demuestra de la misma manera que el corolario 651. (a + b) : a = (c + d) : c. ❑ 691 Teorema Una recta paralela a un lado de un triángulo que divida a sus otros dos lados. El resultado se obtiene entonces del teorema 690.105. AB Se tiene que demostrar que C ′ A′ = r. r3 . 4. los divide proporcionalmente. Si r es irracional. . 5. . Demostración: En la figura 7. 1. an : bn = cn : d n . 692 Presunción Si una recta divide a dos lados de un triángulo en segmentos proporcionales entonces la recta es paralela al tercer lado. . convergiendo a r y aplíquese el resultado ya obtenido.106. supóngase que AB = CD.6 Proporcionalidad y semejanza 688 Definición Una proporción es una aseveración acerca de la igualdad de dos razones. Así. también biseca a A′C′ y A′ B′ = C′ D′ por el teorema 652. Por lo tanto. el cociente de dos enteros positivos. también cortarán a la otra en segmentos de igual longitud. 2. 3. 689 Presunción Si a : b = c : d y si n > 0 entonces. (a + b) : (a − b) = (c + d) : (c − d). Se tiene que demostrar que A′ B′ = C′ D′ . (a − b) : a = (c − d) : c. Demostración: En la figura 7. Divídase a C′ A′ en a b ð Ł partes y a A′ B′ en b partes iguales. Luego BB′ biseca a AC. por hipótesis que BB′ AA′ . . b d Las siguientes aseveraciones son fácilmente demostrables. r2 . Luego bCA = aAB. Si en una de estas rectas. escribimos a : b = c : d ⇐⇒ a c = . las paralelas cortan segmentos de igual longitud. el △FAB es isósceles en A. Además BE = CH y AF = CI. Desde H se traza una recta paralela a [AB] intersecando al [CA] en I. Entonces 693 Teorema (Teorema de la bisectriz) En el △ABC sea D la intersección de la bisectriz angular del A BD AB = . 1+ AC DC de donde se destila el resultado. Desde F se traza una recta paralela a [CA] intersecando al [BC] en H. se tiene AF AG CH CI BJ BD BE = = = = = = . b con [BC]. intersecando al [BC] en E. BC FC = . De aquí. se traza una recta paralela al [AC]. . Desde E se traza una recta paralela a [AB] intersecando al [CA] en F. D Chapter 7 A′ F A C B′ B Figure 7. BC = a y CA = b. Sobre D ∈ [AB]. reflex- iones. AB BC AC AB CB CA AB luego BJ = AG = BD = t.106: Teorema 691. ❑ 694 Ejemplo Los lados del △ABC son AB = c. Por el teorema 691. ◮Resolución: Como los lados paralelos de un lado del triángulo cortan a los otros lados en segmentos proporcionales.. dilataciones o contracciones se puede hacer a la una coincidir con la otra. de donde D y J coinciden. ‘ Luego FA = AB.107. Ahora bien. BD AB = 1+ . La constante de dilatación o contracción utilizada se llama coeficiente de homotecia o de semejanza. Figure 7.← → D′ 132A B C ← → D′ ← → D ← → D B C A′ B′ A C′ Figure 7.105: Teorema 690. Desde I se traza una recta paralela a [BC] intersecando al [AB] en J. ya que por ser ángulos alternos internos FBA Así FC = FA + AC = AB + AC. Hallar AG y BJ en términos de BD = t. AB FC = 1+ AC AC y BC BD + DC BD = = 1+ .107: Teorema 693. Si S y T son semejantes se escribe S ∼ T . AC DC ‘ = BAD. como en la figura 7. traslaciones. CD AC Demostración: Trácese [BF] AD. ◭ 695 Definición (Semejanza) Dos figuras S y T se dicen semejantes si mediante una serie de rotaciones. DC DC DC Por lo tanto. ángulos homólogos son congruentes. áreas homólogas llevan una razón de 1 : λ 2 .❑ La siguiente presunción es ahora inmediata. Demostración: Sólo es necesario mostrar la suficiencia. los lados homólogos son paralelos. b B b Superpóngase al △A′ B′C′ sobre el △ABC. [△ABQ] QM Demostración: Se observan cuatro casos. 696 Teorema (Criterio AAA) Dos triángulos son semejantes si y sólo si sus ángulos homólogos son congruentes. Considérese el triángulo △A′ B′C′ en donde A Como los ángulos coinciden. 698 Presunción (Criterio LAL) Dos triángulos son semejantes si y sólo dos de sus lados lados homólogos son proporcionales y los ángulos comprendidos entre estos lados son congruentes. 3. Las siguientes aseveraciones son ahora evidentes.109 a 7. [△ABQ] [△AMP] [△AMQ] [△ABQ] AM QM AB QM ❑ . Así [△ABP] [△AMP] [△AMQ] AB PM AM PM [△ABP] = · · = · · = . 699 Presunción Si dos figuras tienen razón de semejanza 1 : λ . El resultado ahora se sigue del teorema 691. ❑ 697 Teorema (Criterio LLL) Dos triángulos son semejantes si y sólo si sus lados homólogos son proporcionales. entonces: 1. Sin pérdida de generalidad presúmase que las áreas involucradas no son degeneradas. volúmenes homólogos llevan una razón de 1 : λ 3 . 700 Teorema Si las rectas AB y PQ se intersecan en M entonces PM [△ABP] = . segmentos de recta homólogos llevan una razón de 1 : λ . “′ = Bb y C “′ = C.108: Ejemplo 694.112. como en las figuras 7. Demostración: El teorema es resultado inmediato de la presunción 692. 4.D E G Proporcionalidad y semejanza H 133 C A F I Figure 7. “′ = A. 2. CP intersecan a los lados BC. CA. Demostrar que KF KG = . BP. = + + = [△ABC] [△ABC] [△ABC] [△ABC] CC′ AA′ BB′ gracias al teorema 700. Si las áreas de resultantes dentro del △ABC son [△EKM] = R. demuéstrese que √ triángulos √ √los tres [ABC] = ( R + S + T )2 . AB en los puntos A′ .112: Teorema 700.115).P P P P Q 134 Chapter 7 Q M B A M A M B B B A M A Q Q Figure 7. AA′ BB′ CC′ ◮Resolución: Se tiene 1= [△ABP] + [△BCP] + [△CAP] [△ABP] [△BCP] [△CAP] PC′ PA′ PB′ + + . Entonces R S T EM 2 MF 2 PN 2 . C′ respectivamente. ◮Resolución: Obsérvese que △EKM ∼ △MQF ∼ △PMN. Figure 7.109: Teorema 700.110: Teorema 700. Figure 7. FL [△DBL] [△KBL] [△DBL] CL AD [△ACL] [△ACD] [△ACL] GL ◭ 702 Ejemplo Sea P un punto en el interior del △ABC.111: Teorema 700. concurren en el punto M (figura 7.114. . . 701 Ejemplo ABCD es un cuadrilátero convexo tal que DA y CB se intersecan en K. FL GL ◮Resolución: Aplicaciones sucesivas del teorema 700 producen KF [△DBK] [△DBK] [△KBL] CD AK [△ACD] [△ACK] [△ACK] KG = = · = · = · = = . Las semirrectas AP. cada una de ellas paralelas a los lados del △ABC. Demostrar que PA′ PB′ PC′ + + = 1. = = = [ABC] AC2 [ABC] AC2 [ABC] AC2 . B′ . Figure 7. DB y KL se intersecan en F. AB y DC se intersecan en L. AC y KL se intersecan en G. ◭ K D C A B G L F Figure 7. como en la figura 7.113: Ejemplo 701 703 Ejemplo Tres líneas rectas. [△MQF] = S y [△PMN] = T . Si [△ABL] = s. se tiene [△LBC] = s+ s y [△ALC] = 3s.Proporcionalidad y semejanza 135 Luego € EM = R AC. B . y MF = NC. respectivamente. C′ son puntos en BC. por el teorema 700. MF = [ABC] € S AC. Esto conlleva a EM + PN + MF = AP + PN + NC = AC. N B A C′ Figure 7.114: Ejemplo 702. De esta última igualdad se desprende que € € € R S T AC + AC + AC = AC. EM = AP. al cancelar AC y resolver para [ABC] que √ √ √ [ABC] = ( R + S + T )2 . M son las intersecciones de las rectas AA′ y CC′ .115: Ejemplo 703. [ABC] [ABC] [ABC] de donde se colige. CC′ y BB′ . ′ C B AC B A 3 K. [ABC] A causa del paralelismo de rectas. BB′ y AA′ . encuentre el área del △KLM. ◮Resolución: Observe que 1 = [△ABC] = [△ABL] + [△LBC] + [△ALC]. L. B′ . ◭ 704 Ejemplo En el △ABC. PN = [ABC] € T AC. A′ . M E ′ B Q K ′ P B Figure 7.116: Ejemplo 704. 3 3 s + 3s = 1 =⇒ s = . 13 ◭ C C C A P B′ A C′ F M A′ K L G A Figure 7. Luego 13 [△KLM] = [△ABC] − [△ABL] − [△BCM] − [△CAK] = 4 . Si el área del △ABC es 1. Así. CA. 3 13 Con razonamientos semejantes se obtiene [△BCM] = [△CAK] = 3 . AB respectivamente tales que BA′ CB′ 1 AC′ = ′ = ′ = . Igualdad ocurre si y solamente si ABC co-cíclicos en este orden. si AC y BD se cortan en el interior del círculo. Γ3 . Se tiene (OP + r)(OP − r) = OP2 − r2 . de donde PA PC = . Γ2 . De la desigualdad triangular aplicada al △BCE se tiene CE ≤ CB + BE. Si A = B se considera la recta tangente al círculo en A. Si dos de los ejes tienen un punto en común. los triángulos △PAC y △PDB son semejantes. en este orden. Si los círculos son tangentes en A entonces su eje radical es la tangente que los separa. que existe una homotecia directa enviando △ABE a △ADC. si dos de los ejes se confunden.C. E son alineados. y se Por otra parte. se tiene EAC AB AD BD sigue que CE = AC · . Considérense dos círculos de centros O y O. esto es. esto AB CD EB = . como en la figura 7. Entonces se cumple la desigualdad AC · BD ≤ AB ·CD + BC · DA. B.136 Chapter 7 705 Teorema (Ptolomeo) Sea ABCD un cuadrilátero.❑ . o bien paralelos. Se tiene ‘ = BAC ‘ = BDC ‘ = PDB. ← → ← → Igualdad ocurre si y solamente si ABCD es un cuadrilátero cíclico simple. B. se ‘ = π − ADC. PD PB de donde se sigue el resultado. Entonces sus ejes radicales ∆1 . Por lo tanto. este punto yace en el tercer eje también. 707 Teorema (Teorema de los ejes radicales) Considerénse tres círculos Γ1 . ❑ Demostración: 706 Teorema (Potencia de un punto con respecto a un círculo) Considérese un círculo de centro O y radio r y un punto P en el plano. o bien concurrentes. A. Demostración: Considérese cualquier otra recta pasando por P y cortando al círculo en C y D. como en la figura 7. el producto PA · PB depende solamente de P y del círculo y no de la recta. En reemplazando BE y CE por los valores obtenidos. ∆2 . Se tiene CD AD AD ‘ = BAD ‘ y AE = AC . Sea E el punto único en el plano tal que △ABE y △ADC sean directamente semejantes. D son ‘ = π − ABE halla la desigualdad pedida. es. con igualdad AD si y solamente si C. ∆3 son o bien se confunden. como se puede verificar en utilizando el teorema de Pitágoras. de donde BE = AB · . llamada el eje radical ← → de los círculos. Demostración: Un punto en dos de los ejes radicales tiene la misma potencia con respecto a los tres círculos. El conjunto {P ∈ R2 : PO2 − r12 = PO′2 − r22 } ←→ es una recta perpendicular a la recta OO′ .119. ‘ esto es. de rayos respectivos r1 y r2 . Dada cualquier recta pasando por P y cortando el círculo en A y B. ‘ PAC Así.118. también lo hace el tercero. de donde los triángulos △ACE y △ADB son semejantes. ❑ El producto PA · PB se llama la potencia de P con respecto al círculo. Obsérvese que si los círculos se cortan en dos puntos A y B entonces su eje radical es la recta AB. A B . AL = MB = 2. Figure 7. y si BF AE = = 2. 712 Problema En el triángulo agudo △ABC. hallar MA HA . Si AC = 15. BB′ y CC ′ .117: Teorema 705. 14 y 15 unidades. R. El △A′ B′C ′ está dentro del △ABC con lados paralelos al △ABC y a 2 unidades de distancia de los lados de éste. Tarea ← → ← → 708 Problema En la figura 7. C D N C O M E D A L B Figure 7. 715 Problema (Canadá. hallar el radio del círculo. El punto E es el punto de intersección de los segmentos [AD] y [BC]. EF AB CD 714 Problema (AIME. AD = 16 y BD = 12. Dado que EC = 1. Además. concurren en O y que la suma AO BO CO AO BO CO + + = 92. Hallar el área del △A′ B′C ′ . hállese el producto · · . D ← → 709 Problema ABCD es un paralelogramo. Únase [OE] y extiéndase [OE] de tal manera que corte al círculo en C.120: Problema 708. Si E yace sobre [AD] y F sobre [BC]. del △ABC. Sea O el centro del círculo. El punto E está sobre la recta CD más allá de D. 711 Problema El △ABC tiene lados que 13. Demuéstrese que PR = QS y que PR ⊥ QS. Demostrar que 1 1 1 = + . como en la figura adjunta. de la región sombreada cruzada dentro del cuadrado. AC = 8 y ma = 1.121: Problema 713. Hallar el área. Si AB = 11. hállese el área del △ABE. Dado que AA′ . Figure 7. Se traza el segmento [BE]. S los centros de los respectivos cuadrados. BG BF BE O 710 Problema ABCD es un trapecio en el cual AB = 7 y CD = 10. La bisectriz angular de b A corta el segmento [BC] en D. 713 Problema En la figura 7. OA′ OB′ OC ′ OA′ OB′ OC ′ C A E F B Figure 7. considérese la altura [AHA ] y la mediana [AMA ]. CA y AB respectivamente. Q.122: Problema 715. ABCD es un cuadrado.118: Teorema 706. ED FC B E F Figure 7. Demostrar que 1 1 1 = + . [AN] [LC] y [OB] [DM].123: Problema 717.C ′ están en los lados BC. 1971) [DB] es una cuerda de un círculo. 1992) Los puntos A′ . 717 Problema En la figura 7.120. intersecando [AD] en F y la diagonal [AC] en G. cuadrados en cada lado del cuadrilátero ABCD siendo P. 716 Problema Se construye. hallar EF. exteriormente.121.123. AD ∩ BC = E y [AB] [EF] [CD]. △ABC es rectángulo en A y △ADB es rectángulo en D. E es un punto sobre esta cuerda para el cual DE = 3 y EB = 5. B′ . en unidades cuadradas. D C Figure 7.119: Eje radical.E Tarea O C 137 A B O P B O′ L D C D E Figure 7. 125. 2. P. trácese un arco de centro P. ¿De cuántas unidades cuadradas está pintado en azul y amarillo? AE : EF : FB = 1 : 2 : 3. F es el punto medio del segmento [AC] y G es el punto medio del segmento [FC]. amarillo y azul.124: Problema 719. Hallar la razón en que M divide al lado [BC]. Márquese un punto P. Sin cambiar la anchura del compás. donde se presume que las cevianas concurren.127. 718 Problema En el △ABC. . el rectángulo ABCD de área a. Sin cambiar la anchura del compás. con E entre A y F. E y F yacen sobre el [AB]. que será un extremo de la copia. hallar la razón DH : HE.125: Problema 720. 722 Problema En la figura 7. rojo (90 unidades). 720 Problema En la figura 7. ciánico (105 unidades). como en la figura 7. cosa de que la otra punta descanse sobre B. ◮Resolución: Para copiar el segmento [AB] en el segmento congruente [PQ].126: Problema 721. Se satisface CG : GD : DB = 4 : 3 : 2. Se satisface además 719 Problema Un penacho triangular esta coloreado en verde (252 unidades).124. se trazan paralelas a los lados [AC] y [AB] a través de un punto M que yace sobre el lado [BC]. △FDC. póngase una punta sobre P. A F G B C 7. Figure 7. respectivamente.138 Chapter 7 Figure 7. △GEC es isósceles. [CD] y [AD]. Hállese la fracción . magenta (120 unidades). b Q D C M P R B D E A Figure 7. M es el punto medio del a segmento [QR]. La anchura del compás es ahora AB. 5. copiarlo. El perímetro del △ABC es 84. Ajústese la anchura del compás. se observarán los siguientes pasos: 1.126. 2x B F E 3x G H K 4x y 2y 3y D A C Figure 7.127: Problema 722. como en la figura 7. 4. Sea b el área del triángulo △APM. Si[FG] interseca al [ED] en H. 3. Hállese el perímetro del cuadrilátero sombreado DEGF. E es el punto medio del segmento [DC]. Q y R son los puntos medios de los lados [BC]. cada uno de los triángulos △ABC. Además AB = 3AC. Los puntos G y D yacen sobre [CB] con G entre C y D.7 Construcciones con regla y compás Se recogen aquí algunas construcciones fundamentales. El área del paralelogramo resultante es del △ABC. Póngase una punta del compás sobre A. 723 Construcción Dado un segmento de recta. D es el punto medio del segmento [BC]. 5 del área 18 721 Problema En el △ABC. trace el segmento de recta [PR]. 1. Ponga la punta del compás sobre C y el otro extremo sobre B. La construcción funciona porque △PQR ∼ = △ABC ya que PQ = AB. 9. Llámese R a la intersección de los arcos. La intersección de estos últimos dos arcos es el vértice R. siendo ahora la anchura del compás la distancia CB. la anchura del compás ahora siendo AB. se observarán los siguientes pasos: 1. el compás tiene ahora una anchura AB. copiarlo. una recta L′ pasando por R y ← → paralela a L . ‘ es dado y se le quiere copiar en el ángulo QPR. Trácese una recta de P a Q.Construcciones con regla y compás 139 6. póngase el otro extremo del compás sobre B. construir △PQR. 5. ◭ 724 Construcción Dado un ángulo. Se tiene QPR ◭ 725 Construcción Dado un triángulo. 4. Para trazar PQ RS. ‘ ◮Resolución: El ángulo CAB 1. Sin ajustar el compás. 4. Abra el compás a la anchura de la distancia AC y sin ajustarlo. Usando la regla. QR = BC y RP = CA. 8. Escójase un punto Q sobre el arco. 7. ‘ 8. Póngase P en el lugar deseado y trácese una recta a través de P en la dirección deseada. Ponga la punta del compás en C y el otro extremo en A. 3. 2. Con la punta del compás sobre A. Ponga el vértice P en cualquier lugar deseado. 7. Trace un arco con centro P y radio CA cerca de donde se quiera colocar el vértice R. Ponga la punta del compás en A y el otro extremo en B. ← → ← ← → ← → ← → → ◮Resolución: Sean P y Q puntos sobre L . Trácese una recta pasando por R y cortando a PQ en un punto arbitrario que se llamará J. 7. ponga la punta sobre Q y trace un arco cruzando el arco previamente trazado. construir. que será el otro extremo del segmento. con △PQR ∼ = △ABC. 8. 2. Ponga la punta del compás en B y el otro extremo en C. el compás tiene ahora una anchura BC. Llámese Q al punto de intersección de la recta y el arco. ◮Resolución: Dado el triángulo △ABC. 3. Trace un arco con centro Q y radio BC cerca de donde se quiera colocar el vértice R. Sin ajustar el compás. ponga la punta sobre P y trace un arco sobre la recta. Marque un punto Q en el arco. ◭ ← → ← → 726 Construcción Dada una recta L y un punto R no en L . el compás tiene ahora una anchura CA. con regla y compás. ‘ = CAB. 6. copiarlo. póngalo sobre P y trácese un arco sobre la recta. Observe que PQ = AB. 6. Trace un arco con centro P y radio AB cerca de donde se quiera colocar el vértice Q. 5. . e tal manera que PQ = L . Se concluye AB = PQ. → ← → ← 6. 5. en realidad no importa cuanto. 1. ← → ← → ◮Resolución: Se erige sobre el punto K de la recta AB. cortando a RJ en B′ . en ambos rayos terminando en K. 2. Sin cambiar la anchura del compás. Entonces KR ⊥ AB. 5. pero ahora con Q como centro. póngase un punto sobre J y ← → ← → trácese un arco cortando a PQ en A y a RJ en B. ◭ 727 Construcción Construir la mediatriz de un segmento. se observarán los siguientes pasos: 1. cortando al previo arco superior en A y al arco inferior previo en B. AB es la mediatriz de [PQ]. Con el compás abierto un poco más de la mitad de la distancia entre R y J. Ajústese ahora el compás tan ancho como la distancia AB. Sin cambiar la anchura del compás. cortando ← → a AB en P y Q. Sin cambiar la anchura del compás.140 Chapter 7 2. 4. Abra el compás a un poco más de la mitad de la distancia de R a AB. Póngase una punta del compás sobre B′ . ◭ 729 Construcción Construir una perpendicular a una recta que pase por un punto fuera de la recta. 1. [PQ] y las diagonales de un rombo son mutuamente perpendiculares. ← → 2. ◭ 728 Construcción Dada una recta y un punto sobre ella. Sin cambiar la anchura del compás. ◮Resolución: Para trazar la mediatriz del segmento [PQ]. PQ RS. ← → 5. tal como en ← → el segundo paso. erigir una perpendicular a la recta desde el punto. Sin cambiar la anchura del compás. ← → ← → ◮Resolución: Sea R el punto externo a la recta AB. trácense arcos por encima y por abajo del segmento. 3. ponga una punta del compás sobre Q y marque ahora un arco encima K con centro en Q. Se construirá una perpendicular a AB desde R. ′ RS = ‘ La construcción funciona porque la igualdad de los ángulos correspondientes B‘ BJA. trácense arcos por encima y por abajo del segmento con P como centro. Trácese la recta RS ← → ← → 7. Una ahora K con R. Este arco interseca ahora el arco previo con centro en P en el punto R. Con centro en P marque ahora un arco encima de K. 4. 2. Poner una punta del compás sobre R. 3. colóquese una punta del compás sobre R y trácese un arco. Abra el compás a una anchura promedio. La construcción funciona ya que PAQB es un rombo con diagonales [AB]. . Abra el compás a un poco más de la mitad de la distancia entre P y Q. Ensanche el compás hasta casi el doble de su actual anchura. 4. Trácese la recta AB ← → 6. 3. Trácese un arco ahora que cruce el arco existente de centro R en el punto S. Póngase una punta del compás sobre P. marque dos arcos en la recta. una recta perpendicular a AB pasando por K. ← → 6. Note que P y Q equidistan de K. ◭ 731 Construcción Dado un ángulos. Llámese D al punto de intersección. Haga ahora lo mismo poniendo la punta en B. trace n arcos consecutivos sobre el nuevo segmento. 2. Sin alterar la anchura del compás. 6. 3. creando n puntos sobre ella. 9. Trace una recta desde el vértice hasta el punto de intersección de los arcos. Comenzando desde A. su radio. 5. Únanse los puntos correspondientes a lo largo de [AC] y [DB]. el centro de su círculo inscrito y r. ◮Resolución: Dado el △ABC construir I. [QS] es la bisectriz buscada. Con el compás abierto a la anchura BC trace un arco con centro en A debajo de A. QS = QS y por lo tanto AQS ◭ 732 Construcción Construir el incentro e el círculo inscrito de un triángulo dado. Desde el punto A. 4. La intersección de las bisectrices es el incentro I. 4. Llame a la intersección de los arcos el punto S 5. Con el compás abierto a la anchura AC trace un arco centro en C intersecando al arco del paso anterior. 1. 7. 8. ya que QA = QB. Llámese al último punto C. trácense ahora n − 1 arcos consecutivos comenzando en D a lo largo de [DB]. AS = BS. 3. El centro del k-ésimo arco es el k − 1-ésimo punto. intersecando a AB en P y Q. trace dos arcos uno con centro en P y otro con centro en Q. Ponga la punta del compás en A y trace un arco en el interior del ángulo. 1. 141 . ◭ 730 Construcción Dividir a un segmento de recta en n sub-segmentos congruentes. Trace dos arcos con centro en R sobre AB a entrambos lados de R. Usando la misma anchura que la utilizada para trazar arcos en el segmento AC. intersecando al rayo [QP[ en A y al rayo [QR] en B. trace un arco en cada rayo del ángulo. La inclinación exacta de la recta no importa en tanto el ángulo sea agudo. La anchura exacta no importa. ◮Resolución: Para dividir el segmento [AB] en n partes iguales: 1. construir su bisectriz angular. 4. ‘ = BQS. Ponga el compás sobre el vértice Q del ángulo. Ponga la punta del compás en el punto A y abra el compás a una anchura un poco menor que de la longitud n del nuevo segmento de recta. 1 2. Ajuste el compás a una anchura promedio. Trace el segmento [DB]. 6. ‘ ◮Resolución: Se desea bisecar al ángulo PQR. La recta RS es la perpendicular a AB buscada. dibujése una recta en un ángulo agudo con el segmento de recta. Con la misma anchura en el compás. ‘ La construcción funciona porque △QAS ∼ = △QBS. Construya las bisectrices de dos de los ángulos del triángulos utilizando la construcción 731. intersecándose en S. Esto divide a [AB] en n partes iguales.Construcciones con regla y compás ← → ← → 3. ← → ← → 5. ◭ 734 Construcción Dado círculo. 1. 3. La construcción funciona porque △POA es rectángulo en A y △POB es rectángulo en B. 3. 3. ◮Resolución: Dado el △ABC construir O. Trácese el segmento OP. utilizando la construcción 727. Trácese el círculo de centro I y radio r. construir las tangentes al círculo desde el punto. ◮Resolución: 1. constrúyase la mediatriz de cada lado. R′ . Entonces R = OA. ◭ 733 Construcción Construir el circuncentro y el circuncírculo de un triángulo dado. Hállese el punto medio de OP. construya una perpendicular a cualquiera de los lados. 2. Constrúyase un círculo de centro O y radio R − R′.142 Chapter 7 2. ◮Resolución: Considérense círculos de centro O. . Así el radio OA es ← → ← → perpendicular a la recta PA y por lo tanto PA es tangente al círculo de centro O. construir tangentes a este círculo. 1. Escogiendo cualesquiera dos de los lados. El punto de intersección de esta perpendicular y el lado se denominará L. ◭ 735 Construcción Dado un punto fuera de un círculo y un círculo. Esto resultará en dos tangentes externas a los círculos originales. que se denominarán A y B. Póngase la punta del compás en O y el otro extremo en cualquier vértice del triángulo. por ejemplo. utilizando la construcción 729. su radio. A y P son los puntos de tangencia desde P. 4. 4. en A. Desde I. Trácese el círculo con centro M y radio MO. Observe que este círculo interseca al círculo de centro O en dos puntos. utilizando la construcción 727. O′ y de respectivos radios R. Con una regla. 2. Se arguye de la misma manera para el punto B. Utilizando la construcción 735. 2. el centro de su circuncírculo y R. 2. en el círculo. Construya la mediatriz de cada uno de las cuerdas. cámbiense los roles de O y O′ . 3. que existen si OO′ > R − R′. Entonces r = IL. 3. dígase [AB]. hallar su centro. Se desplazan ahora estas tangentes R′ unidades arriba y abajo de los centros. Trácese el círculo de centro O y radio R. trácense dos cuerdas. ◭ 736 Construcción Construir las tangentes mutuas de dos círculos. tan paralelas como sea posible. El punto de intersección de las mediatrices es el centro del círculo. Sea O el punto de intersección de estas dos mediatrices. Presúmase que R > R′ y en el caso contrario. Llámese M a este punto medio. 1. ◮Resolución: Se considera un círculo de centro O y un punto P fuera del círculo. Esto resultará en dos tangentes internas a los círculos originales. ◮Resolución: Dados segmentos de longitud a. 8. 6. 2. ← → 6. Localicése un punto B tal que AB = b. Únase A y C. Utilizando la construcción 735. Usando la construcción 726. 3. b y c. 7. En la misma recta. que se denominará X. localicése un punto A tal que OA = a. Sobre una recta. 3. La construcción funciona gracias al teorema 690. AX = ab. que existen si OO′ > R + R′ . 4. localicése un punto B tal que AB = b. ◮Resolución: 1. En la otro recta. ◭ 738 Construcción Dados un segmento de longitud a y otro de longitud b construir un segmento de longitud √ ab. Se desplazan ahora estas tangentes R′ unidades arriba y abajo de los centros. se quiere hallar x tal que x a = . escójase un punto arbitrario O. que se denominará O.Tarea 143 4. 5. construir un círculo que pase por los dos puntos y tangente a la recta. c b 1. √ 5. ◭ 737 Construcción Construir la cuarta proporcional de tres segmento de recta dados. constrúyase una paralela a [AC] pasando por B. Construya dos rectas suficientemente largas formando un ángulo conveniente. 2. ← → 4. Eríjase una perpendicular a OB en A. construir tangentes a este círculo. . Trácese un semicírculo de centro M y radio MO. Localicése un punto A tal que OA = a. localicése un punto C tal que OC = c. que se denominará X. En una de las rectas. Esta recta intersecará a OC en un punto. Esta recta intersecará al semicírculo en un punto. Se tiene x = CX. 740 Problema Dada una recta y dos puntos del mismo lado de ella. Localicése el punto medio M del segmento [OA]. si la longitud de sus lados es dada. 5. Se construye ahora un círculo de centro O y radio R + R′ . Trácese [BX]. intersecándose en un punto. ◭ Tarea 739 Problema Construir un triángulo equilátero con regla y compás. formando el segmento [AC]. 129.144 Chapter 7 7. como en la figura 7. Se definen entonces las razones trigonométricas seno. 741 Definición Se △ABC un triángulo rectángulo en A. la medida del segmento [AB] es AB = c. la medida del segmento opuesto al vértice se indicará con la minúscula del vértice correspondiente.8 Repaso de Trigonometría C C Cˆ Cˆ a b a b Aˆ Bˆ Bˆ A B c A Figure 7. la del segmento [BC] es BC = a y la del segmento [CA] es CA = b. Así pues.129: Triángulo rectángulo. c B Figure 7. como se indica en la figura 7.128. coseno. secante. tangente. Tanto el ángulo en el vértice V como su medida algebraica se indicará por Vˆ .128: Notación para triángulos. cosecante y cotangente del ángulo Bˆ de la manera siguiente: sen Bˆ = cos Bˆ = tan Bˆ = csc Bˆ = sec Bˆ = cot Bˆ = longitud del cateto opuesto a Bˆ longitud de la hipotenusa longitud del cateto adyacente a Bˆ longitud de la hipotenusa longitud del cateto opuesto a Bˆ longitud del cateto adyacente a Bˆ longitud de la hipotenusa longitud del cateto opuesto a Bˆ longitud de la hipotenusa longitud del cateto adyacente a Bˆ longitud del cateto adyacente a Bˆ longitud del cateto opuesto a Bˆ b a c a b c a b a c c b = = = = = = = = = = sen Bˆ cos Bˆ 1 sen Bˆ 1 cos Bˆ cos Bˆ sen Bˆ Las razones trigonométricas del ángulo Cˆ se definen de manera semejante: sen Cˆ = cos Cˆ = tan Cˆ = csc Cˆ = sec Cˆ = cot Cˆ = longitud del cateto opuesto a Cˆ longitud de la hipotenusa longitud del cateto adyacente a Cˆ longitud de la hipotenusa longitud del cateto opuesto a Bˆ longitud del cateto adyacente a Cˆ longitud de la hipotenusa longitud del cateto opuesto a Cˆ longitud de la hipotenusa longitud del cateto adyacente a Cˆ longitud del cateto adyacente a Cˆ longitud del cateto opuesto a Cˆ = = = = = = c a b a c b a c a b b c = = = = sen Cˆ cos Cˆ 1 sen Cˆ 1 cos Cˆ cos Cˆ sen Cˆ = cos Bˆ = sen Bˆ = cot Bˆ = sec Bˆ = csc Bˆ = tan Bˆ . Dado el triángulo △ABC. 60◦ = y 90◦ = . 743 Teorema (Fórmulas de periodicidad) Sea θ un ángulo arbitrario. cos π 2  − θ = sen θ . son deducidos de inmediato al 6 4 3 2 considerar varios triángulos equiláteros o isósceles. 746 Ejemplo Dado que 3 sen x + 4 cosx = 5. 30◦ = .Repaso de Trigonometría 145 θ sen θ cos θ tan θ 0 0 π 6 π 4 π 3 π 2 1 √2 2 2 √ 3 2 1 √ 3 2 √ 2 2 1 2 0 √ 3 3 1 0 ∞ 1 √ 3 Table 7. Entonces sen π 2  − θ = cos θ . cos(θ + 2nπ ) = cos θ . medido en radianes. cot2 θ + 1 = csc2 θ .1. estas razones trigonómetricas se extienden a un ángulo de magnitud arbitraria mediante las siguientes relaciones. tan π 2  − θ = cot θ . medido en radianes y n ∈ Z un entero arbitrario. 45◦ = . En- tonces sen(θ + 2nπ ) = sen θ . de donde se obtiene el teorema siguiente. π π π π Los valores del seno. . Son utilizados frecuentemente y vale la pena el memorizarlos. 1 + tan2 θ = sec2 θ . Entonces sen2 θ + cos2 θ = 1. encontrar sen x y cos x. Como es sabido. Las funciones seno y tangente son impares: sen(−θ ) = − sen θ .1: Ángulos especiales. 745 Teorema (Fórmulas Pitagóricas) Sea θ un ángulo arbitrario. La función coseno es par: tan(−θ ) = − tan θ . coseno y tangente de 0◦ = 0. 742 Teorema (Fórmulas de simetría) Sea θ un ángulo arbitrario. cos(−θ ) = cos θ .129 se cumple la relación de Pitágoras b 2 + c2 = a 2 . En el triángulo △ABC de la figura 7. tan(θ + nπ ) = tan θ . Consúltese el cuadro 7. 744 Teorema (Fórmulas de complementareidad) Sea θ un ángulo arbitrario. ❑ [△ABC] = 748 Teorema (Fórmulas de la adición) Sean α y β ángulos arbitrarios. Denótese por ha .146 Chapter 7 ◮Resolución: Se tiene 3 sen x + 4 cosx = 5 ⇐⇒ sen x = Utilizando las relaciones pitagóricas se obtiene Å ã 5 − 4 cosx 2 cos2 x + = 1 =⇒ 3 cos2 x + 5 − 4 cosx . sen x = 5 − 4 cosx 5 − 16 3 5 = = . [BHB ] y [CHC ]. 3 3 5 ◭ 747 Teorema (Fórmulas de área) Sea △ABC un triángulo arbitrario. Entonces [△ABC] = ab sen Cˆ bc sen Aˆ ca sen Bˆ = = . [CA] y [AB]. ’C y β = H ’ dejando el resto a cargo del lector. hb y hc las medidas de las alturas [AHA ]. con α = ACH CCB. α + β < π. Denótese por [△ABC] el área del △ABC. tan(α ± β ) = tan α ± tan β . β > 0. 2 2 2 Demostración: Denótese por HA . 3 25 − 40 cosx + 16 cos2 x =1 9 =⇒ 9 cos2 x + 25 − 40 cosx + 16 cos2 x = 9 =⇒ 25 cos2 x − 40 cosx + 16 = 0 =⇒ (5 cos x − 4)2 = 0 =⇒ cos x = 4 5 Además.8. respectivamente. α > 0. [△ABC] = 2 Pero de la figura 7. Luego ab sen(α + β ) 2 = [△ABC] = [△AHCC] + [△CHC B] = ahc sen α hc b sen β + . cos(α ± β ) = cos α cos β ∓ sen β sen α . 1 ∓ (tan α )(tan β ) Demostración: Tan sólo se demostrará que sen(α + β ) = sen α cos β + sen β cos α . Entonces sen(α ± β ) = sen α cos β ± sen β cos α . 2 2 probando una de las fórmulas. chc ca sen Bˆ = . HB y HC los pies de las perpendiculares a los segmentos [BC]. Las otras fórmulas se obtienen en permutando los lados. 2 2 .8 se ve que hc = a sen Bˆ y por tanto. Entonces chc . Cˆ = α + β . respectivamente. En la figura 7. Presúmase ahora que Bˆ > π . Se tiene 2 c2 = (AHC − BHC )2 = (AHC )2 + (HC B)2 − 2(AHC )(HC B) = ˆ b2 − (hc )2 + a2 − (hc )2 − 2(AHC − c)(b cos A) = b2 − (hc )2 + a2 − (hc )2 + 2bc cos Aˆ − 2bAHC cos Aˆ = b2 + a2 + 2bc cos Aˆ − 2(AHC )2 − 2(hc )2 = b2 + a2 + 2bc cos Aˆ − 2b2 = ˆ −b2 + a2 + 2bc cos A. Las otras identidades se obtienen en permutando los lados.8. Demostración: Pitágoras Presúmase primero que 0 < Bˆ ≤ c2 π . lo cual implica ˆ a2 = c2 + b2 − 2bc cos A. b a como se quería demostrar.131.Repaso de Trigonometría 147 lo cual implica que sen(α + β ) = hc hc sen α + sen β = sen α cos β + sen β cos α . como en la figura 7. ❑ . lo cual implica ˆ a2 = c2 + b2 − 2bc cos A. como en la figura 7. ˆ b2 = c2 + a2 − 2ca cos B. ˆ c2 = a2 + b2 − 2ab cos C. ❑ 749 Teorema (Regla del coseno de Al-Kashi) En el triángulo △ABC se observan las siguientes relaciones: ˆ a2 = b2 + c2 − 2bc cos A. Obsérvese que por el teorema de 2 = (AHC + HC B)2 = (AHC )2 + (HC B)2 + 2(AHC )(HC B) = ˆ − AHC ) b2 − (hc )2 + a2 − (hc )2 + 2(b cos A)(c = b2 − (hc )2 + a2 − (hc )2 + 2bc cos Aˆ − 2AHC b cos Aˆ = b2 + a2 + 2bc cos Aˆ − 2(AHC )2 − 2(hc )2 = b2 + a2 + 2bc cos Aˆ − 2b2 = ˆ −b2 + a2 + 2bc cos A. ˆ de donde y así 16[ABC]2 = 4a2 b2 sen2 C. ❑ C b A C hc HC c a a b A B c Figure 7. cos2 Cˆ = 4a2 b2 De aquí se obtiene 16[ABC]2 = 4a2 b2 − (c2 − a2 − b2 )2 = (2ab − c2 + a2 + b2)(2ab + c2 − a2 − b2) = ((a + b)2 − c2 )(c2 − (a − b)2) = (a + b + c)(a + b − c)(c + a − b)(c − a + b) = (2s)(2s − 2c)(2s − 2b)(2s − 2a). Demostración: Se tiene 1 ˆ [ABC] = ab sen C. ˆ sen A sen Bˆ . y al dividir por 16 y tomar raíces cuadradas se obtiene el resultado. Entonces 2 a sen Bˆ = hc = b sen Aˆ =⇒ a b = . sen2 Cˆ = 1 − cos2 C. (c2 − a2 − b2 )2 . = ˆ ˆ sen A sen B sen Cˆ Demostración: Se considerarán por separado los casos cuando Bˆ es agudo u obtuso. hc B HC Figure 7. Por la regla del coseno. como en la figura 7.131: Regla de Al-Kashi. π Presúmase primero que 0 < Bˆ ≤ . ˆ 16[ABC]2 = 4a2b2 (1 − cos2 C).130: Fórmulas de área. 751 Teorema (Regla del seno) En el triángulo △ABC se observan las siguientes relaciones: b a c = .8.148 Chapter 7 750 Teorema (Fórmula de Herón) Si s = a+b+c es el semi-perímetro del △ABC. entonces su área viene dada por 2 » [ABC] = s(s − a)(s − b)(s − c). 2 ˆ Ahora bien. Recuérdese que la sección áurea τ satisface √ τ 1 1+ 5 τ > 0. . 5 5 ◮Resolución: Considérese un pentágono regular ABCDE.133. Erigiendo una perpendicular desde F hasta [AB]. Sea x la longitud de uno de sus lados.131. como FCE 3 5 5 ‘ = FAB ‘ = π . se tiene x BA t x 1 FA x = =⇒ = =⇒ x = t x =⇒ = τ . Esto quiere decir que Ahora bien. = =⇒ τ = . sen Cˆ π Presúmase ahora que Bˆ > . Como [AC] [DE] . = ˆ sen A sen Bˆ obteniendo nuevamente el resultado. el pentagrama regular obtenido al unir alternadamente los vértices del pentágono. FC CE x t +x 1 + t t t ‘ = CED ‘ y BCA ‘ = FCE. a sen Bˆ = b sen A =⇒ b a . en observando que τ 2 = τ + 1. Así FC = CD = x. y a su vez. se deduce del △FAB. como en la figura 7. Poniendo t = AF y observando que CE = CA = x + t. a sen(π − B) Pero por las fórmulas de la adición ˆ = a sen π cos Bˆ − sen Bˆ cos π = a sen Bˆ a sen(π − B) y así. τ 1+τ 2 ‘ = CED ‘ y por lo tanto Denótese por F el punto de intersección de [AC] and [BE]. cos  τ 2 2π π τ2 τ2 − 2 τ − 1 = 2 cos2 − 1 = 2 −1 = −1 = = = 5 5 2 2 2 2 Cabe además notar que cos √ 5−1 . 4 √ Å ã 2π 3π 1− 5 2π = cos π − = . ‘ = 3π y luego ABF FCE 5 5 √ x x 1+ 5 π τ 2 cos = = = = . FCE △FCD ≡ △DEC. Se tiene 2 ˆ ˆ = hc = b sen A. = − cos 5 5 5 4 ◭ 753 Ejemplo Considérese un pentágono regular inscrito en un círculo de radio 1. Obsérvese que △FAB es isósceles y semejante al △FCE.Repaso de Trigonometría 149 Que estas dos cantidades son iguales a c se observa al considerar. ‘ se tiene BCA ‘ = FCE ‘ = CED ‘ = 1 · 3π = π . ❑ 752 Ejemplo Hállese el valor exacto de cos 2π π y cos . como en la figura 7. la altura BHB . 5 t 2t 2 4 De aquí se sigue. por ejemplo. usando la regla de los cosenos. Nótese que tal triángulo es isósceles.132: 752. Así pues. ◮Resolución: El área del pentágono es 5 veces el área de cualquier triángulo formado por el centro del pentágono y dos de sus vértices consecutivos. 2π sen = 5 … 1 − sen2 de donde el área del pentágono es à 2π = 5 Ç√ 1− 5−1 4 å2 √ 5+ 5 . Calcule el perímetro del pentágono. y viendo que los lados iguales miden 1. 2 5 Por el ejemplo 752. = 8 √ 25 + 5 5 .133: 753. el área del pentágono es medida 1 y con el ángulo entre estos lados de medida 5 2π 1 5 · · 1 · 1 · sen . 2. con ambos lados iguales de 2π . Ejemplo Figure 7. se obtiene. = x = 1 + 1 − 2(1)(1) cos 5 4 2 2 ◭ C A P B Figure 7. Considerando el triángulo isósceles formado al unir los extremos de este lado con el centro del pentágono. 16 Para obtener el perímetro del pentágono. Calcule el área del pentágono. 3. .134: Teorema de la bisectriz angular. sea x la medida de uno de sus lados.B 150 Chapter 7 C A F D E Figure 7. Calcule el área del pentagrama. Ejemplo 1. que € Ç√ å √ √ 5 − 2 π 5 − 1 5 5− 5 2 2 2 = 2−2 =⇒ x = . Hallar el área del cuadrilátero curvilíneo formado por la intersección de los arcos de círculo. Véase la figura 7. AO = y AOE = porque el arco BD 2 6 AFE es así π π 1 (1) = . PC CA sen PAC ‘ Demostración: Úsese la ley de los senos en el △APB: BP AB = ‘ ‘ sen BAP sen APB y en el △APC: AC PC = . Se trazan cuatro cuartos-de-círculo con centros en los vértices del cuadrado y de radio 1. siendo radios de círculos unitarios. ◮Resolución: Obsérvese √ que AF = AE = 1. PC ‘ AC sen PAC ❑ 755 Ejemplo El cuadrado ABCD tiene lado 1. se tiene sen APB Ya que APB ‘ BP AB sen BAP = . si O es el centro de la 2 ‘ π ˆ es trisecado por [AE] y [AF]. se obtiene Ahora bien. Entonces ‘ BP BA sen BAP = . ‘ ‘ sen PAC sen CPA Luego ‘ sen APB ‘ BP AB sen BAP/ = .135. El área del sector circular diagonal [AC]. 8 3 . de | sen x| = 2 € p √ 1 − cos π6 π 2− 3 sen = = . 2 6 12 … 1 − cos2x . ‘ Cancelando ‘ + CPA ‘ = π . PC ‘ sen CPA ‘ AC sen PAC/ ‘ = sen CPA.Repaso de Trigonometría 151 754 Teorema (Forma trigonométrica del teorema de la bisectriz) En el △ABC sea P un punto en la recta BC. 2 2 12 8 8 El área buscada es finalmente Ç 4 ◭ π −2· 12 å √ 3−1 π √ = − 3 + 1. 12 2 2 El área del △AEO es √ √ p √ √ 1 2 π 2 2− 3 3−1 · · 1 · sen = = . Además. b = 3a. 3 De igual manera. 3 3 π π π π + a) + sen2 ( + c) − 2 sen( + a) sen( + c) cos(b). 3 de donde BP = 2R sen(3a) sen(c)/ sen( π − a) . Demostración: Póngase A BC = 2R sen(3a). Por la ley de los cosenos.3) . PR2 = BP2 + BR2 − 2BP · BR cos(b) π π π π = 64 sen2 (a) sen2 (c)[sen2 ( + a) + sen2 ( + c) − 2 sen( + a) sen( + c) cos(b)] . 3 BR = 8R sen(c) sen(a) sen( π + c) . se colige que PR = RQ = QP as claimed.2) (7. BP/ sen(c) = = BC/ sen(π − b − c) = 2R sen(3a)/ sen(b + c) π 2R sen(3a)/ sen( − a).136: Teorema 756. 756 Teorema (Morley) Los pares adyacentes de las trisectrices de los ángulos de un triángulo siempre se encuentran en los vértices de un triángulo equilátero.152 B A Chapter 7 Z Q O P X E R Y B F A D C Figure 7. Por la ley de los senos. Bb = 3b y Cb = 3c. 3 Combinando esto con la identidad anterior. sen(3a) = 4 sen(a) sen( π π + a) sen( − a) 3 3 y así se obtiene BP = 8R sen(a) sen(c) sen( π + a) . Sea R el circunradio del △ABC. Como esta expresión es simétrica en a. 3 3 3 3 Pero como ( se sigue que sen2 (b) = sen2 ( π π + a) + ( + c) + b = π . ❑ Tarea (7. b y c. Aplicando ahora dicha ley al △BPC. 3 3 3 3 de donde PR = 8R sen(a) sen(b) sen(c) . Figure 7.135: Ejemplo 755. 2 760 Problema Si α . Demostrar que 1 sen b A = 1 b sen B + 1 b sen C . La intersección resulta en un octágono convexo de lados congruentes. Hallar R. Demúéstrese que 1 − cos α + cos β + cos γ = 4 sen α β γ cos cos . . Determine la razón de la diagonal mayor a la diagonal menor de tal manera que el octágono sea regular. Dado que tan(α − β ) + tan(β − γ ) + tan(γ − α ) = 0. tal como en la figura 7. 2 2 2k 2 k=0 771 Problema (AHSME. d ′ . entonces estas dos medianas son perpendiculares. 2 2 2 2 2 2 761 Problema (E˝otv˝os.138: Problema 764. Dedúzcase Sn = Xn 1 cos(kθ ) cos((k + 1)θ ) k=1 Figure 7. A 763 √ Problema Un rombo tiene lados de longitud s y diagonales de longitud d. Q P B A′ C Figure 7. β .Tarea 153 757 Problema Dos círculos de radio R son tangentes externamente.139: Problema 771. demostrar que 1 α β γ (sen )(sen )(sen ) ≤ .137. ä α − sen α . β . 2 2 2 8 ¿Cuándo se verifica la igualdad? 762 Problema Sean α . hallar la medida del ángulo agudo entre los lados del rombo. Si s= dd ′ . 3n si y solamente si △ABC es un triángulo rectángulo. Ambos círculos 765 Problema Demostrar que son también tangentes internamente a un triángulo rectángulo de catetos de longitud 3 y 4. β . 1991) El triángulo equilátero △ABC se ha doblado de tal man- era que ahora el vértice A descansa sobre el punto A′ del segmento [BC]. 758 Problema El △ABC es tal que la altura desde A mide lo mismo que la suma de las longitudes de las alturas desde B y C. γ reales tales que α + β + γ = π . γ son los ángulos internos de un triángulo. 2 2 2 y que tan C B α β β γ γ α tan + tan tan + tan tan = 1. como en la figura 7. 772 Problema Demostrar que en el △ABC se satisface sen2 Aˆ + sen2 Bˆ = sen2 Cˆ Figure 7. tan 51◦ − cot 51◦ = 2 tan 12◦ . Si BA′ = 1 y A′C = 2. 764 Problema Dos rombos congruentes son sobrepuestos de tal manera que la diago- nal mayor del uno yace sobre la diagonal menor del otro. 759 Problema Demostrar que si un triángulo de área A es tal que el producto de las longitudes de dos sus medianas es 768 Problema Demostrar que 3 A. β . γ son los ángulos internos de un triángulo arbitrario. los ángulos internos de un triángulo.139. 1897) Si α . hállese la longitud del doblez PQ. γ . A 766 Problema Sean α . (1 + tan 1◦ )(1 + tan 2◦ ) · · · (1 + tan 44◦ )(1 + tan 45◦ ) = 223 . 2 2 2 2 2 2 767 Problema Calcúlese tan p − tan q. demostrar que el triángulo es isósceles. demostrar que α β γ α β γ cot + cot + cot = cot cot cot .137: Problema 757. con θ ∈ R. 769 Problema Demostrar que Xn−1 3k sen3 Ä α 3k+1 ä = 1 4 Ä 3n sen k=0 770 Problema Probar que cot x − 2 cot 2x = tan x y que Xn 1 1 α α tan k = n cot n − 2 cot 2α . o la longitud o norma del segmento [AB]. esto es.9 Repaso de Geometría Analítica En esta sección se desarrollarán las ecuaciones canónicas de círculos y rectas en el plano. es la cantidad AB = d (x1 .154 Chapter 7 773 Problema Sean α . y1 ). a = 5.140. β ángulos agudos. π . b = 4 y cos (A − B) = cos Cˆ = 774 Problema Hallar el valor exacto de sen2 1◦ + sen2 2◦ + · · · + sen2 90◦ . α = AB. Demostrar que 775 Problema Sea x + y + z = 90◦ . y0 ) C(x2 . 780 Definición El plano cartesiano R2 es el conjunto de pares ordenados R2 = {(x. B(x2 .141: El círculo. y1 ). y2 ) en el plano.” Por tanto se utiliza la conjunción “y” y no “e. y1 ) |x2 − x1 | (x0 . β = AC. 7. Construyendo los segmentos [CA] y sec BC con C = (x2 . γ = AD. Gracias a la fórmula de la distancia se deduce el teorema siguiente. como en la figura 7. Aquí x es la abscisa y3 y la ordenada. y2 ) R |y2 − y1 | A(x1 . probar que a−b sen A − senB = . y ∈ R}. y) : x ∈ R. Determínese 31 . y2 ) puntos en el plano cartesiano. y1 ). (x2 . demostrar que 32 Considérense las distancias 1 1 1 = + . Figure 7. 776 Problema Si a + b + c = 36◦ . la distancia de A a B.” (7. Un punto del plano A es un par ordenado de números reales A = (a1 . 3 Se lee la letra como “ye” y no “i griega. y2 ) = » (x1 − x2 )2 + (y1 − y2 )2 . demostrar que tan 5a + tan 5b + tan 5c − (tan 5a)(tan 5b)(tan 5c) = 0. La distancia euclídea entre A y B. 1 and that c = 6.140: Distancia entre dos puntos. B = (x2 .4) . Esto motiva la siguiente definición: 781 Definición Sean A(x1 . a+b sen A + senB sen2 α + sen2 β = sen2 (α + β ). se puede hallar la longitud del segmento [AB]. Considérese dos puntos A = (x1 . y1 ) Figure 7. en utilizando el Teorema de Pitágoras: » AB2 = AC2 + BC2 =⇒ AB = (x2 − x1 )2 + (y2 − y1 )2 . Demostrar que 777 Problema En el △ABC. y1 ). a2 ). 2 si y sólo si α + β = ’ 778 Problema Si en el △ABC. B(x2 . 8 779 Problema Sea ABCDEFG un heptágono regular. α γ β (tan x)(tan y) + (tan y)(tan z) + (tanz)(tan x). y recorre paralelamente al eje de x. y) (x1 . y1 ) x − x1 x2 − x1 Figure 7. 785 Definición La ecuación canónica de una recta vertical es x = a. B. El conjunto {(a.141.143: Recta horizontal.❑ 783 Definición Tres puntos A. (7. y2 ) y − y1 (x. y0 ) es (x − x0 )2 + (y − y0)2 = R2 . Sean a y b constantes reales. Esto conlleva a la siguiente definición. en donde b ∈ R es una constante. y) pertenece a un círculo de radio R > 0 si y sólo si su distancia al centro del círculo es R. (x0 . y recorre paralelamente al eje de y. . en donde a ∈ R es una constante. Se determinará ahora la ecuación de una recta inclinada. y0 ) » (x − x0 )2 + (y − y0)2 = R = R . 784 Definición El punto B está entre los puntos A y B si los tres puntos son colineales y AB + BC = AC.144: Teorema 786. Figure 7. La ecuación canónica de una recta horizontal es y = b.Repaso de Geometría Analítica 155 782 Teorema La ecuación cartesiana canónica de un círculo de radio R > 0 y centro (x0 . y2 − y1 (x2 . el conjunto {(x. Véase la figura 7. y).142: Recta vertical. De igual manera. Figure 7.C son colineales si los tres están la misma recta. b) ∈ R2 : x ∈ R} tiene ordenada constante. y) ∈ R2 : y ∈ R} tiene abscisa constante.5) Demostración: El punto (x. (x − x0 )2 + (y − y0)2 = R2 obteniendo el resultado. Luego se requiere ⇐⇒ ⇐⇒ ⇐⇒ d (x. 156 Chapter 7 786 Teorema La ecuación cartesiana de toda recta no vertical en el plano es de la forma y = mx + k, en donde m ∈ R y k ∈ R son constantes. Recíprocamente, los puntos del plano ligados por la ecuación y = ax + b, en donde a, b son constantes reales, forman una recta. Demostración: Si la recta fuere paralela al eje de x, esto es, si fuere horizontal, entonces su ecuación sería de la forma y = b, end donde b es una constante real y entonces se puede tomar m = 0 y k = b. Considérese ahora una recta no paralela a ninguno de los ejes, como en la figura 7.144, y pertenezcan los puntos (x, y), (x1 , y1 ), (x2 , y2 ) a dicha recta. De triángulos semejantes se observa que y2 − y1 y − y1 = , x2 − x1 x − x1 y masajeando un poco se colige que Å y= de donde se puede tomar m= ã ã Å y2 − y1 y2 − y1 x − x1 + y1 , x2 − x1 x2 − x1 ã Å y2 − y1 y2 − y1 , k = −x1 + y1 . x2 − x1 x2 − x1 Recíprocamente, considérense números reales x1 < x2 < x3 . Póngase P = (x1 , ax1 + b), Q = (x2 , ax2 + b) y R = (x3 , ax3 + b), que pertenecen a la gráfica de ecuación y = ax + b. Se demostrará que d P, Q + d Q, R = d P, R , esto es, los puntos P, Q y R son colineales. Como los puntos P, Q, R son arbitrarios, se deduce pues que la gráfica de la ecuación y = ax + b es una recta. En efecto, se tiene » p p d P, Q = (x2 − x1 )2 + (ax2 − ax1 )2 = |x2 − x1 | 1 + a2 = (x2 − x1 ) 1 + a2, » p p (x3 − x2 )2 + (ax3 − ax2 )2 = |x3 − x2 | 1 + a2 = (x3 − x2 ) 1 + a2, » p p d P, Q = (x3 − x1 )2 + (ax3 − ax1 )2 = |x3 − x1 | 1 + a2 = (x3 − x1 ) 1 + a2, d Q, R = de donde d P, Q + d Q, R = d P, R . ❑ y2 − y1 en el teorema 786 es la pendiente o gradiente de la recta que pasa por (x1 , y1 ) y x2 − x1 (x2 , y2 ). Como y = m(0) + k, el punto (0, k) es el intercepto del eje de y de la recta que pasa por (x1 , y1 ) y (x2 , y2 ). 787 Definición La cantidad m = ☞Una recta horizontal tiene pendiente 0. Una recta vertical tiene pendiente infinita. De las figuras 7.145 y 7.146 se puede percatar que si la recta L tiene ecuación cartesiana L : y = mx + k y si θ es el ángulo que L forma con la parte positiva del eje de x, entonces m = tan θ , Si θ = π , entonces m = ∞. 2 θ ∈ [0; π [. (7.6) ← → ← → 788 Definición Sean A y B puntos distintos en el plano. La dirección de la recta AB es el ángulo que AB forma con la parte positiva del eje de x. Repaso de Geometría Analítica 157 Las observaciones anteriores proporcionan de inmediato el siguiente teorema. 789 Teorema Dos rectas son paralelas si y solamente si tienen la misma pendiente. La condición para perpendicularidad es un poco más difícil de demostrar. 790 Teorema Dos rectas inclinadas son perpendiculares si y sólo si el producto de sus pendientes es −1. 1 Demostración: Se quiere demostrar que L1 : y = mx+k es perpendicular a L2 : y = m1 x+k1 si y sólo si m1 = − . m Refiérase a la figura 7.147. Ya que el trasladar rectas no afecta el ángulo entre ellas, se podrá asumir sin pérdida de generalidad que tanto L1 : y = mx + k como L2 : y = m1 x + k1 se intersecan en (0, 0), en cuyo caso k = k1 = 0. Ahora bien, la recta y = mx se encuentra con la recta vertical x = 1 en (1, m) y la recta y = m1 x se encuentra con esta misma vertical en (1, m1 ). Si las rectas fuesen perpendiculares el Teorema de Pitágoras daría (m − m1 )2 = (1 + m2) + (1 + m21), que simplificando resulta en mm1 = −1. La recíproca se obtiene de la recíproca del Teorema de Pitágoras. ❑ y = mx θ θ • (1, m) • (1, m1 ) y = m1 x Figure 7.145: Ángulo con el eje de x. . Figure 7.147: Teorema 790. Figure 7.146: Ángulo con el eje de x. . . Se desarrollará ahora una serie de resultados que culminarán en la demostración de la desigualdad triangular en el plano. 791 Lema (Desigualdad de Cauchy) Sean a, b, x, y números reales. Entonces (ax ± by)2 ≤ (a2 + b2)(x2 + y2 ), con igualdad si y sólo si (a, x) y (b, y) son proporcionales. Demostración: Se tiene (a2 + b2 )(x2 + y2 ) − (ax ± by)2 = a2 x2 + a2 y2 + b2 x2 + b2 y2 − a2 x2 ∓ 2axby − b2y2 = a2 y2 + b2 x2 ∓ 2abxy = (ay ± bx)2 ≥ 0. 158 Chapter 7 a x = , esto es (a, x) es proporcional a (b, y). Si b = 0, b y entonces ay = 0x, y, o bien a = 0 o bien y = 0. En uno y otro caso es Igualdad ocurrirá si y sólo si ay = ±bx. Si by = 0, entonces (a2 + b2 )(x2 + y2 ) = (ax ± by)2, y se obtiene el resultado. Si y = 0 es también claro el resultado. ❑ 792 Lema Póngase A = (a1 , a2 ), B = (b1 , b2 ) y C = (c1 , c2 ). Entonces A, B y C son colineales si y sólo si (c2 − b2 , b2 − a2 ) y (c1 − b1, b1 − a1 ) son proporcionales. Demostración: =⇒ Pase la recta de ecuación y = mx + k por A, B y C. Luego a2 = a1 x + k, b2 = b1 x + k, c2 = c1 x + k, y así b2 − a2 = m(b1 − a1 ), c2 − b2 = m(c1 − b2 ) =⇒ (c2 − b2 , b2 − a2 ) = m(c1 − b1 , b1 − a1 ), estableciendo la proporcionalidad. ⇐ Supóngase ahora que (c2 − b2, b2 − a2 ) y (c1 − b1 , b1 − a1 ) son proporcionales, dígase (c2 − b2 , b2 − a2) = t(c1 − b1, b1 − a1 ). Hay dos casos, o bien 0 = c1 − b1 = b1 − a1 en cuyo caso c1 = b1 = a1 y A, B, C están alineados en la misma recta vertical al tener todos la misma abscisa, o bien uno de entre c1 − b1 y b1 − a1 no es cero. Súpongase que c1 − b1 = 0, siendo el caso b1 − a1 = 0 de manejo idéntico. Se tiene pues c2 − b 2 = t, c1 − b 1 b2 − a2 = t(b1 − a1) Luego los puntos B y C pasan por la recta de pendiente t, dígase y = tx + k. Cuando x = b1 , deberá ser y = b2 y así b2 = tb1 + k =⇒ k = b2 − b1t. Luego B y C están sobre la recta de ecuación y = tx + b2 − b1t. Falta ahora demostrar que A también está en esta recta. Pero para x = a1 se tiene y = a1t + b2 − b1t = b2 − t(b1 − a1 ) = b2 − (b2 − a2) = a2 , que quiere decir que (a1 , a2 ) = A también está sobre la recta, terminando la demostración. ❑ 793 Teorema (Desigualdad del triángulo) Sean A, B, C puntos en el plano (no necesariamente colineales). Entonces AB + BC ≥ AC. La igualdad ocurre si y sólo si los tres puntos son colineales. Tarea 159 Demostración: Póngase A = (a1 , a2 ), B = (b1 , b2 ) y C = (c1 , c2 ). Por el lema 791, » (c1 − b1)(b1 − a1) + (c2 − b2 )(b2 − a2 ) ≤ ((c1 − b1 )2 + (c2 − b2)2 )((b1 − a1 )2 + (b2 − a2 )2 ). Igualdad ocurre si y sólo si (c2 − b2 , b2 − a2 ) y (c1 − b1 , b1 − a1 ) son proporcionales, y en vista del lema 792, igualdad ocurre si y sólo si A, B y C son colineales. Ahora bien, AC2 = (c1 − a1 )2 + (c2 − a2 )2 = (c1 − b1 + b1 − a1)2 + (c2 − b2 + b2 − a2 )2 = (c1 − b1 )2 + 2(c1 − b1 )(b1 − a1 ) + (b1 − a1 )2 + (c2 − b2 )2 + 2(c2 − b2 )(b2 − a2 ) + (b2 − a2 )2 = = (c1 − b1 )2 + (c2 − b2 )2 + 2(c1 − b1 )(b1 − a1 ) + 2(c2 − b2 )(b2 − a2 ) + (b1 − a1 )2 + (b2 − a2 )2 » (c1 − b1 )2 + (c2 − b2 )2 + 2 ((c1 − b1)2 + (c2 − b2)2 )((b1 − a1 )2 + (b2 − a2 )2 ) + (b1 − a1 )2 + (b2 − a2 )2 » 2 » (c1 − b1 )2 + (c2 − b2 )2 + (b1 − a1 )2 + (b2 − a2 )2 = (AB + BC)2, ≤ demostrando el teorema. Véanse las figuras 7.148 y 7.149. ❑ B A B C C A Figure 7.149: AB + BC > AC. Figure 7.148: AB + BC = AC. Tarea 794 Problema Osama la cucaracha comienza a viajar desde el punto (−1, −1) y quiere llegar al punto (2, 1). En cada cuadrante y también, en los ejes, se mueve a razón de 1 de distancia por minuto, excepto en el segundo cuadrante, en donde se mueve a razón de media unidad de distancia por minuto. ¿Qué ruta deberá tomar Osama para minimizar el tiempo del recorrido? ¡La respuesta no es una recta de (−1, −1) a (2, 1)! puntos en el plano, entonces p (a + c)2 + (b + d)2 ≤ p a2 + b2 + p c2 + d 2 . Ocurre igualdad si y sólo si ad = bc. 795 Problema (Desigualdad de Minkowski) Demuéstrese que si (a, b), (c, d) son 796 Problema (Generalización de la desigualdad de Minkowski) Sea ninguno de encuentre el valor de t y Lt que satisface las condiciones estipuladas. 1 ≤ k ≤ n sobre los ejes. Considérese Sn = min X n p P ➍ Lt es paralela a la recta de ecuación x − 2y − 6 = 0. → − u → − u A → − − u +→ v − −2→ u → −v B → − u → −v C 1→ − u 2 Figure 7. el vector AB asociado al segmento dirigido [AB] es 2 3 − → 6 b1 − a1 77 AB = 64 5. D = (2. es un entero y hállese cual lo es. 2 . En los casos siguientes . . para no importa qué valor de t? 7. 1).160 Chapter 7 los puntos del plano (ak . 1991) Sea P = {a1 . S3 . . . B = (3. asóciese la recta Lt de ecuación !2 Xn bk Lt : (t − 2)x + (t + 3)y + 10t − 5 = 0.150: Suma de vectores. 2). 4). a2 .151: Multiplicación escalar de vectores.10 Vectores − → 799 Definición Dados dos puntos A(a1 . k=1 en donde el mínimo es tomado sobre todas las particiones P. 3). ➏ ¿Existe acaso un punto (x0 . 3). k=1 ➊ Lt pasa por (−2. . a2 ) y B(b1 . 1 ➎ Lt es normal (perpendicular) a la recta de ecuación y = − x − 5.Sn . Por ejemplo. . . b2 − a2 − → − → Obsérvese también que AB = −BA. 4 (2k − 1)2 + a2k . 0 < a1 < a2 < · · · < an < 17. 800 Ejemplo Dos segmentos diferentes pueden asociarse al mismo vector. 797 Problema (AIME. . . . Demuéstrese que exactamente una de entre S2 .an } una colección de puntos con ➌ Lt es paralela al eje de y. a2 an a1 = = ··· = . entonces 2 3 − → 6 37 −→ AB = 64 75 = CD. bk ). Figure 7. b2 ) en el plano. C = (0. si A = (1. . b1 b2 bn ➋ Lt es paralela a al eje de x. . . y0 ) que pertenece a todas las Lt . Entonces Xn p à a2k + b2k ≥ !2 Xn ak k=1 + k=1 Igualdad ocurre si y sólo si 798 Problema Al parámetro real arbitrario t. → − − −c .→ siguientes propiedades: . La multiplicación de vectores por un escalar tiene interpretación semejante. Se notarán aquí. dirección (en las coordenadas del vector) y sentido (desde el punto inicial hasta el punto final). b la instrucción es “desplazarse a unidades en el eje de x.150. 804 Teorema (Regla de Chasles vectorial) Dados tres puntos arbitrarios A.151.C en el plano. − → − → − → AB + BC = AC. u = 64 λ→ 5 λ u2 − −v se busca Nótese que geométricamente la suma y el producto de vectores se interpreta de manera siguiente. y b unidades en el eje de y. b . al apelar a las coordenadas de los vectores. Véase la figura 7. u2 y el escalar λ ∈ R. u +→ 4 5 u 2 + v2 y la multiplicación de vectores por un escalar 2 3 6 λ u1 7 − 7. vectores en el plano y sean α y β escalares. Entonces se cumplen las 805 Teorema (Álgebra de vectores) Sean → a .Vectores 161 ☞Se interpreta pues la noción de vector como un conjunto de “instrucciones”: si el vector 2 3 6 a7 → − u = 64 75 .” Esto es. La demostración es inmediata. se define la suma de vectores 3 2 3 2 3 u + v2 7 → − −v = 66 1 7. 2 3 6 07 → − 7 801 Definición El vector cero es 0 = 6 4 5. Finalmente. Lo arriba indicado pone en evidencia la Regla de Chasles. un vector es un objeto que nos indica magnitud (norma). Véase la figura 7. Luego se busca al (segmento único) [BC] tal que v = BC. 2 803 Definición Dados vectores v → −v = 66 1 77 . B. para futura referencia. 4 5 v2 6 u1 7 → − u = 64 75 . la suma se obtiene por la regla del paralelogramo → − → − → → − −v = − u +→ AB + BC = AC. Para sumar → u a→ − → → − → − → − un segmento arbitrario [AB] tal que AB = u . algunas propiedades del álgebra vectorial. 0 802 Definición Se llamará escalar a todo número real λ ∈ R. . .9) → − − − − − − ∃ −→ a tal que → a + (−→ a ) = (−→ a )+→ a = 0 (7.” . Si A = B. esto es. . De manera semejante.10) → − → − − − a +α b α (→ a + b ) = α→ (7. si y = mx + k es una recta cartesiana. ni porvenir. la recta afín es una recta cartesiana con pendiente . no tengo edad. A = (a1 .14) − − − −v || . 4 5 4 5 4 5 y m k 4 Y por eso canta: “No soy de aquí.15) → − u → − → − − − Si → u = 0 .12) − − a = α (β → (αβ )→ a) (7. Si u1 = 0. Obsérvese que si λ ∈ R. ni soy de allá. y = a2 + tu2. y) ∈ R2 : x = a1 + tu1. La recta afín con vector director 808 Definición Sea → u = 0 . la − → ← → ← → dirección del vector AB es la dirección de la recta AB.4 807 Definición A un vector de norma 1 se le llama vector unitario. entonces y gracias a la desigualdad del triángulo.7) → − → − − − − − (→ a + b )+→ c =→ a +( b +→ c) (7. Si u1 = 0. π [ que la recta AB hace con el eje de x. ||→ u +→ u || ≤ ||→ u || + ||→ (7. → − − − − u : λ ∈ R} y sea A ∈ R2 . entonces (x − a1 ) x − a1 = t =⇒ y = a2 + u2 . entonces el vector → tiene norma 1 y la misma dirección del vector → u . El vector nulo 0 tiene − || u || norma 0 y no posee dirección.13) • Asociatividad • • • Distributividad • Distributividad • þþ− þþ þþ →þþ − → 806 Definición La norma o longitud del vector AB es simplemente þþABþþ = » (b1 − a1)2 + (b2 − a2)2 = AB.11) − − − (α + β )→ a + β→ a a = α→ (7. 793 ☞ − − ||λ → u || u || = |λ | ||→ (7. u1 u1 u2 esto es.162 Chapter 7 • Conmutatividad → − → − − → − a + b = b +→ a (7. u1 entonces 2 3 2 3 2 3 6 x7 6 1 7 6 07 6 7 = 6 7 t +6 7 . la recta afín arriba definida es vertical. el ángulo θ ∈ [0. ya que x es constante. Póngase R→ u = {λ → 2 3 6 u1 7 → − u = 64 75 y pasando por A es el conjunto de puntos en el plano u2 − A + R→ u = {(x. t ∈ R}. a2 ).8) → − − → − − → − a + 0 =→ a + 0 =→ a (7. Supóngase. = XB AX Los puntos A. → − − −v tiene el mismo sentido que el vector → − 810 Definición Dado un vector → u = 0 . X = A. Cuando X → ±∞. Además AX = AP ⇐⇒ AX + PA = 0 ⇐⇒ PX = 0 ⇐⇒ X = P. si X está estrictamente a la izquierda de A (y por consiguiente. La medida algebraica será particularmente útil al considerar razones de longitudes de segmentos. y se puede tomar al vector 64 75 como su vector director. entonces AX > 0.152. el punto medio del segmento [AB].16) La relación de Chasles muestra que AA = AB + BA = AB − AB = 0.17) . ← → Sea X ∈ AB. XB < 0. para cualquier punto A. XB > 0. Nótese que si X = M. entonces AX = AB y XB .Vectores 163 2 3 6 17 esto es. se dice que el vector → u si existe algún escalar − −v . siendo positiva la dirección de AB. sin perdida de generalidad. Se denotará esta por AB− u . fijos y X variando sobre la recta AB. entonces AX = XB. − u es paralelo a → u → paralalos si existe un escalar λ ∈ R tal que → u = λ→ ☞ →−0 es paralelo a todo vector. La gráfica de la función Si X = B. El vector → −v tiene sentido opuesto al vector → − λ > 0 tal que → u = λ→ u = λ ′→ u si existe algún escalar λ ′ < 0 tal que → − → − − u . entonces AX = 0 y XB . AX → −1. (7. a la izquierda de B). Obsérvese que AB = −BA. Así pues. o si el vector u se sobrentiende. Si X está estrictamente entre A y B. = 0. B. toda recta cartesiana es también una recta afín. A = B. Se indicará que → − −v mediante la notación → − −v . = AB. entonces λ es la distancia dirigida o 811 Definición Sean A y B puntos en el plano y → u un vector unitario. Si AB = λ → → − − → medida algebraica del segmento [AB] con respecto al vector → u . por AB. a la izquierda de B). XB > 0. si X está estrictamente a la derecha de B (y por consiguiente.153. que yacen sobre una recta − → horizontal. − −v . como en la figura 7. Finalmente. Considérese tres puntos ← → colineales A y B. (7. X = B. Entonces −→ −→ AX XB . m − −v son paralelos si las rectas R→ − −v son paralelas.C están alineados si y sólo si se cumple la Relación de Chasles: AB + BC = AC. entonces AX > 0. De aquí. Si X = A. XB X→ AX XB aparece en la figura 7. dos vectores → − −v son 809 Definición Dos vectores → u y→ u y R→ u y→ −v . entonces AX < 0. AC = AD + DC = DC − DA. AB ·CD + AC · BD + AD · BC = (DB − DA) ·CD + (DC − DA) · BD + AD · (DC − DB) = DB ·CD − DA ·CD + DC · BD − DA · BD + AD · DC − AD · DB = DB ·CD − DA ·CD − DC · DB − DA · BD + AD · DC + AD · BD = 0.154: Teorema de Thales. ❑ D A B C D′ D′ D C A′ B′ A C′ Figure 7. (eje vertical) contra X (eje horizon- 812 Teorema (Teorema de Euler) Sean A. B. Demostración: Utilizando la relación de Chasles. XB < 0 164 Chapter 7 X B A AX < 0. XB > 0 AX > 0. De aquí. Entonces AB ·CD + AC · BD + AD · BC = 0. XB > 0 tal).AX > 0.153: Razón Figure 7. B A′ B′ Figure 7. D cuatro puntos alineados. AX XB X =B X =A Figure 7.152: Distancias dirigidas. AB = AD + DB = DB − DA.C. BC = BD + DC = DC − DB.155: Corolario del teorema de Thales . B. la igualdad anterior revela que ← → ←→ AC A′C′ = 0. Sean A. − AA′ CC′ ⇐⇒ AB A′ B′ mostrando el teorema. ← → ← → Ya que AA′ BB′ . Ensamblando estos resultados. dos transversales) Si tres o más parale-le-le-las Si tres o más parale-le-le-las Son cortadas. Entonces ← → ←→ AC A′C′ = . por la Regla de Chasles. −→′ − −−→ − → −→ −−→ → −→ BB = BA + AA′ + A′ B′ = (A′ B′ − AB) + AA′ .. A′ = B′ .. existe un escalar λ ∈ R tal que −→ −− → − → A′ B′ = AB + λ AA′ . son cortadas (por dos transversales. = .C′ ← → puntos distintos en D′ . A = A′ .. B′ . A′ .. A′ . Sean A.5 ❑ Del teorema anterior. Dos segmentos de una de estas. . AB AC A ′ B ′ A ′C ′ Por otro lado.) 814 Corolario Sean D y D′ dos rectas distintas. dos segmentos cualesquiera Dos segmentos de una de estas son proporcionales A los segmentos correspondientes de la oootraaa. −− → −−→ → − → − A ′ B ′ A ′C ′ AB AC = . C = C′ . A′ B′ AB A′ B′ = = ← → ← → Como AA′ no es paralela a AB. son cortadas. Entonces ← → ← → CB CB′ = . puntos en D. se obtiene de inmediato el siguiente corolario.10.C puntos distintos en D. por ser vectores unitarios en la dirección de una misma recta.. intersecándose en el punto único C. AA′ BB′ ⇐⇒ CA CA′ 5 “Si tres o más paralelas (Si tres o más parale-le-le-las) Si tres o más paralelas (Si tres o más parale-le-le-las) Son cortadas.” Les Luthiers.10. puntos en D′ . dos transversales) Son cortadas. B. B = B′ . Sea AA′ ← → BB′ . Por un lado. AA′ CC′ ⇐⇒ AB A′ B′ Demostración: Refiérase a la figura 7. A = B. B′ . son cortadas Son cortadas. (Véase la figura 7. −−→ − → −→ −−→ CC′ = CA + AA′ + A′C′ −→ AC − → −→ A′C′  − → − · AB + AA′ + ′ ′ AB + λ AA′ AÇ B Ç AB å å → A′C′ AC − A′C′ −→′ AB + 1 + − λ AA .Vectores 165 813 Teorema (Teorema de Thales de Mileto) Sean D y D′ dos rectas distintas. son cortadas (por dos transversales. Se demostrará que BC = −−→ − → −−→ − → −−−−→ 2MC MB . −→ −−→ BG = 2GMB . llamémosle G. de donde se deduce el resultado. sea MC el punto medio de [AB]. las rectas AMA y BMB no son paralelas y por tanto se −→ −−→ encuentran en un punto. etc. − → ◮Resolución: Sean los puntos medios de [AB] y [CA]. Se verán algunos ejemplos. respectivamente. Se tiene 2AMC = AB y 2AMB = AC. Luego AG y GMA son paralelos. CMC = 2 −−→ −−→ ◮Resolución: Como AMC = MC B. demostrar que −→ −−→ AG = 2GMA . Así − → BC = − → − → BA + AC = − → − → −AB + AC = −−→ −−→ −2AMC + 2AMB = −−→ −−→ 2MC A + 2AMB = −−→ −−→ 2(MC A + AMB) = −−−−→ 2MC MB . Pruébese que → − →ä −−→ 1 Ä− CA + CB . Demuéstrese que el segmento de recta que une a los puntos medios de dos de los lados es paralelo al tercer lado y que mide la mitad de éste. ◮Resolución: Utilizando la Regla de Chasles varias veces: → − − → − → − → −→ −→ −→ 0 = AA = AB + BC + CD + DE + EA. ←−→ ←−→ ◮Resolución: Como el triángulo no es degenerado. y por lo tanto existe un escalar a tal que . − → − → −→ −→ −→ 815 Ejemplo Dado un pentágono ABCDE. MC y MB . ◭ 818 Ejemplo Si las medianas [AMA ] and [BMB ] del triángulo no degenerado △ABC se intersecan en el punto G. como se quería demostrar. hállese AB + BC + CD + DE + EA. se tiene − → − → −−→ −−→ −−→ −−→ CA + CB = CMC + MC A + CMC + MC B = −−→ −−→ −−→ 2CMC − AMC + MC B = −−→ 2CMC .166 Chapter 7 El cálculo vectorial hasta ahora desarrollado es particularmente útil al resolver problemas que demandan demostrar si tal recta es paralela a otra recta. ◭ 817 Ejemplo En el △ABC. ◭ 816 Ejemplo Sea △ABC un triángulo en el plano. como se había de demostrar. −−→′ GG = Resulta pues que −→ −−→′ GB + BG = −−−→ −−→ −2GMB + 2G′ MB = −−→ −−−→ 2(MB G + G′ MB ) = −−→ 2G′ G. Demostrar que NPQR es un paralelogramo. Q′ . −−→ −−−→ CG′ = 2G′ MC . R′ . respectivamente. −→ −−→ AG = 2GMA . Por el ejemplo ya citado. El punto de concurrencia G se llama el baricentro o centroide del triángulo. Encuéntrense ←−→ ←−→ las rectas CMC y BMC en G′ . [BC]. los puntos medios de los lados [AB]. − → BA −−−−→ 2MA MB = = −→ −→ BG + GA = −−→ −−→ bGMB − aGMA = −−→ −−→ −−−−→ bGMA + bMAMB − aGMA . De igual manera. P. Se tiene que demostrar que CMC también pasa por G. Así.) D C N R P Q 6 No suele frecuentar antros de vicio. −−→′ −−−→ BG = 2G′ MB . (Véase la figura 7. . R. existe un escalar b tal que BG = bGMB . las medianas concurren. de donde 2 − b = 0. ←−→ ◮Resolución: Sea G como en el ejemplo 818. P′ . MA MB y GMA no son paralelos. P′ . =⇒ a = b = 2. Constrúyase ahora puntos N. y así −−→ −−−−→ (2 − b)MAMB = (b − a)GMA. Q′ . [DA]. demostrando lo pedido.156. Sean N ′ . ◭ M Tarea B A 820 Problema Dados en el plano son el cuadrilátero ABCD y el punto M. −−−−→ −−→ Al no ser △ABC degenerado6. b − a = 0. ◭ 819 Ejemplo En todo triángulo no degenerado △ABC. Por el ejemplo 816 . −−→′ −−→ −−→ → −−→ → − − GG = −2GG′ =⇒ 3GG′ = 0 =⇒ GG′ = 0 =⇒ G = G′ . R′ . que son las imágenes al ser M reflejado con respecto a los puntos N ′ .Tarea 167 −→ −−→ −→ −−→ AG = aGMA . [CD]. Q. −→ −−→ BG = 2GMB . los puntos medios de los segmentos [AB] . 1. ➋ [AC] y [BD] se bisecan. Entonces −v es independiente de O. F. C ′ respectivamente. B. Demostrar que las condiciones siguientes son equivalentes: 827 Problema Sea S el punto medio del segmento finito AB. [BC]. . C ′ puntos simétricos a → − → − A.157. y sea n = p + q. A todo punto O del plano se le asocia la suma ponderada de vectores −→ −−→ −−→ → −v = a − 1 OA1 + a2 OA2 + · · · + an OAn . . Si a1 + a2 + · · · + an = 0. Demostrar que EK = FL. A. . 824 Problema Sean I. Figure 7. → v y −v no depende de O.156: Problema 820. OO′ = OAk − O′ Ak y luego −′ −−→ −−→ −−→ −−→ −−→ −−→ −−→ → −v − → v = a1 (OA1 − O′ A1 ) + a2(OA2 − O′ A2 ) + · · · + an (OAn − O′ An ) = (a1 + a2 + · · · + an )OO′ . el punto G tal que OG = → v es independiente de O. ← → ← → ← → ← → ➌ AB DC y AD BC. La recta trazada a través de B y paralela al lado [DA] interseca la diagonal [AC] en el punto N. 828 Problema Dos rectas mutuamente perpendiculares intersecan los lados [AB]. entonces sean G y G′ puntos tales que OG = → v y OG = v′ . MA2 + MB2 = 2SA2 + 2SM 2 . . 821 Problema Dado un cuadrilátero ABCD. A2 . 7. con respecto a I. [BC] y [AD]. del cual se han trazado rectas paralelas a las medianas AMA y BMA .An n puntos distintos (dos a dos). no tres de ellos alinea- − → − → dos. y los otros q puntos en el otro semiplano. Si a1 + a2 + · · · + an = 1. → v = OO . Demostración: Para algún otro punto O′ póngase → −′ −−→ −−→ −−→ v = a 1 O′ A 1 + a 2 O′ A 2 + · · · + a n O′ A n . B′ . . P y Q son. N. a2 . Demuéstrese que ➊ ABCD es un paralelogramo. . etc. → −→ − 2. 822 Problema Sean A. Sean a1 . B. y sean A′ . E y el baricentro G del △ABC están alineados. B. lo cual establece que G es independiente de O. Sean E y F los puntos medios de las diagonales [AC] y [BD] respectivamente. −−→ −−→ −−→ Por la regla de Chasles. Se dirá que ABCD es un paralelogramo si AB = DC. Demostrar que EQFP es un paralelogramo. A2 . y sea M cualquier punto que yace en la recta infinita que contiene al segmento AB.11 Baricentros 829 Lema Sean A1 .157: Problema 828. . La recta trazada a través de A y paralela al lado [BC] interseca la diagonal [BD] en el punto M.C.C puntos en el plano. ninguno de los puntos yaciendo en L. si a1 + a2 + · · · + an = 0. 825 Problema Sobre el lado [AB] del △ABC resta el punto P. Demostrar → ←→ ← que MN CD.❑ . . Sea E el punto medio de A′ B′ . [CD]. De igual manera. . esto es. que intersecan a los lados del triángulo en los puntos A′ (sobre [BC]) y B′ (sobre [CA]). . . (Figura 7. −′ −−→′ − −→ − −−→′ → −v − → Si a1 + a2 + · · · + an = 1.) B 823 Problema En un cuadrilátero arbitrario no degenerado ABCD. An n puntos en el plano. q enteros estrictamente positivos. an n números reales. Demostrar que F C K E −→ − → − → −→ −→ AB + AC + AB′ + AC ′ = 2AA′ . K y L respectivamente. Sean A1 . [CD] y [DA] del cuadrado ABCD en E. Demostrar que existe una recta L en el plano tal que exactamente p de los puntos yacen en un semiplano de ésta. A L D Demostrar también que para un punto arbitrario del plano M se cumple −→ −→ −→ −→′ −→′ −−→′ − → MA + MB + MC + MA + MB + MC = 6MI. AI = IA′ . 826 Problema Sean p. Demostrar que P. Luego −−→′ −→ −−→′ −− → −−→ − → − → − GG = GO + OO + O′ G′ = OO′ − (→ v − v′ ) = 0 . D cuatro puntos distintos en el plano. M.168 Chapter 7 Figure 7. −′ → −v = → De aquí. respectivamente. → −→ −→ 1 − Esto implica que AG = GB = AB y por lo tanto G es el punto medio del segmento [AB]. (An . x). . y esto implica que −−→ −−→ −−→ → − → − (kx1 )GA1 + (kx2 )GA2 + · · · + (xan )GAn = k 0 = 0 . un sistema de puntos ponderados. . también llamado centro de masa. an = xn . x1 ). denotado por ê Ü G = bar A1 A2 ··· An x1 x2 ··· xn . xn ) se tiene −−→ → − −−→ −−→ x1 GA1 + x2 GA2 + · · · + xn GAn = 0 . 1) es el punto G del plano tal que −→ −→ → − GA + GB = 0 . Esto es.. si k = 0 es una constante real. x1 ). (An .. x1 ). x) se conoce como iso-baricentro. (A2 . . de donde se destila el resultado. . 832 Lema El baricentro no cambia si se reemplazan sus coeficientes por coeficientes proporcionales. . ê Ü bar A1 A2 ··· An x1 x2 ··· xn ê Ü = bar A1 A2 ··· An kx1 kx2 ··· kxn . esto es. Demostración: Por definición del baricentro de los puntos ponderados (A1 . x1 + x2 + · · · + xn 831 Definición El punto G definido por el lema 830 es llamado el baricentro de los puntos ponderados (A1 . (A2 . . . . x2 )... Si x1 + x2 + · · · + xn = 0 entonces existe un punto único G tal que −−→ → − −−→ −−→ x1 GA1 + x2 GA2 + · · · + xn GAn = 0 . (A2 . (An . x). El baricentro de (A. 1) y (B. xn ).Baricentros 169 830 Lema Sea (A1 . ❑ . xk ∈ R. . x1 + x2 + · · · + xn Entonces a1 + a2 + · · · + an = y el resultado se deduce del lema 829. entonces el baricentro de (A1 . . . Demostración: Póngase a1 = x1 . x2 ). . (An . Si x1 = x2 = · · · = xn = x. x1 + x2 + · · · + xn x1 + x2 + · · · + xn = 1. xn ). . . con Ak ∈ R2 . x2 ). si en el punto Ak se pone una masa de ak unidades. (A2 . 833 Ejemplo Sean A y B dos puntos distintos del plano. ❑ El baricentro coincide con la noción física de centro de gravedad. 2 . el baricentro es el punto de equilibrio de todas estas masas. . x1 + x2 + · · · + xn a2 = x2 . . se puede interpretar a X como el baricentro de los puntos ponderados (A. (7. El baricentro de (A. an ) son las coordenadas baricéntricas de G con respecto a A1 . . 1) (A. a2 ). Véase la figura 7. . . 1) G (B. 1) y (B. si el punto X = B está sobre la recta AB. . a2 . .” ya que no son únicas. 4) (A. . yaciendo sobre [AB]. De igual manera. entonces aA + bB está entre los puntos A y B. a) y (B. tal como en la siguiente definición. . Se puede percatar entonces que para dos puntos A ∈ R2 . a) y (B. g2 ) es el baricentro de los puntos ponderados (A1 . (An . . 7 +· · ·+an 6 7 +a2 6 a1 64 5 4 5 4 5 4 5 a1 + a2 + · · · + an 0 yn − g 2 y2 − g 2 y1 − g 2 g2 = a 1 y1 + a 2 y2 + · · · + a n yn . como el baricentro no cambia si se reemplazan sus coeficientes por coeficientes proporcionales. 1]}. .158: Varios baricentros. b ∈ R. XB esto es. El baricentro está a b unidades de A y a unidades de B. an ) son las “coordenadas baricéntricas. B ∈ R2 fijos. An y se escribe G = a 1 A1 + a 2 A2 + · · · + a n An . la colección de baricentros ← → no es otra cosa si no la recta AB. . −1) (B. −k). . A2 . . . 7) Figure 7.18) esto es. 835 Definición (Coordenadas baricéntricas) Si G es el baricentro de los puntos ponderados (A1 . se observa que −→ − XA −→ → = k ⇐⇒ XA − kXB = 0 ⇐⇒ XA − kXB = 0 . a2 . .158 para varios ← → ejemplos. Se ve entonces que [AB] = {tA + (1 − t)B : t ∈ [0 . yk ) y G = (g1 . an ) entonces 3 2 3 2 3 2 3 2 6 xn − g1 7 6 07 6 x2 − g 1 7 6 x1 − g 1 7 7 = 6 7 =⇒ g1 = a1 x1 + a2 x2 + · · · + anxn . a2 ). Una manera de obtener unicidad es imponiendo una condición extra. . b) es el punto G del plano tal que −→ −→ → − aGA + bGB = 0 . se puede demostrar fácilmente que si ab ≥ 0. a1 ). (A2 . . a+b a+b El ejemplo anterior provee un algoritmo geométrico para localizar el baricentro de dos puntos (A. a1 + a2 + · · · + an En general. el segmento [AB] no es otra cosa que una colección de baricentros de sus extremos. b): se divide al segmento [AB] en a + b partes iguales. con coeficientes que suman a 1. −→ Esto implica que AG = a − b − → −→ → ← → AB y BG = BA. entonces se dice que (a1 . an ) y a1 + a2 + · · · + an = 1. . a1 ). . (An .170 Chapter 7 834 Ejemplo Sean A y B dos puntos distintos del plano y a + b = 0. . (A2 . . {G ∈ R2 : G = aA + bB. Obsérvese que G está en la recta AB. a ∈ R. Inclusive. 2) G (B. a + b = 0} Si Ak = (xk . no se puede decir que (a1 . G (A. (C. . los baricentros aA + bB + cC son puntos en el plano interiores. el baricentro de tres puntos no alineados (A. (A2 .❑ 837 Ejemplo Por el teorema anterior. Ü ê Ü ê A B C A H = bar bar 1 1 1 = G. 1 2 Véase la figura 7. (B. . a1 + a2 + · · · + am ). 1). a2 ). Esto es. . . . En otras palabras. .159.159: Baricentro de un triángulo. . de donde −−→ −−→ −−→ −− → a1 GA1 + a2 GA2 + · · · + a pGA p = (a1 + a2 + · · · + am )GH. o exteriores al △ABC. an ). Luego −−→ → −−→ → −−→ −−→ −−→ −−→ − − a1 HA1 + a2 HA2 + · · · + an HAm = 0 . se puede definir el baricentro H de los puntos ponderados (A1 . 1).160. de donde −−→ → − −− → −−−−→ (a1 + a2 + · · · + am )GH + am+1 GA p+1 + · · · + an GAn = 0 y por lo tanto G es también el baricentro de los n − m + 1 puntos ponderados (H. 1) y (C. el baricentro de los tres puntos (A. a>0 b<0 c<0 C / A H / a>0 b>0 c<0 G A B a>0 b>0 c>0 a>0 b<0 c>0 C a<0 b>0 c<0 Figure 7. am ). . (An . (A2 . Véase la figura 7. sobre. 1) y (H. Luego se halla el baricentro de (A.Baricentros 171 836 Teorema (Asociatividad de baricentros) El baricentro de n puntos ponderados no cambia si se reemplaza una parte de ellos por su baricentro (si existiere) guardando los mismos coeficientes de los puntos restantes. 1). 1). que no es otra cosa si no el punto medio del segmento [BC]. −− → −−→ −−→ Utilizando la regla de Chasles para 1 ≤ k ≤ m. que está a 2 unidades de A y 1 de H. (Am+1 . 2). B a<0 b>0 c>0 a<0 b<0 c>0 Figure 7. (Am . Si a1 + a2 + · · · + am = 0. a1 ). 2) de (B. 1) es en efecto el baricentro del triángulo △ABC. Los signos son determinados de la manera siguiente: para saber el signo . 1) se puede hallar de la siguiente manera: primero se halla el baricentro (H. a1 ). am+1 ). 1). (B.160: aA + bB + cC. (An . GH = GAk − Ak H. (C. Si el triángulo △ABC no es degenerado y si a + b + c = 0. a2 ). . a1 GA1 + a2GA2 + · · · + anGAn = 0 . . . . Demostración: Sea G el baricentro de los puntos ponderados (A1 . a + b + c = 1. an ). N puntos menelaicos de los lados [BC]. b). E. Demostrar que dos triángulos en el plano son copolares si y sólo si son coaxiales. 1. Tarea 842 Problema (Teorema de Menelao7 ) A un punto P que yaga sobre la recta determinada por un lado del △ABC se le llama punto menelaico de este lado. a2 ). Sea M un punto del lado [BC]. Sea F la intersección de las diagonales [AC] y [DB]. demostrar que E es el baricentro de (A. 2. Sean L. −x). Sobre la recta a = 0. en el semiplano que no contiene a A se observa que a < 0. G. La asociatividad de baricentros provee el siguiente criterio para alineación de puntos: dados tres puntos ponderados (A. la asociatividad de baricentros provee el siguiente criterio para concurrencia de rectas: tengan tres puntos ponderados (A. 1). [CA] y [AB] del △ABC. b ) y (C. [△MAC]) y (C. b). Póngase BC = a. Sea A′ el pie de la perpendicular desde A hasta el lado [BC]. Además. Las rectas AD y BC se cortan en E.172 Chapter 7 de a. (C. Demostrar que M es el baricentro de (B. Demostrar que E es también el baricentro de (B. M. Si EA ED = x. Las tres cevianas [AA′ ]. 3). (B. Demostrar que A es el baricentro de (B. c ). c). 2). 3. 1) y (C. 846 Problema (Teorema de Desargues) Dos triángulos △ABC y △A′ B′C ′ se dicen ← → ← → ←→ copolares si las tres rectas AA′ . 1). M (intersección de CA y C ′ A′ ). −3). b) y (C. a) y (B. 4. −x). 2. entonces los puntos A. [CA] y [AB] del △ABC. • F es el baricentro de (B. M. N puntos menelaicos de los lados [BC]. Si AM es la bisectriz del B AC. Si (a + b)(b + c)(c + a) = 0 y si • E es el baricentro de (A. con [AB] [DC]. Póngase BC = a. 1) y (D. c) baricentro G. 1). ← → ← → ← → entonces las rectas CE. ¿a qué conclusión se llega? ” 3. a). L. (B. c) con G como su baricentro. (C. L. a). ← → ← → ➌ Demostrar que AL BC BL CM AN = −1. ➋ Construir el baricentro K de (A. AB = c. considérese el punto A y la recta del triángulo opuesta a A. Demostrar que I. [BB′ ] y [CC ′] de un △ABC concurren si y sólo si AC ′ BA′ CB′ · · = +1. c). con la propiedad LAC 844 Problema (Menelao trigonométrico) Sean L. (B. ¿a qué conclusión se llega? 4. M. Sea L el punto medio de [AA′ ]. F están alineados. (B. BB′ y CC ′ son concurrentes. Los triángulos son coax← → ←→ ← → ←→ iales si los tres puntos L (intersección de BC y B′C ′ ). 1). b2 ) y (C. 1. −x). (C. b) y (C. a). 841 Problema El △ABC tiene todos sus ángulos agudos. (B. F están alineados. c) es el punto de concurrencia de las bisectrices del △ABC. N son colineales si y solamente si 838 Problema Dado un triángulo no degenerado ABC: ➊ Construir el baricentro K de (A. En el semiplano que contiene al punto A se observa que a > 0. b). (C. 847 Problema (Teorema de Ceva. La ceviana es propia si no coincide con un lado del triángulo. 1. b). J. 843 Problema En el △ABC sea L un punto en el lado [BC]. −x) y (D. C ′ B A′C B′ A . 3). CA = b. De igual manera se determinan los signos de b y c. a) y (B. ′ 2 2 = ” AB sen B AL ” CA sen LAC . CA = b. Si el punto no es un vértice del triángulo entonces es un punto menelaico propio. • K es el baricentro de (A. positiva si medido en sentido levógiro Aquí LAC ” = −CdAL. [△MAB]). Si M es el punto medio de [BC]. b). Esta recta divide al plano en dos semiplanos: uno conteniendo al punto A y el otro no. M. 1678) Una recta que pasa por el vértice de un trián- gulo se llama ceviana de este vértice. c2 ). AF y BK son concurrentes en G. (C. y negativa en sentido dextrógiro. N son colineales si y solamente si dBM sen AdCN ” sen B AL sen C · = −1. Deducir que E es el baricentro de (A. Sea I el punto medio de [AB] y J el punto medio de [DC]. AB = c. (B. Deducir que el baricentro de (A. N ← → ←′→′ (intersección de AB y A B ) son colineales. (C. 840 Problema △ABC es rectángulo en A. Demostrar que − → − − → − → → −→ EA + EB − xED − xEC = 0 . (B. Demostrar que BL LC 2. d BA sen NdCB ” sen M sen LAC 845 Problema Demostrar que las rectas tangentes al circuncentro de un triángulo en sus vértices se cortan en tres puntos alineados. Si b + c = 0 y F es el baricentro de (B. ” denota la medida del ángulo dirigido. · · LC MA NB ← → ← → 839 Problema ABCD es un trapecio. Demostrar que L es el baricentro de (A. Así pues. esto es. ◭ D C A O′ O A M Figure 7. [MB] y [BC] del △MBC. 849 Ejemplo Dadas las longitudes AB. ′ −−→ − → tal que MM ′ = → u . conmutativa T u ◦ T− → u y la ley de inversos T−− u = T− v ◦ T− v +− u = T− u +− v = T− v = T− u Es fácil de verificar que la traslación satisface las siguientes propiedades: ð Ł 1. La traslación transforma a un círculo en otro igual. construir un trapecio ABCD.C. la identidad. La traslación envía a un segmento [AB] a otro A′ B′ . B B Figure 7. → es una función que no mueve ningún punto. de bases AB y DC. La traslación transforma a un ángulo en otro ángulo. respectivamente. La traslación transforma un triángulo △ABC en otro igual △A′ B′C′ siendo A′ .C′ homólogos. paralelos. siendo A′ homólogo a A y B′ homólogo a B. Además se observa la ley de composición Se observa que T− 0 −1 → → → → → → → → − → . B. Supuesto resuelto el problema.162: Ejemplo 850. es longitud conocida. siendo los vértices puntos homólogos y los lados de correspondientes de los ángulos. La traslación transforma a una recta a otra paralela a la original. 850 Ejemplo Sea Γ un círculo dado y D una recta dada. una posible construcción es la siguiente: C. .12 Transformaciones geométricas − 848 Definición Una traslación por un vector → u es una función R2 → T− u : M → R2 → M . con T− ◮Resolución: Véase la figura 7.161: Ejemplo 849. se trasladan los puntos homólogos M y C por un vector de longitud min(þþDCþþ. þþABþþ) y con −→ dirección y sentido del vector BM.161. → (D) = → . CD y DA. preservando los radios y siendo los centros de los círculos puntos homólogos. a A. B′ . Construir una cuerda de Γ paralela a D de longitud l dada.Transformaciones geométricas 173 7. considérese la traslación T− DC DC → (A) = M. 5. BC. 2. 3. 4. se construye este triángulo. Obsérvese que CM = DA. igual en longitud y paralelo a éste. El punto M ′ se llama punto homólogo del punto M con respecto a la traslación T− u. þþ−→þþ þPara þ →þþobtener þþ þþ þþ− þþ los vértices A y D. T− DC conocidas las longitudes de los lados [CM]. La recta ∆ paralela a D y pasando por A → cumple las condiciones del problema. Considérese la imagen Γ′ de ′ → Γ mediante la traslación T− u . A′ B′ ) = α . como requerido. 2. siendo A′ homólogo a A. En efecto. igual en longitud y paralelo a éste.α y la ley de inversos GO. La rotación transforma a un ángulo en otro igual e igualmente orientado.α .164: Ejemplo 853.α . La rotación transforma un triángulo △ABC en otro igual △A′ B′C′ siendo A′ .163: Ejemplo 852. La rotación transforma a un círculo en otro igual. Sea → u un vector director de D. La rotación transforma a una recta en otra. equilátero. B′ . 3. si T− u (A) = C.α +α ′ = GO. tal que OM = þþOM ′ þþy MOM Se observa que GO.α : M → R2 → M .α ◦ GO. El punto M ′ se llama punto homólogo del punto M con respecto a la rotación GO. hallar un triángulo △ABC.0 es una función que no mueve ningún punto. 5. entonces AC = l y ||AC|| = l.162.C. que tenga el vértice B sobre L y el vértice C sobre L′ . esto es. Además se observa la ley de composición conmutativa GO. La rotación envía a un segmento [AB] a otro A′ B′ . . de longitud l. 60◦ B A 60◦ A L L L′ C C M B N Figure 7. B. una solución si l = 2R y ninguna si l > 2R. B′ Ÿ − → −−→ homólogo a B y (AB.C′ homólogos.α ′ +α = GO.−α = G−1 O. nótese que hay dos soluciones si l < 2R. siendo los centros de los círculos puntos homólogos. la identidad. ◭ 851 Definición Una rotación (o giro) de centro O y ángulo de giro α es una función R2 GO. 852 Ejemplo Dado un punto A y dos rectas L y L′ . 4. Sean A y B las intersecciones de Γ y Γ . Es fácil de verificar que la rotación satisface las siguientes propiedades: ð Ł 1.174 Chapter 7 − ◮Resolución: Véase la figura 7.α ′ = GO. a A. Figure 7. ′ þ −−→ þþþþ−−→þ þþ ÷′ = α . respectivamente.α ′ ◦ GO. siendo el ángulo entre ellas α . Si R es el radio del círculo. 164. ◭ 854 Ejemplo Se construye. N. cuadrados en los lados AB y BC del triángulo △ABC. Considérese la recta GA. 855 Definición Una reflexión con respecto al punto O o simetría central de centro O es una función R2 RO : M → R2 → M .165.60◦ (M). Sea C la intersección de N y GA. ¿Cómo encontrar C? Sea C la intersección de la recta L′ y GA. Fíjese un punto A arbitrario de L. ◭ U P V C Q A B M Figure 7. ′ ð Ł tal que O es el punto medio de MM ′ . Nótese que una rotación de 90◦ en torno a C toma al △ACV al △UCB. Demuéstrese que PM = QM y que PM ⊥ QM.60◦ (M). exteriormente. ☞ Una simetría central no es otra cosa que el giro G O. ◮Resolución: Véase la figura 7. PM y MQ son iguales y perpendiculares. Por lo tanto. MQ es la mitad de larga y paralela a PV .165: Ejemplo 854.60◦ (C) = B. ◭ 853 Ejemplo Considérese rectas paralelas L. Póngase ahora GA.Transformaciones geométricas 175 ◮Resolución: Supuesto el problema resuelto. En △PUB vemos que we see PM es la mitad de larga y paralela a UB.60◦ (B) = C. Es fácil percatarse que C y B cumplen las condiciones requeridas. .′ 60◦ (C) = B. Luego PV y UB son iguales y perpendiculares. Se tendrá GA. B yaga sobre M y C yaga sobre N. como en la figura 7.60◦ (L). Sea M el punto medio del lado AB. respectivamente.60◦ . Constrúyase un triángulo equilátero △ABC tal que A yaga sobre L. De igual manera. El punto M ′ se llama punto homólogo del punto M con respecto a la simetría central RO . M. Póngase ahora GA. siendo P y Q los centros de los respectivos cuadrados. ◮Resolución: Sean U y V las esquinas de los cuadrados opuestas a A y B. considérese la rotación GA.180◦ y por lo tanto tiene las mismas propiedades de éste. ◭ Dado un polígono de n lados. ◮Resolución: Constrúyase la recta ℓ′ . Mn de los lados.169: Ejemplo 859 . llamémosle A. . Para hacer esto. El siguiente ejemplo considera el problema recíproco. . 857 Ejemplo Dados los puntos M1 .167: Ejemplo 862. va existir un punto fijo. elíjanse dos puntos cualesquiera de ℓ y constrúyanse sus simétricos con respecto a X. Nótese que X está a igual distancia de ℓ y ℓ′ . Nótese además que ya que una reflexión central es un giro con ángulo de rotación π . Obsérvese que A1 es un punto fijo de la composición de reflexiones centrales R = RMn ◦ . . Sea S la intersección de ℓ′ con el otro lado del ángulo. . Esto resuelve el problema. luego trácese la recta que los une. . homólogo a S con respecto a una simetría central de centro X. . . ¿existe acaso un polígono cuyos puntos medios de los lados son los Mk ? ◮Resolución: Sea RMk una reflexión de centro Mk . Figure 7.168: Ejemplo 856 Figure 7. los otros Ak quedan determinados. si la composición es impar. Si la composición es par. Obsérvese que S′ está sobre ℓ. está claro que se puede construir otro polígono cuyos vértices son los puntos medios M1 . Así pues. . un número par de composiciones es efectivamente una traslación.A A′ 176 C′ B′ Chapter 7 A O′′ O O′ O′′′ E′ B D′ C A′′ D E Figure 7. Mn .◭ O′ S X S′ G′ O ℓ′ ℓ F B A S G O′′ Figure 7. Póngase RMk (Ak ) = Ak+1 . 856 Ejemplo Por un punto X. situado en el interior de un ángulo.166: Ejemplo 861. mientras que un número impar es una reflexión central. trácese una recta cortando a los lados del ángulo en dos puntos equidistantes de X. Una vez A1 es elegido. . con An+1 = A1 . Se verá ahora como elegir a A1 . RM2 ◦ RM1 . simétrica con respecto a X de la recta ℓ (uno de los lados del ángulo). Se pondrá entonces A1 = A. tal punto fijo no existe necesariamente y el problema no tiene solución. Constrúyase ahora S′ . . ya que hay cuatro tangentes.Transformaciones geométricas 177 858 Definición Una simetría axial con eje ℓ. B′ . B. Si λ > 0 la homotecia se dice directa y si λ < 0. 3. a A.C. siendo A′ homólogo a A. La simetría axial transforma un triángulo △ABC en otro igual △A′ B′C′ siendo A′ .λ : . 4. . ′ ð Ł tal que ℓ es la mediatriz de MM ′ . −−→ −−→ tal que OM = λ OM ′ . respectivamente. ◭ 860 Definición Llámase homotecia de centro O y constante de similitud λ a una función R2 → R2 M → M′ HO. Se construyen ahora tangentes a los círculos O y O′′ . hallar los centros y las constantes de similitud de las homotecias que transforman el uno al otro. 3. o reflexión con respecto a la recta ℓ es una función R2 Sℓ : M → R2 → M . 861 Ejemplo Dados dos círculos de centros O y O′ . inversa. O′ y una recta L. La simetría axial transforma a un círculo en otro igual. 859 Ejemplo Dadas dos circunferencias de centro O.169. Es fácil de verificar que la simetría axial satisface las siguientes propiedades: ð Ł 1. Por ser opuestos al vértice ′ SB = BSG. El punto M ′ se llama punto homólogo del punto M con respecto a la simetría axial Sℓ . 2. La simetría axial envía a un segmento [AB] a otro A′ B′ . 4. hallar sobre L un punto del cual las tangentes trazadas hasta las circunferencias desde este punto formen ángulos idénticos con L. La simetría axial transforma a un ángulo en otro igual pero contrariamente orientado.−1 es una reflexión de centro O. 2. Es fácil de ver que HO.C′ homólogos.1 es la identidad y que HO. Está claro que la solución se simplifica si ambos círculos yacen en lados opuestos de la recta. pero cambia el sentido de orientación de los vértices. Se observa que Sℓ ◦ Sℓ es la identidad. siendo los centros de los círculos puntos homólogos. Se observan además las siguientes propiedades: 1. G ‘ de donde se deduce que S es el punto buscado. Hay cuatro BSG soluciones. Un triángulo es homotético a otro triángulo semejante al primero. Una recta es homotética a otra recta. ‘ Por ser reflexión axial. La simetría axial transforma a una recta en otra. Un ángulo es homotético a otro ángulo igual al primero. igual en longitud. 5. B′ homólogo a B. Se refleja el centro O′ a través de la recta L y se construye un círculo homólogo con centro O′′ . Un círculo es homotético a otro círculo concéntrico. ◮Resolución: Véase la figura 7. En la solución se consideró el caso cuando ambos círculos yacían en el mismo lado de la recta. ‘ ‘ = FSA. al igual que los puntos A. camináis igual distancia. þ ′ · ′ · ′ þ= C B AC BA [△PCB] [△APC] [△ABP] ⇐ Presúmase que AA′ y BB′ se intersecan en P. camináis hasta el pino. 864 Problema Encontráis un antiguo mapa de tesoro en el baúl del tatarabuelo. BB′ y CC′ de un △ABC concurren si y sólo si AC′ BA′ CB′ · · = +1. en uno y otro círculo. intersecándola en C′′ . Sea HA. respectivamente. Sean D′ y E ′ las ← → ← → intersecciones de las rectas AD y AE con el lado [BC].167 ◭ Tarea 863 Problema Dado un punto A. R′ es una homotecia directa que transforma al círculo de centro O y radio R en el círculo de centro O′ y radio R′ . . B. Las rectas Les BN y CM se cortan en P. 867 Problema Dados son tres círculos disjuntos dos a dos. una recta L y un círculo Γ. Véase la figura 7. Trácense ahora dos radios OA y O′ A′ paralelos ← → ←→ y en el mismo sentido. Ahora se construye la sucesión de puntos P1 . Trácense los tres puntos de 865 Problema Sea △ABC isósceles en A. clavando una estaca aquí. de donde B′C′ D′ E es el cuadrado deseado. y claváis una estaca marcada con una Y . Sea O′′ la intersección de las rectas AA′ y OO′ . Defínase As = As−3 para s ≥ 4. P3 . simplemente trácense dos radios OA y O′ A′′ paralelos y en distinto R sentido y sígase el procedimiento anterior. Demostrar que → ←→ ← dPM = A” MN BC ⇐⇒ A PN.λ (E) = E ′ . BB′ y CC′′ son concurrentes. pero. Únase CP y extiéndase hasta AB. Al llegar a la costa de tan pequeña isla. El tesoro se halla en el punto medio del segmento [XY ]. ◮Resolución: Constrúyase primero el cuadrado BCDE exterior al lado [BC] del triángulo. alquiláis un barco y viajáis hasta los mares del sur. entonces △A1 A2 A3 es equilátero. otro al lado [AC] y los otros dos al lado [BC]. . ¡ay caramba!. Por construcción. Véase la figura 861. los puntos A. Sea B′C′ D′ E ′ la imagen del cuadrado BCDE bajo esta homotecia.λ (D) = D′ y HA.120◦ (Pk−1 ) = Pk . B′C′ D′ E ′ cuadrado es. Luego regresáis al muelle.13 Teoremas de Ceva y de Menelao 868 Teorema (Teorema de Ceva. ¿Cómo encontraréis el tesoro? 866 Problema Considérese el △A1 A2 A3 y sea P0 un punto en el plano. C ′ B A ′C B ′ A . Es fácil ver ahora que HO′′ .− R′ . R Para obtener la homotecia inversa HO′′′ . Ahora bien. por la mitad del teorema ya demostrada se tiene AC′′ BA′ CB′ · · = +1. El dia- grama indica que partiendo del muelle debéis caminar hasta el roble. sóis capaz de encontrar el roble y el pino.C′ . mediante regla y compás. P2 .λ la homotecia de centro A tal que HA. Como encontráis fáciles las instrucciones. C ′ B A ′C B ′ A Demostración: =⇒ Usando el teorema 700 se obtiene þAC′ BA′ CB′ þ [△APC] [△APB] [△PCB] þ þ · · = 1. dobláis a la izquierda 90◦ . marcada con una X .C.178 Chapter 7 ◮Resolución: Trácese la recta pasando por los centros O y O′ . del muelle no hay rastro porque se lo ha comido el salitre. ◭ 862 Ejemplo En el △ABC construir. . doblar a la derecha 90◦ y caminar igual distancia. constrúyase un triángulo △ABC tal que B yaga sobre L y C yaga sobre Γ. un cuadrado tal que uno de sus vértices pertenezca al lado [AB]. Demuéstrese que si P2004 = P0 . 7. B′ están alineados. Sean M y N puntos sobre los lados [AB] y intersección de las tres tangentes exteriores comunes. Como AA′ . ← → ←→ [AC]. Demuéstrese que esto puntos están alineados. de tal manera que GAk+1 . 1678) Las tres cevianas AA′ . = ′ AC XC CA sen A‘ ’′ CY BC sen CBB = . Entonces ‘′ sen CBB ’′ sen ’ ACC′ sen BAA · · = 1. ′ CB C B de donde se colige que C′ = C′′ . N son colineales si y solamente si BL CM AN · · = −1.CC′ son concurrentes. þ= þ · LC MA NB [△CXY ] [△AXY ] [△BXY ] .Y puntos arbitrarios de la recta LMN. C ′ B A ′C B ′ A Esto significa que AC′′ AC′ = ′′ . 869 Teorema (Forma Trigonométrica del Teorema de Ceva) Sea P un punto arbitrario en el plano y en el △ABC sea A′ el punto de intersección de las rectas AP y BC. N puntos menelaicos de los lados BC. AB respectivamente.CA. y si ‘′ sen CBB ’′ ’′ sen BAA sen ACC · · =1 ′ AC sen B ′ BA ′CB sen A ‘ ‘ ’ sen C entonces AA′ . B′ . BB′ . L. AZ CA sen ’ ACC′ .C′ son puntos en los lados BC. Demostración: Se quiere demostrar que ‘′ sen CBB ’′ sen BAA ’′ sen ACC · · =1 ′ AC sen B ′ BA ′CB sen A ‘ ‘ ’ sen C sí y sólo si AZ BX CY · · = 1. LC MA NB Demostración: =⇒ Sean X. M. y C′ el punto de intersección de las rectas CP y AB. la demostración anterior es válida aun cuando el punto P yace fuera del triángulo △ABC.Teoremas de Ceva y de Menelao 179 Pero por hipótesis AC′ BA′ CB′ · · = +1. B′ el punto de intersección de las rectas BP y CA. ❑ 870 Teorema (Teorema de Menelao) Sean L. M. CA y AB del △ABC. ′ AC sen B ′ BA ′CB sen A ‘ ‘ ’ sen C De manera recíproca. ❑ ☞Como el teorema 700 no depende de la posición de los puntos involucrados. si A′ . Entonces þBL CM AN þ [△BXY ] [△CXY ] [△AXY ] þ þ · · · = 1. = ′CB ZB BC sen C ’ ‘′ BX AB sen BAA . ′ BA YA AB sen B‘ Multiplicando y cancelando en estas tres igualdades se obtiene el resultado. ZB XC YA Ahora bien. ◮Resolución: Únase BE intersecando AC en P. entonces AC · BD A′C′ · B′ D′ . NC r √ 3 2 − 2r 1 1 − r · · = 1 =⇒ r = . △B′ PB. PB = AB cos ABP 1 MP r − 2 2 4 BE 4 Además de donde se colige que EN 1−r = . B′ . CC′ . Vêase la figure 7. △BPB′ intersecándose con las rectas CC′ . AC PC′ A′ O · · = 1. L′C MA NB Pero por hipótesis BL CM AN · · = −1. AC CE Determínese r si B. D son puntos distintos sobre L1 y si A′ . △A′ PA. Aplicando el teorema de Menelao a △ABA′ . CP C′ A′ OA . ❑ 871 Ejemplo (IMO 1982) Las diagonales AC y CE de un hexágono regular ABCDEF son dividas interiormente en los puntos M y N. DD′ . respectivamente. DD′ se intersecan en O y si P es el punto de intersección de L1 y L2 (si L1 ||L2 . CC′ . DD′ son concurrentes. C′ . N son colineales. LC MA NB Así BL BL′ = . 2r − 1 4 r 3 ◭ C B M P N D E A F 872 Teorema [Invariancia bajo perspectiva] Sean L1 y L2 dos rectas distintas sobre el plano. = .170. Por la mitad del teorema ya demostrada se tiene BL′ CM AN · · = −1. B. BB′ . y DD′ respectivamente. C. = CB · DA C′ B′ · D′ A′ Demostración: Si AA′ . BB′ . Aplícando el teorema de Menelao al △CPE y a la recta BMN. CC′ . ′ L C LC de donde L = L′ . de tal manera que AM CN = = r. D′ son puntos distintos sobre L2 y si las rectas AA′ . entonces P es el punto en infinito). Si A. se tiene 1−r CM ‘ = AB = BE =⇒ PB = 1 . M.180 Chapter 7 ⇐ Presúmase ahora que la recta MN corta a AC en L′ . Figure 7. ‘ de donde se consigue el resultado. E. los pies de las perpendiculares de A a BC. F son colineales. Q. Arguyendo de manera semejante se demuestra que Q. Nótese que AF = PA cos PAF. ‘ DC = PC cos PCD.171: Teorema 873. D. ‘ Por lo tanto. Demuéstrese que P. N. M. Entonces C. F son. DP D′ B′ OB En multiplicando estas cuatro igualdades se colige AC · A′ D′ · B′C′ · BD = 1. M. E. · · D′ P DA OA′ BD PD′ B′ O · · = 1. Sean D. Trácense las rectas perpendiculares de D a AB. Demostración: Para demostrar que D. Demostración: El cuadrilátero BDHF es cíclico y la recta de Simson de D pasa por P. FB DC EA (cos PBF)(cos ‘ ‘ ‘ PCD)(cos PAE) ‘ = PCD. En otras palabras P. respectivamente. se necesita demostrar que AF BD CE · · = 1. CF y sean P.170: Teorema 872. PAF C E O P C A′ D′ B′ Q D C′ E D N B A F M A D B C A P Figure 7. F P B Figure 7. Q. PA cos PAE.172. B a CA y de C a AB. C′ A′ · DA ·CB · D′B′ de donde AC · BD A′C′ · B′ D′ . M. M.172: Ejemplo 874. E. E. = CB · DA C′ B′ · D′ A′ ❑ 873 Teorema [Teorema de Simson] Sea P un punto en el circuncírculo del △ABC. ‘ PBD ‘ PCE ‘ = PBF. FB DC EA ‘ FB = PB cos PBF. ‘ CE = PC cos PCE. ❑ . ❑ Ahora bien. N son colineales. F son colineales. N son colineales.Teoremas de Ceva y de Menelao 181 A′ D′ PD AO = 1. CA. F los pies de las perpendiculares de P a BC. ‘ = PAE. ‘ ‘ ‘ (cos PAC)(cos PBD)(cos PCE) AF BD CE · · = . 874 Ejemplo Refiérase a la figura 7. CA y AB respectivamente. BE. ‘ EA = ‘ BD = PB cos PBD. N los respectivos pies de las perpendiculares. N son colineales. M. 182 Chapter 7 875 Ejemplo Sea H el ortocentro, O el circuncentro y R el circunradio del △ABC. Sea D la reflexión de A a través de la recta BC, E la reflexión de B a través de CA y F aquella de C a través de AB. Demuéstrese que D, E y F son colineales si y sólo si OH = 2R. (Figura 7.173.) Demostración: Sea △PQR el triángulo cuyo triángulo de medianas es △ABC (esto es A es el punto medio de QR, B el de RP y C el de PQ). Desde O, trácense rectas perpendiculares a QR, RP y PQ, cuyos pies de las perpendiculares son D′ , E ′ , F ′ respectivamente. Por el teorema de Simson, D′ , E ′ , F ′ son colineales si y sólo si O yace en el circuncírculo del △PQR. Nótese que el circuncentro del △PQR es el ortocentro del △ABC, esto es, H. Así, O yace en el circuncírculo del △PQR si y sólo si OH es igual al △PQR que es 2R. Véase la figure 7.174 ❑ 876 Teorema (Teorema de Ptolomeo) Dados cuatro puntos cualesquiera en posición general (esto es, ninguno de ellos coin- cide, y no tres de ellos son colineales), AB ·CD + AD · BC ≥ AC · BD. La igualdad se cumple si y sólo si el cuadrilátero ABCD es cíclico. Demostración: Sean L, M, N, respectivamente, los pies de las perpendiculares de D a BC, CA, AB. Como ‘ = CMD ’ = 90◦ , los puntos L, C, D, M son cocíclicos. La figura 7.175 muestra uno de los casos posibles. De CLD todas maneras se tiene ‘ = CD · AB , LM = CD sen BCA 2R en donde R denota el circunradio del △ABC. De la misma guisa, MN = AD · BC , 2R LN = BD · AC . 2R En virtud de la desigualdad del triángulo, LM + MN ≥ LN. Utilizando las expresiones para LM, MN y LN se obtiene CD · AB AD · BC BD · AC + ≥ . 2R 2R 2R Esto demuestra la desigualdad deseada. La igualdad es satisfecha si y solamente si L, M, N son colineales y gracias al teorema de Simson esto sucede si y solamente si D yace en el circuncírculo del △ABC.❑ ˆ Demuéstrese que PA = PB + PC. 877 Ejemplo Sea P un punto en el circuncírculo del △ABC yaciendo en el arco BC. ◮Resolución: El resultado se consigue de inmediato al aplicar el teorema de Ptolomeo al cuadrilátero cíclico ABPC (figura 7.176). ◭ 878 Ejemplo Si un círculo pasando por A corta dos lados y una diagonal del paralelogramo ABCD en los puntos P, Q, R tal como en la figura , demuéstrese que AP · AB + AR · AD = AQ · AC. H H D D Teoremas B de Ceva y de Menelao E′ B R C A O C D′ A E 183 P E Q O F ′ Figure 7.174: Ejemplo 875. Figure 7.173: Ejemplo 875. ◮Resolución: Aplíquese el teorema de Ptolomeo al cuadrilátero cíclico APQR obteniendo AP · RQ + AR · PQ = AQ · RP. Véase la figura 7.177. Obsérvese que △ABC ∼ △RQP. Al multiplicar esta última igualdad por la constante AB RQ se obtiene AP · AB + AR ·CB = AQ · AC. Reemplazando CB por AD, se colige el resultado AP · AB + AR · AD = AQ · AC. ◭ B D D P C R Q NA C M A O C B L A P B Figure 7.175: Teorema 876. Figure 7.176: Ejemplo 877. Figure 7.177: Ejemplo 878. 879 Ejemplo Sean A, B, C, D, E, F, G vértices, nombrados en orden, adyacentes de un heptágono regular. Demuéstrese que 1 1 1 + = . AC AD AB 184 Chapter 7 ◮Resolución: En la figura 7.178, aplicando el teorema de Ptolomeo al cuadrilátero cíclico ABCF AC · BF = AB ·CF + BC · FA. Substituyendo BF por AD, BC por AB y FA por AC en la última desigualdad se obtiene AC · AD = AB · AD + AB · AC =⇒ 1 1 1 + = , AC AD AB obteniéndose el resultado. ◭ 880 Ejemplo En el △ABC, BC > CA > AB. D yace en el lado BC y E yace en la prolongación de BA producida más allá de A de tal manera que BD = BE = CA. Si P yace en el lado AC de tal manera que E, B, D, P son cocíclicos y sea Q el otro punto de intersección de BP con el cicuncírculo del △ABC. Demuéstrese que AQ +CQ = BP. Véase la figura 7.179. ◮Resolución: Obsérvese que △AQC ∼ △EPD porque ‘ = CBQ ‘ = DEP ‘ CAQ ‘ = 180◦ − ABD ‘ = EPD. ‘ Por otra parte, el teorema de Ptolomeo implica que y AQC BP · DE = BE · DP + BE · EP. Así pues BP = BE · EP CQ AQ DP + BD · = CA · +CA = AQ +CD. DE DE CA CA ◭ E A E D F Q C P G A B Figure 7.178: Ejemplo 879. Tarea B Figure 7.179: Ejemplo 880. D C Puntos y rectas notables de un triángulo 185 881 Problema (Teorema de Van Aubel) Demostrar que si el △ABC tiene cevianas AA′ , BB′ ,CC ′ que concurren en P entonces CP CA′ CB′ = ′ + ′ . PC ′ AB BA 7.14 Puntos y rectas notables de un triángulo 882 Definición Dado un △ABC, si MA , MB , MC son, respectivamente, los puntos medios de los lados BC, CA y AB, los segmentos AMA , BMB , CMC son las medianas del triángulo. 883 Teorema Las medianas de un triángulo concurren. Demostración: Denótense los puntos medios de los lados BC,CA, AB por MA , MB , MC . Refiérase a la figura 7.180. Por definición de las medianas BMA = MAC, CMB = MB A, AMC = MC B y así AMC BMA CMB · · = 1. MC B MAC MB A Luego las medianas concurren gracias al Teorema de Ceva (868). ❑ 884 Definición El punto de concurrencia de las medianas de un triángulo se llama el centroide o baricentro. 885 Teorema Todo triángulo es dividido por sus medianas en seis triángulos de área igual. Demostración: Como los △GBMA y △GMAC tienen bases de igual longitud y las misma altura, se tiene [△GBMA ] = [△GMAC] = x, digamos. De la misma manera [△GCMB ] = [△GMB A] = y, [△GAMC ] = [△GMC B] = z. Ahora bien [△CAMC ] = [△CMC B] =⇒ 2y + z = z + 2x =⇒ x = y, [△ABMA ] = [△AMAC] =⇒ 2z + x = 2y + x =⇒ y = z, y por lo tanto x = y = z.❑ 886 Corolario El baricentro G del △ABC triseca cada mediana. En efecto, AG : GMA = BG : GMB = CG : GMC = 2 : 1. Demostración: Por el teorema 885, [△BMA G] = 2[△BMA A]. Como ambos triángulos tienen la misma altura, [△BMA G] = 2[△BMAA] =⇒ MA G = 2GA =⇒ AG : GMA = 2 : 1. Las otras relaciones se demuestran de manera semejante.❑ 887 Teorema Sea G el baricentro del △ABC y L una recta. Entonces GG′ = AA′ + BB′ +CC′ , 3 en donde A′ , B′ ,C′ denotan, respectivamente, los pies de las perpendiculares desde A, B,C en L. El área del triángulo medial es un cuarto del área del triángulo original. de donde 4GG′ = 2MM ′ + 2NN ′ = (AA′ +CC′ ) + (BB′ + GG′ ) y por lo tanto. B = (b1 . si A = (a1 . el radio del circuncírculo del triángulo 2 . 3GG′ = AA′ + BB′ +CC′ . Se sigue. las alturas del △ABC son el doble de las alturas las alturas del △MA MB MC . Entonces → − →ä −→ 1 Ä− → − PA + PB + PC . .181. entonces Å ã a 1 + b 1 + c1 a 2 + b 2 + c2 G= . MB MC C B MA G B MA C Figure 7. 3 3 Demostración: El resultado se obtiene de inmediato del teorema 887. Entonces 2GG′ = MM ′ + NN ′ . b se tiene que Demostración: Como 2AMC = AB y 2AMB = AC. Ahora bien. 2MA MC = AC. como en la figura 7.182: El triángulo medial. b2 ). 891 Teorema En todo triángulo.181: Teorema 887. PG = 3 Aún más. y 2MB MC = BC. Teorema 883. ◊ ‘ △AMC MB ∼ △ABC. 890 Teorema Los lados del triángulo medial son paralelos a los lados del triángulo original y miden la mitad de los lados correspondientes en el triángulo original.186 Chapter 7 Demostración: Sea M el punto medio de CA y N el punto medio de BG. 2MC MB = CB.180: El baricentro. R . el radio del circuncírculo de su triángulo medial es original. c2 ). a2 ). ❑ 889 Definición El triángulo medial de un triángulo dado es el triángulo obtenido al unir los pies de las medianas del triángulo dado. y los triángulos △AMC MB y △ABC comparten el A. ❑ 888 Corolario Sea G el baricentro del △ABC y sea P un punto arbitrario. Luego AM C MB = ABC y por lo tanto BC y MC MB son segmentos paralelos. C = (c1 . además que △MA MB MC ∼ △ABC y que 2MA MB = AB. Figure 7. Así pues [△ABC] = 4[△MA MB MC ]. como △MA MB MC ∼ △ABC.❑ A A MC N G A M B MB C B′ ′ ′ N ′ AG M′ C′ Figure 7. Un argumento semejante demuestra que AB MA MB y AC MA MC . D b = sen(π − A) b = − sen(−A) b = sen A. 2R b porque ángulos opuestos inscritos en un cuadrilátero son suplementarios. Sea R el radio del círculo circunscrito al △ABC. b is agudo u obtuso. demostrando el teorema en el caso A b es obtuso. 2R y sen Cb = c . Entonces CBD ‘ = π al estar inscrito en un Presúmase primero que A 2 semicírculo. sen A 2R De la misma manera se demuestra que sen Bb = b . b = A. ❑ A A B D C O D C O B Figure 7. b sen(π − A) se deduce que b = sen(π − A) b = sen D b= sen A a .Puntos y rectas notables de un triángulo 187 Demostración: El resultado sigue inmediatamente del teorema 890.184: Generalización de la ley de los b obtuso. Entonces [△ABC] = abc . Ahora bien. 2R b agudo. A Figure 7.183: Generalización de la ley de los b agudo. sen D DC 2R b ya que ambos subtienden el mismo arco. Como b = π − A. senos. b = AC y c = AB. b = AC y c = AB. ❑ 892 Teorema (Generalización de la ley de los senos) En el △ABC póngase a = BC. Por lo tanto. Entonces a b sen A = b sen Bb = c sen Cb = 2R. En este caso. A 893 Teorema Sea R el radio del círculo circunscrito del △ABC y sean las longitudes de sus lados a = BC. Se tiene que Presúmase ahora que A b= sen D a . Demostración: Se considerarán dos casos: si A b es agudo agudo. Sea DC un diámetro. 4R . Así a a b= = . D b = sen D b= a . senos. 2R de donde queda demostrado el teorema. Entonces [△ABC] = sr. 2 2 2 ❑ 900 Teorema Las cevianas a los puntos de contacto de un triángulo con su círculo inscrito concurren. . b HC B cot Bb HAC cot Cb HB A cot A HC B HAC HB A Luego las alturas concurren gracias al Teorema de Ceva (Teorema 868). Luego AF BD CE · · =1 FB DC EA y las cevianas concurren gracias al Teorema de Ceva (teorema 868). Sea s= a+b+c el semi-perímetro. =⇒ = = = · · = 1. ❑ 898 Definición El punto de concurrencia de las bisectrices angulares de un triángulo se llama el incentro. ❑ 894 Teorema Las alturas de un triángulo concurren. 899 Teorema Sea r el radio del círculo inscrito del △ABC y sean las longitudes de sus lados a = BC. 2 4R por la ley de los senos. . 897 Teorema Las bisectrices angulares de un triángulo concurren. 896 Definición El triángulo órtico del △ABC es el triángulo cuyos vértices son los pies de las alturas del △ABC. FB = BD y CE = EA. 2 Demostración: De a figura 7.79 se tiene [△ABC] = [△IBC] + [△ICA] + [△IAB] = ar br cr + + = sr. Demostración: Obsérvese que b BHA CHB cot Cb AHC BHA CHB cot Bb AHC cot A . CB AC AB AB BC AC′ BA′ CB′ AC = = = . =⇒ · · = 1. ya que tangentes desde un punto a un círculo son congruentes.186. Demostración: En la figura 7.188 Chapter 7 Demostración: Se tiene [△ABC] = ab sen Cb abc = . AC′ C′ B BA′ A′C CB′ B′ A C′ B A′C B′ A y el resultado se obtiene por el Teorema de Ceva (teorema 868). AF = EA. ❑ . . ❑ 895 Definición El punto de concurrencia de las alturas de un triángulo se llama el ortocentro del triángulo. Demostración: Gracias al Teorema de la bisectriz (Teorema 693). b = AC y c = AB. 188: Círculo de nueve puntos de Euler-Feuerbach. Esto implica que G′ = G. Demostración: Como BC es un lado común al △ABC y △HBC. C B K HA MA C Figure 7. 903 Teorema (Recta de Euler) En un triángulo dado. 902 Definición El triángulo de Gergonne del △ABC es el triángulo cuyos vértices son los pies de las las cevianas a los puntos de contacto de los lados del △ABC. MA MB IJ π ÷ y MC MA KI son rectángulos.❑ A A I HB HC HC H MC H MC MB J O HA MB N G B HB MA Figure 7. el baricentro de △ABC. ❑ 904 Teorema Los pies de las tres alturas de cualquier triángulo. MB MC JK es un rectángulo. 901 Definición El punto de concurrencia de las cevianas a los puntos de contacto de los lados de un triángulo con su círculo inscrito es el punto de Gergonne.185: El incentro. senC senC Si OH y AMA se intersecan en G′ entonces △AG′ H ∼ MA G′ O y AG′ = 2G′ MA . K ambos segmentos MB MC y JK son paralelos a BC y 2MB MC = 2JK = BC. AH es lado común a △BAH y △CAH. Luego AH = AHB c cos A = = 2R cosA = 2OMA .186: Punto de Gergonne y triángulo de Gergonne. De igual manera. el circuncentro O y el baricentro G son colineales y se satisface OG 1 = . Teorema 897. Luego MA I.A A F F E ′ C Puntos y rectas notables de un triángulo E L B′ I 189 B B D A ′ D C C Figure 7. Como M . y los puntos medios de los tres segmentos de los vértices al ortocentro todos yacen en el mismo círculo. De igual manera. MB y J.187: Recta de Euler. el ortocentro H. el A HA I = 2 círculo con MA I como diámetro pasa por HA . llamado el círculo de nueve puntos de Euler-Feuerbach. MB J y MC K son tres diámetros de un círculo. De igual manera se demuestra que pasa por HB y por HC . MB MC JK es un paralelogramo y como BC ⊥ AH. De aquí. . Figure 7. GH 2 Demostración: Sea HB la proyección del ortocentro H en el lado AC. los puntos medios de los tres lados. cuyos otros dos lados son bisecados respectivamente por MC . de donde MC J y MB K son paralelos a AH y 2MC J = 2MB K = AH. De igual manera se demuestra que AE = NB por lo tanto MN = AE + AF. ya que el radio del circuncírculo del triángulo medial es . Demuéstrese que B ese el incentro del △AEF y que MN = AE + AF. Se construye una recta paralela a EF a través de B cortando a C1 en M y a C2 en N. AO 1 B = 2AFB y MO1 F = 2MBF = 2EFB. en tanto el conjunto de puntos que satisface la segunda condición es un arco. como en la figura . el incentro del triángulo.❑ 2 906 Ejemplo Dos círculos C1 (con centro O1 ) y C2 (con centro O2 ) se intersecan en los puntos A y B. Al extender la recta O1 B. La intersección de estos dos lugares geométricos es obviamente única. Al extender la recta O2 B.190: Teoremas 907 y thm:criterio-ortocentro. R . Por otra parte. esta se interseca con C2 en E.189: Ejemplo 906. Como B yace el bisector angular interior del AFE. ◭ 907 Teorema Un punto P dentro del △ABC es el incentro del triángulo si y sólo si ‘y 1. Figure 7. de donde se ha demostrado que B es el incentro del △AEF. P yace en el bisector angular del ángulo CAB ◦ ‘ ‘ = 180 + CAB .191: Teorema 909. Figure 7. BPC 2 Demostración: Es claro que el incentro de un triángulo satisface las condiciones mencionadas.190 Chapter 7 905 Teorema El radio del círculo de nueve puntos es del triángulo medial. 2. Por simetría. esta se interseca con C1 en F. ◮Resolución: Obsérvese que △O1 AO2 ≡ O1 BO2 y por tanto ◦ ◦ ÷ ÷ ’ ÷ O 1 AO2 = O1 BO2 = 180 − O1 BF = 180 − O1 FO2 y así AO1 FO2 es un cuadrilátero cíclico. ❑ B B A D P O1 O2 C M F P A C A B E N Figure 7. la mitad del radio del círculo circunscrito. . ’ ‘ ÷ ‘ ’ ‘ Ahora bien. Su centro es el circuncentro 2 Demostración: Al pasar el circuncírculo medial y el círculo de nueve puntos por MA . E también yace en este círculo. ambos círculos R coinciden. de donde se deriva la aserción. MB y MC . se ha demostrado que AF = MB. el conjunto de puntos en el plano que satisface la primera condición es un segmento de recta dentro del triángulo. Demostrar que J es el incentro del △CEF. La mediatriz D el punto medio del arco AB perpendicular de OA interseca con C en E y F. 908 Teorema Un punto P dentro del triángulo agudo ABC es el ortocentro del triángulo si y sólo si 1. △OAE es equilátero. F yace en el arco ‘ = 1 AOE ‘ = 30◦ = ACF. Ya que △AOC > 60◦. 2 Luego AJ = DO = AO = AE. J es el incentro del △CEF. Demuéstrese que F es el ortocentro del △ABC si y sólo si HD CF y H yace en el circuncírculo del △ABC. ‘ de donde J yace en el bisector ı Se sigue que ACF ‘ = AEF ‘ = 1 = 30◦ y ACE menor AC. D es el circuncentro del △BPC. Sea 910 Ejemplo (IMO 2002) Sea BC un diámetro del círculo C con centro O. ◭ A F D E B A A E F J C O B D C B P C D Figure 7. Sea H el ortocentro b D es un punto en BC tal que ADB 911 Ejemplo (CMO 1999) En el triángulo agudo ABC. ‘ luego AC DO y por lo tanto ADOJ es un paralelogramo.Puntos y rectas notables de un triángulo 191 La siguiente aserciones son evidentes. CAB ‘ Un punto P en el segmento AD es en el 909 Teorema Sea D un punto en el circuncírculo del △ABC tal que AD biseca al CAB. CF ⊥ AB ‘ + BFA ‘ = 180◦. ◮Resolución: Por el teorema 908 se necesita demostrar que 1. ‘ es obtuso. ◮Resolución: Como EA = EO y OE = OA. Sea A un punto en C tal que AOC ı que no contiene a C. La recta a través de O paralela a DA interseca a AC en J.194: Teorema 912.192: Ejemplo 910. AP ⊥ BC y ‘ + CPB ‘ = 180◦. incentro del triángulo si y sólo si DB = DP (o lo que es equivalente. b > B. 2. ‘ > 60◦ . En este caso. Por el teorema 909. 2 2 ‘ Ahora bien. C del △ABD. 2. BCA . El punto F está en el interior del △ABC y en el circuncírculo del △ABD. DC = DP). angular ECF. Figure 7. ACB ‘ = 1 AOB ‘ = DOB.193: Ejemplo 911. Figure 7. M =. respectivamente. Se necesita demostrar ‘ = ADB ‘ = ahora que (2) es satisfecha si y sólo si H yace en el circuncírculo del △ABC. De igual manera. satisfaciendo AB en P y al lado AC en Q. [BC] y [CA]. entonces A. Obsérvese que PAE ◦ ‘ = 90 y PE = PF = 2. Ejemplo 916 Teorema (Intersección de cuerdas) Sea C un círculo y P un punto no en C . Recíprocamente. Teorema Figure 7. 915 Problema Por el baricentro G de un triángulo se traza una recta que corta al lado 914 Problema En el △ABC.❑ Tarea 913 Problema Demuéstrese que el circuncentro de un triángulo coincide con el orto- centro de su triángulo medial. B. Luego. Así. Sea L una recta pasando por P que interseca a C en los puntos A y B.15 Potencia de un punto con respecto a un círculo A A A P ′ T E D P B′ A′ A B B F C B B′ Figure 7. AB BC CA 7. P es además el ortocentro del triángulo medial del △ABC y el circunradio del △DEF es 2.196: 916.195: 916. Teorema Figure 7. lo que implica que A yace en el segmento EF y PA es la mediatriz perpendicular de PAF EF. ‘= Demostración: Sean D. B. esto es. los puntos L. el circunradio del △ABC es 1. son cuatro puntos no alineados. N yacen sobre los segmentos [AB]. se tiene HD ⊥ AB. ′ . B y C. y por lo tanto el circunradio del △ABC es la mitad del circunradio del △DEF. PA QA 4 AL BM CN = = . Demostrar que los baricentros de los triángulos △ABC y △LMN coinciden. Demuéstrese que PB QC 1 · ≤ . F los puntos en cada círculo diametralmente opuestos a P. si L′ es otra recta que pasa por P y que interseca a C en A′ y B′ . A′ . Demuéstrese que P es el ortocentro del △ABC y que circunradio de △ABC es 1. se tiene PA × PB = PA′ × PB′. Entonces la cantidad PA × PB es independiente de L. si las rectas AB y A′ B′ se encuentran en P y si PA · PB = PA · PB′ como segmentos dirigidos. Obsérvese que AFB ◦ ‘ ‘ 180 − AHB. si A. Así (1) es equivalente a HD ⊥ CF. PB y PC son las mediatrices perpendiculares de PD y DE respectivamente. que equivale a decir que H yace en el circuncírculo del △ABC. E.192 Chapter 7 Como H es el ortocentro del △ABD. Como P es el circuncentro del △DEF. ◭ 912 Teorema (Teorema de los tres círculos de Carnot) Tres círculos de radio unidad se encuentran en el punto P y las otras intersecciones de cada dos de ellos son los puntos A. A′ . En adición. (2) se convierte en ACB = AHB. B′ yacen en el mismo círculo. el △ABC es el triángulo medial del △DEF. B.197: 921. Sea T un punto de tangencia desde P. dando PA/PB = PA /PB. Entonces SC = SI si y sólo si SI · IA = SC · IA = 2R sen α · = 2rR.198: Teorema 918. completando la demostración. Entonces OI 2 = R2 − 2Rr. Entonces BC es tangente al círcul menor si y sólo si IO = R(R − 2r). respecti- vamente. Así PT PB = . PA′ PA Recíprocamente. Si R > 2r entonces existe un número infinito de triángulos no semejantes teniendo a R como circunradio y r como inradio. Demostración: Sea S un punto en el círculo mayor tal que AS es la bisectriz angular del BAC. En virtud del teorema . PA · PB = PA′ · PC =⇒ PB′ = PC. b BC es tangente al círculo menor si y sólo si BCI A su vez. se tiene B′ = C. Además. C S I M B N O A Figure 7. . d = ICA. ‘ Además. y sean AB y AC dos»cuerdas en el círculo mayor. ❑ 917 Definición La cantidad PA × PB en el teorema 916 se llama potencia de P con respecto al círculo C . ❑ 919 Corolario (Teorema de Euler) Sea I el centro del círculo inscrito y O el centro del círculo circunscrito al triángulo △ABC. si y sólo si R ≥ 2r. Como PB′ y PC son segmentos dirigidos. Entonces △PTA ∼ PBT y △PBA′ ∼ △PB′ A. Supóngase ahora que P está en el exterior del círculo. lo que equivale a d 2 = R2 − 2rR. Luego PA · PB = PA′ · PB′ . esto es PA · PB = PA · PB . en donde d = IO. así △PAA ∼ △PB B. PA PT PB PB′ = =⇒ PT 2 = PA · PB = PA′ · PB′ . esto sucede si y sólo si SCI ya que d = ICA d + IAC d = ICA d + SCB. SCI d = CIS d si y sólo si SC = SI. ambas tangentes al círculo menor.Potencia de un punto con respecto a un círculo 193 ′ P = PBB ‘ ‘′ Demostración: Supóngase primero que P yace en el interior del círculo. De la figura 7. Sea MN = 2R el diámetro del círculo CIS r mayor que pasa por I y O. Luego se tiene SI · IA = 2rR si y sólo si (R − d)(R + d) = 2rR. ‘ Trácese CI y CS. 918 Teorema Un círculo de radio r con centro I está dentro de un círculo de radio R > r y centro O. en donde sen α ‘ α = CAS. 920 Corolario Dos reales r > 0 y R > 0 son el radio del círculo inscrito y el radio del círculo circunscrito del △ABC. sea C el circuncírculo del triángulo △ABA′ . como en la figura 7. d d = CIS. SI · IA = MI · IN = (R − d)(R + d). La recta PA′ corta C en un punto C tal que PA·PB = PA′ · PC. Sea A un punto arbitrario en el círculo mayor.196. R = 2r si y sólo si el △ABC es equilátero.195 se tiene AA ′ ′ ′ ′ ′ ′ ′ ′ ‘ ‘ y APA = BPB . La recta BP interseca al círculo con diámetro BD en los puntos B y N. intersecando XY en E ′ . Sea P un punto en XY distinto de Z. D cuatro puntos alineados en este orden. CA CB CF AB × CD pero esta última cantidad es 1. Obsérvese que ZE ′ ZE ′ ZP ZA ZC = · = · . ◭ 922 Ejemplo Sean A. La recta XY interseca la recta BC en el punto Z. Demuéstrese que las rectas AM.19) Por el teorema 916 ZA · ZC = ZX · ZY = ZB · ZD. Demuéstrese que AE = CF. Ejemplo Figure 7. . DN y XY son concurrentes. AD AE = .C. Trácese AE ′ paralelo a BN. Las cuerdas A1 B1 y A2 B2 se intersecan en el punto medio P de AB. ZE ZP ZE ZB ZD (7. por el Teorema de la bisectriz angular. Los círculos con diámetros AC y BD se intersecan en los puntos X y Y . XY son las alturas del △ADE y por lo tanto. Se concluye que AM.199: 922. concurrentes. Se quiere demostrar que E = E ′ . La recta CP interseca al círculo con diámetro AC en los puntos C y M. ◮Resolución: Trácese DE paralelo a CM. AC AB CD CF AE AD × CB = =⇒ = = 1. Demuéstrese que C1C2 AB. B. Las tangentes al círculo desde A1 y B1 se intersecan en C1 y las tangentes desde A2 y B2 se intersecan en C2 . ◮Resolución: Por el teorema 916.194 Chapter 7 b del △ABC con D yaciendo en AC. DN.200: 923. intersecando XY en E.19) da ZE = ZE ′ . Ejemplo 923 Ejemplo AB es una cuerda de un círculo que no es un diámetro. El circuncírculo del △BDC 921 Ejemplo (SPCMO 1996) Sea BD la bisectriz angular del B interseca a AB en E y el circuncírculo del △ABD interseca a BC en F. Por lo tanto (7. La recta CP interseca al círculo con diámetro AC en los puntos C y M. ◭ E M N X A Z B O B2 P N D C B1 M P K A A1 B A2 C2 C1 Y Figure 7. Evidentemente.Tarea 195 ◮Resolución: Sea O el centro del círculo. las mediatrices perpendiculares ’ = ONP ‘ = 90◦ . P. respectivamente. C2 . ◭ Tarea 924 Problema Demostrar que si las cuerdas A1 B1 and B1C1 en el circuncírculo del △ABC son tangentes al incírculo. Esto implica que de A1 B1 y A2 B2 . la cuerda C1 A1 (en el circuncírculo) es también tan- gente al incírculo. por lo que O. OM y ON son. OC2 interseca a A2 B2 en N y OC1 interseca a AB en K. . completando la demostración. ÷ ’ ‘ Se tiene pues que OC 1C2 = ONM = OKA. dígase que OC1 interseca a A1 B1 en M. M. N yacen en el mismo círculo. N yacen en el mismo círculo. Así. C1 . de donde OM · OC1 = OA21 = OB22 = ON · OC2 . Ahora se demostrará que M. Obsérvese que △OA1C1 y △OB2C2 son triángulos rectángulos. OMP ◦ ’ ’ ’ ‘ ONM = OPM = 90 − MOP = OKA. 2n}. . Luego si se sacan 27 + 25 + 1 = 53 boletas. al menos 3 tendrán la misma suma. desigualdad imposible al ser 0 < α < 1. y como d + b + bc = 2. digamos 2a m y 2b m. Entonces como bd = 2. 2. Cada una de las otras 25 sumas aparece al menos tres veces. Presúmase que a. {2n − 1. . En cuadrando. 23 20 24 Divídase al cuadrado en 4 subcuadrados congruentes. . 20 Decompóngase el conjunto en los n pares {1. Pero como la distancia máxima en un subcuadrado es su diagonal de 2/2 unidades de longitud. lo que implica que α (1 − α ) ≤ 0. 2n}. dos tendrán la misma parte impar. Las sumas 1 y 27 se obtienen de manera única (en 100 y 999). . de donde d es par. lo que hace tanto a b como a c nones. Luego d + b sería non. contradicción. d son íntegros. Como solamente hay n enteros impares en el conjunto {1. bc debería ser non. 3}. . tendrá A √ 9 Si a ≤ α entonces α ≤ α 2 . {2. contradicción.Appendix A Indicaciones y respuestas 8 Presúmase que AC ≥ BC y colóquese D en el segmento de recta AC de tal manera que AD = BD. se tiene 102000 104000 1− 1 102000 2 1 + < α2. Dos de los cinco puntos caerán en un √ subcuadrado. Luego si a < b se tendrá que 2a m dividirá a 2b m. Luego △ADB es isósceles en D y se b = Bb. 102000 1− Ya que − 1 1 + < 0. b. Así bd = 2. de los n + 1 enteros tomados. Como siempre se tomarán dos enteros consecutivos. 21 Cada entero es de la forma 2a m en donde a ≥ 0 es entero y m es impar. 2}. ad + bc = 2. a + c = 2. estos serán relativamente primos. . el uno par y el otro non. Se dará otra solución en el problema 47 22 Hay n residuos posibles diferentes cuando se divide a un entero por n. el resultado se cumple. 10 Se tiene 1 − 1 < α < 1. así entre n + 1 enteros diferentes habrá dos dejando el mismo residuo al dividirse por n. Así pues ad es par y ad + bc = 2 no puede ser. Su diferencia será divisible por n. c. b y d deben ser de paridad opuesta. con lados paralelos al cuadrado original. 102000 104000 < 1− 1 102000 − 1 102000 + 1 104000 < α 2. ya que ad es par y bc non. . . 38 Hay 27 sumas distintas. 196 . d + b + bc = 2. 11 Se tiene x4 + 2x2 + 2x + 2 = (x2 + ax + b)(x2 + cx + d) = x4 + (a + c)x3 + (d + b + ac)x2 + (ad + bc)x + bd. de aquí que es verdadera para todos los conjuntos de 2n números 1. 2. . Mn+1 contiene tanto a 2n − 1 como a 2n. La primera proposición se deduce del hecho de que de los números en Mn−1 . En utilizando P(n − 1). Como k2 − 1 = (k2 − 1)(k2 + 1). .Indicaciones y respuestas 197 √ √ √ √ √ 45 Sea P(n) la aseveración: “(1 + 2)2n + (1 − 2)2n es par y (1 + 2)2n − (1 − 2)2n = b 2 para algún b ∈ N. basta demostrar (I) Que ningún número en Mn−1 es divisible por n y (II) Que n no es divisible por número alguno en Mn−1 . .e. . Esto es √ √ √ √ (3 + 2 2)(1 + 2)2n−2 + (3 − 2 2)(1 − 2)2n−2 . . . 3. el resultado es el mismo que en los casos 1 y 2. Presúmase que P(n − 1) es cierta para n > 1. Se demostrará que si así occurriese sería posible seleccionar de entre los 2n − 2 números 1. Quítese el número 2n − 1 del conjunto Mn+1 . ninguno de los cuales es mayor que 2n − 2. Aquí se dará una solución por inducción. 2n. estableciendo la veracidad de P(n). 2n − 2. . Para hacerlo. 2. 2n. Hágase la hipótesis que 2n+2 |k2 − 1. 2n. y 2n es divisible entre n. y demostremos que 2n+3 |k2 − 1. aún cuando se agrega el número n al conjunto Mn−1 . La segunda se deduce del hecho de que 2n no es divisible por número alguno de en Mn−1 . en caso contrario. Se observan los siguientes casos: 1. Quítese un número arbitrario del conjunto Mn+1 .. la aseveración es cierta. Como el conjunto Mn+1 no contuvo dos números de los cuales uno fuera divisible por el otro. Mn+1 contiene a 2n pero no a 2n − 1. ninguno será divisible por cualquier otro. . . presúmase que √ √ (1 + 2)2(n−1) + (1 − 2)2(n−1) = 2N para algún entero N y que √ √ √ (1 + 2)2(n−1) − (1 − 2)2(n−1) = a 2 para algún entero positivo a. ninguno es mayor que 2n − 2 y ninguno de ellos es divisible por otro cualquiera. . se han encontrado n + 1 números tales que ninguno de ellos es divisible por cualquier otro. . vemos que 2n+2 divide a(k2n − 1). . Antes que todo. A continuación agréguese el número n al conjunto Mn−1 . un conjunto conteniendo n números tales que ninguno de los n números sea divisible por cualquier otro. i. De aquí que. de donde el problema se reduce a demostrar que 2|(k2n + 1). Así se ha demostrado que si la proposición es falsa para los 2(n − 1) números 1. Veamos 1. con n ≥ 2 . es sólo necesario el demostrar que no existe dos números tales. Mn+1 contiene a 2n − 1 pero no a 2n. obsérvese que el número n no puede pertenecer al conjunto Mn+1 . 2. que es par y que √ √ √ (1 + 2)2 − (1 − 2)2 = 4 2. . . 2. Pero esto es obvio ya que el impar k2n torna a k2n + 1 par. 2. donde n es un número natural. 4. y por lo tanto divisible n n+1 n+1 n n por 8. . Ninguno de éstos es divisible por cualquier otro. . . . Entonces quedan n números ninguno de los cuales es mayor que 2n − 2. el conjunto Mn+1 contendría a dos los números n y 2n. también debe ser verdadera para los 2n números 1. 2. 46 Para n = 1. 3. Quítese ahora los dos números 2n − 1 y 2n del conjunto Mn+1 . . Denótese por Mn−1 al conjunto de los n − 1 números que quedan. obteniendo de este modo un conjunto de n números. entre los n números restantes. Así P(1) cierta es. 2. 2n − 2. . si la proposición es verdadera para los 2(n − 1) números 1. 4. Por lo tanto. . Falta demostrar que de estos n números. . 47 Este es el problema 47. La proposición es verdadera para los dos números 1 y 2. Considérese ahora √ √ √ √ √ √ (1 + 2)2n + (1 − 2)2n = (1 + 2)2 (1 + 2)2n−2 + (1 − 2)2 (1 − 2)2n−2 . de nuevo. Quítese el número 2n del conjunto Mn+1 . Nótese que la solución utilizando el principio de las pichoneras es más sucinta. 2. un entero par y de manera semejante √ √ √ √ √ (1 + 2)2n − (1 − 2)2n = 3a 2 + 2 2(2N) = (3a + 4N) 2. 2n − 2. el conjunto Mn−1 tampoco contendrá tales números. . ninguno es mayor que 2n − 2.” Si n = 1. . Supóngase que de entre los 2n números 1. Denótese este conjunto de n + 1 números por Mn+1 . esto simplifica a √ √ 12N + 2 2a 2 = 2(6N + 2a). . Mn+1 no contiene al número 2n − 1 ni al número 2n. entonces vemos que √ √ (1 + 2)2 + (1 − 2)2 = 6. ya que k2 − 1 = (k − 1)(k + 1) es el producto de dos enteros pares consecutivos. Efectivamente. Ahora. 10 = 5 + 5. múltiplo de 9. 2(k + 2) estableciendo el resultado para k + 1.198 Anexo A 48 Para n = 1. y por la hipótesis de inducción. Presúmase que k3 + (k + 1)3 + (k + 2)3 = 9N. n − 2 y n − 1. 49 Utilizarase inducción robusta. esto es. en virtud de la hipótesis de inducción. y n − 1 pesos. Obsérvese que 8 = 3 + 5. Esto significa que fn fn+2 = fn ( fn+1 + fn ) = fn fn+1 + fn2 = fn fn+1 + fn−1 fn+1 − (−1)n = fn+1 ( fn + fn−1 ) + (−1)n+1 = fn+1 fn+1 + (−1)n+1 . fn2 = fn−1 fn+1 − (−1)n . 3x2 + 5y2 = n − 1 =⇒ 3(x2 + 1) + 5y2 = n + 2. de donde la aserción es cierta para n = 1. también se puede pagar las cantidades n. y que la aserción es cierta para n. esto es fn−1 fn+1 = fn2 + (−1)n . Usando fn+2 = fn+1 + fn . n + 1 y n + 2. n− 1 se pueden pagar. La aseveración queda demostrada por inducción robusta. ya que 03 + 12 + 23 = 9. . Presúmase que para k > 1 4 2(1 + 1) 1 4   1− Å 1 1 ··· 1− 9 (k + 1)2 ã = k+2 .  1 1− 4   Å 1 1 1− ··· 1− 9 (k + 2)2 Å ã = = = = ã 1 k+2 1− 2(k + 1) (k + 2)2 Å 2 ã k + 4k + 3 k+2 2 2(k + 1) Å (k + 2) ã (k + 1)(k + 3) k+2 2(k + 1) (k + 2)2 k+3 . Demostrarase que también se pueden obtener soluciones para 3x + 5y = k con k = n. de donde se puede pagar 8. Presúmase que se puede pagar n − 3. o 10 pesos con las susodichas monedas. 51 Para n = 0 esto es cierto. en donde N es un entero. como se quería demostrar. 2(k + 1) Por la hipótesis de inducción. 3x + 5y = n − 3 =⇒ 3(x + 1) + 5y = n. 50 El resultado es inmediato para n = 1 ya que 1 −  1− 1+2 1 = . que 3x + 5y = k tiene soluciones no negativas para k = n − 3. Se demostrará que (k + 1)3 + (k + 2)3 + (k + 3)3 es también un múltiplo de 9. Esto es 9N + (k + 3)3 − k3 = 9N + (k3 + 9k2 + 27k + 27) − k3 = 9N + 9k2 + 27k + 27 = 9(N + k2 + 3k + 3). n − 2. 9. Supongamos que n > 1. n+ 2. de donde se colige el resultado. 9 = 3 + 3 + 3. 3x1 + 5y1 = n − 2 =⇒ 3(x1 + 1) + 5y1 = n + 1. y así si las cantidades n− 3. Pero (k + 1)3 + (k + 2)3 + (k + 3)3 = 9N + (k + 3)3 − k3 . tenemos 0 · 1 = f0 f1 = 12 − (1)1 = f12 − (1)1 . n− 2. n+ 1. Tenemos dos casos: o bien todos los elementos de F son pares o bien todos nones. . De cierto. y tienen la misma paridad. Si a1 > b1 entonces (a1 − b1 ) + a2 + · · · + an = b2 + · · · + bn . se sigue que m > 1. ′ a = b. . podemos aplicar la hípótesis de inducción. = c2n+1 = 1. . . . sea x ∈ C y sean A (con suma de elementos a) y B (con suma de elementos b) dos subconjuntos con n elementos cada uno verificando ′ C \ {x} = A ∪ B. Ahora demostraremos la igualdad de todos los elementos por inducción. . . . f2 /2. ( f2n+1 + 1)/2} porque max ( fi + 1)/2 = (t + 2)/2 < t 1≤i≤2n+1 y al ser todas las ( fi + 1)/2 idénticas también lo serán las fi . . Como n ≤ a1 + a2 + · · · + an < mn. . . De igual manera se demuestra que n > 1. b2 + · · · + bn < mn − . pues se podrán suprimir estos términos y se logrará una suma restante idéntica. . 54 Razonamos por inducción sobre m + n. f2 . . . Para m + n = 4 se tiene m = n = 2. donde n se mantendrá fijo y se inducirá en el máximo de los elementos de C = {c1 . . Si max 1≤i≤2n+1 = 1 entonces c1 = c2 = . Si a1 = b1 no cabrá nada que demostrar. c2 . Si todos los elementos de F son pares. por ser todos los elementos estrictamente positivos. . f2n+1 } un conjunto de enteros positivos no nulos verificando la propiedad del enunciado con max 1≤i≤2n+1 fi = t + 1. sea y ∈ C y sean A (con suma de elementos a ) y B (con suma de elementos b′ ) dos subconjuntos con n elementos cada uno verificando C \ {x} = A′ ∪ B′ . A ∩ B = ∅.Indicaciones y respuestas 199 52 El enunciado es obvio para n = 1. A′ ∩ B′ = ∅. de donde se colige el resultado. f2n+1 /2} porque max 1≤i≤2n+1 fi /2 = (t + 1)/2 < t y al ser todas las fi /2 idénticas también lo serán las fi . Si c es la suma de todos los elementos en C entonces c = x + a + b = x + 2a y también c = y + a′ + b′ = y + 2a′ . ( f2 + 1)/2. Si 1+ 1 1 1 +···+ 2 < 2− 2 n 2 n para n > 1 entonces 1+ 1 1 1 1 +···+ < 2− + =. n (n + 1)2 n+1 Pero esto de inmediato resulta de la desigualdad n(n + 1) < (n + 1)2 =⇒  1 1 1 1 1 1 − = 1− − < 2 n n(n + 1) n n n + 1 (n + 1)  = 1 . aplica la hipótesis de inducción a {( f1 + 1)/2. y mn = n(m − 1). y las igualdades posibles son 1 + 1 = 1 + 1 y 1 + 2 = 2 + 1. donde m + n = k + 1. c2n+1 }. n+1 53 Primero demostramos que todos los elementos de C poseen la misma paridad. . aplica la hipótesis de inducción a { f1 /2. ′ De manera semejante. Sea F = { f1 . Si todos los elementos de F son nones. n (n + 1)2 22 (n + 1)2 Se demostrará que para n > 1 2− 1 1 1 < 2− + . Presumáse pues que el enunciado es cierto cuando max 1≤i≤2n+1 ci = t > 1. Supóngase que el resultado es cierto para k = m + n ≥ 4 y considérese a1 + a2 + · · · + an = b1 + b2 + · · · + bn < mn. obteniendo el resultado. . . a′ = b′ . m Como n + (m − 1) = k. Así y − x = 2(a′ − a) y x. Sin perdida de generalidad se puede presumir que a1 es la mayor de todas las ai y que b1 es la mayor de todas las bi . Si ninguno de ellos es el centro del disco. dado n. .3 proveen una descomposición en 4. lo que se puede demostrar con otra inducción. lo que es imposible. 82 52 84 Pista: 22225555 + 55552222 = (22225555 + 45555 ) + (55552222 − 42222 ) − (45555 − 42222 ). Por la ley de los cosenos de Al-Kashi ‘ AB2 = OA2 + OB2 − 2OA · OB cos A OB =⇒ AB < 1. y 10 subtriángulos. . . 196 Los factores de 295 son 1. 2. lo que es una contradicción. . Figure A. A7 los siete puntos en cuestión. Luego 219 = 524288 < 1000000 < 1048576 = 220 . digamos 2r−1 < s < 2r . 180 2 181 $0. respectivamente. 6. . . Figure A.1: 4 subtriángulos equiláteros.200 Anexo A 55 Las figuras A. 4.1. se puede construir o bien n+ 3 triángulos o n+ 5 triángulos. una potencia de 2. . Si s yace estrictamente entre dos potencias de 2. . . a la manera del problema 49. 161 63 167 Pista: Demuestre primero que 320 ≡ 1 mod 100. Por lo tanto.2. entonces s < 2r < 2s < 2r+1 . Figure A. Como claramente se tiene am+1 + am > am + am−1 se deberá tener am+1 − am < am − am−1 para m ≥ 2. son los factores 220 . 197 En utilizando . entonces el menor ◦ ’ entre los ángulos A i OA j es estrictamente inferior a 60 . 81 Ponga x = 123456789. 48 son 33 en total y están del mismo lado del diámetro que une al punto 15 con el 49. . . 159 Pista: n2 + 15n + 122 ≡ n2 + 3n + 2 = (n + 1)(n + 2) mod 6. Así pues. A.3: láteros. Para cada uno de estos hay un punto correspondiente y opuesto en la circunferencia. 22 . Observemos que 210 = 1024 y por lo tanto 220 = 1048576. Obsérvese ahora que toda n ≥ 6 puede ser escrita de la manera 3x+ 5y = n. .2: 6 subtriángulos equiláteros. . 17. Estos constituyen un total de 95 − 20 + 1 = 76 factores. 2s] contiene a 2r . 295 . 3. 61 Si la tuviere entonces a2m+1 − a2m = a2m − a2m−1 m = 2.73 195 Los puntos 16. 10 subtriángulos equi- 56 Si s fuese una potencia de 2 entonces no hay nada que demostrar. Esto se puede escribir como a2 − a1 > a3 − a2 > a4 − a3 > . . . . y sean A1 . 295 los mayores de 1000000. por lo que el intervalo [s. . . Entonces x2 − (x + 2)(x − 2) = 4. y A. A2 . Sean A y B los puntos correspondientes a este ángulo. 221 . Así pues hay un total de 2 · 33 + 2 = 68 puntos en total. . una sucesión infinita de enteros positivos estrictamente decreciente. . 65 Sea O el centro del disco. el razonamiento anterior nos dice que d(n) = (a1 + 1)(a2 + 1) · · · (as + 1) y σ (n) = pa11 +1 − 1 pa22 +1 − 1 pas +1 − 1 · ··· s . Así pues. contamos 567 páginas a partir de la 100. p1 − 1 p2 − 1 ps − 1 199 Para escribir las primeras nueve páginas. el número de divisores positivos de n y la suma de los divisores positivos de n. se utilizaron nueve dígitos. 198 Presumiremos conocido el que los enteros naturales se pueden factorizar en factores primos de una manera única. 90 · 2 = 180 enteros de 2-dígitos. Nos quedan 1890 − 189 = 1701 dígitos que usar. 900 · 3 = 2700 enteros de 3-dígitos. Entonces pues. Por lo tanto. hay (1 + 8)(1 + 9)(1 + 3) = 320 factores. Hasta ahora hemos utilizado 189 dígitos. Si el libro llegase hasta la página 999. se utilizaron 2 · 90 = 180 dígitos. Para escribir las 99 − 10 + 1 = 90 páginas entre la 10 y 99 inclusas. al expandir el producto (1 + 2 + 22 + · · · + 28 )(1 + 3 + 32 + · · · + 39 )(1 + 5 + 52 ) obtenemos todos los factores de 28 39 52 y sólo factores de este número. . La suma de los divisores la obtenemos sumando las tres series geométricas anteriores: 29 − 1 310 − 1 53 − 1 · · = 467689684.Indicaciones y respuestas 201 9·1 = 9 enteros de 1-dígito. el tercer dígito de 4027. 2−1 3−1 5−1 En general. hay tantos factores como términos en el producto. el 3000-avo dígito es el tercer dígito del 28-avo entero positivo de 4-dígitos. Así pues. así pues el 3000-avo dígito debe de pertenecer a los enteros positivos de 4-dígitos. el 2. Quedan pues 3000 − 2889 = 111 dígitos para ser usados y como 111 = 4 · 27 + 3. si n = pa11 · pa22 · · · pas s . el respectivamente. que es mucho más que la cantidad de dígitos prescrita. las 999 − 100 + 1 = 900 páginas de tres dígitos utilizarían 3 · 900 = 2700 dígitos. Así pues. σ (n) denotan. 201 800 270 3030 271 93 273 5973 1993 309 4 310 a = −998 = b 311 2400 312 9 313 1 315 384 316 580 318 Pista: x3 ± 1 = (x ± 1)(x2 ∓ x + 1) 320 Pista: Demuestre primero que csc 2x = cot x − cot 2x. es decir. Esto quiere decir que el libro tiene 666 páginas. el número de páginas es un número de tres dígitos. esto es. un total de 9 + 180 + 2700 = 2889 dígitos han sido utilizados. donde las p’s son primos distintos y si d(n). que nos dan para 1701/3 = 567 páginas más. 344 Se tiene ( f (x))2 · f de donde  1−x 1+x   1−x 2 ( f (x))4 · f Substituya x por 1−x . . 341 Pista: Poner un = cos vn . 3. Resuelva para m. 1 + ab √ √ 1 √ . . = 642 x2  1−x 1+x  1+x 1−x . de donde se destila el resultado. ya que x es positivo. . 30 · 31 · 32 · 33 + 1 = 312 + 31 − 1 = 991. 2 1 − y 1−y 1 − y2 325 Pista: De la identidad tan x − tan y = tan x − tan y 1 + tan x tan y deduzca que arctan a − arctan b = arctan 326 Pista: De la identidad a−b . 363 Pista: cos2 x = 1 − sen2 x 371 Observe que p Así √ (x − 1)x(x + 1)(x + 2) + 1 = p (x2 + x)(x2 + x − 2) + 1 = p (x2 + x)2 − 2(x2 + x) + 1 = p (x2 + x − 1)2 = x2 + x − 1. x. k Poniendo n = 1000000 se obtiene el resultado. f (x)3 = 64x2 = 64x. .202 Anexo A 321 Pista: Observe que 1 1 y − = . . 373 Se tiene q » x+ x+ p √ √ x + · · · = 2 =⇒ x + 2 = 2 =⇒ x + 2 = 4 =⇒ x = 2. k Haciendo k = 2. n. k+1− k = √ k+1+ k deduzca √ √ √ √ 1 2 k + 1 − 2 k < √ < 2 k − 2 k − 1. = 2 =⇒ x2 = 2 =⇒ x = √ 2. 2 n+1−2 2 < k k=2 √ √ √ Como 2 2 < 3 y n + 1 > n se sigue que X √ 2 n−2 < n k=1 √ 1 √ < 2 n − 1. 372 Se tiene xx . 377 Pista: Ponga y = mx y divida las ecuaciones así obtenidas. Entonces 1+x f  1− x 2 1+x 1+x ) f (x) = 64 Divida (I) por (II). luego Xn 1 √ √ √ √ < 2 n − 2. . (I) (II) . Esto da A2 = A · A < A · B. . Póngase entonces O = (0. Esto implica que x x−1 < x x+1 en tanto cada uno de los cuatro factores sea positivo. Por la desigualdad del triángulo en el plano. 442 Sea A= 9999 1 3 5 · · ··· 2 4 6 10000 B= 2 4 6 10000 · · ··· . b + d). A a+A < . Luego 1/2 < 2/3 3/4 < 4/5 5/6 < 6/7 . 2 4 6 10000 3 5 7 10001 o sea. L = (a. 6.. b+B B a+A a < . B(a + A) = aB + AB < b b+B Por otra parte. b) y M = (a + c. se multiplican ahora una y otra columna para obtener 9999 2 4 6 10000 1 3 5 · · ··· < · · ··· . 1 2 3 4 5 6 7 9999 10000 1 · · · · · · ··· · = . ocurriendo igualdad si y sólo si los puntos son colineales. ¿Podría ser q menor? Obsérvese que > y que < . De manera semejante. Ab + AB = A(b + B) y así. 384 Pista: Ponga u = x + 2. Así. 2. . . 437 Es suficiente demostrar la desigualdad cuando a. 0). 7 11 7 18 11 7 25 18 11 < =⇒ < < =⇒ < < < . Divida una ecuación por la otra.Indicaciones y respuestas 203 378 Pista: Escriba la ecuación como (x2 − 9x − 1)10 − 10x9 (x2 − 9x − 1) + 9x10 = 0. 10 15 10 25 15 10 35 25 15 5 11 4 7 25 5 = . x2 − 1 < x2 para todo real x. 1 < k < n. OM ≤ OL + LM. 4. b d 438 Como aB < Ab se tiene a(b + B) = ab + aB < ab + Ab = (a + A)b así. A < B.. c. Así. 3. 385 Pista: Ponga u = x + y. b. Se deduce que A < 1/ 10001 < 1/100. considerando caso a caso con denominadores Como 35 7 6 15 6 10 q = 1. Pero entonces. Ahora bien. . 3 5 7 10001 y Evidentemente. d son todos estrictamente positivos. 441 Obsérvese que para un entero k. Luego entonces el menor denominador es 7.. v = y + 3. A·B = . 2 3 4 5 6 7 8 10000 10001 10001 √ y en consecuencia A2 < A · B = 1/10001. 5. 2 n! = (1 · n)(2 · (n − 1))(3 · (n − 2)) · · · ((n − 1) · 2)(n · 1) > n · n · n · · · n = nn . v = x − y. . k(n − k + 1) = k(n − k) + k > 1(n − k) + k = n. p p p (a + c)2 + (b + d)2 = OM ≤ OL + LM = a2 + b2 + c2 + d 2 . 9999/10000 < 10000/10001 Como todos los números involucrados son positivos. a c e igualdad ocurre si y sólo si = . se tiene q ≤ 7. se ve que ninguna de estas fracciones yace en el intervalo deseado. . no hay nada que demostrar. de donde se destila el resultado. 2( b + a)2 1 2 2  + d− 1 2 2 < 0. O bien Tp−1 − Tp = 2|a p |. 444 Póngase X Tm = ak − 1≤k≤m X ak . . pues una suma de cuadrados reales no puede ser estrictamente negativa. contradicción. se tiene a+b √ − ab 2 = = = √ a + b − 2 ab √ 2√ ( b − a)2 2 (b − a)2 √ √ . entonces 4 a − b2 − 1 1 1 1 + b − c2 − + c − d 2 − + d − a2 − > 0 =⇒ 4 4 4 4  a− 1 2 2  + b− 1 2 2  + c− contradicción. . . por lo que no puede ser reemplazada por un valor 2 mayor. entonces 2|a p | = |Tp−1 − Tp | > 2 max |ak |. 1 462 Si por el contrario todas estas cantidades fuesen > . Como t − s ≤ 0. o bien Tp − Tp−1 = 2|a p |. T1 . . X n | ai |=| (n − 1) ai − (−ai ) |=| Ð ai − a j |≤ i= j X i= j | ai − a j | Sumando sobre i. n X Xn i=1 | ai |≤ 2 i< j | ai − a j |. . Se asevera que þ þ þ þÐ min þTp−1 þ. Como la sucesión T0 . (r − s + t)2 − t 2 ≤ (r − s)(r + s) = r2 − s2 lo que resulta en (r − s + t)2 ≤ r2 − s2 + t 2 . Presúmase que a = b. Como √ √ √ √ (b − a)2 = ( b − a)2 ( b + a)2 .204 Anexo A 443 Para i fijo. . n Obsérvese que para a1 = a2 = . 464 Si a = b. elíjase un índice p tal que Tp−1 y Tp tengan signos diferentes. el resultado es inmediato. Tn cambia de signo. = an−1 = x y an − (n − 1) x. se tiene. T0 = −Tn . 461 De las igualdades dadas se deduce Ð 1 (x1 − x2 )2 + (x2 − x3 )2 + · · · + (xn−1 − xn )2 + (xn − x1 )2 = 0. k=1 Como una suma de cuadrados es 0 si y sólo si cada término es 0. 2 Como una suma de cuadrados es 0 si y sólo si cada término es 0. r − s + 2t = r + s + 2(t − s) ≤ r + s y así. se obtiene igualdad. m<k≤n Claramente. el resultado es inmediato. þTp þ =≤ max |ak | . 463 Se tiene (r − s + t)2 − t 2 = (r − s + t − t)(r − s + t + t) = (r − s)(r − s + 2t). 1≤k≤n 1≤k≤n 460 De las igualdades dadas se deduce que Xn 1≤k≤n (x2k − xk )2 = 0. 1≤k≤n þ þ þ þ Si de caso contrario se tuviese þTp−1 þ > max |ak | y þTp þ > max |ak |. b. 468 Como el cuadrado de todo real es positivo (a − 1)2 ≥ 0 =⇒ a2 + 1 ≥ 2a. . se tiene. b ≥ 0. Ð a2 − ab + b2 1 ≥ ⇐⇒ 3 a2 − ab + b2 ≥ a2 + ab + b2 ⇐⇒ a2 + b2 ≥ 2ab. b2 + 1 ≥ 2b. Multiplicando estas tres desigualdades. 470 De la desigualdad de la media para tres números. Entonces x+y √ √ xy ≤ =⇒ xy ≤ 50 =⇒ xy ≤ 2500. 471 Usando la identidad x3 + y3 = (x + y)3 − 3xy(x + y) dos veces. entonces a + b + c ≥ 0 y además como a2 + b2 + c2 − ab − bc − ca ≥ 0 por la desigualdad 6. 1=  a b c  1/3 · · b c a a b c + + b c a =⇒ 3 ≤ a + b + c . 3 La desigualdad deseada se obtiene en poniendo u = a3 . √ √ √ ( a + b)2 = a + b + 2 ab. De aquí se colige que a3 + b3 + c3 ≥ abc. como se quería demostrar. v = b3 .Indicaciones y respuestas 205 √ √ √ √ Se nota ahora que 2 a ≤ b + a ≤ 2 b y por lo tanto 1 (b − a)2 (b − a)2 1 (b − a)2 · ≤ √ √ 2 ≤ · 8 b 8 a 2( b + a) 465 Notar que 3a4 − 10a2 + 9 = (a2 − 1)2 + 2(a2 − 2)2 > 0. 2 de donde el máximo producto es 2500. w = c3 . 469 Sean x ≥ 0. De la misma manera. y ≥ 0 con x + y = 100.6. Se tiene igualdad si y sólo si a = b. c son positivos. a3 + b3 + c3 − 3abc = (a + b)3 + c3 − 3ab(a + b) − 3abc = (a + b + c)3 − 3(a + b)c(a + b + c) − 3ab(a + b + c) = (a + b + c)((a + b + c)2 − 3ac − 3bc − 3ab) = (a + b + c)(a2 + b2 + c2 − ab − bc − ca) Si a. 466 Para a ≥ 0. (a2 + 1)(b2 + 1)(c2 + 1) ≥ 8abc. Las desigualdades deseadas se siguen ahora de las obvias √ a + b ≤ a + b + 2 ab ≤ a + b + a + b = 2(a + b). 467 Se tiene. 2 2 3 a + ab + b que es siempre cierta. ≤ 3 b c a demostrando la desigualdad deseada. c2 + 1 ≥ 2c. Habrá igualdad si y sólo si p > : y = x2 + 1 475 Por el ejemplo 453. 477 La igualdad siniestra equivale a esto es. Entonces 4 (ab + bc + ca) − (a + b + c)2 = c2 − (a − b)2 + a2 − (b − c)2 + b2 − (c − a)2 > 0. … 4 9x2 sen2 x + 4 = 9x sen x + ≥2 x sen x x sen x  (9x sen x) 4 x sen x  √ = 2 9 · 4 = 12. que es trivial. 44 1 entonces 4 contradicción. b≥ Así pues (a + b) (b + c) (c + a) ≥ 8abc. c. abc ≥ (a + b − c) (b + c − a) (c + a − b) . c son las longitudes de los lados de un triángulo. |a − b| < c. 2 De aquí se obtiene el resultado. 1 [(a − b)2 + (b − c)2 + (c − a)2 ] ≥ 0. 476 La desigualdad propuesta es equivalente a Å x y − y z ã2  y + z − z x 2 Å z + x − x y ã2 ≥ 0. a2 − (b − c)2 = √ √ b + a − c b + c − a. como a. √ √ a + b − c a − b + c. de donde x2 + y2 = x2 + y2 + 2. . c≥ √ √ c + a − b c + b − a. b.206 Anexo A 472 Como x → x sen x es estrictamente positiva en dicho intervalo. a≥ De la misma manera. |b − c| < a. x sen x 36 6 ≥ 0. se tiene » Además. pero como esto es imposible. Así. habrá siempre desigualdad. (a + b + c)2 − 3 (ab + bc + ca) ≥ 0. 8 p > < x = y2 + 1 . que se obtiene cuando 9x sen x = 473 Ver que x2 − x +  1 1 = x− 4 2 2 Si todos los productos fuesen > 1 1 4 =⇒ x2 sen2 x = =⇒ x sen x = . con igualdad si y sólo si a = b = c. Para la desigualdad diestra. 44 a(1 − b)b(1 − c)c(1 − d)d(1 − a) > 1 . a(1 − b)b(1 − c)c(1 − d)d(1 − a) ≤ 1 . Nótese que esta igualdad es válida cualesquiera sean los valores estrictamente positivos a. Luego el valor mínimo es 12. |c − a| < b. de donde se deduce el resultado. Obsérvese que igualdad ocurre si y sólo si x = y = z. 474 Se tiene Ä 0 ≤ x− p y2 + 1 p de donde x2 + y2 + 1 ≥ x ä2 Ä + y− p p y2 + 1 + y ä2 x2 + 1 x2 + 1. b. obteniendo la desigualdad deseada. 480 Supóngase primero que cada factor siniestro es positivo. c = z + x. Sumando se obtiene la desigualdad deseada. de donde nuevamente se obtiene la conclusión. ⇐⇒ a = b = c. La igualdad se cumple si y sólo si x = y = z. 2 2 2 a= Nótese que √ √ Ð2 √ x+ y . Luego. 1 2  ac b +  ba c =a b2 + c2 ≥a 2bc con igualdad si y sólo si b = c. ≤ (abc)2 = 1. Es suficiente entonces demostrar que a + b > c. si por ejemplo. b c a de donde se obtiene el resultado. √ √ √ b + c − a + c + a − b ≤ 2 c. y > 0. b c a = = Ahora bien. b > 0. c > 0 y x2 − z2 y2 − x2 z2 − y2 + + y+z z+x x+y (a − b) c (b − c) a (c − a) b + + b c a ac ba bc + + − a − b − c. Sumando se obtiene ac ba bc + + − a − b − c ≥ 0.  b−1+ 1 c  c−1+  c−1+ 1 a 1 a  2 a−1+ 1 b  ≤ ac2 . esto es. también lo habrá de ser el cuarto. z > 0 y y+x z+y x+z . sólo b c a uno de los factores es negativo. Ya que tres de los factores son positivos.Indicaciones y respuestas 207 478 Póngase x = a + b − c. de donde se llega a la conclusión. y = b + c − a.b = . 479 Póngase a = x + y. Entonces a > 0. a − 1 + < 0. Hay igualdad si y sólo si a = b = c. Entonces x > 0. √ √ √ c + a − b + a + b − c ≤ 2 a. esto es. si x = y = z. Se tiene.  a−1+ 1 b 1 1 1 = b 1− + c b bc  b−1+ 1 c     = b 1+a− Å  = b a2 − 1 − 1 b 1 . 481 Sin pérdida de generalidad se puede suponer que a ≤ b ≤ c. Ð Ð2 a2 + b2 + c2 > 2 a4 + b4 + c4 si y sólo si (a + b + c) (a + b − c) (b + c − a) (c + a − b) > 0. De la misma manera 1 2  ac b + bc a  1 2 ≥ c.  b−1+ En consecuencia. 2 (x + y) ≥ x + y + 2 xy = cumpliéndose la igualdad si y sólo si x = y. b = y + z. b  2ã ≤ ba2 . 1 1 1 Ahora bien. Procediendo de igual manera con los otros factores se deduce que  b−1+ De aquí 1 c  c−1+  a−1+ 1 b 1 a  ≤ cb2 . De la misma manera √ √ √ √ √ √ √ a + b − c + b + c − a = x + y ≤ 2 x + y = 2 b. z = c + a − b.c = .  ba c + bc a  ≥ b. . Ahora bien. Bajo estas condiciones b − 1 + > 0 y c − 1 + > 0. entonces a < 1 y b > 1. 2a … a2 + con igualdad si y sólo si a4 = 3 ≥2 4a2 3 4 1 3 yb=− . Así. 492 Por la de sigualdad de las medias. n 2 habiendo desigualdad estricta cuando n > 1. 1 2 + ai = 1 + 1 + ai ≥ 3 (ai ) 3 . . n: n!1/n = (1 · 2 · · · n)1/n < 1+2+··· +n n+1 = . . 4 2a 497 Para toda i. 493 Por usando las desigualdades de las medias aritmética y geométrica y las medias armónica y geométrica. √ x1 + x2 + · · · + xn ≥ n n x1 x2 · · · xn y Multiplicando se obtiene la desigualdad deseada. . 4a b+ 1 . Ð 101 Ð 101 logrando igualdad cuando a = b = c = d = 1. Yn ak (1 − ak ) = k=1 Yn k=1 ak × Yn (1 − ak ) k=1 n 1X ak n ≤ !n k=1 (n − 1)n . 495 Por la desigualdad de las medias. !n n 1X (1 − ak ) n k=1 Ð 61 . a2 + b2 + c2 + d 2 + ab + ac + ad + bc + bd + cd ≥ 10 a2 b2 c2 d 2 abacadbcbdcd = 10 a5 b5 c5 d 5 = 10. n2n ≤ 496 Se tiene a2 + b2 + b 1 + a2 a  = ≥ habiendo igualdad si y sólo si b = −  1 2 3 + a2 + 2 2a 4a 3 a2 + 2 . 2. . n x x ···x x1 x2 xn n 1 2 494 Se tiene Ð (a + b + c)3 = a3 + b3 + c3 + 6abc + 3 a2 b + a2 c + b2 a + b2 c + c2 a + c2 b ≥ a3 + b3 + c3 + 6abc + 3 × 6 a6 b6 c6 = a3 + b3 + c3 + 24abc. Yn i=1 (2 + ai ) ≥ 3n Yn i=1 ! ai 1 3 = 3n . Pero por la desigualdad de las medias.208 Anexo A 491 Aplicando la desigualdad de las medias a 1. 1 1 n 1 + +··· ≥ √ . 1 (xyz) 3 Å = 1+ 1 (xyz)1/3 ã3 .. x y z xy yz zx xyz Por la desigualdad de las medias. f (x1 . . con igualdad si y sólo si x = y = z = . Entonces para todo k. x2 .. j = 1.an ) n 3 n (n − 1) . x2 . x y z (xyz) 3 Además... . 499 Póngase  P = 1+ Entonces P = 1+ 1 x Å 1+ 1 y ã  1 . xn ) ≥ con igualdad si y sólo si x1 = x2 = . = xn . 3 Así. xj 498 Se consideran los índices módulo n.... = xn = 1. j = Yn αk... xn ) = X x9i + x9j x6 + x3i x3j + x6j 1≤i< j≤n i y ai = x3i para toda i. j. .. Xn a j s − x j xj j=1 Ð = Xn aj ! Xn−1 j=1 αk.. Pero por la desiguadad de las medias 3 1 ≥ =3 x+y+z (xyz)1/3 1 y se deriva que P ≥ 43 = 64. j j=1 k=1 k=1 Xn−1 ≥ Ñ n é n1 Yn aj . se toma αk. j=1 k=1 con igualdad si y sólo si x1 = x2 = . 1 1 1 + + ≥3 xy yz zx Å Se concluye que P ≥ 1 + 3 1 (xyz)1/3 ã Å +3 1 (xyz)1/3 ã2 Å + 1 (xyz)1/3 Ç ã3 å2 1 . y usando el problema 467. Para enteros k.. Luego a1 a2 . j X n Xn−1 = a j αk...Indicaciones y respuestas 209 x j+k . n (n − 1) . z 1+ 1 1 1 1 1 1 1 + + + + + + .. 3 500 Se pone f (x1 . 1 1 1 1 + + ≥3 1 . 3 å .an = 1. f (x1 . j=1 Así. x2 .. xn ) = X a3i + a3j a2 + ai a j + a2j 1≤i< j≤n i = Ç X ai + a j 1≤i< j≤n ≥ = 1 X 3 ai + a j Ð a2i − ai a j + a2j a2i + ai a j + a2j Ð 1≤i< j≤n n n−1 X ai 3 i=1 ≥ = 1 n (n − 1) (a1 a2 .. c 2 2 ya que f es simétrica. Por la desigualdad de CBS. d) = abc + bcd + cda + dab −  176 176 abcd = bc (a + d) + ad b + c − bc . Esto es ≤f  b+c a+d a+d b+c 2 . . b. b. es suficiente demostrar que a1 . 2 Iterando. Entonces. = . 27 27 Obsérvese que f (a. . (1 · x1 + 1 · x2 + · · · + 1 · xn )2 ≤ 12 + 12 + · · · + 12 Ð Ð x21 + x22 + · · · + x2n . (n − 1) a41 + a42 + . 510 El caso n = 3 ya se ha visto en el problema 481. + a4n Ð < = ≤ a21 + a22 + . c. a3 son las longitudes de los lados de un triángulo. 508 Usando CBS en Xn (ak bk )ck una vez.. c. b. a+d . . Por simetría. d) ≤ bc (a + d) ≤  13 = 3 1 . a+d 2    = f b. se obtiene k=1 Xn k=1 Usando CBS de nuevo Xn ! 1/2 Xn ak bk ck ≤ ! 1/2 Xn a2k b2k k=1 c2k .210 Anexo A 501 Póngase  f (a. c. 2 4 4 2 ≤f  1 1 1 1 1 . . Supóngase ahora que n ≥ 4. Presúmase primero que b + c − 27 f (a. + a2n Ð2 a21 + a22 + a23 a21 + a22 + a23 X + + 2 2 (n − 1) a21 + a22 + a23 4 Ð2 !2 n a4k k=4 a2 + a22 + a23 + 1 4 Ð2 + Xn k=4 ! a4k . . Entonces Presúmase ahora que b + c − 27 f (a. . 176 bc ≤ 0. 27 asegurando el resultado en este caso. c. . . . a2 . c. de donde se obtiene el resultado. a+d a+d . 176 bc > 0. b. 2 . b. b. . c. 2 2 =f  a+d b+c a+d b+c 2 . k=1 ! 1/2 a2k b2k se obtiene k=1 Xn ≤ ak bk ck k=1 ≤ Xn ! 1/2 a2k b2k ! 1/4 k=1 Xn a4k k=1 Xn k=1 Xn ! 1/2 c2k ! 1/4 b4k ! 1/2 Xn c2k . k=1 k=1 lo que resulta en la desigualdad deseada. b. . 2 2 . 2 =f 4  b+c 1 1 a+d  . 2 . c. d) ≤ f  a+d 2 . 509 Por CBS. d) es simétrica. . 4 4 4 4 27 lo que resulta en la desigualdad. f (a. . 2 ≤f  1 b+c a+d 1 4 . d) ≤ bc (a + d) +  a+d 2  2  176 bc = f 27 b+c−  a+d 2  . Gracias a CBS. se tiene lo que quiere decir que 1 ≥ (a + b + c)2 = a2 + b2 + c2 + 2 (ab + bc + ca) . obteniendo el resultado. n−1 Se sigue que S2 − a2i 1 ≥ (S1 − ai ) S1 − ai n−1 y entonces X n S2 − a2 k k=1 525 Por la desigualdad del reordenamiento. . b = . 2 2 2 Esto quiere decir que 1 c3 d 3 + ≥ ≥ 1. es equivalente a + + ≤ 1. ab + bc + ca ≤ p a2 + b2 + c2 p b2 + c2 + a2 . Ð 2 a41 + a42 + a43 < a21 + a22 + a23 Ð2 . 511 Sin pérdida de generalidad presúmase que a2 + b2 = 1. k2 ≥ Xn 1 k=1 k . Así pues. x−1 y−1 z−1 + + = 3− x y z Å x−1 y−1 z−1 + + x y z 1 1 1 + + x y z ã = 1. Ahora bien. x y z x y z xyz ≥ 3 (x + y + z) ⇐⇒ ab + bc + ca ≤ 1. De aquí. a b ac + bd 512 De CBS se obtiene … p √ √ √ x−1+ y−1+ z−1 ≤ x+y+z Ahora bien. x y z 3 esto es. llegando a la conclusión. 2 1 1 1 1 1 1 513 Póngase a = . Habrá igualdad si y sólo si x−1 y−1 z−1 = 2 = 2 . 514 Por CBS se tiene S2 − a2i ≥ 1 (S1 − ai )2 . 1 ≥ 3 (ab + bc + ca) . c = . Entonces c2 + d 2 = 1.Indicaciones y respuestas 211 De aquí. de donde se deduce la conclusión. a + b + c ≤ 1. Así. a2 + b2 + c2 ≥ ab + bc + ca. Å Ahora bien. x2 y z 1 1 1 + + = 2. Ð a2 + c2 b2 + d 2 1 2 + = a + b2 + c2 + d 2 = 1. si x = y = z = . n−1 ≥ k=1 X n aˇk k=1 ya que aˇk ≥ k y las a’s siendo enteros distintos estrictamente positivos. ac + bd ≤ c3 d 3 + a b ã (ac + bd) ≥ c2 + d 2 Ð2 = 1. S1 − ak ≥ X n ak k=1 k2 n 1 X (S1 − ak ) = S1 . La restricción xyz ≥ xy + yz + zx. Por otra parte. i=1 que se sigue inmediatamente de la desigualdad de reordenamiento. Entonces an ≥ bn ≥ cn y desigualdad de reordenamiento y an bn cn an bn cn + + ≥ + + b+c c+a a+b a+b b+c c+a bn cn an bn cn an + + ≥ + + . y = c + d + a. an + bn ≥ de donde De igual manera. Ð 1 2 a + b2 + c2 + d 2 (a + b + c + d) . S≥ 4 x y z t Usando otra vez Chebyshev. x y z t Ahora bien. 48 3 xi yi .t = a + b + c. 529 Por simetría se puede suponer. 528 Póngase an+1 = a1 . 1 1 1 ≥ ≥ . que a ≥ b ≥ c. b+c c+a a+b c+a a+b b+c En resumen. 4 De a2 + b2 + c2 + d 2 ≥ 1 y 3 (a + b + c + d) = x + y + z + t se concluye. a2 + b2 + c2 + d 2 ≥ ab + bc + cd + da = 1 y por la desigualdad de Chebyshev Å ã Ð 1 1 1 1 1 3 a + b3 + c3 + d 3 + + + . sin pérdida de generalidad. a3 + b3 + c3 + d 3 ≥ Å S≥ 1 1 1 1 1 (x + y + z + t) + + + 48 x y z t 527 La desigualdad deseada es equivalente a Xn i=1 Xn xi zi ≤ ã ≥ 16 1 = . bn cn 1 an + + ≥ b+c c+a a+b 2 Por la desigualdad de Chebyshev. Å an + bn bn + cn cn + an + + a+b b+c c+a Ð 1 n−1 a + bn−1 (a + b) .212 Anexo A 526 Póngase S= b3 c3 d3 a3 + + + . z = d + a + b. por la desigualdad del reordenamiento. Entonces an ≥ bn ≥ cn ≥ d n para un entero n > 0 y 1 1 1 1 ≥ ≥ ≥ . 2 Ð 1 n−1 an + bn ≥ a + bn−1 . Por la b+c c+a a+b . a+b 2 Ð bn + cn 1 n−1 ≥ b + cn−1 b+c 2 ã . De la desigualdad de reordenamiento se tiene. Por simetría y sin pérdida de generalidad se puede presumir que a ≥ b ≥ c ≥ d. i=1 dando la conclusión. b+c+d c+d +a d +a+b a+b+c x = b + c + d. Xn i=1 X a2i ≥ ai+1 n i=1 a2i X = ai n ai . + + ≥ a3 × (1 + y) (1 + z) (1 + z) (1 + x) (1 + x) (1 + y) (1 + a)3 Luego pues. 360◦ 602 Mídanse los ángulos en sentido dextrógiro. Por la desigualdad de Chebyshev ! ! n Xn Xn 1 X xi ln xi . asegura la conclusión. xi ln xi ≥ n i=1 i=1 i=1 o lo que es lo mismo. a las 12 : 00. Entonces también lo será (ln xi ). Cuando el minutero está en el 8. éste ha viajado 60 3 2 360◦ = 240◦ 3 y el horario se ha movido 2 del camino desde el 4 al 5. (1 + y) (1 + z) (1 + z) (1 + x) (1 + x) (1 + y) Por la desigualdad de Chebyshev. x3 y3 z3 6 . +∞[. siendo el origen (0◦ ). i=1 531 Por simetría se puede suponer. 3 Å 1 3a = x + y + z ≥ 3 (xyz) 3 = 3. sin pérdida de generalidad que (xi ) la sucesión es creciente. que x ≤ y ≤ z. habiendo viajado 3   4+ 2 (30◦ ) = 140◦ . sin pérdida de generalidad. el percatarse de que es estrictamente creciente sobre el intervalo ]0. el ángulo entre una y otra manecilla es 240◦ − 140◦ = 100◦ . Entonces 1 1 1 ≤ ≤ . así. y 60 40 2 = de la circunferencia. esto es. Así. Luego por la desigualdad de las medias. . Cada minuto corrido por el minutero cuenta = 6◦ . 4 (1 + a) 3 Ya que a ≥ 1 y como  a → 6 1− 1 1+a 6a3 (1 + a)3 3  = 6 1− 1 1+a 3 . 3 x3 + y3 + z3 ≥ a3 . 3 Luego. las manecillas viajan 5(6◦ ) = 30◦ . x3 y3 z3 + + (1 + y) (1 + z) (1 + z) (1 + x) (1 + x) (1 + y) x3 + y3 + z3 1 1 1 [ + + ] 3 (1 + y) (1 + z) (1 + z) (1 + x) (1 + x) (1 + y) 3+x+y+z x3 + y3 + z3 × 3 (1 + y) (1 + z) (1 + x) ≥ = Póngase x+y+z = a.Indicaciones y respuestas 213 y Ð 1 n−1 cn + an ≥ c + an−1 c+a 2 y así. Å Yn 1 n Yn i=1 xxi i ≥ xi Pn ã xi i=1 . an bn cn an−1 + bn−1 + cn−1 + + ≥ . b+c c+a a+b 2 530 Por simetría se puede suponer. es suficiente demostrar que 6a3 3 ≥ . (1 + y) (1 + z) (1 + x) ≤ (1 + x) + (1 + y) + (1 + z) 3 ã3 = (1 + a)3 . de hora a hora. La otra se obtiene de manera semejante. Así 180◦ − 162◦ ‘ ‘ ‘ = 360◦ − 108◦ − 90◦ = 162◦ . Como △AEF es isósceles. Como los ángulos del hexágono son 120◦ cada uno. el △XY Z es equilátero. se desprende que E‘ = 9◦ . que es 20◦ . AC = 20 + r y CB = 6 + r. Luego ZA + AF + FX = XE + ED + DY. de donde. . 604 De la figura A. 612 Las tangentes a un círculo desde un punto son congruentes. AB + AF + EF = EF + DE +CD =⇒ AB − DE = CD − FA. Cada uno de los ángulos internos mide pues 540◦ ÷ 5 = 108◦ .4 se percata que ADH T A B H C G D F E Figure A. 360◦ . El ángulo exterior debe ser lo suficientemente pequeño para que n 360◦ divida al máximo común divisor de 40◦ y cuando la figura rote 40◦ o 60◦ . como en la figura A.5.4: Problema 604. 613 π 7 614 Extiéndanse los lados del hexágono. Luego ADH CH = 45◦ . Como r > 0 se tiene r = 4. se obtenga una figura homóloga. dando una de las igualdades. Así.214 Anexo A 603 La suma de los ángulos internos de un pentágono es 3 × 180◦ = 540◦ . Esto se satisfará cuando n 360◦ = 20◦ tiene la solución n = 18. y en restando los lados de los triángulos menores. éste es el mínimo requerido. La igualdad de áreas revela que     1 1 1 (20 + r)(6 + r) = r2 + 2 (20)(r) + 2 (6)(r) 2 2 2 =⇒ 120 − 26r − r2 = 0 =⇒ (30 + r)(4 − r) = 0. Como n 605 Sea n el número de lados requerido. Cada ángulo exterior mide 611 Se tiene AB = AD + DB = AE + DB = AC − EC + DB = AC −CF + DB = AC − (BC − FB) + DB = AB − BC + DB + DB = AB − BC + 2DB. 60◦ . en resolviendo para DB. se obtiene el resultado. A EF = 360◦ − A ED − EDF AF = 2 ‘ =A ‘ ‘ + A‘ CT y que △T HC es rectángulo isósceles. Entonces PF = a. [MC] son todos congruentes (en efecto. M BA = MAB. 2 2 2 . siendo h1 la altura del trapecio ABNM y h2 la altura del trapecio MNCD. B BA + MCA dAC = se deduce B π . [AH]. la longitud de cualquier cateto es menor que la longitud de la hipotenusa. BHB < BC. OMC = OMB . 674 Sean D. MN = x y CD = b. ‘ Así pues 636 Como △BAM es isósceles. 2 637 Obsérvese primero que por el teorema de Pitágoras. ‘ ‘ = MAB ‘ + MAC ‘ = BdAC. radios del circuncírculo del △AHC) y por lo tanto ‘ = 30◦ .5: Problema 614. Así BMC = CD y AB = AC. PE = 2a. E. gracias al teorema 653 y BOM = △COD. A MC MB O C B Figure A. los pies de las perpendiculares a los lados [BC]. BO = CO. Por lo tanto dandoa = √ 3 . Luego AHA < AB. [CA] y [AB] respectivamente. Entonces h a+b = 1. ‘ ‘ De la misma manera. 673 7. PF = 3a. △MAC es isósceles y MCA ‘ = MAC. 4 1 ·3· 2 … 32 −  32 2 = 1 1 1 · 3 · a + · 3 · 3a + · 3 · 2a. MHC ’C = COM ’B . △BOE ∼ 659 En la figura A. 669 Póngase AB = a.Z B A Indicaciones y respuestas 215 C F X Y E D Figure A. 639 El △AHC es rectángulo en H. M BA + MCA Pero como la suma de los ángulos internos del △ABC es dAC + M ‘ ‘ = π. CHC < CA =⇒ AHA + BHB +CHC < AB + BC +CA. Sea h = h1 + h2 la altura del trapecio. Por lo tanto. F.6: Problema 659. y [ABC] = [APB] + [APC] + [CPA]. [AM]. dando el resultado. 2(a + x) h a+b h a+b a+b = 2 =⇒ = 1− =⇒ x = 2(b + x) h 2(a + x) 2(b + x) … a2 + b2 . 2 √ 670 6 3 − 2π .6. Así. Se tiene [△AOB] = [△B′ OA]. r el área de la región roja y b el área de la región azul. La información dada estipula que las cuerdas dividen a A en dos regiones: una de área y otra de 4 4 Divídase el área A del círculo exterior en nueve regiones: cuatro rincones como el área sombreada que se denominará 4P. El largo del rectángulo es pues (2c + 1) + (c + 1) + (c + 2) = 4c + 4. Continuando este proceso se deduce que c6 tiene lado 2c + 1. Conclusión: el rectángulo tiene área 33 × 32 = 1056. Si x es el lado del cuadrado central entonces P = S x2 π x2 = . El cuadrado c7 tiene lado c + 3 + 4 = c + 7. dos de área 2P + Q y una de área 2Q + S. El cuadrado c8 tiene lado c + 7 + 4 = c + 11. Se colige que 4 4 2 P = I x2 4 π x2 2 = 1 . Entonces se ve que c2 tiene lado c + 1. [△BOC] = [△COB′ ]. 2π 679 Sea y en la figura adjunta. 4 . 4 y substitutyendo. dos lados opuestos del rectángulo tienen dimensiones 4c + 4 y (c + 7) + (c + 11) = 2c + 18. 2 8 La última igualdad resulta en 2y = π R2 − 2b. 4 y+b = π (R/2)2 π R2 = . Como han de ser iguales se deduce que 4c + 4 = 2c + 18 =⇒ c = 7. Se quiere r + b. que se denominará S. 3A A . Obsérvese que [ABCA′ B′C′ ] = 2[△AOB] + 2[△BOC] + 2[△COA′ ]. [ABCA′ B′C′ ] = 2([△B′ OA] + [△COB′ ] + [△AOC]) = 2[△ACB′ ] = 2. [△COA′ ] = [△AOC]. cuatro entre los rincones. r + b + 2y = π R2 . 678 Se A el área del circulo exterior. 4 Luego 2Q + S = 2(2P + Q) =⇒ S = 4P. 2 2P + Q = A . El ancho del rectángulo es pues (c + 7) + (c + 3) + (c + 2) = 3c + 12. que se denominará 4Q y el cuadrado. Así A = 4P + 4Q + S. c3 tiene lado c + 2 y c4 tiene lado c + 3. De aquí se sigue que c5 tiene lado 4. 677 Sea O el centro del círculo. El área de la banda central y una de las bandas exteriores se pueden ahora expresar como 2Q + S = A . Las cuerdas dividen ahora al círculo mayor en tres bandas. π R2 − 2y = 2b. r+b = de donde r = b. El área del círculo interior es I = . el área de la región amarilla. Ahora bien.216 Anexo A 675 Supóngase que c1 tiene lado c. Finalmente. El área deseada es pues 100π . 2 . El punto B. La mitad del área de la porción azul se puede hallar en substrayendo del área OCD el área del △OCD:  2 R b 1 = ·π 2 4 2 − R2 (π − 2) 1 R R · · =⇒ b = .Indicaciones y respuestas 217 Se desea ahora hallar el área de la región azul. 2 4 2+r 685 Sea [AD] tangente al círculo P en E. AB = 2 − r. AC = 2 + r. Por el teorema de Pitágoras. Además. 4 A D O B C 684 Únanse los centros A y C como en la figura 7. Entonces DE = DB y √ AE 2 = AP2 − PE 2 = 36 − 4 = 32 =⇒ AE = 4 2.101. 20 R2 − r2 = ( )2 = 100. Como ‘ ABC = y ACB 2 4 BC = 2 − r. Evidentemente. que a su vez √ AD2 = BD2 − AB2 =⇒ (AE + BD)2 = 64 + BD2 =⇒ (4 2 + BD)2 = 64 + BD2 . respectivamente. Por el Teorema de Pitágoras. a los lados del cuadrado que pasan por A y C. △ABC es isósceles. Esto simplifica a r2 − 12r + 4 = 0. Se puede también obtener a r observando que √ 2 π 2−r = sen = . 686 Sean R y rlos radios del círculo externo e interno. El área buscada es π (R2 − r2 ). directamente abajo del punto A. Para lograr esto. 2 2 (2 − r) + (2 − r) = (2 + r)2 . obsérvese que △OCD es rectángulo en C. es la intersección de las de rectas paralelas π d π = . 2 2 2 8 El área buscada es entonces b + r = 2b = (π − 2)R2 . √ de donde r = 6 − 4 2. escogiendo la raíz menor que 2. √ √ =⇒ 32 + 8BD 2 + BD2 = 64 + BD2 =⇒ BD = 2 2. Luego a = . Sea x = HD. en donde r es el radio del círculo menor y a es la medida del segmento de recta fuera del círculo menor desde el centro del círculo mayor. Finalmente. 2 84 = 21. al descartarse una raíz extraña.25. B . ya que a > 0. DM = 1. se tiene CM = 9.218 Anexo A O A C 687 Únase los centros de ambos círculos. [△ABC] = 84. Ahora bien. En utilizando el teorema de Pitágoras. Use el teorema 652. Como [AD] es la bisectriz angular de A CD 8 = .7: Problema 712. como en la figura. 711 Gracias a la fórmula de Herón. 3 3 A C HA DMA Figure A. paralelos a los dos radios del círculo mayor. Así pues 1 84k2 = 84 − 41(k + 1) =⇒ k = . Como CM = MB. se tiene por el teorema de la bisectriz. El área buscada es entonces 4 b. DB 11 Sea CD = 8a y DB = 11a para alguna constante a. △ABC ∼ △A′ B′C′ y por lo tanto [△A′ B′C′ ] = 84k2 donde k es la constante de semejanza. obteniendo un cuadrado de lado r. R = a + 2r = ( 2 − 1 + 2)r =⇒ r = √ 2+1 710 Trace [KL] [AB] entre [KL] y [EF]. Como DM = 1 se tiene 16 22 2 y DB = . 712 Véase la figura A. El área del △A′ B′C′ también puede obtenerse al substraer él área de los tres trapezoides de altura 2 formados al unir los vértices correspondientes de los triángulos. Luego CD = 3 3 3 16 22 82 − ( − x)2 = AHA2 = 112 − ( + x)2 =⇒ HA MA = 1 + x = 2. √ r2 + r2 = (a + r)2 =⇒ a = ( 2 − 1)r.5a. Resulta que EF = 9.7. Obsérvese que R = a + 2r.5a. √ √ R = ( 2 − 1)R. Trácense dos radios del círculo menor. Como la hipotenusa de estos triángulos está en √ 135 15 135 290 15 . Ahora. 716 Sea M el punto medio de BD. 3k = 34. AC AB 2 2 Para completar. proporción. Se sigue que △ABE es isósceles en E. 2 2 2 714 Póngase KA = [BCO] KB = [CAO] KC = [ABO]. PR y QS son iguales y perpendiculares. AB = p AD2 + BD2 = p 162 + 122 = p 42 · p 42 + 32 = 20. OC′ KC Se deduce que 92 = KB + KC KA + KC KA + KB + + =⇒ KA2 KB + KA2 KC + KB2 KA + KB2 KC + KC2 KA + KC2 KB = 92KA KB KC . Del ejemplo 854. Así. AE = E B = 10. Since BD 3 AC = = . OA′ OB′ OC′ KA KB KC KA KB KC ‘ ya que son opuestos por el vértice.D E Indicaciones y respuestas 219 A B E′ 713 Sea E ′ la proyección perpendicular de E sobre el segmento [AB]. de donde r = 8. Por lo tanto. Entonces [ABO] [CAO] [ABO] + [CAO] KB + KC AO = = = = . Así 715 En la figura 7. B AE = A BE y por lo tanto AE = EB. Ak2 = . cada uno de estos pequeños triángulos es semejante a los triángulos rectángulos mayores (de catetos 5 y 3). 3 1 de donde FE = 15. [△AEB] = 1 15 1 · AB · EE ′ = · 20 · = 75. B △EFB ∼ EDC. Un giro de 90◦ toma al △PMR al △QMS. Esto resulta en A = 2 = 2 68 17 17 2k 718 5 : 1 . 717 Sea A el área de uno de los triángulos rectángulos de los rincones (con hipotenusa 3). Por Pitágoras. y también MQ y MR son iguales y perpendiculares. DC = B FC. AD 4 AB ‘ ‘ △ABC ∼ △DAB. El área buscada es 25 − 4A. △AE ′ E ∼ △BAC y por lo tanto EE ′ AE ′ 1 15 = = =⇒ EE ′ = . Si r es el radio del círculo entonces 2r = FC = FE + EC = 16. Por otro lado. Luego FE EB = ED EC o FE 5 = . ya que subtienden el mismo arco. Como EE ′ es la altura a la base de 20 de ′ ′ un triángulo isósceles. para alguna constante k. También B‘ ‘ ‘ EF = CED. Finalmente 25 − 4A = 25 − = . Luego. MP y MS son iguales y perpendiculares. OA′ [BOA′ ] [OA′C] [BOA′ ] + [OA′C] KA De manera semejante se puede demostrar que KA + KC BO = OB′ KB CO KA + KB = . KA KB KC Así KA2 KB + KA2 KC + KB2 KA + KB2 KC + KC2 KA + KC2 KB + 2KA KB KC AO BO CO KB + KC KA + KC KA + KB · · · = · = = 94.122. ← → 721 Tómese K sobre FG tal que [DK] [EF]. De triángulos semjantes. De aquí FC = = 6 y FG = 3. b = 168. y a [AB] en su punto medio. 10 722 Sea N el punto medio del segmento [PQ]. CD DA y DN = . 105 120 + 105 + 90 Luego y = 210. Para esto. A. Observe que hay dos posibilidades para ← → ← ← → ← → ← → → P. de donde se infiere que MN = Obsérvese que a = (AD)(DC) = (AB)(BC) y que [ABCD] = = = = = Así. HE EF 2y 2 3y 2 5x 10 9 . Entonces x + 3x + 3x = 84 =⇒ x = 12. 4 4 [AMND] + [△APM] + [CPMN] + [△APB]  DA + MN   CP + MN  AB × PB NC + 2 2 2  3CB  3CD AB × BC  5DA  CD +b+ + 8 4 8 4 4 9a a 5a +b+ + 32 32 4 22a +b 32 DN + b + a 16 = . se sigue que La razón requerida es DK DK 3 DK 3 3x 9 DH = = = · = · = . RD = 2MN. constrúyase la mediatriz de [AB]. AC BC 720 Sea x = AC. BF BG 5 Como △HFE ∼ △HKD. llámese Q. 120 + 90 + b b = 252 252 + 105 + y y 252 + y + b y = . b 5 N D Q C M P R B A ← → ← ← → → ← → 740 Sean los dos puntos A y B y la recta L . ← → ← → Si AB no es paralela a L entonces las dos rectas se intersecarán en un punto. Esta intersecará a L en algún punto. Se deduce que el 2 2 perímetro buscado es 39. llámese T . B. . El círculo de centro O y radio OP cumple las condiciones deseadas. Eríjase ahora la perpendicular ℓ1 a L desde P y la mediatriz ℓ2 del [AB]. Además DC = = 18. El centro del círculo buscado es O = ℓ1 ∩ ℓ2 . obsérvese que P debe satisfacer T P2 = TA · T B. se sigue que DG 9y DK = =⇒ DK = . Si AB L . Sea O el punto medio de [PQ].220 Anexo A 719 Sea b el número de unidades cuadradas pintadas en azul y y el número en amarillo. dígase P. Por triángulos semejantes. conocidos T. luego P se construye con el proceso usado para construir la media proporcional. Se tiene que localizar el punto de tangencia P del círculo con la recta. Ya que △GDK ∼ △GBF. De aquí. CQ = CQ′ . Sean Q y Q′ las proyecciones perpendiculares del centro del círculo superior sobre los segmentos [AC] y [BC] respectivamente. Obsérvese que △ABC ∼ △OT O′ . R 2 b b C A + 2R + R cot = 5 =⇒ R = 2 2 5 b Cb A cot + cot + 2 2 2 . 2 [△O′ BC] = 2R.Indicaciones y respuestas 221 757 Sean P y P′ las proyecciones perpendiculares del centro del círculo superior sobre los segmentos [AB] y [AC] respectivamente. R 2 Luego. 2 2 5 10 Finalmente. Ahora bien. 6= 5 39R 5R + + 2R =⇒ R = . En considerando triángulos rectángulos apropiados se obtiene b x A = cot . R = » » Cb 1 cosC 1 3 b cos 2 C 2+ 2 2 + 10 =» = 2. Sea S la proyección perpendicular de O′ sobre [AB] y sea T la proyección perpendicular de O sobre [SO′ ]. BC AC 4 5 5 Así. 2+3+2 7 Aliter: Tengan los círculos centros O y O′ como en la figura adjunta. 1 Se hallará ahora [△O′ AB] = (AB)(SO′ ). Porque el círculo de centro O′ es tangente a los lados [BC] y [AC]. x + 2R + y = 5 =⇒ R cot Además. Se tiene 6 = [△ABC] = [△O′ BC] + [△O′ AC] + [△O′ AB]. b y C = cot . =» 2 b 2 1 1 cos A A − − 2 2 2 5 sin 2 De la misma guisa. se tiene 5 [△O′ AC] = R. Obsérvese que AP = AP′ . 10 2 7 A A P′ O P Q O′ S Q T B Q ′ C B Q′ C . SO′ = ST + T O′ = R + 8R 13R = 5 5 y 1 3 13R 39R [△O′ AB] = (AB)(SO′ ) = · = . » b » A 1+2 1 + cos A b cos 2 A 2 5 2 2 cot = =» = 3. Póngase AP = AP′ = x y CQ = CQ′ = y. 2 O′ T OO′ O′ T 2R 8R = =⇒ = =⇒ O′ T = . AP′ + 2R +CQ = 5. cot = =» 2 b 3 1 1 − cosC C 2 2 2 − 10 sin 2 5 5 = . p p p (a + c)2 + (b + d)2 = OM ≤ OL + LM = a2 + b2 + c2 + d 2 . 0). en donde ocurre igualdad si y sólo si los puntos son colineales. 0) hasta (2.√b > 0 entonces la recta que une a (−b. Así pues. póngase O = (0. Esto es porque si a > 0. en tanto a+b ≤ a2 + b2 . dos de los ángulos del rombo serán 8 4 las diagonales de un rombo se bisecan en ángulos rectos y como bisecan los ángulos en los vértices. s2 sen A = 1 dd ′ π =⇒ sen A = =⇒ A = . 0) hasta (a. y luego desde (0. 1) hasta el origen (0.222 Anexo A 763 Si A es este ángulo. Pero entonces. Pero por la desigualdad de la media 2ab ≤ a2 + b2 =⇒ (a + b)2 = a2 + 2ab + b2 ≤ 2a2 + 2b2 =⇒ a + b ≤ √ p 2 a2 + b2 . Observe que cuatro de los vértices del octágono provienen directa8 8 3π π y los otros dos serán . 7 sen θ sen 3θ sen 3θ sen 2θ 794 Osama deberá viajar sobre dos segmentos de recta: primero desde (−1. Para esto. Por la desigualdad del triángulo OM ≤ OL + LM. Pero en el camino sobre los ejes desde (−b. b + d). 2 2 6 (8 − 2)π ◦ 3π = . 2 la cucaracha deberá evitar el segundo cuadrante a toda costa. Entonces − = = . b d . √ π 1 1 2− 2 2 − 2 cos 4 1 2 + 21 cos d ′ /2 cos π /8 d/2 sen(p − q) . esto es. 0) y (a. Sean d > d ′ las diagonales de los rombos. b. Como mente de los rombos. 7√ 21 20 779 Poner θ = 1 π 1 2 cos 2θ 1 . evitando a toda costa el segundo cuadrante. 0) es y mide a2 + b2 unidades. hallando el área de dos maneras distintas. si = . d sean positivos. La cucaracha pasa aquí unos 2 a + b unidades de largo y la cucaracha sólo invierte unos a + b minutos aquí. se sigue 2 2 2 lo anunciado. se obtiene de la figura adjunta que 764 Cada ángulo interno de un octágono regular mide q π d π 8 = cot = π =q d′ 8 sen 8 π p √ 4 = p 2 + 2 = √2 + 1. 0). Ya que < √ . 0) yace en el segundo cuadrante p a2 + b2 minutos. 1). c a y la igualdad ocurre si y sólo si los puntos son colineales. 1 1 1 lo que significa que en tanto la velocidad de la cucaracha en el segundo cuadrante sea < √ será mejor evitarlo. 795 Es suficiente demostrar la desigualdad en el caso de que todos a. L = (a. 767 tan p − tan q = cos p cos q 8 > > > > > < Sn = tan((n + 1)θ ) − tan θ sen θ si θ ≡ 0 mod 2π n > > > > > : si θ ≡ π −n 771 si sen θ = 0 mod 2π . b) y M = (a + c. c. Así. En este caso la recta es x = −7. Luego. −→ −−→ −→ MR = MD + MA. El valor buscado es S12 . Por lo tanto (x0 . y0 ) = (−7. Las rectas serán perpendiculares cuando = 4 or t = −2. −→ −→ −−→ NR = MR − MN. −→ −→ −→ MP = MB + MC. y por construcción. o sea. −→ −→ NR − PQ = −−→ −→ −→ −−→ (MR − MN) − (MQ − MP) = −→ −−→ −→ −→ −−→ −→ −→ −→ (MD + MA − MA − MB) − (MC + MD − MB − MC) = → − 0.Indicaciones y respuestas 223 797 Úsese la generalización del teorema de Minkowski y el hecho que 172 + 1442 = 1452 . 3) entonces (t − 2)(−2) + (t + 3)(3) + 10t − 5 = 0. ➌ Lt será paralela al eje de y si el coeficiente de y es cero. 8 de donde t = − . ➏ Si tal punto existiese. 11 11 11 ➋ Lt será paralela al eje de x si el coeficiente de x desaparece. por definición de N ′ . MN ′ = N ′ N. 798 Se tiene ➊ Si Lt pasa por (−2. entonces pasaría por las rectas verticales y horizontales de la familia de Lt arriba obtenidas. NPQR es un paralelogramo. se deduce. En ➎ La recta y = − x − 5 tiene pendiente − y Lt tiene pendiente 4 4 t +3 t +3 este caso la recta pedida es −4x + y − 25 = 0. AN ′ = N ′ B. En este caso la ecuación de la recta es 1 2−t 2−t 1 y Lt tiene pendiente . Así. En este caso la recta es y = −3. Las rectas serán paralelas cuando = or 2 t +3 t +3 2 5 10 5 − x + y − = 0. −3) es un candidato para tal punto. . t = 2. −→ −→ De manera semejante se establece que NP = RQ. lo cual necesita t + 3 = 0. Como = −−→′ −−→ MN + N ′ N = −−→′ −−→′ MN + MN = −→ −−→′ −→ −−→′ MA + AN + MB + BN = −→ −→ MA + MB −−→ −→ −−→ MQ = MC + MD. En este caso la recta es 11 − 30 25 135 x+ y− = 0. t = −3. −−→ −−→ −−→ −−→ 820 Se tiene. o sea. −→ −−→ −→ PQ = MQ − MP. −3) pasa por todas las rectas Lt no importa cual valor de t. Como (t − 2)(−7) + (t + 3)(−3) + 10t − 5 = −7t + 14 − 3t − 9 + 10t − 5 = 0 el punto (−7. ➍ La recta de ecuación x − 2y − 6 = 0 tiene pendiente t = 1/3. lo que ocurrirá cuando t − 2 = 0. −−→ MN Igualmente. 3 3 3 1 2−t 2−t 1 . Se recoge que los triángulos △ABC y △A′ B′C′ son copolares en O. esto es los puntos A. · MA A′ O C′C AN BB′ OA′ BL CM AN · · = −1 =⇒ = −1. A′ . i = j son Å ã n en número. N. b. (BC)2 NB CM (BC)2 . Luego A′ B A ′C =− 1−a . existe una recta en el plano. Encuéntrense BB′ y CC′ en O. · · ′ ′ NB B O A A LC MA NB de donde L.C′ ) y al △OAB (con puntos menelaicos N. M. 847 Por ser baricentros de sus respectivas rectas. ‘ (AB)2 BL AB sen B AL AB sen Cb =± . c con −→ −− → −− → −→ −− → −− → → − 0 = aA′ B + (1 − a)A′C = bB′C + (1 − b)B′ A = cC′ A + (1 − c)C′ B. se debe tomar el signo negativo. llamémosla D′ que no es paralela a D. Se tiene â 2 3 6 L7 6 7 6 7 6 M7 = 6 7 4 5 N ì2 3 0 a 1−a 1−b 0 b c 1−c 0 6 A7 6 7 6 7 6 B7 . Desplazando D paralelamente a si misma en la dirección de D′ se pasan los puntos Ai uno a uno. De igual manera (CA)2 AB =− . M. B′ . M = bC + (1 − b)A. a B ′C B′ A =− 1−b . 845 Se tiene por la ley de los senos. · LC C′ O B′ B CM AA′ OC′ · = −1. Se procede ahora de manera formal. =− (AB)2 MA El resultado se deduce ahora en multiplicando estas razones y en utilizando el Teorema de Menelao.224 Anexo A ←−→ 826 Como las rectas Ai A j . c con L = aB + (1 − a)C. 842 Por ser baricentros de sus respectivas rectas. O son colineales. De 2 igual manera. b C′ A C′ B =− 1−c c . ← → ←→ Recíprocamente. 6 7 4 5 C Para este sistema tener solución no trivial es necesario y suficiente que ì â det 0 a 1−a 1−b 0 b c 1−c 0 = 0 ⇐⇒ abc + (1 − a)(1 − b)(1 − c) = 0. b. = a LC CM 1−b . Luego. 846 Sean △ABC y △A′ B′C′ copolares en O. llamémosla D que no es paralela a ninguna de ellas. Ahora bien. Entonces BL CC′ OB′ · = −1. sean △ABC y △A′ B′C′ coaxiales en L. = c NB de donde se obtiene el resultado. existen reales a. △MCC′ y △NBB′ son copolares en L. existen reales a. Es suficiente detenerse cuando se hallan pasado p puntos. Aplíquese el Teorema de Menelao al △OBC (con puntos menelaicos L. A′ ). B′ ). existe una recta en el plano. A′ . por lo ya demostrado en la primera parte de este problema. N son colineales. Como BL 1 − a . △OCA (con puntos menelaicos M.C′ . = b MA 1−c AN . = =± (CA)2 LC CA sen L ‘ CA sen ‘ ABC AC Como la división del segmento [BC] es externa. N = cA + (1 − c)B. dichos triángulos son coaxiales. ′ PC B A ·C′ B . Así pues −→ −→ −→ −→ −→ −→ OR + OP r(OR − OP) OX − OY = + 2 2 2 es independiente de la posición del muelle. −x − (1 − c)y = c − 1. bx + y = b. De aquí CB′ · AB CP ·C′ B = B′ A PC′ y así CP CB′ · AB = ′ . abc como se quería demostrar. bx + y = b =⇒ x = ab . · · = +1 ⇐⇒ abc A ′C B ′ A C ′ B −→ −→ Se distingue dos casos. el roble y el pino.Indicaciones y respuestas y 225 −→′ − → − → AA = aAB + (1 − a)AC. −→ −→ −→ −−→ OY = OP − r(OP − OM). está en 2 ′ ′ BC. ab + 1 − a Estas soluciones deberán satisfacer la tercera ecuación y por lo tanto. Ahora bien. dependiendo de si AA′ BB′ o no. BB′ y CC′ si y sólo si (1 − a)x − ay = 0. −→ −→ −→ −−→ Si AA′ BB′ entonces ab = a − 1 lo que implica que b(c − 1) + 1 = 0. entonces ab = a − 1. Este es un sistema de dos variables y tres ecuaciones. − ab b(1 − a)(1 − c) − = c−1 ab + 1 − a ab + 1 − a ⇐⇒ −ab − b(1 − a)(1 − c) = (c − 1)(ab + 1 − a) ⇐⇒ ab − b + bc − 2abc − c + ca + 1 − a = 0 ⇐⇒ (1 − a)(1 − b)(1 − a) − abc = 0 ⇐⇒ (1 − a)(1 − b)(1 − c) = +1. Un punto P tal que AP = xAB + yAC yacerá sobre las tres rectas AA′ . Esto proporciona un algoritmo para encontrar el tesoro: tomad a P como el origen. Las condiciones del problema requieren vectores OX y OY que satisfagan −→ −→ −→ −−→ OX = OR + r(OR − OM). Sea r una rotación de 90◦ −→ −→ hacia la derecha. OR y OP vectores. −−→ −→ −→ 864 Sean OM. Obsérvese que de las igualdades arriba obtenidas se desprende que BA′ CB′ AC′ (1 − a)(1 − b)(1 − c) = +1. (1 − a)x − ay = 0. −→ −→ ← → ← → ←→ − → − → − → Si AA′ no es paralelo a BB′ . respectivamente. luego el tesoro − → − → PR + r(PR) . −→′ − → − → BB = −AB + bAC. desde un origen O arbitrario hasta el muelle. ab + 1 − a y= b(1 − a) . que a su vez implica que AA′ CC′ . por lo que una de ellas es redundante. Luego dBB y ψ = B‘ 881 Sea φ = A y CB′ CB sen ψ = B′ A AB sen φ en el △ABC CB sen ψ CP = PC′ C′ B sen φ en el △C′ BC. −−→ − → − → CC′ = (1 − c)AB − AC. 226 Anexo A Como AB = AC′ +C′ B. luego de donde se obtiene el resultado. B′ A ·C′ B A′ B . en substituyendo se tiene CB′ (AC′ +C′ B) CB′ · AB = ′ ′ B A ·C B B′ A ·C′ B ′ CB · AC′ CB′ ·C′ B + = ′ B A ·C′ B B′ A ·C′ B CB′ · AC′ CB′ + = ′ B A ·C′ B B′ A Por el Teorema de Ceva. CB′ AC′ BA′ · · =1 B′ A C′ B A′C CB′ · AC′ CA′ = .
Copyright © 2024 DOKUMEN.SITE Inc.